0% found this document useful (0 votes)
7K views552 pages

Engineering Mechanics - Singer, Ferdinand Leon, 1907 - 1954 - New York, Harper - Anna's Archive

Copyright
© © All Rights Reserved
We take content rights seriously. If you suspect this is your content, claim it here.
Available Formats
Download as PDF, TXT or read online on Scribd
0% found this document useful (0 votes)
7K views552 pages

Engineering Mechanics - Singer, Ferdinand Leon, 1907 - 1954 - New York, Harper - Anna's Archive

Copyright
© © All Rights Reserved
We take content rights seriously. If you suspect this is your content, claim it here.
Available Formats
Download as PDF, TXT or read online on Scribd
You are on page 1/ 552

‘MSE

repeated

eases
Ht i

|
5 é ; ; il antes 2! ae es

SeaeiahetRe ;. y
REY MnO eaten ea
*' 7 | ¥ 7 4 4 nie ‘ " "7 : Ni
vr
rer
eri
, a)
-
« "
4 . ‘ ae ’ ; av * 7v ie, y
i uy sags. p S ! ry PAA ee 4? Ae .
‘ t { rire he bi é fle

E tae
at
Pe ie .

mM. JAn O. Lone

Rm 248 EaseB fds 10


Pai,

eel
Di
De
ee

/
Digitized by the Internet Archive
in 2022 with funding from
Kahle/Austin Foundation

https://archive.org/details/engineeringmecha0O0O00sing
ENGINEERING MECHANICS
ENGINEERING
MECHANICS
pee Oe ND oe ee ee ee ee) |

Ferdinand L. Singer
ASSOCIATE PROFESSOR OF ENGINEERING MECHANICS
NEW YORK UNIVERSITY
COLLEGE OF ENGINEERING

HARPER & BROTHERS, PUBLISHERS


NEW YORK
ENGINEERING MECHANICS, SECOND EDITION
COPYRIGHT, 1941, 1943, 1954, BY FERDINAND L. SINGER
PRINTED IN THE UNITED STATES OF AMERICA

ALL RIGHTS IN THIS BOOK ARE RESERVED.


NO PART OF THE BOOK MAY BE USED OR REPRODUCED
IN ANY MANNER WHATSOEVER WITHOUT WRITTEN PER-
MISSION EXCEPT IN THE CASE OF BRIEF QUOTATIONS
EMBODIED IN CRITICAL ARTICLES AND REVIEWS. FOR
INFORMATION ADDRESS HARPER & BROTHERS
49 BAST 33RD STREET, NEW YORK 16, N.Y.

B-D

Library of Congress catalog card number: 54-6834


Contents

PAGE

Preface to the Second Edition xl


List of Symbols and Abbreviations XV

PART I—STATICS
Chapter I. Principles of Statics
. Introduction
. Fundamental Creepis an PN ;
. Force Systems .
. Axioms of Mechanics .
. Introduction to Free-Body ae ams
Scalar and Vector Quantities
7. Parallelogram Law.
—8. Triangle Law
. Solution of Problems .
. Dimensional Checks
. Conversion of Units WwW
FW
COONA

. Numerical Accuracy — =>

Chapter II. Resultants of Force Systems


. Introduction
. Forces and C Ppeucnte oe
3. Resultant of Three or More Concurrent Ponies ’
. Moment of a Force ; oe
. The Principle of Moments. eae s Theorem
. Resultant of Parallel Forces SK
OP
Oe
nds
Fe
Wn
SS

. Couples . 36
. Resolution of a Rarece aia aWe fue a Coagle: 39
. Resultant of Non-Concurrent Force Systems 41
Summary 47

Chapter III. Equilibrium of Force Systems


. Definition and Meaning of Equilibrium. 49
3-2. Free-Body Diagrams . 49
Vi CONTENTS

. Equilibrium of Concurrent Force Systems .


. Conditions of Equilibrium from Moments .
. Three Coplanar Forces in Equilibrium Are Coneunene
. Equilibrium of Parallel Forces
. Equilibrium of Non-Concurrent Force cate
Summary

Chapter IV. Analysis of Structures


. Introduction :
. Construction of Sa spree
. Method of Joints
4. Method of Sections .
. Redundant Members; Callnes eos : 102
. Definition and Characteristics of Three-Force Membees 104
. Method of Members Applied to Frames Containing Three-
Force Members . 105
Summary 112

Chapter V. Friction
. Friction Defined 114
. Theory of Friction 114
. Angle of Friction 116
. Laws of Friction 118
. Further Problems in eee 124
. Wedges . ; 130
. Square-Threaded ee be ; 134
. Belt Friction 136
. Rolling Resistance 140
Summary 141

Chapter VI. Force Systems in Space


. Introduction : 143
. The Three Mutually Berner Coe of a For ce . 143
. Resultant of Concurrent Force Systems in Space . 146
.Moment of a Force about an Axis 148
. Equilibrium of Concurrent Space Forces 152
. Equilibrium of Non-Concurrent Space Forces . 157
Summary 166
ContTENTS vii

Chapter VII. Centroids and Centers of Gravity

7-1. Introduction 168


7-2. Center of Gravity of a Flat Plate 168
7-3. Centroids of Areas and Lines. 169
7-4. Importance of Centroids and Moments of nee ilar
7-5. Centroids Determined by Integration Lh
7-6. Centroids of Composite Figures . 176
7-7. Theorems of Pappus ae aes eat, we ok 185
7-8. Center of Gravity of Bodies. Centroids of Volumes . 188
Summary 193

Chapter VIII. Moments of Inertia


8-1. Definition of Moment of Inertia . 195
8-2. Polar Moment of Inertia . 196
8-3. Radius of Gyration 197
8-4. Transfer Theorem for Meer of rey tia 197
8-5. Moments of Inertia by Integration . 199
8-6. Moments of Inertia for Composite Areas 202
8-7. Product of Inertia . 210
8-8. Product of Inertia Is Zero w ith eenen toee i prea 211
8-9. Transfer Formula for Product of Inertia 211
8-10. Moments of Inertia with Respect to Inclined Axes 215
8-11. Mohr’s Circle for Moments of Inertia 217
8-12. Maximum and Minimum Moments of Inertia. ferret nye 220
8-13. Moments of Inertia of Bodies 222,
8-14. Radius of Gyration 223
8-15. Transfer Formula for Mass Nomments of Tee 223
8-16. Mass Moments of Inertia by Integration 224
8-17. Moments of Inertia of Composite Bodies 228
Summary 232

PART II—DYNAMICS
Chapter IX. Principles of Dynamics
9-1. General . ; 237
9-2. Kinematics and netics ; 238
9-3. Motion of a Particle 238
9-4. Newton’s Laws of Motion for a Torre 241
ConrTENTS

. Fundamental Equation of Kinetics for a Particle .


. Effective Force on a Particle. Inertia Force
. D’Alembert’s Principle
. Motion of the Center of Gravity of ie ea
. Applying the Principles of Dynamics
Summary

Chapter X. Rectilinear Translation

rO—L: Definition and Characteristics of Translation .


10=2: Rectilinear Motion with Constant Acceleration
10-3. Freely Falling Bodies, Air Resistance Neglected
10-4. Rectilinear Motion with Variable Acceleration.
10-5. Motion Curves a EL ital
O62 Kinetics of Rectilinear Tr Sod Sreviee: as a Particle .
1057; Dynamic Equilibrium in Translation. Analysis as a Rigid
Body .
Summary

Chapter XI. Curvilinear Translation

pn Curvilinear Motion of a Particle. Fundamental Concepts


IN Velocity in Curvilinear Motion
fieoe Rectangular Components of Acceleration
11-4. Flight of Projectiles. Air Resistance Neglected
Ti: Tangential and Normal Components of Acceleration .
Pi=6- Kinetics of Curvilinear Translation. Dynamic Equilibrium
IEP Banking of Highway Curves .
Summary

Chapter XII. Rotation


2b Rotation. Definition and Characteristics
12-2. Kinematic Differential Equations of Rotation
12-3. Rotation with Constant Angular Acceleration
12-4. Rotation with Variable Angular Acceleration
12-5. General Kinetics of Rotation .
ate Centroidal Rotation :
Veil Non-Centroidal Rotation. Deans Baan é
Summary
CoNTENTS 1x

Chapter XIII. Plane Motion


PAGE

13-1. Definition and Analysis of Plane Motion 338


13-2. Interpretation of the Kinematic Equations. 341
13-3. Relation Between Translation and Rotation for Freely eine
Disks and Spheres 343
13-4. Instantaneous Center and Eucsamianevin ee of Plauen 354
13-5. The Instant Center Does Not Have Zero Acceleration 356
13-6. Equations of Plane Motion 363
13-7. Rolling Bodies . : 365
13-8. Dynamic Equilibrium in Bane Mipfon 373
Summary 382

Chapter XIV. Work and Energy


14-1. Introduction . 384
14-2. Fundamental Work- Ty, Teqeation for Ree Treran
tion : 384
14-3. Meaning of W ot ; : 386
14-4. Application of the Work-Energy Method: Gthsiant Ferces 387
14-5. Resultant Work. Variable Forces 393
14-6. Power. Efficiency , 398
14-7. Work Energy Applied to ane (ir Tenet 400
14-8. Fundamental Work-Energy Equation for Rotation 403
14-9. Work-Energy Method with Constant Moments 405
14-10. Work-Energy Method with Variable Moments 409
14-11. Work-Energy Applied to Plane Motion. 416
Summary 424

Chapter XV. Impulse and Momentum


15-1. Introduction 427
15-2. Fundamental Pree Maeninns mata for reerelarion 427
15-3. Special Cases of the Fundamental Equation . 428
15-4. Linear Impulse-Momentum Applied to Translation 429
15-5. Displacement from Force-Time Curve 432
15-6. Dynamic Action of Jets 435
15-7. Conservation of Momentum 439
15-8. Elastic Impact . , 442
15-9. Fundamental Impulse- Renn Biriation in Potation 445
15-10. Rate of Change of Angular Momentum . 447
‘ CoNTENTS

15-11. Angular Impulse-Momentum Applied to Rotation . 447


15-12. Conservation of Angular Momentum 451
15-13. Impulse-Momentum Equations for Plane Molen : 454
15-14. Gyroscope 458
Summary 462

Chapter XVI. Mechanical Vibrations


16-1. Introduction. Definitions and Concepts 464
16-2. Simple Harmonic Motion. Free Vibrations 464
16-3. Simple Pendulum . 469
16-4. Compound Pendulum . 470
16-5. Torsion Pendulum. 473
16-6. Vector Representation of Snes iaenerie Motion 475
16-7. Free Vibrations Without Damping. General Case. 477
16-8. Free Vibrations Analyzed by Work-Energy Method . 484
16-9. Forced Vibrations . 486
Summary 494

Chapter XVII. Graphic Methods


17-1. Introduction 497
17-2. Application of Paraleoee ntaes 497
17-3. Resultant of Coplanar Force Systems 498
17-4. String Polygon . 499
17-5. Equilibrium of Force Sgerene i Gr fstie Methods 505
17-6. Graphic Analysis of Trusses. Maxwell Diagrams 508
Summary 516

Index 519
Preface to the Second Edition

The original edition endeavored to show how a few basic concepts—the


relation between a force and its components, the principle of moments, and
Newton’s laws of motion—may be combined and applied to a wide variety
of practical situations encountered by engineers. Another purpose was to
help the student to develop the logical, orderly processes of thinking which
characterizes the engineer. These objectives have been emphasized to an
even greater extent in this second edition.
While retaining the general plan and features of the original edition, the
text has been completely rewritten and embodies the suggestions received
from many users of the book. These are, to mention only a few, an ex-
panded discussion of free-body diagrams; a unified and coordinated treat-
ment of plane and space statics; motion curves discussed in more detail;
greater stress upon the motion of the center of gravity as a guiding principle
in deriving the equations of kinetics; simplification of the method of dy-
namic equilibrium; and the direct application of the work-energy method to
connected systems of bodies. Also the analysis of plane motion has been
simplified and amplified for both its kinematics and kinetics.
It is hoped that these topics, as well as the others in the text, are presented
in a manner that will relieve the instructor of the burden of detailed ex-
planations. This should permit him to expand and develop those details he
wishes to emphasize without sacrificing fundamental principles. But. pri-
marily, the student’s point of view has been retained and his special prob-
lems kept in mind. Words have not been spared to make a fundamental
principle perfectly understood. On the other hand excess verbiage has
been eliminated. Similar phrases have been used to express similar ideas
to enable the student to follow a new idea in terms of its predecessor. Al-
though this may make for undue repetition, it is repetition of a sort which
drives an idea home rather than disguises it in a new dress of different
phraseology.
Analytical methods are emphasized throughout the book, but graphic
methods have not been neglected. Graphic methods appear frequently in
the portion on dynamics; however, all graphic methods for statics have been
grouped together in a unified treatment to form the concluding chapter.
It is perhaps uncommon to do this, but this grouping of methods affords an
opportunity to coordinate them as well as to enable the student to find
quickly the particular graphic method he is seeking. Another variation
».¢ |
xl PREFACE

from conventional arrangement is the relegation of all the material on simple


harmonic motion and various types of pendulums to the chapter on vibra-
tions. It is felt that this is the natural place for these items and strengthens
the continuity of development. For the same reason, the kinematics and
kinetics of translation, rotation, and plane motion are developed concur-
rently in individual chapters in order to give ‘greater understanding of
kinematics by immediately applying it to the kinetics of a particular
motion.
Wherever possible, equations have been interpreted in terms of their
geometrical equivalents; the student is given a picture of what is being
discussed. The ability to visualize the equations results in practically no
need to memorize them. <A distinct effort is made to eliminate the memoriz-
ing of traditional rules of sign. Thus in dynamics, all rules of sign have been
reduced to one common denominator; namely, the initial direction of mo-
tion determines the positive sense of forces, moments, displacements,
velocities, and accelerations for all motions. Furthermore, the szgnzficance
of the sign of a quantity is emphasized so that the student can choose for
himself whatever systems of signs he finds most convenient. Also of im-
portance is the summary appended to each chapter. These summaries are
intended to give the student a coordinated picture of what he has studied.
It is hoped that they will be useful in review and post-college work.
Although the text material in this second edition has not been enlarged
to any appreciable extent, the number of problems has been greatly in-
creased. It is hoped that an instructor can thereby make an adequate
selection of problems without repeating more often than every three or four
years. Only the most stimulating problems of the original edition have
been retained, and all have been carefully selected to give wide variety in
type and scope. As before, they have been arranged approximately in their
order of difficulty, and answers to two-thirds of them have been given. No
problem is presented that is not preceded by an adequate text presentation
and, as a rule, numerical values were chosen which simplify the arithmetical
computations.
Numerous illustrative problems (many of them new) again show in de-
tail how principles are applied. The explanations are complete—nothing is
taken for granted. Throughout, the equation or theory to be applied is
stated at the left-hand side of the page. In the solution, numerical values
are substituted in the respective order in which the symbols appear in the
equation. This procedure enables the student to follow readily the various
steps of the solution without continually referring to the body of the text.
In addition, many supplementary explanations are given on the use of the
equations—why they were selected and what method to apply to their
solution.
PREFACE Xill

The numbering plan used in this revision enables one to locate quickly
any cross-reference. With this plan, all articles, figures, equations, tables,
and problems are preceded by the chapter numeral in which they appear
and are numbered consecutively through each chapter. Figures for as-
signed problems are given the same number as the problem to which they
refer in order to simplify correlation of a problem figure with the correspond-
ing problem data.
The author wishes to acknowledge his indebtedness to his colleagues all
over the nation for the many valuable suggestions for this revision which
they so generously offered to him. He is grateful also for the assistance
given in preparing the original edition by his former associates Dean
Emeritus William R. Bryans and Professor Charles EK. Gus. For the second
edition, special mention goes to Dr. Herbert F. Marco of the United States
Air Force Institute of Technology, not only for his many comments but for
the care with which he checked the problems. Grateful acknowledgment in
the checking of problems is due also to Mr. Peter H. Basch and Dr. Tobey
Yu. It is hoped that with their help the number of errors has been reduced
toaminimum. However, it is inevitable that some errors still exist, and the
author will appreciate being informed about these, as well as welcome any
other comments on the contents of the book.
FERDINAND L. SINGER
New York, New York
January, 1954

a

' L ral |

aided ee
me -
y)
A ath
bibs eal daveau why
- gut eit a,
=
nee P Paha i ; Le ey sAt uk F teitiee

uth |(it bibrwewy hema pl


nt
yoo ities meinen :
iit

ib to « ied onct ale “A aye


J ryan crag tt ersel ervey Weycolin A aviqunles
ane

i” tiGghwa Vike

io rape
x
” i
5 1 erry viet

4 ay Li
~~

! ve] ey
™ ;
LU

ao | pve s cael) alte ie fra


. ae =
} 4 bt g /S silico «any Boat
. ie '

leat nt!al selnt:beetvaath he Re


.

bo atc en Rg
ot 2 Sirs —_ dat! }ieiviinEed, ernme
= OP iF ato
(walt Be sen Wz
abi

CHT ae, cnstesimy wht ‘fiw -


.
’ ra vais 4
=
‘|

= =
»

= 0

“.

2 A

eo

We

: ay
List of Symbols and Abbreviations*

oe area, amplitude of vibration


acceleration of center of gravity
acceleration
normal acceleration
tangential acceleration
breadth, width
centroid, instant center
depth, diameter, distance, moment arm
modulus of elasticity in tension or compression
cats
£Sece
Go
— coefficient of restitution, natural base of logarithms
d© * static elongation
force, friction
coefficient of friction, frequency
modulus of elasticity in shear
gravitational acceleration (32.2 ft per sec?)
angular momentum
height
moment of inertia
centroidal moment of inertia
polar moment of inertia
ae
De
ics
enya
SQ
AN centroidal polar moment of inertia
kinetic energy
radius of gyration, spring constant
length
moment of force
normal component of force, normal pressure
revolutions per minute
te dea: ws forces or loads, concentrated
2inwedy
e products of inertia
ba reaction, resultant force
Tike radii
* With very few exceptions, these symbols and abbreviatians agree with those approved
by the American Standards Association.
XV
XV1 List or SYMBOLS AND ABBREVIATIONS
i

RW resultant work
displacement, arc length
period of vibration, tangential component of force
thickness, time
linear momentum
rectangular coordinates
volume
velocity of center of gravity
velocity
work
weight
orthogonal projections of a force
rectangular coordinates
coordinates of centroid or center of gravity
angular acceleration
angles
phase angle
weight per unit volume
angular displacement
polar coordinates
angle of friction
angular velocity
natural circular frequency (2 af)
angular velocity of precession
degrees
free-body diagram
feet
horsepower
hour
inches
kips (1000 Ib)
kilowatts
pounds
mph miles per hour
rad radians
rev revolutions
rpm revolutions per minute
rps revolutions per second
sec seconds
shm simple harmonic motion
Per I

STATICS
Chapter I.
Principles of Statics

1-1. Introduction

Engineering mechanics may be defined as the science which considers the


effects of forces on rigid bodies. The subject divides naturally into two
parts: statics and dynamics. In statics we consider the effects and distribu-
tion of forces on rigid bodies which are and remain at rest. In dynamics
we consider the motion of rigid bodies caused by the forces acting upon
them.
ENGINEERING MECHANICS
Statics Dynamics

Force Systems Applications Kinematics Kinetics


os ———

Concurrent Trusses Translation Translation


Parallel Centroids Rotation Rotation
Non-Concurrent Friction Plane Motion Plane Motion
Fic. 1-1.— Outline of engineering mechanics.

A visual introduction to the subject of engineering mechanics is repre-


sented in Fig. 1-1. The subject has two main divisions: statics and dynamics.
These are subdivided into two subbranches. In statics we consider first the
various types of force systems, then their application to the various condi-
tions shown. Not all phases of statics, however, are represented in the dia-
gram, merely the more common elements. Dynamics is similarly divided;
there are kinematics (which deals with the pure motion of rigid bodies)
and kinetics (which relates the motion to the applied forces). Each of these
subdivisions deals primarily with the rigid body motions of translation,
rotation, and plane motion. These terms are discussed at length in Part II.
For the present, we confine ourselves to statics.

1-2. Fundamental Concepts and Definitions


Rigid Body. A rigid body is defined as a definite amount of matter the
parts of which are fixed in position relative to each other. Actually, solid
bodies are never rigid; they deform under the action of applied forces. In
many cases, this deformation is negligible compared to the size of the body
3
4 a PRINCIPLES OF STATICS [Chap. I
cd aU kien ep SRS ee ee
and the body may be assumed rigid. Bodies made of steel or cast iron, for
example, are of this type. The study of strength of materials, however, is
based on the deformation (however small) of such bodies.
Force. Force may be defined as that which changes, or tends to change,
the state of motion of a body. This definition applies to the external effect
of aforce. The internal effect of a force is to produce stress and deformation
in the body on which the force acts. External effects of forces are con-
sidered in engineering mechanics; internal effects, in strength of materials.
The characteristics of a force are (1) its magnitude, (2) the position of its
line of action, and (3) the direction (or sense) in which the force acts along
its line of action.
The principle of transmissibility of a force states that the external effect
of a force on a body is the same for all points of application along its line
of action; i.e., it is independent of the point of application. The znternal
effect of a force, however, is definitely dependent on its point of application.
The unit of force commonly used in the United States is the pound, or
multiples of the pound such as the kip (1000 pounds) or ton (2000 pounds).
Units such as the gram and kilogram are also used. In this book we shall
use the ‘‘foot-pound-second”’ system of units; i.e., the common unit of
length is taken as the foot, of force as the pound, and of time as the second.
If other units happen to be specified in problems, it is generally desirable
to convert them into the foot-pound-second system before solving for the
answer.

1-3. Force Systems


A force system is any arrangement where two or more forces act on a
body or on a group of related bodies. When the lines of action of all the
forces in a force system lie in one plane, they are referred to as being
coplanar; otherwise they are non-coplanar. The coplanar system is obvi-
ously simpler than a non-coplanar system since all the action lines of the
forces lie in the same plane. We shall consider first a discussion of coplanar
systems; it will then be a relatively simple step to the discussion of non-
coplanar or space systems of forces.
The force systems are further classified according to their lines of action.
Forces whose lines of action pass through a common point are called con-
current; those in which the lines of action are parallel are called parallel
force systems; and those in which the lines of action neither are parallel
nor intersect in a common point are known as non-concurrent force systems.

1-4. Axioms of Mechanics


The principles of mechanics are postulated upon several more or less
self-evident facts which cannot be proved mathematically but can only be
Art. 1-5] Introduction to Free-Body Diagrams 5

demonstrated to be true. We shall call these facts the fundamental axioms


of mechanics. The axioms are discussed at length in subsequent articles
as they are used. At this time we shall merely collate them for reference
and state them in the following form:
1. The parallelogram law: The resultant of two forces is the diagonal
of the parallelogram formed on the vectors of these forces.
2. Two forces are in equilibrium only when equal in magnitude, opposite
in direction, and collinear in action.
3. A set of forces in equilibrium may be added to any system of forces
without changing the effect of the original system.
4. Action and reaction forces are equal but oppositely directed.

1-5. Introduction to Free-Body Diagrams


One of the most important concepts in mechanics is that of the free-body
diagram. This concept is discussed in detail in Chapter III where we first
really use it. It is introduced here to help the beginner distinguish between
action and reaction forces. To do so, it is necessary to zsolate the body being
considered. A sketch of the isolated body which shows only the forces
acting upon the body is defined as a free-body diagram. The forces acting
on the free body are the action forces, also called the applied forces. The
reaction forces are those exerted by the free body upon other bodies.
The free body may consist of an entire assembled structure or an isolated

T =100 lb

ie
W=100 lb

(b) E, (c)
Free-body diagram Free-body diagram
of point C of derrick
Fic. 1-2. — Free-body diagrams.

part of it. For example, consider the derrick shown in Fig. 1-2a. The free-
body diagram of pin C (Fig. 1-2b) shows only the forces acting upon C.
These forces consist of the weight, the pull 7’ exerted by the cable, and the
force P exerted by the boom. If the free-body diagram of the entire derrick
6 PRINCIPLES OF STATICS [Chap. I
anneete tt RS e
were desired, it would show only the forces acting on the derrick as in Fig.
1=2¢e:

1-6. Scalar and Vector Quantities


Scalars. Imagine two groups of marbles, one consisting of 10 marbles
and the other of 5. If a common group is formed by mixing them, the
resultant number will be 15 marbles, a result obtained by arithmetical
addition. Quantities which possess magnitude only and can be added
arithmetically are defined as scalar quantities.
Vectors. At point C of the derrick (Fig. 1-2b) suppose the weight W and
the tension 7 were each 100 lb. What is the force P in the boom? By
arithmetical addition the answer is 200 lb. This result, however, is incor-
rect, as can be determined by means of a measuring device placed in the
boom. Actually the force in the boom would vary as the boom was lifted.
The error is due to the fact that arithmetical addition was applied to
quantities which possess direction as well as magnitude. Such quantities
can be combined only by geometric addition, usually called vector addition.
A vector of a quantity can be represented geometrically (i.e., graphically)
by drawing a line acting in the direction of the quantity, the length of the
line representing to some scale the magnitude of the quantity. An arrow
is placed on the line, usually at the end, to denote the sense of the direction.

1-7. Parallelogram Law


The method of vector addition is based on what is known as the paral-
lelogram law. The parallelogram law cannot be proved; it can only be
demonstrated by experiment. It is one of the fundamental axioms of
mechanics. One method of demonstrating the law is by means of the ap-
paratus shown in Fig. 1-3. Tie three cords together and fasten the weights
P, Q, and W to the free ends. (The sum of P and Q must be greater than
W.) Place the cords to which
P and Q are attached over the
smooth pegs as shown and allow
the system to reach a position
of equilibrium.
The tensions in these cords
will then be equal to the weights
P and Q. Draw vectors P and
@ to scale from point A where
the cords are tied together and
construct a parallelogram with
these vectors as the initial sides.
Fic. 1-3. — Parallelogram law. It will be found that the diago-
Art. 1-8] Triangle Law i
nal R of the parallelogram scales exactly to the value of W and is in line
with the vector representing IW.
From Axiom 2 which states that two equal, opposite, collinear forces
are in equilibrium, we conclude that weight W will be perfectly supported
by the force R. In other words, the net effect of the forees P and Q may be
replaced by a single force R. Such a force is called a resultant. Therefore
the resultant of two forces is the single force which will produce the same
effect as the original forces.
The parallelogram law may now be stated as follows: The resultant of
two forces is the diagonal of the parallelogram formed on the vectors of these
forces.

1-8. Triangle Law


If we examine closely the parallelogram formed by forces P and Q as in
Fig. 1b, we observe that side BC is parallel and equal to side AD. If the

Cc Cc
Q
P B

R R
P
D
Q i

(a) (b) (c)


Fic. 1-4. — Triangle law.

triangle A BC were drawn alone as in Fig. 1—4c, the resultant R joining A to


C would have the same magnitude and direction as the diagonal of the
parallelogram ABCD. In this instance force Q has been represented by
the free vector BC. A free vector is defined as one which does not show
the point of application of the vector, as distinguished from a localized
vector which does. a) |
It is also evident that, as DC is equal and parallel to P, the triangle ADC
in Fig. 14a may also be used to determine FR. In this case, P is taken as the
free vector whereas Q is the localized vector.
We may now state the triangle law as a convenient corollary of the
parallelogram law: If two forces are represented by their free vectors
placed tip to tail, their resultant vector is the third side of the triangle,
the direction of the resultant being from the tail of the first vector to the
tip of the last vector.
Special Case. If the angle between two forces becomes zero or 180°,
the forces act along the same line; i.e., the forces are collinear. By taking
8 PRINCIPLES OF STATICS [Chap. I

one direction as positive and the other direction as negative, it will be


apparent that the resultant of two collinear forces is their algebraic sum.
This application of the triangle law is extensively used in analytical solu-
tions.

1-9. Solution of Problems .

One of the first things a student should acquire is the ability to organize
his work in a neat and orderly fashion. Properly arranged work not only
helps to eliminate personal errors but also permits easy checking by another
person — a frequent occurrence in engineering offices. To aid the student
to achieve orderly work habits the following suggestions are offered:
1. After identifying the problem, start by constructing a neat diagram
of the quantities involved. This diagram should be of sufficient size so
that pertinent data and dimensions may be added without affecting its
legibility. A freehand sketch is usually acceptable, although the use of a
straightedge is preferred and will take little additional time. Some students
use different-colored pencils to distinguish between known and unknown
quantities.
2. State as concisely as possible what data are given and what informa-
tion is required. Students who fail to realize what is required often find
themselves in the unenviable position of obtaining the right answer to the
wrong question.
3. Errors are frequently caused by mental substitution in equations
and subsequent failure to include the term in the equation. For this reason,
write out the equation you intend to use before substituting in it. This will
also make the process apparent to any person who may check it. If an
equation is not used, write a short note indicating the principle used or
the operation performed. This short statement of theory — be it equation,
principle, or operation — may be put at the left side of the sheet and the
numerical work placed in line with it at the right of the page. In this book,
whenever an equation or principle is used in the solution of a problem, it is
stated at the left of the page in brackets and is followed by the solution in
the same line. You cannot be urged too strongly to use this ‘“theory-
solution” technique when solving problems.
Experience has also shown that many students have difficulty in obtain-
ing accurate numerical results even though they have correctly applied
the principles. To indicate the way to more accurate computation, the
illustrative problems discuss the technique of solution as well as the applica-
tion of principles. The following articles offer several additional items
for the student’s guidance.
ATEn 1 | Conversion of Units 9

1-10. Dimensional Checks

The equations used in engineering computations must be dimensionally


homogeneous; that is, the units on each side must be of the same dimen-
sional form. An easy way to check the dimensions in an equation is to
substitute the dimensional equivalents of each term and then multiply or
divide these equivalents as though they were algebraic quantities. This
process determines the dimensional unit of each term.
For example, consider the equation v? = v,2 + 2.as where v and v, are
in feet per second, a in feet per second squared, and s in feet. The numeral 2
is a dimensionless number.' Substituting dimensionally in the equation,
we get
ft? Lt? ft
sec aay
sec sec as 1

which checks the equation since each term is in the same dimensional units.
A similar process may be used to determine the unit of an expression.
For example, determine the dimensional unit of kinetic energy if it is

expressed by the relation KE = dire where W is in pounds, v in feet


g
per second, and g in feet per second squared. Substituting dimensionally,
we have

Ib ft? Ib iiBc ects 4 i .

KE =
=
fp CU ons ft-lb Ans
ft per sec? oe eine

Even the definition of acceleration, a = = (Art. 9-3), can be checked

dimensionally. Here d?s is a second differential of length, whereas dé? is a


square of a differential of time. With units of length in feet and time in
. seconds, dimensional substitution in the definition for a yields
ft
Cee Ans.
sec

1-11. Conversion of Units


Occasionally it is necessary to convert a term from one system of units to
another to make an equation dimensionally correct. The conversion is
accomplished by multiplying the given term by unity where unity is a ratio
of units (of the same kind but different in size) containing the required units
and those given.
1[—n Art. 10-2, the numeral 2 is shown to be a factor of integration.
10 PRINCIPLES OF STATICS [Chap. I
eee
For example, convert a velocity of 60 miles per hour to units of feet
per second. Begin by writing

To express v in feet per second, multiply the right side by the following
ratios, each of which has the value of unity.
a miles (eee f ( hour )
© Dalmaitiout mile 3600 sec
Canceling out like units, we obtain

5280 ft
v = 60 X a609 = 885 Ans.

The ratio 328° is the conversion factor by which miles per hour must
be multiplied to yield feet per second. It is not necessary to remember
conversion factors once the method is mastered.
As another example, consider the equation 0 = wt + $ at? where w, is
in revolutions per minute, ¢ in seconds, and a@ in revolutions per minute
per second. It is required to express 6 in radians. Substituting dimension-
ally, we have
= rev uf rev ; ‘
6 (rad) = a ee) X t (sec) + 5 Xa Coy,| x ? (sec?)

Converting by multiplying by ratios having the value unity gives

i rev _, 27 rad min


2 Meath oes (ee * rev ao 60 sec eee)
1
=
rev 27 rad min 2 2
2; 7 Gs (=. xX sec rev x 60 in x # (sec?)

MRD ero an agniat pat x an Gadimeine


Each term is now expressed in units of radians. In this example, @ was
obtained in radians by multiplying each right-hand term by the factor
2
60
1-12. Numerical Accuracy
Significant Figures. It frequently happens that numerical work is
computed to a greater degree of accuracy than is warranted by the given
data. The accuracy of the final result depends upon the least accurate
figure used in the computation. For example, if the value of the gravita-
Art; 1-12] Numerical Accuracy ilu)

tional constant g is taken as 32.2, the answer should be computed to three


significant figures only. No increase in accuracy is attained by carrying
more figures through a computation than those in the least accurate data.
If for some reason four figures are carried through a computation where
the data in the example are accurate to only three figures, the answer
should be rounded off to three figures.
If a result is found to be 24.2, for example, the number indicates that
the result is greater than 24.15 but less than 24.25; the last 2 of the result is
doubtful. On the other hand, 24.20 means that the result is greater than
24.195 but less than 24.205; obviously, the last zero should not be added
unless it is a significant figure.
Location of the Decimal Point. In sliderule work difficulty is sometimes
experienced in locating the decimal point. It is not advisable to rely upon
systems that locate this point by counting the number of times the slide
extends to the left or right, or by counting the significant figures, ete. A
better method is to compute an approximate answer for the problem by
rounding off the figures and canceling as far as possible. This method not
only will locate the decimal point but will check the sliderule work.
For example, compute the value of

2.196 X 48.2 XK 289


3420 X 68.2
Rounding off figures gives approximately
Be 50 B00 6 6) 0.12
3500 X70 4950
By the sliderule, the numerals in the result are 131; hence, to resemble the
approximate value, the answer must be 0.131.
Solution of Quadratic Equations. Quadratic equations may be solved by
formula, sliderule, completing the square, etc. Completing the square is
probably as fast and accurate as any other method. It is obtained as
follows:
Consider a quadratic equation of the form x? + az — b = 0. Rearrang-

eras ()-(+s
ing this, we get

whence 2 2

from which the value of z is easily found.


12 PRINCIPLES OF STATICS [Chap. I

For example, solve for the roots of


a? — 22.82 — 31.38 = 0
From the above discussion, we obtain
x? — 22.84 + (11.4)? = (11.4)? + 31.3
(¢ — 11.4)? = 180 + 31.3 = 161.3
zg — 11.4 = 412.7
ap = OM I eralap == ile} Als

Trigonometric Functions. When a table of trigonometric functions is


not available, it is useful to know how to compute the value of sin 86°,
let us say, by means of a sliderule. This may be done as follows:
Since sin 20 = 2 sin 6 cos 6
we have sin 86° = 2 sin 438° cos 43°
== 4 < (VOSA < O77 = O08 Azan.

For angles less than 6° a similar technique is available. Since sin 6 = 6


for small angles (where @ is in radians), the value of sin 4°, for example, is
given by the equation

ree 2 7 rad = T rad


= =-=+deg X
Therefore, 2 2 360 deg 360
Sate 1
DUG 360 ~ 0.00873 Ans.

2
Likewise for cosine functions of small angles, we note that cos@ = 1 — ee
Hence, in computing the value of cos 3° we get

il lf x7*\

ll 0.9999618 Ans.
Art. 1-12] Numerical Accuracy 13

TRIGONOMETRIC FUNCTIONS

Angle 159 30° 45° 60° (ae

sin 0.259 4 = 0.500 |4+/2 = 0.707 |4-+/3 = 0.866 0.966


cos 0.966 |3+/3 = 0.866 |2+/2 = 0.707 1= 0.500 | 0.259
tan 0.268 |4+/3 = 0.577 1.000 V3 = 1.732 3.732

sin (—#) = —sin@; cos(—@) = cos8; tan (—@) = —tané


sin? @+ cos?@=1; 1+ tan?@ =sec?6; 1-+ ctn?@
= esc? é
sin? @ = +—icos26; cos??=i+34co0s26
sin 2 6 =
2 sin @ cos 8
cos 2 6 =
cos? @ — sin? @ = 1 — 2sin?@ = 2 cos? 9 —1
sin (0+ ¢)
sin#cos¢+ = cos @sing
cos (9 + ¢)
cos@cos¢ = F sind sing
tan@ + tang
ee
A tan
. 6 «95
sind=O0-J +5
-—at
feline!
cos 0 = 1 aL edt. Ole |

a(“) Hae Jered


a d(w) dv du v dx dx
Sana ae =u—+v—;} SS eee aad
dx dx dx dx dx v

grt dx

ht, = >
ip* dz Ee aa . =
— g logsa-C

sin za = — cose +¢ Hiepseas = nec

rims 2 dr = 2 9872 0c [peer oper 2 7

[ud =» — foae
Chapter LI.
Resultants of Force Systems

2-1. Introduction
The effect of a system of forces on a body is usually expressed in terms
of a resultant. The value of this resultant determines the motion of the
body. As we shall see, if the resultant is zero, the body will be in equilibrium
and will not change its original state of motion. This is the province of
statics. If the resultant of a force system is not zero, the body will have a
varying state of motion, thereby creating a problem in dynamics. In this
chapter we shall consider the technique for determining the resultant
effect of various types of coplanar force systems. When these coplanar
types are mastered, it is only a simple step to the more general case of non-
coplanar or space systems of forces which is discussed in Chapter VI.

2-2. Forces and Components


Consider the example of a car driven due east for 4 miles, then turned
sharply and driven due north for 3 miles. What resultant distance has the
car covered? The answer can be obtained by drawing the vectors of the
travel as in Fig. 2-la, from which the resultant distance is 5 miles.

miles
3

4 miles 4 miles
(a) (b)
Fic. 2-1. — Components.

In the illustration just cited, the 4-mile and 3-mile distances may be
called the components (meaning parts) of the resultant. Since these com-
ponents were at right angles to each other, their resultant was easily
computed by means of the Pythagorean theorem. If the car travels the
3 miles in a northeasterly direction, the resultant distance cannot be
so readily computed, although the resultant can be scaled from the vector
addition shown in Fig. 2-1b. In Ue ae the 4-mile and 3-mile distances
Art. 2-2] Forces and Components 15
ee CC‘
are also components of the resultant distance although not mutually
perpendicular. A resultant may be resolved into any pair of components,
but it is generally most convenient to use rectangular components.
In engineering, forces are not generally at right angles. While the
resultant of such forces may be found graphically, it is not always con-
venient to do so. It is frequently desirable to resolve each force into a pair
of right-angle components for analytical calculation.
Consider Fig. 2-2 in which force F acts upon the given body. The effect
of the force is to move the body rightward and upward. Choosing these
directions as the positive directions of perpendicular X and Y reference
axes, we project the force F upon them to obtain the perpendicular com-
ponents F, and F,. The relations between these components and F is de-
termined by the basic definitions of sine and cosine of the angle 6, between F
ee aS. : PF ae :
and the X axis, i.e., sin 6, = = and cos 6; = = Which are usually rewritten

in the following form:


Fa=F oe ‘i (2-1)
F, = F sin 0,
The components F, and F, are considered positive if they act in the
positive directions of the X and Y axes, and negative if directed in the
negative sense of the reference axes. The choice of the X and Y axes is
arbitrary; they may be in any
nF in
convenient position; the rela- Free vector of F,
tions given above are indepen-
dent of the orientation of the Bae = pisces cooley
X axis. If desired, the angle
between F and the Y axis, des-
ignated as 6,, may also be used;
whence the components are then
given by F,=F sin 6, and
Hg
Ff, = Ff cog ,. ae
It is obvious that the mag-
nitude, inclination, and direction Fe oe Se etatoular components.
of a force can be derived when .
its rectangular components are known. For example, assuming values of
F, and F, to be known, we obtain from Fig. 2-2 the following equations:
F=v Cp. zm (Pa)
F (2-2)
tan 6, = —

The direction of F is determined by the signs of its components; this is


. .
F,

clearly explained in the table which follows.


16 RESULTANTS OF ForcE SYSTEMS [Chap. IT

Direction of F
Sign of Sign of with Respect to Diagram
F, Py Origin O

iy a

+ ~ Up to right

+ ~ Down to right

va

= + Up to left

eX

xX

a — Down to left
Art. 2-2] Forces and Components Ve

ILLUSTRATIVE PROBLEMS
201. A force of 200 lb is directed as shown in Fig. 2-3. Determine the X and Y
components of the force. @AY
Solution: By projecting the force upon i
the axes, we discover that the sign of F,
is minus and of F, positive. Applying Eq.
(2-1), we obtain
[F, = F cos 6,] F, = —200 cos 30° =
—200 X 0.866
F, = —173.2 lb Ans:
[Fy = F sin 0] F, = 200 sin 30° =
200 X 0.5
F, = +100lb Ans. Fig. 2-3.
202. Determine the components of the 300-lb force directed down to the right
at a slope of 2 to 3 as shown in Fig. 2-4a.
Solution: The major difference between this problem and the preceding one is
that the direction of the force is defined by its slope instead of its angle. We can
compute 6, from its tangent and then substitute its sine and cosine functions into

@
(Pr ee 3.61 F=3001b
2 F,

3
F=3001b 7

(a) (b)
Fig. 2-4,

Eq. (2-1), but it is simpler and more direct to compute the hypotenuse of the slope
triangle as +/ (2)? + (8)? = 4/13 = 3.61 and then apply the definitions of sine and
cosine as follows:

[F. = F cos 0] F, 300 ae


361 = 249 lb

2
[F, = Fsin?;] F, = — 300 X =~ = =166 lb
: 3.61
An even better procedure is to note the similarity between the slope triangle and
the force triangle in Fig. 2-4b whose corresponding sides are proportional to each
other. This gives
18 RESULTANTS OF ForcE SYSTEMS [Chap. IT

FF, F,— 300


3 Pi 5i Ya Si6l
whence
F, = 249lb and F, = —166lb
203. The components of a certain force are defined by F, = 300 lb and F, =
—200 lb. Determine the magnitude, inclination with the X axis, and pointing of
the force.
Solution: The magnitude of the force is found by applying the first of Eq. (2-2).
[? =V/ (Fz)? + ry | F =~/(300)? + (200)? 1? = BXOil Mey Ali

The inclination with the X axis is determined by the second part of Eq. (2-2).
1D 200 _ x e
tan i. = 7 | tan 6, = 300 > 0.667 i = Bre ADs,

Note particularly that by neglecting the given signs of the components the angle
found is the acute angle between the force and the X axis. The direction of the
Y force is found by sketching a tip-to-tail
| summation of the components as shown
in Fig. 2-5, or by visualizing it mentally.
Note that the minus sign of F, indicates
it to be directed downward. Hence the
force F points down to the right.
This technique of determining a force
eliminates the necessity of remembering
certain arbitrary conventions. For ex-
ample, a mathematical convention de-
fines an angle as always measured in a
Fig. 2-5.
counterclockwise sense from the X axis.
Accordingly in the given example, 6, might be defined as —33.7° or as +326.3°.

PROBLEMS
204. Determine the X and Y components of each of the forces shown in Fig.
P-204. Ans. T, = —307 lb; T, = —257 lb

F=3901b a Yi
T= 722 lb
P=2001b

7a ore P=3001b poe


Fia. P-204. Fie. P-205.
Art. 2-3] Resultant of Three or More Concurrent Forces 19

205. Compute the X and Y components of each of the forces shown in Fig.
P-205. Ans. T, = —400 lb; Ty, = +600 Ib
206. The triangular block shown in Fig. P-206 is subjected to the loads P = 1600
Ib and F = 600 lb. If AB = 8 in. and BC = 6 in., resolve each load into com-
ponents normal and tangential to AC.
Ans P= 12801b\,; P; = 960 Ib /:
F,, = 360 lb \; Fy = 480 lb /
207. Rework Prob. 206 if 6 = 60°.
P=1600 lb
208. The horizontal and vertical components
of several forces are: (a) P, = —200 lb and
P, = 100 lb; (b) F;, = 300 lb and F, = —200
lb; (ec) T, = —50 lb and T» = —90 lb. Deter-
mine each force.
Ans. (a) P = 224lb up to the left at 6, = 26.6°
209. Repeat Prob. 208 if (a) P, = 150 lb and
Fic. P-206 and P-207.
P, = —200 lb; (b) F, = —240 lb and F, = 360
lb; (ec) T, = —500 lb and T, = —300 lb.
Ans. (c) T = 583 lb down to the left at 6, = 31°
210. In Fig. P-210, the X component of the force P is 140 lb to the left. De-
termine P and its Y component.

te eo
Fic. P-210. Fic. P-211.

211. The body on the 30° incline in Fig. P-211 is acted upon by a force P in-
clined at 20° with the horizontal. If P is resolved into components parallel and
perpendicular to the incline and the value of the parallel component is 400 lb, com-
pute the value of the perpendicular component and that of P.

9-3. Resultant of Three or More Concurrent Forces


The determination of the resultant of three or more concurrent forces
that are not collinear requires determining the sum of three or more vectors.
There are two ways of accomplishing the addition of three or more vectors:
graphically and analytically.
Graphically. Two vectors can be added to give a resultant; this resultant
in turn can be added to a third vector, etc., until all the vectors have been
added together to give an overall resultant. These vectors can be added
in any order.
20 RESULTANTS OF ForRcE SYSTEMS [Chap. II

Consider the system of three concurrent forces shown in Fig. 2-6. I the
parallelogram method of vector addition is used, forces F and P may be
combined to give a resultant R, as shown in Fig. 2-6b. Since A, is equiva-

(a) (b)
Fic. 2-6. — Resultant determined by parallelogram law.

lent to and replaces F and P, the original system of three forces now consists
of only two: R; and Q. These may also be combined by the parallelogram
method to give the final resultant R. If the original system consists of more
than three forces, this same technique can be extended to include the addi-
tional forces.
The same result can be more readily obtained by the use of free vectors
and the application of the triangle law.
Thus in Fig. 2-7, by using the free vec-
tor P, the resultant of F and P (ie.,
R;) is easily obtained. To this result-
ant the free vector Q is added to give
the final resultant R. Observe that R;
need not be drawn at all, the total re-
sultant of the system being obtained by
joining the tail of the first vector (F)
with the tip of the last vector (Q). The
same result would be obtained if the
Fig. 2-7. — Resultant determined by order of addition had been P, F, and Q.
triangle law. In fact, any convenient order of tip-
to-tail vector addition may be used.
Analytically. The vectors can be resolved into components that coincide
with arbitrarily chosen axes. The components of each vector with respect
to these axes can be added algebraically, and the resulting additions will
be the components of the overall resultant vector.
Art. 2-3] Resultant of Three or More Concurrent Forces 21

Figure 2-7 can be redrawn as in Fig. 2-8 to show the X and Y components
of each force by projection upon the reference axes. It is apparent that
R., the X component of R, is equivalent to the algebraic sum of the X com-
ponents of F, P, and Q; also that R, is equivalent to the algebraic sum of

Ne

Fic. 2-8. — Rectangular components of resultant.

the Y components of F, P, and Q. Denoting such algebraic summations of


the components of the forces by 2X and LY respectively, we have
Re 2k
lige
Having thereby computed the components of the resultant R, we can
now determine its magnitude and inclination by using the method discussed
in Art. 2-2. Doing this transforms Eq. (2-2) into

R = (2X)? + (ZY)? (2-3)


tan 6, = zs (2-4)
The pointing of R is determined by the signs of its rectangular components
DX and LY as described in the table on page 16.

ILLUSTRATIVE PROBLEM
212. Determine completely the resultant of the concurrent force system shown
in Fig. 2-9.
Solution: We first determine the components of the resultant from the algebraic
summations of the components of the given forces. Knowing the rectangular
components of the resultant, we can easily find R.
22 RESULTANTS OF ForcE SYSTEMS [Chap. IT

Since the X component of a force F is given by F cos 6, and the Y component by


F sin 6,, we obtain
[R, = 2X) LX = 200 cos 30° + 100 cos 45° — 400 cos 0° — 300 cos 60°
= 173.2 + 70.7 — 400 — 150 ZX = —306.1 lb

[Ry = ZY] LY = 200 sin 30° + 300 sin 60° — 50 sin 90° — 100 sin 45°
= 100 + 259.8 — 50 — 70.7 ? ZY = +239.1 lb

|
The magnitude and inclination of R
Y are now found from
300 lb

| 200 1b
[x = vexyt
ery |
|
R = \/(806.1)? + (239.1)?
R = 388lb Ans.

- x XV

[tanhe, = 4

239.1
50Ib —-1001b Nereis UTM
Fic. 2-9. ae 38° Fy

Since the signs of 2X and YY were neglected in finding the inclination of R, it


follows that 6, is the acute angle between R and the X axis. The direction of R is
obtained from a mental tip-to-tail addition of 2X and ZY. Since ZX is minus
(i.e., directed left) and YY is positive (i.e., directed up), R points up to the left at
an angle of 38° with the X axis.
If desired, a check may be obtained graphically by plotting the given forces to
scale in tip-to-tail fashion.

PROBLEMS
213. Determine the resultant of the concurrent forces shown in Fig. P-213.
Ans. & = 486 lb up to the left at 6, = 70.6°
Y
400 Ib Y
4000 Ib :

200 Ib 5000 Ib
Hire, P=213) Fig. P-214.
Art. 2-3] Resultant of Three or More Concurrent Forces 23

214. Determine the resultant of the concurrent system of forces shown in Fig.
P-214.

215. Find the resultant of the concurrent force system shown in Fig. P-215.
Ans. R = 477 lb down to the right at 6, = 61.6°

300 Ib | 300 lb 300 1b 200 lb


Fie. P-215. Fig. P-216.

216. A concurrent force system is shown in Fig. P-216. Determine the resultant.

217. Compute the value of the resultant of the concurrent forces shown in Fig.
P-217. Ans. R = 149 lb down to the right at 6, = 14.2°

Y 300 Ib

300 Ib

200 Ib 224 |b 390 Ib


Fie. P-217. Fie. P-218.

218. The body shown in Fig. P-218 is acted on by three forces. Determine the
resultant.
219. Determine the resultant of the four forces acting on the body shown in
Fig. P-219.
Ans. R = 572 |b up to the right at 6, = 53.5°
24 RESULTANTS OF ForcE SYSTEMS [Chap. I

600 lb

361 Ib ;
Fic. P-219. Fig. P-220.

220. The resultant of a certain system of forces has the X and Y components
shown in Fig. P-220. Determine the components of this resultant with respect to
N and T axes rotated 30° counterclockwise relative to the X and Y axes.
AGS, lis = S00 lo: ii = BAS lis
221. The resultant of the concur-
rent forces shown in Fig. P-221 is 300 lb
pointing up along the Y axis. Com-
pute the values of F and 6 required to
give this resultant.
Ans. F = 512 |b up to the right
at 6, = ODeoe

| 240 Ib 222. Repeat Prob. 221 if the result-


Fic. P-221 and P-222. ant is 400 lb down to the right at 60°
with the X axis.
223. The block shown in Fig. P-223 is W=200lb
acted upon by its weight W = 200 lb, a
horizontal force Q = 600 lb, and the pressure
P exerted by the inclined plane. The result-
ant R of these forces is up and parallel to the
incline thereby sliding the block up it. De-
termine P and R. Hint: Take one axis
parallel to the incline. Ans. R = 293 lb
224. Two horses on opposite banks of
Fie. P=223.
a canal pull a barge moving parallel to
the banks by means of two horizontal ropes. The tensions in these ropes are 200
lb and 240 lb while the angle between them is 60°. Find the resultant pull on the
barge and the angle between each of the ropes and the sides of the canal.
Ans. Ru= 382 lb:0 ="837 %ase-n2 ie
2-4. Moment of a Force
The moment of a force about an axis or line is the measure of its ability
to produce turning or twisting about the axis. The magnitude of the
Art. 2-4] Moment of a Force 25

moment of a force about an axis which is perpendicular to a plane con-


taining the line of action of the force is defined as the product of the force
and the perpendicular distance from the axis to the line of action of the
force. For example, in Fig. 2-10, the moment of the horizontal force F
about the vertical axis Y equals F times
ee
d or Fd. The distance d is frequently
called the moment arm of the force.
In our present discussion of coplanar
forces, the paper represents their plane
of action. Consequently the axis of mo-
ments, which is perpendicular to the
plane of the forces, appears as a point
commonly called the center of moments.
Then the moment arm of the force be- ly
comes the perpendicular distance from
Fig. 2-10. — Moment of a force.
this moment center O to the line of ac-
tion of the force. It should always be remembered, however, that the
F center of moments is really the intersec-
rues? “<u” tion of the axis of moments with the plane
| of the forces.
Day NN SG Another method of defining the mo-
rd er SY> ment of a force is to say that the moment
KIN is equal to twice the area of the triangle
WANS ‘ a lee
yr formed by joining the center of moments
O with the ends of the force, as in Fig. 2-11.41
Fic. 2-11. — Moment equals twice Take F as the base of the shaded triangle;
the shaded area. ;
the altitude, measured from the moment
center O, is equal to the moment arm d. The area of the shaded triangle is
the product of one-half the base mul-
tiplied by the altitude; i.e., the area
equals $ Fd. Obviously Fd, which is
the moment of F about O, is twice this
area.
This concept of the moment of a
force can be used to demonstrate the
principle of transmissibility, namely,
that the external effect of a force is
independent of where it is applied along Fia, 2-12. — The moments of F, Fi, and
its line of action. Thus in Fig. 2-12, F, about 0 are all equal.
1 This is a graphic interpretation of the ‘‘cross-product”’ used in vector analysis. The
product M = F X d (the X is read as cross) is there defined as equal to the area of the
parallelogram formed on the sides of the vectors; in this instance f and d. The triangle
mentioned above is one-half of this parallelogram.
26 RESULTANTS OF FoRCE SYSTEMS [Chap. II

the rotational effect of force F upon a body free to rotate about its axle O
is the moment Fd which equals twice the area of the shaded triangle. How-
ever, an equal force /;, applied along the action line of but acting at the
right edge of the body, produces the same turning effect about the axle O
and so does the equal force F', acting as shown on the left edge. The mo-
ments of these three applications of the same force are equal inasmuch as
the areas of the triangles joining the ends of F'; and F2 with O are each equal
to the area of the shaded triangle.
Units and Signs. Because a moment is the product of a force and a
distance, the unit of moment is correspondingly the product of the dimen-
sional units of force and distance. Expressing force in pounds and distance
in feet, the unit of moment is pound-feet (lb-ft).2 With other dimensional
units, moments are expressed differently, for instance as pound-inches
(Ib-in.), gram-centimeters (gram-cm), ton-feet (ton-ft), etc.
Since a force can cause rotation about an axis in either of two directions,
a convention is necessary to describe the direction of rotation. Some
engineers take counterclockwise tendency to rotate as the positive sense
of moment and others consider clockwise moment effects as positive.
Both are correct; they are merely selecting the convention that best suits
their purpose. In this book, we shall assume to be positive whichever
direction (clockwise or counterclockwise) that happens to be more con-
venient. We must be careful, however, to use only one convention through-
out any particular problem. To avoid confusion, it will be helpful to
indicate the positive sense of moments in a particular case by a curved
arrow; thus if moments about a center A are considered positive clock-
wise, the moment sum may be indicated by @ 2M4. The curved arrow to
indicate positive moment will be used whenever confusion about signs
might arise.

2-5. The Principle of Moments. Varignon’s Theorem

It is almost self-evident that the moment of a force is equivalent to the


sum of the moments of its components. This statement is known as Vari-
gnon’s theorem and is demonstrated as follows:
Let the resultant R of two concurrent forces P and F be determined
by the parallelogram law as in Fig. 2-13. With any point O as a moment
center, it is required to show that the moment of R is equal to the sum
of the moments of P and F.
Join the ends of the force vectors to the moment center to form the
triangles oab, oac, and oad. The preceding article showed that the moment
: ? Jt makes no difference whether the unit of force or the unit of length is stated first;
it would be equally correct to use foot-pounds (ft-lb) as the unit of moment if desired.
Art, 2-5] The Principle of Moments. Varignon’s Theorem 27
Saeco eis rele ENT 2 ll a
of a foree is equal to twice the area of the triangle formed by joining the
ends of the force vector with the moment center. Hence the theorem is
proved if we can show that the area of Aoad equals the area of Aoab plus
the area of Aoac.

Fic. 2-13. — Varignon’s theorem.

These triangles all have the common base oa; their altitudes are laid off
on line of drawn perpendicular to line oa. We now have
Area of Aoad = 40a X of

Since cd is equal and parallel to ab, its projection ef is equal to og whence


Area of Aoab = 3 0a X og = 4 0a X ef
Also,
Area of Aoac = 4 0a X oe
Moreover, of =oe+ ef
whence by multiplying by } 0a we obtain
40a X of = 40a X 06 + 500 X ef
or
Area Aoad = Area Aoac + Area Aoab

which proves the theorem. If R were the resultant of more than two forces,
the theorem could be applied to the resultant F, of two of the forces, whence
the procedure is applicable to R, and the next force, ete.
Note carefully that the action line of the resultant and its components
must intersect at a common point. Thus although we have proved that
28 RESULTANTS OF ForcE SYSTEMS [Chap. II

the moment of R is equal to the moment sum of P and F, the principle of


moments would not apply to the moment sum of F and the free vector cd
of P because, even though their vector tip-to-tail sum is R, the areas of
Aaco plus Acdo is obviously not equal to that of Aado.
Applications. In some cases it is more convenient to determine the
moment of a force from the sum of the moments of its components rather
than from the force itself. For example, in Fig. 2-14 suppose a force F,
making an angle @ with the X axis, passes through a point A having the

Fig. 2-14. — Applications of the principle of moments.

coordinates (x,y). In this case it is inconvenient to calculate the moment


arm d. By resolving the force into its components F; and F, at A, the mo-
ment arm of Ff, about O is the coordinate distance y, and the moment arm
of F, about O is the coordinate distance x. Then the moment of F is ex-
pressed by
CMo=F-d=F,-y—F,-2 (a)

from which the value of the moment arm d may be computed if desired.
The intercepts of the line of action of F with the X and Y axes may also
be computed from the principle of moments. Replacing F by its com-
ponents at B and at C in Fig. 2-14, we have

and eC Mo = 1B : ty | (b)

@Mo = F, - izf
Note that Ff, at B and F, at C both have zero moment about O since they
both pass through O and therefore have zero moment arms. Having al-
ready determined the moment of F by means of Eq. (a), the intercepts 7,
and 7, are now readily computed from Eq. (b).
Art. 2-5] The Principle of Moments. Varignon’s Theorem 29

Another example is shown in Fig. 2-15. Suppose it is desired to find the


moment about point A of the force P acting on the roof truss. Ata point B
on the action line of P, resolve the force into its components P, and P,.

Fic. 2-15. — Moment of a force in terms of its components.

Applying the principle that the moment of a force is equal to the moment
sum of its components, we have
@Ma= Ped = PyoAB (c)

Note that P, intersects the moment center A and therefore has no moment
arm.

ILLUSTRATIVE PROBLEM

225. In Fig. 2-16, a force F passing through C causes a clockwise moment of 120
ft-lb about A and a clockwise mo- Y
ment of 70 ft-lb about B. Deter-
a
mine the force and its x intercept 7z. a
Solution: By resolving the force al |
into its components at C, we observe Cc
that since F, passes through A, the &, S
moment of F about A is due only to F, ~S eee line of F
F,, which must act leftwards as a |
shown in order to create a clockwise
moment about A. The magnitude
of F, therefore is

[G=M, = F,-y] 120 = F, (2)


F, = 60 lb left Fic. 2-16.

Considering again the components at C, we see that with respect to B, F, causes


a counterclockwise moment, and hence 7, must act upwards in order to create the
specified clockwise moment of 70 ft-lb about B. By applying the principle of
moments, the value of F, is
[CMs = F,-« — Fey] 70 = F, (5) — (60)(3) F, = 50 lb up
30 RESULTANTS OF FoRCE SYSTEMS [Chap. II

Now that the components of 7 are known, we apply Eq. (2-2) to obtain

[7 = V/(F2)? + A | F = (60)? + (50)? F = 78.2 lb up to the left


50 a
tan = =| eG). = 60 6, = 39.8

To determine the x intercept of F, at D where F crosses the X axis, resolve F into


its components. Since F, at D causes zero moment about B, the specified clockwise
moment about B can be created only by placing /’, to the left of B as shown. Then
we obtain

[@ Mes = F,- x] 70 = 50e e=1.A4ft

whence the z intercept from O is t@=5—e=5—14=3.6it Ans.


Query: How can 7, be found directly from the moment of F about O?
This procedure illustrates the application of Varignon’s theorem, but it would be
simpler in this instance to determine 7, directly, using the slope of the action line
of F as specified by its components. Doing this yields

Be bly 3 50 2 60_
|e F| = 60 or — op 50 > 3.6{ft Ans.

PROBLEMS

226. In Fig. P-226 assuming clockwise moments as positive, compute the mo-
ment of force F = 450 lb and of force P = 361 lb about points A, B, C, and D.
Ans. For F: Mp = 810 ft-lb; for P: Mg =
— 1200 ft-lb
227. Two forces P and Q pass through a
point A which is 4 ft to the right of and 3 ft
above a moment center O. Force P is 200 lb
directed up to the right at 30° with the hori-
zontal and force Q is 100 lb directed up to
the left at 60° with the horizontal. Deter-
mine the moment of the resultant of these two
forces with respect to O.
228. Without computing the magnitude
of the resultant, determine where the resultant
of the forces shown in Fig. P-228 intersects
Fic. P-226.
the X and Y axes.
229. In Fig. P-229, find the y coordinate of point A so that the 361-Ib force will
have a clockwise moment of 400 ft-lb about O. Also determine the X and Y inter-
cepts of the action line of the force.
Ans. ya = 2.67 ft; 4, = 1.33 ft above O; ¢, = 2 ft left of O,
Art. 2-6] Resultant of Parallel Forces dl
a a

Fic. P-228. Fic. P-229,.

230. For the truss shown in Fig. P-230, compute the perpendicular distance
from £ and from G to the line BD. Hint: Imagine a force F directed along BD and
compute its moment in terms of its components about EF and about G. Then equate
these results to the definition of moment M = Fd to compute the required per-
pendicular distances.

G 12’

Fia. P-230.

231. A force P passing through points A and B in Fig. P-231 has a clockwise
moment of 300 ft-lb about O. Compute the |¥
value of P.
232. In Fig. P-231, the moment of a certain ‘)
force F is 180 ft-lb clockwise about O and 90 ft-
lb counterclockwise about B. If its moment 3/
about A is zero, determine the force. B x
Ans. F = 75\b down to the right at 6, = 36.9° O alee SSS

233. In Fig. P-231, a force P intersects the


Fic. P-231, P-232, and P-233.
X axis at 4 ft to the right of O. If its moment
about A is 170 ft-lb counterclockwise and its moment about B is 40 ft-lb clock-
wise, determine its y intercept.

2-6. Resultant of Parallel Forces


A parallel force system is one in which the action lines of all the forces
are parallel. The resultant of such a system is determined when it is known
in magnitude, direction, and position. One of the outstanding differences
32 RESULTANTS OF FORCE SYSTEMS [Chap. II

between a concurrent and a parallel force system is that in the former the
position of the resultant is known by inspection whereas in the latter it is
not.
For example, consider the wheels shown in Fig. 2-17. In Fig. 2-17a the
resultant force acts through the axis of rotation; the wheel does not rotate.
. F

1p |
e
(a) (b)
Fig. 2-17. — Rotational effect of parallel forces.

In Fig. 2-17b the same forces applied to the rim of the wheel cause rotation.
The resultant of the parallel forces in Fig. 2-17b must be so located as to
produce the equivalent moment effect of the system; the posztion of the
resultant must be determined to produce this effect.
At this point we shall restrict ourselves to the analytical determination
R of the resultant of a parallel force sys-
ve y, tem; ina later chapter we shall consider
ym ea graphic methods of solution for the
SY Pe) 4s present case as well as for others.
ig . oY Consider the system of parallel for-
No? ces P, Q, and S shown in Fig. 2-18.
ee row |
Select reference axes as shown, with the
“a x Y axis parallel to the forces so that
the 5 AS none of the forces have an X compo-
xX nent and the Y component of each
Fig. 2-18.
— Resultant of parallel forces. force is its own magnitude. Conse-
quently, 7X = 0 and ZY = =F where
=F is the algebraic summation of the forces. Using the methods of Art.
2-8, we have R = (=X)? + (ZY)? which reduces to

R= =F (2-5)
Art. 2-6] Resultant of Parallel Forces
35
Obviously the line of action of R is parallel to the forces comprisin,__
system. cker
To determine the position of R, we select’some convenient point O as?
moment center and employ the principle that the moment of a resultant
equals the moment sum of its parts. Denoting the moment sum of the
force system by 2Mo and the moment arm of R by d, we then have
R-d=ZMo (2-6)
The relative position of R with respect to O is determined from the fact
that the resultant must produce the same moment effect as the original
system. A sample problem will illustrate the technique used.

ILLUSTRATIVE PROBLEMS
234. Determine the resultant of the parallel force system acting on the bar AB
shown in Fig. 2-19. The forces and po-
sitions are given in the figure. 20Ib 101b 40 1b
Solution: The magnitude of the re-
sultant force is first obtained by applying

ae ieee = 90) > 10-2 30, — 40


R= —40lb Ans.

Upward forces having been assumed to 30 1b


be positive, the negative sign of R indi-
cates it to be directed downward. hee ae he:

Applying the principle that the moment of the resultant is equal to the moment
sum of its parts (Varignon’s theorem, Art. 2-5), we have, taking clockwise moments
about A as positive,
[Me = 2M] @2ZM, = 10% 2—30*5+40
x8
= 190 lb-ft C
[Mr ea >Ma]) 40 da = 190 da = Abaiinn AMOS.

To determine the position of FP relative g! :


to A, draw R acting downward (because ae
of the minus sign) as shown in Fig. 2-20. dp=9.25 >|
Since the moment sum of the original B
forces was found to be clockwise, R must
lie to the right of the moment center A in
order also to produce a clockwise moment. R=40lb
By locating R with respect to B, it is easily ;
ee Fic. 2-20.
shown that the position of the resultant
is independent of the choice of moment center. Thus we have

[Me = 2Mz] CIM, X38


= —20X8-—10X6+30
—130 lb-ft G
[Me = R-d = 2Mz] 40 dp 130 dg = 3.25ft Ans.
32 RESULTANTS OF ForcE SYSTEMS

betw¢, 0 and noting that the sign of 2M, is negative (thereby indi-
poy & o e moment), we see that R must lie to the left of the moment
Vf ivalent counterclockwise moment. Moreover, d4 + dg =
h is the total distance from A to B. Hence the position
e choice of moment center.
to choose the moment center somewhere near the middle
rces in order to simplify calculations by having smaller
__ 48 wise to select the moment center at one of the forces in
order to eliminate the moment effect of that
pigiian nelfae. toasts |Ww force from the computations.
J w Ib/ a 235. <A beam of length ZL supports a load
cle which varies from w lb per ft at the right
end to zero at the left end. Determine
the magnitude and position of the resultant
load.
Solution: The total weight W of the
ae re triangular load shown in Fig. 2-21 is the
Fig. 2-21. resultant of smaller parallel loads ike dW
each of which is the product of an in-
tensity of y lb per ft by the small length of dx ft along which it may be assumed
w
constant. From the proportionality between similar triangles, we have ae zo
w x
yr pr Applying Eq. (2-5) we obtain
iD i
(peri pri [eye Saat|e bal
0 L 0 2
Ans.
The position of this resultant weight from O is obtained from Eq. (2-6):
L i
wl w wL?
[Rid=2M o] ead)
5 Aion
| x(y dx) aa,()ixx? dx eau”
3
whence
d=21L Ans.

PROBLEMS
236. A parallel force system acts on the lever shown in Fig. P-236. Determine
the magnitude and position of the resultant.
Ans. R= 1101b | at 6 ft to right of A
50lb 401b 20 Ib 60 lb
30 1b 60lb 201b 40 lb

Fig. P-236. Fic. P-237.


Art. 2-6] Resultant of Parallel Forces 35

237. Determine the resultant of the four parallel forces acting on the rocker
arm of Fig. P-237. Ans. R= 501b | at 4 ft to right of O
238. The beam AB in Fig. P-238 200 lb/ft
supports a load which varies from an
intensity of 50 lb per ft to 200 lb per ft. 50 lb/ft
Calculate the magnitude and position of
the resultant load. Hint: Replace the
given loading by a uniformly distributed
load of 50 lb per ft plus a triangular load
varying from zero at A to 150 lb per ft
at B. Fic. P-288.
239. The 16-ft wing of an airplane is subjected to a lift which varies from zero
at the tip to 360 lb per ft at the fuselage according to w = 90 2? lb per ft where z is
measured from the tip. Compute the resultant and its location from the wing tip.
Ans. R = 3840 lb at 9.60 ft
240. The shaded area in Fig. P-240 represents a steel plate of uniform thickness.
A hole of 4-in. diameter has been cut in the plate. Locate the center of gravity of
the plate. Hint: The weight of the
plate is equivalent to the weight of the
original plate minus the weight of ma-
terial cut away. Represent the original
weight of the plate by a downward force
acting at the center of the 10 X 14 in.
rectangle. Represent the weight of the
material cut away by an upward force
acting at the center of the circle. Lo-
cate the position of the resultant of
these two forces with respect to the left
edge and bottom of the plate.
Fic. P-240.
Ans. Center of gravity is 6.80 in.
from left edge and 4.90 in. above bottom edge
400 lb

Fic. P-241.
36 RESULTANTS OF ForcE SYSTEMS (Chap. IT

241. Locate the amount and position of the resultant of the loads acting on the
Fink truss shown in Fig. P-241. Ans. R = 3400 lb down at 12.06 ft to right of A
242. Find the values of P and F so that the four forces shown in Fig. P-242
produce an upward resultant of 300 Ib acting at 4 ft from the left end of the bar.
100 lb 12 F 200 lb 40 lb 60 lb

Fig. P-242. Fic. P-243.

243. The resultant of three parallel loads (one load is missing in Fig. P-248) is
30 lb acting up at 10 ft to the right of A. Compute the magnitude and position of
the missing load. Ans. F = 70 lb down at 8 ft to the right of A

2-7. Couples
Sometimes the resultant of a force system will be zero in magnitude and
yet have a resultant moment sum. For example, consider the force system
shown in Fig. 2-22. The magnitude of the resultant is given by
[R = =F| R= — 10 + 20 + 30 — 40 R=0
The student should not leap to the conclusion that a resultant does not
exist, since on taking a moment sum about B, we find
GIMs = — 20 X 3 — 30 X 4+ 40 X 6 2M, = + 60lb-ft

101b 40lb The system shown therefore does


produce some effect; in this case, it is
a tendency to rotate with a magni-
tude of 60 lb-ft. Before discussing
the nature of the resultant in this
case, let us take moment sums about
201b 301b points C, D, EF, and also any other
Fic. 2-22. point A as follows:
GziMe = —10X3-—30X1+40%38 =Mc = + 60 lb-ft
€IMp = —10K44+20K1+40X 2 2Mp = + 60 lb-ft
C2ZMz = —-10X6+20X3+30 X 2 2Mn = + 60 lb-ft
€z2M,=+10XK2—20X5—-—30X6+40*8 2M, = + 60 lb-ft
In this case, the resultant is seen to have the same clockwise moment
effect regardless of where the moment center is chosen.
The special case in which the resultant has zero magnitude but does have
a moment is said to consist of a couple. We define a couple as made up of
Art. 2-7] Couples Bu
two equal, parallel, oppositely directed
forces, as shown in Fig. 2-23. The perpen- F
dicular distance between the action lines
of the forces is called the moment arm of
the couple. Itisevident that the magnitude
of the resultant of these two forces is zero.
Their moment sum is constant and independ-
ent of the moment center. This is proved by
selecting moment centers at A and B to ate Se One
give respectively

GrMsz=f-d
CIM, = Fd+a)—F-a=F-d
We conclude that the moment of a couple C is equal to the product of one
of the forces composing the couple multiplied by the perpendicular distance
between their action lines. This relation is expressed by the equation

C=F-d
In the above example, the resultant is completely determined by specify-
ing it to be a clockwise couple having a magnitude of 60 lb-ft. Since neither
the force nor the moment arm of this couple is known, any pair of forces
separated by a distance which gives a clockwise moment of 60 lb-ft could
comprise the couple. Such couples might consist of two 60-Ib forces 1 ft
apart, or two 30-lb forces 2 ft apart, ete.
Since the only effect of a couple is to produce a moment that is inde-
pendent of the moment center, the effect of a couple is unchanged if

(a) the couple is rotated through any angle in its plane


(b) the couple is shifted to any other position in its plane
(c) the couple is shifted to a parallel plane
For the somewhat special case in which the given force system is com-
posed entirely of couples in the same or parallel planes, the resultant will
consist of another couple equal to the algebraic summation of the moment
sum of the original couples.

ILLUSTRATIVE PROBLEM
244. Transform the couple shown in Fig. 2-24a into an equivalent couple whose
forces are horizontal and act through points C and D.
Solution: From condition ¢ above, we can begin by shifting the couple to the
parallel plane CDEF, as in Fig. 2-24b. Under conditions a and 6 the couple is now
rotated and shifted to the position shown in Fig. 2-24c. When the forces of the
couple act through points C and D, the moment arm of the couple becomes 3 in.
38 RESULTANTS OF ForcE SYSTEMS [Chap. II

Fic. 2-24. — Transformations of a couple.

Since the moment effect of a couple is constant, the forces acting at C and D are
found from
IC = hah C=9x4=FX3
1 == WP Mo}
Therefore the forces acting through C and D in Fig. 2-24d each have the magnitude
of 12 lb.

PROBLEMS
245. Refer to Fig. 2-24a. A couple consists of two vertical forces of 60 Ib each.
One force acts up through A and the other acts down through D. Transform the
couple into an equivalent couple having horizontal forces acting through # and F.
246. Determine the resultant moment about point A of the system of forces
shown in Fig. P-246. Each square is 1 ft on a side.
Ans. M = 561 lb-ft clockwise
40 lb

80 Ib 80 lb

A 40 lb
Fic. P-246. Fie. P-247.

247. The three-step pulley shown in Fig. P-247 is subjected to the given couples.
Compute the value of the resultant couple. Also determine the forces acting at
the rim of the middle pulley that are required to balance the given system.
Ans. C = 760 in.-lb counterclockwise; two 63.3-lb forces
Art. 2-8] Resolution of a Force into a Force and a Couple 39

248. To close a gate valve it is necessary to exert two forces of 60 lb at opposite


sides of a handwheel 3 ft in diameter. Through an accident the wheel is broken and
the valve must be closed by thrusting a bar through a slot in the valve stem and
exerting a force 4 ft out from the center. Determine the force required and draw
a free-body diagram of the bar.
249. Figure P-249 represents the
top view of a speed reducer which is
geared for a four to one reduction in
speed. The torque input at the hori-
zontal shaft C is 100 lb-ft. The torque
output at the horizontal shaft D, be-
cause of the speed reduction, is 400
lb-ft. Compute the torque reaction
at the mounting bolts A and B hold-
ing the reducer to the floor. Hint:
The torque reaction is caused by the
unbalanced torque, which is a couple. Fie. P-249.
Ans. R = 120 |b directed vertically up at A and down at B
250. The cantilever truss shown in Fig. P-250 carries a vertical load of 2400 lb.
The truss is supported by bearings at A and B which exert the forces Ay, Ay, and
B,. The four forces shown constitute two couples which must have opposite mo-
ment effects to prevent movement of the truss. Determine the magnitudes of the
supporting forces. Ans. A, = 2400 lb; A, = 3600 lb; B, = 3600 lb

6!

24001b AS R=1001b
Fia. P-250. Fra, P-251.
251. A vertical force P at A and another vertical force F at B in Fig. P-251
produce a resultant of 100 Ib down at D and a counterclockwise couple C of 200 lb-ft.
Find the magnitude and direction of forces P and F.
Ans. P = 300 lb down; F = 200 lb up

2-8. Resolution of a Force into a Force and a Couple


It sometimes becomes necessary to replace a force acting at a given point
by an equal force acting through some other point. This introduces a
40 RESULTANTS OF ForcE SYSTEMS [Chap. IT
i eh ech as SS
couple. For example, in Fig. 2—25a, let a known force F be acting through
A. By adding twe collinear forces F’ and F’” equal and parallel to P at B,
the effect of F isunchanged.? The forces F and F” form a couple having the
A A
jane

F’

(a) (b)
Fie. 2-25. — Resolution of a force into a force and a couple.

moment arm d. Since the moment of a couple is independent of its moment


center, it is convenient to represent this couple by the curved vector
C = Fd in Fig. 2-25b. F and F” having been disposed of, there remains
only F’ which acts at B. The original force at A has therefore been replaced
by an equal force acting at B plus a couple C having the magnitude Fd.

PROBLEMS

252. A force system consists of a clockwise couple of 480 lb-ft plus a 240-lb
force directed up to the right through the origin of X and Y axes at 0, = 30°. Re-
place the given system by an equivalent single force and compute the intercepts of
its line of action with the X and Y axes.
Ans. 4 ft to the left of O and 2.31 ft above O
253. In Fig. P-253 a system of forces reduces to a downward vertical force of
y 400 lb through A plus a counterclockwise couple
of 800 lb-ft. Determine the single force that will
4! produce an equivalent effect.
La re 9A Ans. R = 400 lb down acting 2.0 ft to right
1 of O
254. Rework Prob. 253 if the system reduces
+: t__X toa leftward horizontal force of 300 lb through
O point A plus a clockwise couple of 750 lb-ft.
Hig) 2253 and P2564. 255. A short compression member carries an
eccentric load P = 200 lb situated 2 in. from the
axis of the member, as shown in Fig. P-255. In strength of materials it is learned
that the internal stresses are determined from the equivalent axial load and couple

* This effect follows from Axiom 8: A set of forces in equilibrium may be added to any
system of forces without changing the effect of the original system.
Art. 2-9] Resultant of Non-Concurrent Force Systems 41

into which P may be resolved. Determine this P=200 1b


equivalent axial load and couple. Le ”
Ans. F = 200 lb; couple = 400 lb-in. clockwise ;
256. A vertical shaft AB is 5 ft long and bolted
to a rigid support at its lower end A. At its upper
end B is attached a horizontal bar BC which is 2 ft
long. At the end of C is applied a foree P = 180 lb.
Force P is perpendicular to the plane containing
points A, B, and C. Determine the twisting effect of
P on the shaft AB and the bending effect at point A.
Ans. Twisting effect = 360 lb-ft; bending effect =
900 lb-ft Z
257. Replace the system of forces acting on the
frame in Fig. P-257 by aresultant R at A anda couple Fig. P-258.
acting horizontally through B and C.
Ans. R = 50 lb down; B = 110 lb right; C = 110 bb left
20 lb

141.4 lb

30 1b 60 lb 861 lb 224 |b
Fig. P-257. Fic. P-258.

258. Replace the system of forces shown in Fig. P-258 by an equivalent force
through O and a couple acting through A and B. Solve if the forces of the couple
are (a) horizontal and (b) vertical.

2-9. Resultant of Non-Concurrent Force Systems


It will be shown in kinetics of rigid bodies (Part II) that the effect of
an unbalanced non-concurrent force system on a body is to give the body
a motion consisting of a combined translation and rotation. The transla-
tional effect is determined by the magnitude of the resultant A whose
components are 2X and YY. The rotational effect is determined by the
moment of the resultant force which may be expressed in terms of the
applied forces by R-d = 2M.
42 RESULTANTS OF FoRCE SYSTEMS [Chap. IT

It is necessary to remember that a resultant of a force system is defined


as a force (or forces if the resultant is a couple) which produces the same
effect as the original set. of forces. The components of the resultant must
therefore equal the component effects of the given force system. This
statement may be expressed by the following relations:
JR Ss BO
Ry, = ZY
Moment
of hi — hd. =

The magnitude and direction of the resultant are determined from the
first two of these relations by the use of Eqs. (2-3) and (2-4) developed in
Art. 2-3, Viz.,
R = VRE + Ry = V@XP + GY (2-8)
tan 0; =
R,
R, =
>Y
ye (2-4)

whereas the third relation determines the moment arm of the resultant
with respect to the origin of moments.
The significance of this discussion is illustrated in Fig. 2-26. The system
of forces in (a) is modified to that in (b) by adding three pairs of balanced
forces acting through the chosen reference point O. Then, as shown in (c),
each of the original forces is equivalent to the same force now acting

P P
P

pO ie
T
F F F
(a) (b)
P
P

= + =
FP.

Fis FE
(c) (d)
Fig. 2-26. — Reduction of non-concurrent system to a foree plus a couple or a single
force.
Art. 2-9] Resultant of Non-Concurrent Force Systems 43

through O plus a couple equal to its moment about O. The resultant of the
concurrent system of the left portion of (c) is the resultant R acting through
O in (d) whereas the system of couples acting on the right portion of (c)
produces the resultant couple shown as 2Mo in (d). If desired, the system
may then be reduced to the single force shown in (e).

ILLUSTRATIVE PROBLEM

259. The rectangular framework shown in Fig. 2—27a is subjected to the indi-
cated non-concurrent system of forces. Determine the magnitude and direction of
the resultant; also its moment arm relative to the origin O.
Solution: To enable the student to follow the moment calculations, it will be
convenient to draw a new diagram and to replace each of the original forces by its
X and Y components. These components must intersect on the line of action of the
force they replace (see Art. 2-5). The moment effect of any force is then found by
the moment sum of its components. This technique and its advantages were dis-

Fic. 2-27.

cussed in Art. 2-5. The resulting duplicate of the original framework on which the
forces are represented by their components is called a “dummy diagram.” Note
that the dummy diagram has the equivalent effect of the original set of forces.
Applying X = F cos6, and Y = F sin 6, we replace the forces in Fig. 2-27a by
their components as shown in the dummy diagram in Fig. 2-27b. The effects of the
resultant are found to be

[R, = 2X] ZX = 17.8 — 70.7 + 60 — 50 2X = —43.4 lb


[Ry = ZY] ZY = 10+ 70.7 — 86.6 — 30 ZY = —35.9 lb
[Mrz = 2Mo]
D)IMo = —17.8 X 3+ 70.7 X 3 + 70.7 X 4 — 60 K 3 — 86.6 XK 4
2Mo = —83.4 lb-ft
44 RESULTANTS OF ForcE SYSTEMS [Chap. II

Having determined the effect of the original system of forces, we proceed to de-
termine the resultant as follows:

[r = JVOEX¥ + GFF | R = /(434) + (85.9)? R=56.3lb Ans.

tan 0, = ==>
ZY = eee
35.9 0, = 39.6° Ans.
|- =X | BERS ore ;
[Mp =R-d=ZMo] 563 Xd = 83.4 = 1.48 ft Ans.
Note that we neglected the signs of the quantities 2X, 2Y, and 2Mo. These
signs have no effect on the magnitudes just found; they determine directions. For
example, )X and LY were both minus; hence # points down to the left as shown
in Fig. 2-28. 6, is the acute angle with the X axis. The minus sign of the moment
component indicates that the moment
uf effect of the resultant is clockwise about
ly the origin. This effect can be produced
i SXeera x only by placing R below O as shown,
rae comme ——_—_ whence the moment arm d appears as
oh ; | indicated.
DY Frequently it is more convenient to
specify the action line of a resultant in
terms of its intercepts with the reference
EX B axes rather than in terms of its moment
SY arm. These intercepts are readily com-
Fig. 2-28. — Location of resultant.puted by applying Varignon’s theorem
—the moment of the resultant equals
the moment sum of its parts. Thus in Fig. 2-28, at A, where R crosses the X
axis, resolve FR into its components. Then we obtain

[Mr = (ZY)tz = 2M} SHO ie = Stl tz = 2.32 ft to right of O. Ams.

Similarly at B, where R crosses the Y axis, resolve R into its components. This
time we obtain

[Mp = (2X)iy = DMol 43.4 ty = 83.4 iy = 1.92 ft below O. Ans.


Note that at point A the X component of the resultant passes through the center
of moments and hence has no moment about O; likewise at B, the Y component of
the resultant has no moment about O.

PROBLEMS

260. The effect of a certain non-concurrent force system is defined by the follow-
ing data: 2X = +90 lb, ZY = —60 1b, and 2Mo = 360 lb-ft counterclockwise.
Determine the point at which the resultant intersects the Y axis.
261. In a certain non-concurrent force system it is found that 2X = —80 lb,
ZY = +160 lb, and 2Mo = 480 lb-ft in a counterclockwise sense. Determine the
point at which the resultant intersects the X axis. Ans. 3 ft to right of origin
Art. 2-9] Resultant of Non-Concurrent Force Systems 45
a ree mania erent re Ye
262. Determine completely the resultant of the forces acting on the step pulley
shown in Fig. P-262.
Ans. R = 1250 lb down to right; 0, = 44.3°; R passes through axle

750 lb

250 lb
Fig. P-262. Fig. P-263.

263. Determine the resultant of the force system shown in Fig. P-263 and its
x and y intercepts.

264. Completely determine the resultant with respect to point O of the force
system shown in Fig. P-264.

Fic. P-264. Fic. P-265.

265. Compute the resultant of the three forces shown in Fig. P-265. Locate its
intersection with the X and Y axes.
Ans. R = 957 |b down to right at 6, = 32.2°; i, = 1.38 ft above O; 1, = 2.19
ft right of O

266. Determine the resultant of the three forces acting on the dam shown in
Fig. P-266 and locate its intersection with the base AB. For good design, this
intersection should occur within the middle third of the base. Does it?
Ans. Yes
46 RESULTANTS OF ForcE SYSTEMS [Chap. II

267. The Howe roof truss shown in Fig. P-267 carries the given loads. The
wind loads are perpendicular to the inclined members. Determine the magnitude
of the resultant, its inclination with the horizontal, and where it intersects AB.
Ans. R = 10,760 lb down to right at 6, = 68.2°; 16.0 ft to right of A.

1120 lb

2240 lb

1120 Ib 2000 lb

3000 lb 2000 lb 1000 lb


Fig. P-267.

268. The resultant of four forces, of which three are shown in Fig. P-268, is a
couple of 480 lb-ft clockwise in sense. If each square is 1 ft on a side, determine
the fourth force completely.
269. Repeat Prob. 268 if the resultant is 390 lb directed down to the right at a
slope of 5 to 12 passing through point A. Also determine the x and y intercepts of
the missing force FP. :
Ans. F = 219 lb down to right at 6, = 43.2°; 7, = 3.26 ft right of O; 7, — 3.06
ft above O
270. The three forces shown in Fig. P-270 are required to cause a horizontal
resultant acting through point A. If F = 316 lb, determine the values of P and T.
Hint: Apply Mr = =Mz to determine R, then Mr = 2M¢ to find P, and finally
either Mp = 2Mpor R, = ZY tocompute T. Ans. P = 475lb; T = —225 |b
Art. 2-9] Resultant of Non-Concurrent
ee eee ee eh ee eeeForce
vey
Systems
oe 47
|¥ 150 lb

Fic. P-268 and P-269. Fic. P-270 and P-271.

271. The three forces in Fig. P-270 create a vertical resultant acting through
point A. If 7 is known to be 361 lb, compute the values of F and P.

SUMMARY
The effect of a system of forces on a body is expressed in terms of a
resultant. This resultant is generally computed in terms of its force com-
ponents along specified X and Y axes and a moment component.
The rectangular components of a single force F in terms of the force
and the angle 6, at which it is inclined with the X axis are expressed by
Fe ea,
F, = F sin 0, teat
These components are considered positive when directed along the positive
directions of the reference axes (refer to Art. 2-2).
When the resultant of three or more concurrent forces is desired (Art.
2-3), each force is resolved into rectangular components, whence the magni-
tude and inclination of the resultant R are given by
R = V(X)? + (SY)? (2-3)
tan panes
0. = Sy 2-4
(2-4)

The position of the resultant of a concurrent system of forces passes through


the point of concurrency.
The moment of a force about an axis which is perpendicular to a plane
containing the force is the product of the force and the perpendicular
distance from the axis to the line of action of the force. In coplanar force
systems, moments are usually taken about a point called the center of
moments. It is important to note that such a moment center represents
48 RESULTANTS OF FoRCE SYSTEMS [Chap. IT

the point in which the axis of moments intersects the plane of the force
system.
A convenient principle of moments, known as Varignon’s theorem (Art.
2-5), permits the moment of a force to be expressed as the moment sum
of its components. This method is especially recommended in all cases
in which the moment arm of the force is difficult to compute and the mo-
ment arms of its components are readily obtained.
The resultant of a parallel force system (Art 2-6) is the algebraic sum of
the forces composing the system. The position of the resultant is deter-
mined from the condition that the moment of the resultant is equal to the
moment sum of its parts. These principles are expressed by the relations

R=xXF (2-5)
R:-d==xM (2-6)
In a special case the magnitude of the resultant may be zero although the
system may produce a moment effect. In such cases the resultant is a
couple C consisting of two equal, parallel, oppositely directed forces having
a moment equal to the product of one of the forces and the perpendicular
distance between them. The moment of the couple is equal to the moment
effect of the given force system (refer to Art. 2-7).
The resultant of a non-concurrent force system (Art. 2-9) must be deter-
mined in magnitude, inclination, and position. Its magnitude and inclina-
tion are expressed in terms of the summations of the components of the
force system by the equations

R =~ (2X)? (CY)?
tan 6, = as

The position of the resultant foree may be obtained by computing its


moment arm d from a reference center O by means of the principle of mo-
ments
Mr = Rd = 2Mo
or its intersection with a reference axis passing through O by means of the
relations
Mr a DA cS 2 3s Mo
Chapter IIT.
Equilibrium of Force Systems

3-1. Definition and Meaning of Equilibrium


The subject matter of statics, as its name implies, deals essentially with
the action of forces on bodies which are at rest. Such bodies are said to be
in equilibrium. Specifically, equilibrium is the term used to designate the
condition where the resultant of a system of forces is zero. A body is said
to be in equilibrium when the force system acting upon it has a zero result-
ant. The physical meaning of equilibrium, as applied to a body, is that
the body either is at rest or is moving in a straight line with constant
velocity.
In this chapter we shall determine and apply the conditions necessary
to produce equilibrium for coplanar force systems. The principles and
techniques developed in this chapter are the basic fundamentals of statics;
the student is urged to master them. Actually what is studied is a method
of reasoning — of learning how to apply the basic concepts of the com-
ponents of a force and its moment effect in the most efficient manner.

3-2. Free-Body Diagrams


Problems in mechanics always involve the interaction of bodies upon
one another. Successful solution of these problems generally requires that
the bodies be isolated from one another so that the forces involved may be
analyzed and unknown forces determined. An isolated view of a body
which shows only the external forces exerted on the body is called a free-
body diagram (frequently abbreviated as FBD). These external forces are
caused either by direct bodily contact or by gravitational or magnetic
attraction.
Table III-1 describes some of the more common types of bodily contact
or support and shows how to represent the action of the body to be removed
upon the isolated free body. The earth pull is always shown as a downward
vertical force equal to the weight W of the free body and passing through
its center of gravity. The action of a flexible cord, rope, or cable is repre-
sented by a tensile pull 7 directed along the cord. The action of a smooth
surface is shown by a force N acting perpendicular to the smooth surface
because there can be no resistance to sliding along such a surface. Simi-
larly a roller support exerts a reaction F that is perpendicular to the surface
49
50 EQuILiBRIUM OF ForcE SYSTEMS [Chap. III

along which the roller can move. At a smooth pin or hinge, however, the
supporting force F can be exerted in any direction; such a force is usually
shown as two independent components which, when known, can be com-
bined to determine the supporting force and its inclination.

TasueE IIJ-1. Frree-Bopy DrAGRAMsS.

Type of Body Sketch of Action of Body Removed


Removed Reacting Bodies upon the Free Body

Earth Cp

pom
: 1p
Flexible cord,
rope, or cable
(weight neglected)

Smooth pin
or hinge

Notice that although forces always occur in pairs (i.e., action and reaction
forces which are collinear, equal, but oppositely directed), a free-body
diagram shows only the forces acting upon the body being considered. It
does not show the forces exerted by the free body upon other bodies.
The steps involved in drawing a free-body diagram are: (1) Draw a
diagram of the body completely isolated from all other bodies. The free
body may consist of an entire assembled structure or any combination or
part of it. (2) Represent the action of each body or support that is re-
moved by a force or its components. (3) Label each force by its magnitude,
if known, or by a symbol, if unknown.
Art. 3-2] Free-Body Diagrams 51

ILLUSTRATIVE PROBLEM
301. In Fig. 3-1, a 200-lb cylinder is supported by a horizontal rod AB and
rests against the uniform bar CD which weighs 100 lb. Draw the free-body dia-
grams (a) of rod AB, (b) of the cylinder, (c) of bar CD, and (d) of the assembled

Fic. 3-1.

cylinder and bar. Assume the pins at A, B, C, D and the rollers to be smooth and
frictionless. The rod AB is assumed to be weightless, a statement which although
physically impossible is often used to mean that the weight is negligible when com-
pared with other loads or forces.

Fic. 3-2. — Free-body diagrams of separate parts of Fig. 3-1.


52 Equrisriu SYSTEMS ee
m oF Force A [Chap. III
jaa pena arecast Fe
Solution: The FBD of rod AB is shown in Fig. 3-2a. The two forces exerted by
pins A and B upon the rod must be equal, opposite, and directed along the rod, or
the rod could not remain in equilibrium in its designated spatial position (see
Axiom 2, page 5). We conclude that a member fixed in space and loaded by two
forces at its ends requires the forces to be equal and their lines of action to coincide
with the centerline of the member. This conclusion is of great importance in the
analysis of pin-connected trusses since it determines the action lines of the forces
existing in the bars of such trusses.
Consider next in Fig. 3-2b the FBD of the cylinder. In addition to its weight of
200 lb, the cylinder is acted upon by the equal but opposite force that it exerted
upon the rod and a push N perpendicular to CD which is exerted by the bar. We
shall shortly learn (Art. 3-3) how to determine the unknown forces P and N holding
this concurrent force system in equilibrium.
The FBD of bar CD (Fig. 3-2c) shows its weight of 100 lb acting vertically at the
midpoint of the uniform bar. The cylinder pushes down to the right with a force
N perpendicular to CD. The roller at C cannot resist any force parallel to the
surface of the roller; its net effect can only be the vertical force C,. The force
exerted by the hinge D can act in any direction; its action is denoted by the com-
ponents D, and D;, which, if known, can be combined to determine the magnitude
and inclination of the hinge force at D.

Fic. 3-3. — Free-body diagram of assembled cylinder and bar.

Observe that the FBD of the assembled cylinder and bar in Fig. 3-3 does not
show the contact force N because here N is internal to the system and its action and
reaction effects balance out. Actually Fig. 3-3 is statically indeterminate (1.e.,
unsolvable using the equations of statics) because the four unknowns shown cannot
be determined directly using only the three equations of equilibrium available for
such a set of forces. It is necessary to take the assembly apart and draw the FBD
of each part as discussed above, even though doing this introduces an additional
unknown force N. Only then will there be available as many independent equations
Art. 3-2] Free-Body Diagrams 53

of equilibrium as there are unknowns; two equations for the concurrent system of
Fig. 3-2b and three for the non-concurrent system of Fig. 3-2c. We shall see later
that many systems which are apparently statically indeterminate may be solved
by taking the system apart and considering the FBD of each separate part.

PROBLEMS
General Instructions: The follow-
ing group of problems is provided
to give practice in isolating and
drawing a free-body diagram of
the various elements of an as-
sembled structure. Denote each
known force by its magnitude and
each unknown force by an ap-
propriate symbol. Later we shall
learn how to determine the magni-
tudes of the unknown forces.
302. The cylinder C in Fig.
P-302 weighs 1000 lb. Draw a
FBD of cylinder C and of rod AB.
303. The uniform rod in Fig.
P-303 weighs 420 lb and has its
center of gravity at G. Draw a
FBD of the rod. Neglect the Fic. P-302.
thickness of the rod and assume
all contact surfaces to be smooth.

304. The frame shown in


Fig. P-304 is supported in pivots
at A and B. Each member
weighs 50 lb per ft. Draw a
FBD of each member.

Fic. P-303. Fig, P-304.


54 EQuILIBRIUM OF ForcE SYSTEMS [Chap. III
a
is fas-
305. A 600-Ib load is supported by a cable which runs over a pulley and
FBD of bars AC and DE and of the
tened to the bar DE in Fig. P-305. Draw a
be smooth and neglect the weight of each bar.
pulley. Assume all hinges to

Fie. P-305. Fic. P-306.

306. Draw a FBD of pulleys # and D and of the bar AD shown in Fig. P-306.
Assume all hinges to be smooth and neglect the weight of each bar.

3-3. Equilibrium of Concurrent Force Systems


The conditions of equlibrium for concurrent force systems are obtained
by determining the equations that produce a zero resultant. In Art. 2-3
it was shown that the magnitude of the resultant of a concurrent force
system is found by means of the equation
R= V (2X)? + (ZY)?
Obviously, the resultant will be zero and equilibrium will exist when the
following equations are satisfied:

SEs
xX =0
ey
These equations are known as the conditions of equilibrium. It is important
to note that with two conditions of equilibrium, only two unknown quanti-
ties can be determined to create equilibrium of a concurrent force system.

ILLUSTRATIVE PROBLEM
307. A system of cords knotted together at A and B support the weights shown
in Fig. 3-4. Compute the tensions P, Q, F, and T acting in the various cords.
Art. 3-3] Equilibrium of Concurrent Force Systems 55
ee eee eee
Solution: We begin by drawing a FBD of knots A and B. Of these two concurrent
force systems, we must first solve that at A. The force system at B is temporarily
indeterminate because it contains three unknown forces and has available only two

Fic. 3-4.

independent equations of equilibrium. Its solution must be postponed until one


of the unknowns, P in this instance, has been determined from the concurrent system
acting at A, where P, exerting an equal and opposite effect to its action on B, is
only one of two unknowns.
Several methods are available for the solution of the concurrent force system at
A. Let us discuss each of these methods so that their individual advantages or OO
disadvantages will enable us to select the most efficient and rapid method to use
in similar problems.

Fic. 3-5. — Method I.

Method I—Using Horizontal and Vertical Azes. This is a routine method requir-
ing no imagination. Selecting reference axes that are horizontal and vertical as
shown in Fig. 3-5, we apply the conditions of equilibrium, Eq. (3-1), to obtain
[2X = 0] P cos 15° — Q cos 30° = 0 (a)
p20) P sin 15° + Q sin 30° — 300 = 0 (b)
56 EqurireriuM OF Forcr SYSTEMS [Chap. ITI
OO a
Solving Eqs. (a) and (6) simultaneously yields
P = 367 |b | Ans.
Q = 410 lb
Method II — Using Rotated Axes. The disadvantage of Method I is the necessity
of solving simultaneous equations. Since the reference axes are arbitrarily selected
in the first place, a better choice of the reference axes will eliminate simultaneous
equations; this simplifies the numerical work and reduces the chance for error.
For example, let the X axis be selected to pass through one of the unknowns, say Q.
In this case Q will have no Y component and will not appear in a Y summation.

Q Ge
P

300 Ib
(a) (b)

Fic. 3-6. — Method II: Using rotated axes.

The method of determining the angles between the forces and the rotated refer-
ence axes is shown in Fig. 3—6a; the final values of the angles are shown in Fig. 3-6b.
When actually solving the problems, only the X axis need be drawn, as in Fig. 3-6a.
The Y axis can be omitted; it is understood to be perpendicular to the X axis.
Since the X axis was chosen to coincide with Q, it is evident that Q has no Y
component. Hence by applying the condition of equilibrium, 2Y = 0, we auto-
matically eliminate Q from the equation. Thus we have
[ZY = 0] P sin 45° — 300 sin 60° = 0 P = 3671lb Ans.
Having determined P, we readily find the second unknown Q by applying the
second equation of equilibrium:

Pax = O 367 cos 45° + 300 cos 60° — Q = 0 Q=410lb Ans.

Note carefully the technique used. When the X axis is chosen so that it coincides
with one of the unknowns, the Y summation determines the other unknown. Then
the X summation determines the remaining unknown.

Method III — Using Force Triangle. When three forces are in equilibrium, the
easiest solution is generally obtained by applying the sine law to the triangle rep-
resenting the polygon of forces. Since forces in equilibrium have a zero resultant,
the tip of the last vector must touch the tail of the first vector. This tip-to-tail
Art. 3-3] Equilibrium of Concurrent Force Systems 57

addition gives the closed polygon of forces shown in Fig. 3-7. Applying the law
of sines to this triangle, we obtain!

300 P Q
sin 45° sin 60° __ sin 75°

whence as before

P= 367lb and Q=410lb Ans.


Fie. 3-7.— Method III:
Using force triangle.
If this solution is compared with 2Y = 0 in Method II, it will be seen to give the
same equation. Indeed, the solution from a force triangle is equivalent to using
two sets of rotated axes, one set drawn so as to pass through one unknown force,
whereas the other set coincides with the other unknown force.
We are now ready to determine the forces F and 7 holding the concurrent
system of forces at B in equilibrium.
A closed polygon of forces for this F
system forms a quadrilateral so that
the sine law cannot be applied. Al-
though a diagonal of this quadrilateral
can be drawn that will subdivide it
into two triangles to which the sine
law can be applied, this procedure is
more cumbersome than the method of
using rotating axes described above
P=367\b 75°
in Method II.
Applying the method of rotated
200 lb
axes to the FBD of B, we draw the
X axis to coincide with 7 as in Fig. pyg, 3-8. — Method of rotated axes applied
3-8, thereby eliminating 7 from a Y to FBD of B.
summation. Hence we obtain F from

(ZY; = 0] F sin 45° — 367 sin 45° — 200 sin 60° = 0 F=612lb Ans.

The remaining unknown 7’ is now determined from

i xe— 0] T + 200 cos 60° — 367 cos 45° — 612 cos 45° = 0
T= SON Moy ANGI)
1 Using a sliderule in which the S (sine) and D scales are aligned with each other, these
proportions are rapidly solved by setting 45° on S opposite 300 on D, whence, as shown
in the following table, the values of P and Q are found opposite 60° and 75° respectively.

S set 45° opposite 60° opposite 75°

D opposite 300 read P = 367 read Q = 410


58 EQuILiBRiuM OF ForcE SYSTEMS [Chap. III
e
OO an ee

PROBLEMS

308. The cable and boom shown in Fig. P-308 support a load of 600 Ib. Deter-
mine the tensile force 7’ in the cable and the compres-
sive force C in the boom.
Ags, fF = 439 lb: C = 538 Ib
309. A cylinder weighing 400 lb is held against
a smooth incline by means of the weightless rod AB

Fig. P-308. Fic. P-309.

in Fig. P-309. Determine the forces P and N exerted on the cylinder by the rod
and the incline respectively. Ans. P = 378lb; N = 418 1b
310. A 300-lb box is held at rest on a smooth plane by a force P inclined at
an angle @ with the plane as shown in Fig. P-310. If @ = 45° determine the value
of P and the normal pressure N exerted by the plane.
Ans, P= 212 lbswVe="410 lb

W=300lb

i \30"
Fig. P-310 and P-311. Fig. P-312.

311. If the value of P in Fig. P-310 is 180 Ib, determine the angle @ at which it
must be inclined with the smooth plane to hold the 300-Ib box in equilibrium.
DAdtS eer Pea.
312. Determine the magnitudes of P and F necessary to keep the concurrent
force system shown in Fig. P-312 in equilibrium.
Ans. “P= 183.6 lbs hf = so.lilb
Art. 3-3] Equilibrium of Concurrent Force Systems 59
ee ae ee ee ition
313. Fig. P-313 represents the concurrent force system acting at a joint of a
bridge truss. Determine the values of P and F to maintain equilibrium of the
forces. Ans. ee = 166lb: 2 =.413 Ib

400 lb
Ere bois. Fig. P-314.

314. The five forces shown in Fig. P-314 are in equilibrium. Compute the values
of P and F.
315. The 300-lb force and the 400-lb force shown in Fig. P-315 are to be held in
equilibrium by a third force F acting at an unknown angle @ with the horizontal.
Determine the values of F and 0.

400 lb 20 lb

300 Ib 30°

be 30 Ib
Fig. P-315.

316. Determine the values of the angles


a and @ so that the forces shown in Fig.
P-316 will be in equilibrium.
Ans. a = 46.6°; 6 = 29°
317. Thesystem of knotted cords shown
in Fig. P-317 support the indicated weights.
Compute the tensile force in each cord.
Ans. A = 846lb;B = 9141b;C = 400
Ib; D = 207 lb
318. Three bars, hinged at A and D
and pinned at B and C as shown in Fig.
P-318, form a four-link mechanism. De-
termine the value of P that will prevent
motion. Ans. P = 304 lb Fic. P-317.
60 EquiLipriumM OF ForcE SYSTEMS [Chap. III

Fic. P-318.

319. Cords are looped around a small


spacer separating two cylinders each
weighing 400 lb and pass, as shown in Fig.
P-319, over frictionless pulleys to weights
of 200 lb and 400 lb. Determine the angle
6 and the normal pressure NV between the
cylinders and the smooth horizontal sur-
face. Ans. 6 = 60°; N = 454 lb

Fic. P-319.

3-4. Conditions of Equilibrium from Moments


We shall now show that it is possible to express the conditions of equilib-
rium in terms of moment summations instead of XY and Y summations:
The advantage of expressing equilibrium in terms of moment summations
is that any particular force can be eliminated by taking moments about a
center on its line of action. This method is especially useful when the force
to be eliminated is specified in direction by its slope.
Q Since the moment effect of a system
of forces is equal to the moment of its
Ba resultant (i.e., 2M = Rd), a moment
eek. summation of a system of forces be-
\\o comes zero if either
le (a) the center of moments is on the
l action line of the resultant, i.e., d =
| 0, or
Ei | (b) the magnitude of the resultant
Ly is zero, which indicates equilibrium.
Fic. 3-9. — Conditions of equilibrium In Fig. 3-9 let the moment sum of
from moments. P, Q, and F be taken about A. If
Art. 3-5] Three Coplanar Forces in Equilibrium Are Concurrent 61
er ge ee eo
2M. = 0, either the resultant passes through A or no resultant exists.
Taking a moment sum about some other point B which is not on the line
OA, assume that 2M, also equals zero. We cannot now assume that the
zero moment sum is also caused by the resultant passing through B. This
would mean that the resultant acts in the two directions OA and OB at
the same time, which is impossible. The only conclusion possible to yield
zero moment sums is that the resultant is zero. Hence two other equations
of equilibrium are
=IM, = 0

xiMs, == 0
(3-2)

It must not be assumed that Eq. (8-2) presents two new or additional
equations of equilibrium for concurrent forces. They are really equivalent
to =X = 0Oand SY = 0. They may be used in place of or in combination
with these equations as may be convenient, thus
EX =0° EX =0 sY=0 =M,=0 ae
Sed SEM A= 0[e eee Me 0 EMS 0 a
To demonstrate that the X and Y summations of equilibrium and the
moment equations of equilibrium are not independent we refer to Fig. 3-10.
Assume the given forces to be in equili-
| Y
brium, and choose a moment center A Q
on the Y axis. Each force may be re-
solved into its X and Y components.
Then 2M, may be found as the sfim
of the moments of the X and Y com-
ponents. As the moment center A is
on the Y axis, moments of all Y com-
ee
ponents are zero. Hence 2M, = 0 is |
equivalent to 2X = 0 since all of the A
X components have the same moment Fic. 3-10.
arm. Consequently, in Fig. 3-9,
points O, A, and B must not lie on the same straight line or else both
~M, = 0 and [Mz = 0 are equivalent to the same force summation.

3-5. Three Coplanar Forces in Equilibrium Are Concurrent


Occasionally equilibrium of a structure is maintained by only three
coplanar forces. We shall now prove that three coplanar forces in equi-
librium must be concurrent (or parallel). Then the methods in the preced-
ing articles may be applied to such cases. We shall see later (Art. 3-7) that
such problems may also be solved by other methods, but the principle
Forcr SYSTEMS [Chap. II]
)62{MAA intr aeilieOFane AES
EQuILiBRiuM ee
that three coplanar forces in equilib-
rium must be concurrent is conven-
ient for determining the directions
of unknown forces.
Let the three forces acting on the
body shown in Fig. 3-11 be in equi-
librium. Prolong F and Q to inter-
sect at O and determine their result-
ant by the parallelogram law.
If the third force P is to hold the
system in equilibrium, it must be
Fig. 38-11.— Three coplanar forces in equal, collinear, and oppositely di-
equilibrium are concurrent. rected to force R (refer to Axiom 2,
page 5). Hence force P must
coincide with R and pass through point O.
We conclude that if three coplanar forces are in equilibrium, their lines
of action must intersect in a common point.

ILLUSTRATIVE PROBLEMS
320. The bell crank shown in Fig. 3-12a is supported by a bearing at A. A 100-Ib
force is applied vertically at C, rotation being prevented by the force P acting at B.
Compute the value of P and the bearing reaction at A.

1001b (b)
Fig. 3-12.
Art. 3-5] Three Coplanar Forces in Equilibrium Are Concurrent 63

Solution: Since the bell crank is in equilibrium, the three forces which act upon it
must pass through a common point. Prolonging the lines of action of the forces to
intersect at D makes the direction of R4 such that it must pass through A and D.
From the geometry of the figure, the distance AZ is found to be 13.67 in., whence
the distance CD = 21.67 in. The direction of R4 is found from

DE 21.67 =
tanan @, = —
| A “4 tan 6 Ss3 2.
P(A 0 = 69° 69° 45’ 45’ Ans

Plotting the polygon of forces that are acting on the bell crank as shown in Fig.
3-12b, we obtain by applying the law of sines

100 POR Aer


sin 24° 45’ sin 20° 15’ _— sin 135°
whence Po — 82.8 lb and aes lOO bye Ans:
If desired, the value of P may be checked by taking moments about A. Then we
have
[2Ma4 = 0] (P sin 75°) X 10 — 100 X 8= 0 P = 82.8lb Check
The moment of P about A was obtained by applying Varignon’s principle (see
Fig. 2-15). By resolving the force P into components parallel and perpendicular to
AB, the parallel component is made to pass through the moment center, whence
the moment effect of P is due only to the perpendicular component.
321. The loads applied to the truss shown in Fig. 3-18 cause the reactions shown
at A and D. A free-body diagram of hinge A forms the concurrent force system
300 lb 300 lb

12’

A,=2001b

A, = 400 1b

Fie. 3-13.
64 EquruipriuM OF ForcE SYSTEMS [Chap. III
isi Il ia aN le i a ee
shown enclosed at A. Determine the magnitudes of the forces P and F, directed
respectively along bars AB and AZ, that maintain equilibrium of this system.

Fig. 3-14. — >Mz = 0 eliminates P while [Mc = 0 eliminates F.

Solution: In Fig. 3-14, the horizontal and vertical components of forces P and
F are shown acting along their extended lines of action at B and C respectively.
The dimensions of the truss determine the forces to have the indicated slopes so that
the relations between the components of force P are

shies
Sy er=e (a)ce
and for force F are

Ee eee
2 ies ©
To determine P, take a moment summation about any point on the line of action
of F, thereby eliminating F from the moment equation. In this case, C is a con-
venient moment center since it not only eliminates / but also the component P,
which passes through C. Thus we obtain
[(DzMe = 0] 8 P, + 200 X 12 — 400 x 16 = 0 P, = 500lb
whence, using the relations (a), we have
P;, = 383 lb and P=601lb Ans.
Observe that any moment center on the line of action of F may be selected to
determine P even though both components of P may then appear in the moment
summation. For example, taking # in Fig. 3-13 as a moment center, both P; and
Art. 3-5] Three Coplanar Forces in Equilibrium Are Concurrent 65
ee
RP a ll ld
P, appear in 2Mx = 0, but we may use the relations between them given by Eq.
(a) to substitute P, = 3 P» or P, = $ Py so that one of the components of P can
be found directly from the moment summation. Of course, if a moment center like
C is available, we would prefer to use it, but we need not waste too much time
looking for it.
To determine F from a moment summation, select B as a moment center, thereby
eliminating P as well as F,. Setting moments about B equal to zero, we obtain
[>rIMz = 0] 8 F, + 200 x 12 — 400 x 8=0 Hee)
100 Ib
whence, from relations (6), we obtain
F,= 183lb and F =167lb Ans.
These results may be easily checked by horizontal and vertical force summations
applied to the FBD of A as follows:
[=H = 0] By IP 412200 =0
133 — 333 + 200 =0 Check
[ZV = 0] F,— PsAse00' 6
100 — 500 + 400 = 0 Check

PROBLEMS
322. The Fink truss shown in Fig. P-322 is supported by a roller at A and a
hinge at B. The given loads are normal to the inclined member. Determine the
reactions at A and B. Hint: Replace the loads by their resultant.
Ans. Ra = 4620 1b; Rg = 4620 lb at 30° with horizontal

1000 lb

2000 lb

2000 lb

-2000 lb

<—— 60! >


Fig. P-322.

323. The truss shown in Fig. P-323 is supported by a hinge at A and a roller
at B. A load of 2000 Ib is applied at C. Determine the reactions at A and B.
Ans. Ra = 2100 lb down to the left at 0; = 34.7°; Rp = 2200 lb
324. A wheel of 10-in. radius carries a load of 1000 lb, as shown in Fig. P-324.
(a) Determine the horizontal force P applied at the center which is necessary to
66 EQuiILiprium OF ForcE SYSTEMS [Chap. I]
ge
a

Fig. P-323.

start the wheel over the 5-in. block. Also find the reaction at the block. (b) If the
force P may be inclined at any angle with the horizontal, determine the minimum
value of P to start the wheel over the block; the angle that P makes with the
horizontal; and the reaction at the block.

Fig. P-324. Fig. P-325.

325. Determine the amount and direction of the smallest force P required to
start the wheel in Fig. P-325 over the block. What is the reaction at the block?
Ans. P = 1893 lb at 71.3° with horizontal; R = 642 lb
W=200 Ib 326. The cylinders in Fig. P-326
have the indicated weights and di-
mensions. Assuming smooth contact
surfaces, determine the reactions at A,
B, C, and D on the cylinders.
Ans. Ra = 346.41lb;Rg = 600 lb;
Re = 400 lb; Rp = 346.4 lb
327. Forces P and F acting along
the bars shown in Fig. P-327 main-
tain equilibrium of pin A. Determine
the values of P and F.
[ 328. Two weightless bars pinned
5:6" together as shown in Fig. P-328 sup-
Fia. P-326. port a load of 350 lb. Determine the
Art. 3-6] Equilibrium of Parallel Forces 67
ee e ee fg a BY

re n

350 lb
Fig. P-327. Fig. P-328.

forces P and F acting respectively along bars AB and AC that maintain


equilibrium of pin A. Ans. P = 901 lb; F = 641 lb

329. Two cylinders A and B,


weighing 100 lb and 200 Ib respec-
tively, are connected by a rigid rod
curved parallel to the smooth cylin-
drical surface shown in Fig. P-329.
Determine the angles a@ and @ that
define the position of equilibrium. 7
Agia e220 one =) 3.0% Fic. P-329.

3-6. Equilibrium of Parallel Forces


The conditions for equilibrium of parallel force systems are determined
from the conditions necessary to create a zero resultant. In Art. 2-6 we
found that the resultant of parallel force systems is determined by the
equations
Raed (2-5)
R-d=2M (2-6)

Since equilibrium means a zero resultant, we conclude that the independent


equations of equilibrium are
=F = 5 (3-4)
=M=0

from which only two unknowns may be determined to hold a parallel force
system in equilibrium.
68 EqQurLisriuM OF ForcE SYSTEMS [Chap. III
ne ee ee
As shown in Art. 3-4, a force summation may be replaced by a moment
summation. Hence the equations of equilibrium for parallel forces may also
be expressed by
EM, = i G-4n)
=M; = 0

where the moment centers A and B connect a line that is not parallel to the
forces. The use of Eq. (3-4a) is usually preferred; the condition ZF = 0 is
reserved for a check. This technique is illustrated in the following problems.

ILLUSTRATIVE PROBLEMS

330. A beam, simply supported at the ends, carries a concentrated load of 300 Ib
and a uniformly distributed load weighing 100 lb per linear foot, as shown in Fig.
3-15. Determine the beam reactions.

300 lb

Fie. 3-15.

Solution: To begin with, the uniformly distributed load is equivalent to a


resultant of 6 X 100 = 600 lb acting at the center of gravity of the uniform load
diagram. This resultant is represented by the dotted-line vector of 600 lb. The
original loading produces the same reactions as the equivalent two concentrated
loads.
Rk, is found by taking moments about a point on the line of action of R:, thereby
eliminating R» from the moment equation.
[E2Me, = 0] 10 RF, — 300 X 8 — 600 X 3 = 0 R, = 420lb Ans.
Similarly, R2 is found by a moment sum about a point on the action line of Ry.
This results in
[DrMer, = 0] 10 R. — 600 X 7 — 300 X 2=0 R,=480lb Ans.
A vertical summation of forces is used to check the results. Thus we have
pee = R, + R. — 300 — 600 = 0
420 + 480 = 300 + 600 Check
Art. 3-6] Equilibrium of Parallel Forces 69
a e nei Tce ae hd
331. The upper beam in Fig. 3-16a is supported by a reaction R3 at D and aroller
at C which separates the upper and lower beams. Determine the reactions Ri, Ro,
and R3.

P=960\lb

Fie. 3-16.

Solution: The FBD of the assembled beams is statically indeterminate. There


are three unknown forces and only two independent equations of equilibrium avail-
able. By taking the assembly apart and drawing a separate FBD-of each beam
as shown in (b) and (c), we expose the contact force F exerted by the roller C as an
additional unknown. As compensation for this additional unknown force, however,
we may write two independent equations for each FBD to obtain a total of four
independent equations of equilibrium. Thus for (b), we obtain
[2Mc = 0] 12 R; — 960 X 9 = 0 Rk; = 720lb Ans.
[2Mp = 0] 12 F — 960 X 3 = 0 F = 240lb
The value of F on AB acting equal and opposite to its action on CD is now used
in (c) to obtain
[=M4 = 0] 12 R, — 240 8 =0 R 160 lb Ans.
[[Mp = 0] 12R, — 240X4=0 Ri l| 80 lb Ans.

A check on these results is available by applying 2Y = 0 to the FBD of the


original assembly in (a). Doing this gives

[ZY = 0] 80 + 160 + 720 — 960 = 0 Check

PROBLEMS

332. Determine the reactions for the beam shown in Fig. P-332.
Ans. R, = 1580 lb; R2 = 520 Ib

333. Determine the reactions R; and R, of the beam in Fig. P-333 loaded with a
concentrated load of 1600 Ib and a load varying from zero to an intensity of 400 Ib
per ft. Ans. R, = 1900 lb; R, = 2100 lb
70 EQUILIBRIUM OF FoRCE SYSTEMS [Chap. IIT

400 lb /ft.
q
300 Ib 400 Ib 1600 lb TT

= 100 1b /ft. 2 a
Pat Th

ps Fea a 12:

R, R,
Fic. P-332. Fic. P-333.

334. Determine the reactions for the beam loaded as shown in Fig. P-334.
Ans. Ri = 372 lb; Ry = 558 Ib

120 1b 150 lb/ft


<j

Sheen
peta Pee Pes oa

Ry R,

Fic. P-334.

335. The roof truss in Fig. P-335 is supported by a roller at A and a hinge at B.
Find the values of the reactions.

200 lb

10’

600 lb 1500 lb
Fie. P-335.

336. The cantilever beam shown in Fig. P-336 is built into a wall 2 ft thick so
that it rests against points A and B. The beam is 12 ft long and weighs 100 lb per ft.
Art. 3-6] Equilibrium of Parallel Forces 71

A concentrated load of 2000 lb is ap- 2000 1b


plied at the free end. Compute the
reactions at A and B.
Ans. A = 12,400 lb; B = 15,600 lb

337. The upper beam in Fig.


P-337 is supported by a reaction at
R; and aroller at A which separates
the upper and lower beams. Deter-
mine the values of the reactions. Fia. P-336.

4000 lb 600 lb 1900 lb

Fie) P-337.

338. The two 12-ft beams shown in Fig. 3-16a on page 69 are to be moved hori-
zontally with respect to each other and load P shifted to a new position on CD so
that all three reactions are equal. How far apart will R, and R; then be? How far
will P be from D? Ans. 6 ft; 8 ft

Fic. P-339. Fig. P-340 and P-341.


72 Equiuiprium oF Force SYSTEMS [Chap. III

339. The differential chain hoist shown in Fig. P-839 consists of two concentric
pulleys rigidly fastened together. The pulleys form two sprockets for an endless
chain looped over them in two loops. In one loop is mounted a movable pulley
supporting a load W. Neglecting friction, determine the maximum load W that
can just be raised by a pull P applied as shown. Aye 7} JAD)
»
D-d

340. For the system of pulleys shown in Fig. P-340, determine the ratio of W to P
to maintain equilibrium. Neglect axle friction and the weights of the pulleys.
341. Ifeach pulley shown in Fig.
600 lb 200 lb P-340 weighs 36 lband W = 720 lb,
find P to maintain equilibrium.
Ans. P = 96 lb

342. The wheel loads on a jeep


are given in Fig. P-342. Determine
the distance « so that the reaction
of the beam at A is twice as great
as the reaction at B.
ALS, a BL aie

343. The weight W of a traveling crane is 20 tons acting as shown in Fig. P-343.
To prevent the crane from tipping to the right when carrying a load P of 20 tons,

Fia. P-343.

a counterweight Qis used. Determine the value and position of Q so that the crane
will remain in equilibrium both when the maximum load P is applied and when
the load P is removed. Ans, @ = 20 tonsa = 640

3-7. Equilibrium of Non-Concurrent Force Systems


In Art. 2-9 it was demonstrated that the resultant of a non-concurrent
force system could be determined from the components 2X, ZY, and =M.
Art. 3-7] Equilibrium of Non-Concurrent Force Systems 13
See ee eile
The resultant will therefore equal zero, and hence equilibrium will exist
only when
arial SRE 0 =M, =0
ZY=0} or ©M,=0$ or ©Mzy=0 (3-5)
=M =0) =M; =0 mM.= 0
The second and third sets of the equilibrium equations are obtained by
replacing a force summation by an equivalent moment summation. The
moment centers may be chosen anywhere provided that a line joining A and
B is not perpendicular to the X axis, and that A, B, and C do not lie on
the same straight line. (Explain these exceptions by reviewing Art. 3-4.)
In applying the moment summations it is best (whenever possible) to select
the moment center at the intersection of two of the unknowns, thereby
eliminating these unknowns from the moment summation. This technique
is illustrated in the following sample problems.

ILLUSTRATIVE PROBLEMS
344. The roof truss shown in Fig. 3-17a is supported on rollers at A and hinged
at B. The wind loads are perpendicular to the inclined members. Determine the
components of the reactions at A and B.

100 lb
400 Ib 320 Ib
N

200Ib 2001b 200 Ib


(a)
Fic. 3-17.

Solution: The roller at A constrains the reaction to be vertical. The reaction


at B is resolved into its components B, and B, as shown in Fig. 3-17b. These three
unknown quantities are determined by applying the equations of equilibrium. To
simplify computations, the symmetrical dead loads have been replaced by their
resultant of 600 lb. The 400-lb resultant of the wind loads has been resolved into
its components acting at C. Its vertical component is 400 cos 6 = 320 lb, and its
horizontal component is 400 sin 6 = 240 lb, since from the small 3-4-5 triangle,
the functions of 6 are sin @ = 2.andcos6@ = 4. Referring to Fig. 3-17b, we now ob-
tain B, from a horizontal summation which thereby eliminates A, and B,. Thus
[2X = 0] 240 — Bh = 0 By, = 240 lb Ans.
74 EQUILIBRIUM OF ForcE SYSTEMS [Chap. III

A moment summation about A eliminates A, and B;, and solves directly for By
as follows:
[Dr2M, = 0] 80 B, — 600 X 40 — 320 X 20 — 240 x 15 = 0
B, = 425 lb Ans.
Finally A, is determined from a moment summation about B which, by eliminat-
ing B, and Ba, gives a result which is independent of the reaction at B.
[CZMe = OJ 80 A, + 240 x 15 — 320 x 60 — 600 xX 40 = 0
A, = 495 lb Ans.
A vertical summation may be used to check A, and B,, but a more reliable check
involving all the forces is obtained from a summation of moments about the LADER D
of the truss, 1.e., 2Mp = 0. :
345. Determine the load P required to hold bar AB in a horizontal position on
the smooth inclines shown in Fig. 3-18. Also determine the reactions at A and B.

ie 400 Ib

Fia. 3-18.

Solution: Applying the conditions of equilibrium 2X = 0, ZY = 0, 2M4 = 0


provides a solution which is quite cumbersome since it leads to a set of three simul-
taneous equations relating the three unknowns. A much simpler solution is ob-
tained by choosing a moment center which eliminates two of the unknowns. This

lb
645

Fic. 3-19.

center O, lying at the intersection of R4 and Rg, is easily located geometrically in


Fig. 3-19a by applying the sine law to triangle AOB:
Art. 3-7] Equilibrium of Non-Concurrent Force Systems 75

Ae ee AO 208
sin 105° sin 45° sin 30°
whence AO = 14.62 ft and OB = 10.34 ft. Then AD = AO cos 30° = 12.68 ft and
DB = OB cos 45° = 7.32 ft whence the moment arms of P and the 400 lb load with
respect to O are 8.68 ft and 5.32 ft as shown in Fig. 3-19a.
Applying a moment summation about O, we now obtain

[=Mo = 0] 8.68 P — 400 X 5.32 = 0 P = 245 lb Ans.


This value of P is used to obtain the force polygon shown in Fig. 3-19b to which
the sine law is applied to give
645 Ra Rp
sin 75° sin 45°" sin 60°

whence
ha —4/2 1b and Re= 578 lb Ans.

PROBLEMS
346. A boom AB is supported in a horizontal position by a hinge A and a cable
which runs from C over a small pulley at D as shown in Fig. P-346. Compute the
tension 7’ in the cable and the horizontal and vertical components of the reaction
at A. Neglect the size of the pulley at D.

200 Ib 1001b
Fic. P-346 and P-347.

347. Repeat Prob. 346 if the cable pulls the boom AB into a position at which it
is inclined at 30° above the horizontal. The loads remain vertical.
Ans. T = 217 lb; A, = 108.3 lb; A» = 112.5 lb
76 EQuILIBRIUM OF ForCE SYSTEMS [Chap. III

348. The frame shown in Fig. P-348 is supported in pivots at A and B. Each
member weighs 50 lb per ft. Compute the horizontal reaction at A and the hori-
zontal and vertical components of the reaction at B.

Fic. P-348.

349. The truss shown in Fig. P-349 is supported on rollers at A and a hinge at B.
Solve for the components of the reactions.
Ans: A, = 740 1b; B, = 260 tb;
By, = 240 lb

400 lb
Fic. P-349.

350. Compute the total reactions at A and B for the truss shown in Fig. P-350.
Ans. A = 1085 lb; B = 1250 lb up to the left at 6, = 76.1°
Art. 3-7] Equilibrium of Non-Concurrent Force Systems dere

600 Ib 1200 1b 500 lb


Fic. P-350.
351. The beam shown in Fig. P-351 is supported by a hinge at A and aroller ona
1 to 2 slope at B. Determine the resultant reactions at A and B.
400 lb

Fig. P-351.
352. A pulley 4 ft in diameter and supporting a load of 200 lb is mounted at B
on a horizontal beam (Fig. P-352). The beam is supported by a hinge at A and
rollers at C. Neglecting the weight of the beam, determine the reactions at A and C.
Ans. A = 180 lb up to right at 0, = 16.1°; C = 50 lb

Fig. P-352.
78 EQuiILisriuM OF Forcr SYsTEMs [Chap. III
i eT era latercrn tax EG pal tN Pate pe
353. The forces acting on a 1-ft length of a dam are shown in Fig. P-353. The
upward ground reaction varies uniformly from an intensity of p1 lb/ft at A to
po lb/ft at B. Determine p; and p; and also the horizontal resistance to sliding.
Ans. p; = 1222 lb/ft; p2 = 1778 lb/ft; F = 4800 lb

Fire. P-353.
354. Compute the total reactions at A and B on the truss shown in Fig. P-354.
Ans. A = 4510 lb up to right at 6, = 48.4°; B = 4625 lb
2240 Ib

1000 lb 2000 Ib 3000 Ib


Fie. P-354.

355. Determine the reactions at A and B on the Fink truss shown in Fig. P-355.
Members CD and FG are respectively perpendicular to AE and BE at their mid-
points. Ans. A = 5360 lb; B, = 6130 lb; B, = 895 lb
1000 1b

2000 Ib

Fig. P-355.
Art. 3-7] Equilibrium of Non-Concurrent Force Systems 79
Se ci lle ile a i ie ih
356. The cantilever truss shown in Fig. P-356 is supported by a hinge at A anda
strut BC. Determine the reactions at A and B.
Ans. A = 2000 lb up to right at 0, = 60°; B = 3460 lb

1000 lb

1000 lb

1000 lb

1000 Ib

Fic. P-356.

357. The uniform rod in Fig. P-357 weighs 420 lb and has its center of gravity
at G. Determine the tension in the cable and the reactions at the smooth surfaces
at A and B,
Ans. N = 254 lb; T = 180 lb; A = 240 lb

Fie. P-357. Fic. P-358.

358. A bar AZ is in equilibrium under the action of the five forces shown in
Fig. P-358. Determine P, FR, and T.
Ans. P = 371 |b right; R = 428 lb down; T = 1285 lb up to left
80 EQUILIBRIUM OF ForcE SYSTEMS [Chap. III

359. A 12-ft bar of negligible weight rests in a horizontal position on the smooth
planes shown in Fig. P-359. Compute the distance x at which load T = 100 lb
should be placed from point B to keep the bar horizontal. Ans. «x = 4.83 ft

P=2001b L

Fic. P-359, P-360, and P-361.

360. Referring to Prob. 359, what value of T acting at x = 3 ft from B will keep
the bar horizontal?

361. Referring to Prob. 359, if T = 300 lb and x = 3 ft, determine the angle 6 at
which the bar will be inclined to the horizontal when it is in a position of equi-
librium.

SUMMARY

Equilibrium is the term used to express the condition existing when the
force system acting on a body has a resultant equal to zero. This definition
determines the equations of equilibrium for various force systems by merely
specifying the conditions required to make the resultant equal to zero.
For a concurrent force system (Art. 3-3), the conditions of equilibrium
are
xX 4 (3-1)
xY=0
Although any number of reference axes may be chosen, no more than two
unknown quantities may be determined because only two independent
conditions of equilibrium exist. A recommended method is to select the
X axis so that it coincides with one of the unknowns; a Y summation then
determines the other unknown, and an X summation determines the first
unknown.
The following relations are a frequently useful variation of the above
equations:
=M, Of (3-2)
x=M;, = 0

To eliminate an unknown force, select the moment center A on its line of


action thereby permitting a direct evaluation of the other unknown force.
Summary 81

Similarly the first unknown is determined from a moment summation about


a center B located on the action line of the second unknown. Note care-
fully that the line joining A and B must not pass through the point of
concurrency of the system.
The conditions for equilibrium of parallel force systems (Art. 3-6) are
= ='0 (3-4
=M=0
It will be found more convenient to replace these conditions by equivalent
moment summations as follows (the condition >F = 0 being reserved as
a check on the results):
=M,=0
(3-4a)
Se — 0

Note carefully that the line joining A and B must not be parallel to the
forces. Observe also that these equations permit only two unknowns to be
determined since there are only two independent conditions of equilibrium.
The conditions for equilibrium of non-concurrent force systems (Art. 3-7)
give three independent equations from which no more than three unknown
quantities may be determined. Any one of the following sets of equations
may be used; the best selection depends upon the specific problem to be
solved. When applying moment summations it is best to select the moment
center at the intersection of two of the unknowns, thereby eliminating
these unknowns from the equation and permitting a direct determination
of the third unknown quantity.
=x =—0 rp, Cet) =M, = 0
=Y=0;7 or =Mz,=0; or =M;z = 0 (3-5)
=M=0 =M; = 0 xM:, =0
In applying these conditions, the line joining A and B must not be per-
pendicular to the force summation axis, nor may A, B, and C lie on the
same straight line.
Chapter IV.
Analysis of Structures
el
>

4-1. Introduction
The analysis of a structure is the process by which we determine how the
loads applied to a structure are distributed throughout the structure. Al-
though there are many kinds of structures, we limit ourselves here to pin-
connected types, i.e., those that consist of assemblages of bars fastened
together by smooth bolts. Our purpose is to determine the forces acting
in the bars and upon the pins or hinges of the structure. In subsequent
courses we shall learn how to determine the dimensions of the various parts
of a structure so they can safely resist these forces.
Two types of structures will be studied — pin-connected trusses and pin-
connected frames. The essential difference between these types is that
in trusses, the internal force in a bar is directed along the axis of the bar,
whereas in frames, the members are subjected to bending action. Occa-
sionally a structure is composed of members some of which are subjected
to axial forces while others are subjected to bending action; these are also
called frames. After we have learned how these types of structures are
constructed, we shall see that the internal forces holding their various mem-
bers in equilibrium create concurrent or non-concurrent systems of forces
in equilibrium. Consequently a force analysis of a structure consists of
applying the conditions of equilibrium studied in the preceding chapter to
determine the internal forces that act in or upon its various members.

4-2. Construction of Simple Trusses


A truss is a structure composed of members fastened together in such a
way as to resist change in shape; it is a rigid structure.' The purpose of a
truss is to support a larger load or span a greater distance than any indi-
vidual member from which the truss may be built. To make a structure
rigid, its members must be fastened together in such a way as to prevent

1 The term rigid is used in the sense of having no deformation. Actual members are,
of course, subject to elastic deformation which can be neglected if small compared with
the dimensions of the truss. There are, however, some types of trusses in which the
deformations must be taken into account; they must be solved by considering the elastic
deformation of the members themselves. These types are beyond the scope of this book;
for their analysis, the student is referred to Structural Theory by Sutherland and Bowman,
or Theory of Structures by Timoshenko and Young.
82
Art. 4-2] Construction of Simple Trusses 83
any movement between them. A pin-connected structure meeting this
condition is shown in Fig. 4-1. This structure is composed of three bars.
A pin-connected structure composed of four bars arranged as shown in
Fig. 4-2 is not inherently rigid; it will collapse as indicated under the action
of the applied forces.

A Cc
Fic. 4-1. — Rigid frame. Fia. 4-2. — Non-rigid frame.

In the construction of trusses, the basic design is three bars arranged


to form a triangle. To this base triangle, ABC in Fig. 4-3, two more bars
may be added to locate a joint D which is rigid relative to the other joints.
Since A and D are now fixed relative to each other, two more bars may be
added at these joints to fasten together at EH, after which two additional
bars determine F. Thus by continuing to add bars at joints rigid to one
another, additional joints can be determined and a truss consisting of many
members may be constructed. Trusses built up in this manner are known
as simple trusses.

B D

E
Fic. 4-3. — Formation of a simple truss.

Trusses are usually supported by anchoring one joint to the foundation


by means of a fixed hinge as at F in Fig. 4-3. Since the truss could rotate
about this hinge, additional support is necessary. This is supplied by
mounting another joint, say A, on rollers in such a way as to prevent that
rotation. Then the truss is completely constrained against any movement.
Assumptions in Simple Trusses. The members of a truss are joined to-
gether by means of pins at their ends (actually large bolts known as pins
which act as pivots) or by riveting or welding them to a common plate
84 ANALYSIS OF STRUCTURES [Chap. IV

known as a gusset plate. If due care is taken to assemble the bars so that
the centerlines of the members intersect in a common point at each joint,
the forces in the members may be calculated as if they were pin-connected,
even though their ends are actually riveted or welded to a gusset plate.
Trusses are so constructed that all applied loads act at the ends of the
members. This construction is illustrated in Fig. 4-4 for a simple bridge.

Fic. 4-4. — Simple bridge.

Loads are carried by the flooring which is supported by stringers. The


stringers in turn are supported by crossbeams which are fastened to the
joints. The net effect is that all loads are applied at the joints of the truss.
Further, it is usually assumed that the weight of the truss members is
negligible in comparison with the applied loads. In case the weight is not
negligible, calculations are made on the assumption that the weight of each
member is divided equally into two forces which act vertically downward
on the end pins of the member.
Since the space position of a truss member is fixed by the rigid construc-
tion of the truss and the loads are applied only at the joints, a free-body
diagram of a typical member would show it acted upon by only two equal,
oppositely directed forces exerted by the pins at its ends. These forces
must be axially directed along the member; otherwise we could not satisfy
Axiom 2 (see p. 5) that two forces are in equilibrium only when equal in
magnitude, opposite in direction, and collinear in action. Such members
held in equilibrium by only two forces are called two-force members. The
fact that the internal force in a two-force member is axial is important be-
cause it determines the action line of the force although its magnitude may
be unknown. Review now the discussion concerning rod AB in Fig. 3-2a
on p. 52.
Art. 4-3] Method of Joints 85
ee
eeee e ee e
Members which are stretched are said to be in tension, while those that
are shortened are said to be in compression. In a typical truss, Fig. 4-5a,
if the member CE were in tension, isolating it from the truss would give the
free-body diagram shown in part (b), and its effect on the joints of the truss
would appear as forces T which pull away from these joints. Another
member, BD, assumed to be in compression would be isolated as shown in
(c) Compression

(a) Original truss (b) Tension


Fic. 4-5. — A tensile member exerts pulls away from its end joints while a compression
member pushes toward them.

the free-body diagram of part (c) while its effect on the joints of the truss
would be represented by forces C which push toward the joints. From
these diagrams, we deduce the following rule: A member in tension causes
forces which pull away from its end joints whereas a member in compres-
sion causes forces which push toward its end joints.

4-3. Method of Joints


The assumption that all members of a truss are two-force members carry-
ing axial loads means that the free-body diagram of any joint is a con-
current force system in equilibrium. Since only two independent equations
of equilibrium can be written for a concurrent system, we must start with a
joint acted on by only two members. The unknown forces in these mem-
bers are determined by using any of the methods previously discussed in
Arts. 3-3 and 3-4. Once these forces have been determined, their effects on
adjacent joints are known, and we continue solving the concurrent force
systems acting at successive joints until the unknown forces in all members
have been found. At no time should there be more than two unknown
forces at a joint.
The method of joints may be summarized in the following steps; these
will then be applied to an illustrative problem.
1. Choose a pin (i.e., joint) on which no more than two members act.
86 ANALYSIS OF STRUCTURES [Chap. IV

When the force in each member has been determined, it is indicated on the
truss diagram by arrows at each end of the member. These arrows act in
the direction appropriate to the force, i.e., toward the pin for compression
and away from it for tension. When the force in a member is determined
and appropriate arrows are marked on the original truss diagram, the mem-
ber is called a marked member. :
2. Draw the free-body diagram of the pin having only two unmarked
members, assuming the forces in the unmarked members to be either ten-
sion or compression. Solve the resulting equilibrium problem of concurrent
forces for the unknown forces. If a negative value is obtained for any
force, the result will be correct in magnitude, but opposite in action to that
assumed.
3. After determining the forces at a pin, mark the original truss diagram
with appropriate arrows at each end of the member whose force you have
found. Remember that the arrows act away from the pins for tension and
toward them for compression.
4. From the original truss diagram, select the next pin at which there
are only two unmarked members. Draw a new free-body diagram and de-
termine the forces. Continue this procedure until you have marked all
the members, thereby indicating that the internal forces in all members
have been found.
5. In many cases it is preferable to work from one end of the truss to the
middle member, and then from the other end of the truss back toward the
same mid-member. A check on the accuracy of the calculations is obtained
if the forces in the mid-member agree as determined with these two in-
dependent methods of analysis.

ILLUSTRATIVE PROBLEM
401. A Fink truss is loaded as shown in Fig. 4-6. Determine the force in each
member of the truss assuming them to be pin-connected.
1000 lb

1000 1b 1000 lb

3500 1b 2000 lb 2000 lb 3500 lb


Fic. 4-6. — Fink truss.
Art. 4-3] Method of Joints 87
Solution: The given truss is symmetrical and also symmetrically loaded so that
the forces need be found in only one-half of it.
After determining the reactions from symmetry, consider joint A which has only
two unmarked members (AB and AC) acting upon it. As shown in Fig. 4-7, we
may use either the FBD of the joint or the equivalent FBD of the pin. Of the two,
the FBD of the pin is preferred since it is simpler to draw. In either diagram, it is
evident that AB denotes compression, i.e., is directed toward the pin, in order that

3500 Ib

(a) FBD of joint A (b) FBD of pin A’

Fic. 4-7. — Free-body diagrams of joint and pin.

its vertical component may balance the upward reaction. Hence AC must be in ten-
sion and pull away from the pin to balance the leftward component of AB. Select-
ing the X axis to coincide with the unknown force AC, we obtain
[ZY = 0] 3500 — AB sin 30° |= 0 AB = 7000 lb C Ans.
[2X = 0] AC — 7000 cos 30° = 0 AC = 6062 lb T Ans.
The positive values obtained for AB and AC confirm the original assumption con-
cerning the direction of these forces. The action of members AB and AC on their
end pins, indicating respectively compression and tension, may now be drawn as
shown in Fig. 4-8. (In an actual problem the arrows would be placed on the

1000 lb

1000 1b 1000 lb

3500 lb 2000 Ib 2000 lb 3500 Ib

Fic. 4-8. — Truss marked to show effect of members AB and AC on their end pins.
88 ANALYSIS OF STRUCTURES [Chap. IV

1000 lb original diagram of the truss, but to indicate the


2% 30° SK marked and unmarked members more clearly, the
of truss is here redrawn.)
a From Fig. 4-8, the next pin at which no more than
z ‘two unmarked members appear is seen to be B. Re-
: peating the technique used at pin A, equilibrium of
AB=7000 lb. ‘BC pin B can be achieved by assuming BD and BC to be
Fig. 4-9. —Free-body dia- 1 compression and therefore acting toward the pin
gram of pin B. as shown in Fig. 4-9. Rotating the X axis to coin-
cide with the unknown force BD, we obtain
[ZY¥e="0] BC — 1000 cos 30° = 0 BC = 8661lbC Ans.
[2X = 0] 7000 — 1000 sin 30° — BD = 0 BD = 6500 lb C Ans.
The positive values obtained for BD and BC confirm the fact that these forces
are compressions. The action of BD and BC upon their end pins may now be
marked on the original truss diagram as in Fig. 4-10.
1000 1b

1000 Ib D 1000 lb

3500 lb 2000 Ib 2000 lb 3500 lb


Fic. 4-10. — Truss marked to show effect of members BD and BC on their end pins.

The next pin at which two unmarked members appear is C. Assume both CD
and CE to be in tension. The FBD of pin C can now be drawn as in Fig. 4-11.
Selecting the X axis to coincide with CE, we have
[sY =0] CDsin 60° — 866sin 60° —
2000 =0 CD =3175lbT Ans, BC=866lb |¥ cp
[=X = 0] CE + 3175 cos 60° + 866 cos 60° —
6062 = 0 OF SA040in nT wine
As mentioned previously, the loading and the
truss are symmetrical so that the forces in all
the members are now determined. If the truss
or loading were not symmetrical, however, the
solution would be continued by proceeding to 2000 lb
the next unmarked pin. This pin is D, but an- Fig 4-11 — Proapecy diagram
other pin having only two unmarked members of pin C.
Art. 4-3] Method of Joints 89

acting upon it is pin G. It is preferable to avoid pin D, start anew from pin G, and
determine the forces in FG and EG. After the action of FG and EG upon their
end pins is indicated in the original truss diagram, the next pin to be selected for
analysis is pin 7. From the FBD of F, the forces DF and EF can be found. Next,
the FBD of pin £ will enable us to find the forces in DE and CE. The force in CE
will then have been determined from the FBD at C and, again independently, from
the FBD at Z. A check on the accuracy of the work is thus obtained if the force in
CE as found from pin C agrees with that found from pin Z.
The final appearance of the original truss diagram after all the forces have been

1000

Fic. 4-12. — Order of taking free-body diagrams. All members marked indicates that
all forces have been determined.

determined is shown in Fig. 4-12. This figure also indicates the order in which the
free-body diagrams of the various pins would be drawn if the truss or the loading
were not symmetrical.

PROBLEMS
402. Joint B of the truss shown in Fig. P-402 is subjected to the forces exerted by
the three members AB, BC, and BD. Members AB and BD are in the same straight
line, but BC is inclined at an angle of @ degrees with this straight line. Show that the
force in BC must be zero. Generalize this result and then show that the force in
members CD, DE, EF, FI, HI, HK, and JK is also zero. [Ans. given on page 90.]

Fic. P-402.
90 ANALYSIS OF STRUCTURES [Chap. IV

Ans. At a joint subjected to the action of three


members but no other load, if two of the
members are collinear, the force in the third
member must be zero.
403. Determine the force in each bar of the truss
shown in Fig. P-403. Hint: First determine which
bars carry no load using the principle developed in
Prob. 402.
Ans. CD*=DE = 0.57(P Cy BF = '0.289P T
404. Determine the forces in the members of the
roof truss shown in Fig. P-404.
Ans. ABT= 100 Wb ‘ClAC = 86.6 1b 1-50 =
100 1b-T; "CD = "86:6 lb-T; BD =.200)1b: G
405. Determine the force in each bar of the truss
shown in Fig. P-405 caused by lifting the 120-lb load
at a constant velocity of 8 ft per sec. What change in
Fie. P-408. these forces, if any, results from placing the roller
support at D and the hinge support at A?

1001b

100 lb
Fig. P-404. Fic. P-405.

406. The cantilever truss in Fig. P-


1000 lb
406 is hinged at D and H. Find the
force in each member.
Ans. AB = 2000: lb T; AC = 1732
lb C; BC = 866 lb C; BD =
2000 lb T; DC = 2020 Ib T;
CH = 3175 |b'©

407. Inthe cantilever truss shown in


Fig. P-407, compute the force in mem-
bers AB, BE, and DE.
1000 lb 1000 Ib
Ans. AB = 1732lbT;BE = DE =
1154 lb C Fic. P-406.
Art. 4-3] Method of Joints 91
1000 lb

1000 1b

1000 lb

Fig. P-407.

408. Compute the force in each member of the Warren truss shown in Fig. P-408.
Ans. AB = 4910 lb C; AC = 2455 lb T; BC = 2600 lb T; BD = 3755 lb C;
CD = 2020 lb T; CE = 2745 lb T; DE = 5480 lb C
2000 lb 3000 Ib

4000 |b
Fic. P-408.

600 lb 1000 lb 400 Ib


Fic. P-409.
92 ANALYSIS OF STRUCTURES [Chap. IV

409. Determine the force in members AB, BD, BE, and DE of the Howe roof
truss shown in Fig. P-409.
410. Determine the force in each member of the Pratt roof truss shown in Fig.
P-410.

1800 lb .

D
1800 1b 1800 1b

a iat sio htepluie rp


Fig. P-410.

411. Determine the force in members AB, AC, BD, CD, and CE of the canti-
lever truss shown in Fig. P-411. If the loads were applied at C and H instead of at B
and D, specify which members would have their internal force changed.
Ansa DD — 180i b alt CDF 9250) ba OF —a3 00M bre

100 lb

Fie. P-411.

412. Compute the force in each member of the truss shown in Fig. P-412. If the
loads at B and D are shifted vertically downward to add to the loads at C and Z, will
there be any change in the reactions? Which members, if any, would undergo a
change in internal force?
Ans. AB = 983 lb C; AC = 440 lb T; BD = 492 lb C; BC = 700 1b T; CD =
1120 lb C; CE = 1440 lb T; EF = 1440 lb T; DE = 2001bT; DF = 1610
Ib C
Art. 4-4] Method of Sections 93
400 lb

413. Determine the force in each


member of the crane shown in Fig.
P-413.
Ans. AB = 12,000 Ib C; AC =
13,100 lb T; BC = 6000 lb C;
BD = 10,400 lb C; CD = 0
414. Determine the force in mem- =,
bers AB, BD, and CD of the truss D 5200 1b
shown in Fig. P-414. Fie. P-413.

300 lb 300 lb 900 Ib


Fia. P-414 and P-415.

415. Solve for the force in members FH, DF, and DG of the truss shown in
Fig. P-414. Ans. FH = 1270 lb C; DF = 947 lb C; DG = 375 lb T

4-4, Method of Sections


We have seen that the method of joints consists of analyzing trusses by
applying the principles of equilibrium to the concurrent force systems
acting at each joint. The principles of equilibrium of non-concurrent force
94 ANALYSIS OF STRUCTURES [Chap. IV
dN neiniaReR MN HORE SE FE
systems may also be applied to truss analysis; the procedure is known as
the method of sections.2 Its use permits us to determine directly the force
in almost any member instead of proceeding to that member by a joint-to-
joint analysis.
In the method of sections, a cutting plane is passed through the entire
truss, separating it into two parts without cutting more than three mem-
bers. If this plane actually severed the members, the truss would collapse.
But suppose, at the instant of severing the members, that an external
force is applied to each side of the cut members exactly equivalent to the
load being transmitted by the members. We shall then have two parts of
the truss, each constituting a non-concurrent system of forces in equilib-
rium under the action of the known loads that act on each part and the
unknown forces (stresses) that the members of one part exert on the other.

eae Plane
D F H J

R,=25001lb 10001lb 1000)b 1000]lb 10001b 10001b R,=2500 1b


Fig. 4-13. — Section through a truss.

For example, consider the truss shown in Fig. 4-13. If a section cuts
through the members DF, EF, and HG, the truss may be separated into
two parts, each of which is held in equilibrium by unknown forces equiva-
lent to the loadg being transmitted by these members. These two parts
are shown in Fig. 4-14. Each part in this figure constitutes a system of
non-concurrent forces in equilibrium. Since the unknowns in either system
are the same, it is generally best to determine the unknowns from the
equations of equilibrium applied to the simpler system. Here part (a) is
obviously the simpler system since it involves fewer forces. The forces in
the uncut members are internal to the free bodies shown. Consequently
the forces in these uncut members occur in equal opposite pairs at their
joints and hence cancel out of any calculation involving the entire free body.
To simplify the calculations, we use a condition of equilibrium that de-
termines each unknown force independently of the other unknowns. Usu-
? Strictly speaking, the method of joints is really a variation of the method of sections:
the section is taken around the joint. It is traditional, however, to use the expression
method of joints to apply to the equilibrium of concurrent forces at a joint, and the ex-
pression method of sections to apply to the non-concurrent forces set up by a section
through the truss.
Art. 4-4] Method of Sections 95

ally a moment summation is best, the center of moments being chosen at


the intersection of the unknown forces to be eliminated from the moment
summation. Thus in Fig. 4—14a, if the force in DF is desired, select E as

eg Cutting Plane
ares H J
EF
EC AG T K ©
R,=2500 lb 10001b 1000 Ib 1000 lb 1000 1b 10001b R,=25001b
(a) (b)
Fie. 4-14. — Each segment of the truss is held in equilibrium by forces equal to the
stresses in the cut members.

the center of moments. To determine the force in EG, select the center of
moments at the intersection of DF and EF. This will be at a point F’
having the location of the original point F with respect to the left section.
However, the moment method cannot be applied to determine EF because
DF and EG, being parallel, intersect at infinity. Nevertheless, a vertical
summation of forces may be taken to eliminate HG and DF from the calcu-
lation to determine FF.
The procedures just described constitute the method of sections. This
method usually permits us to determine the force in any desired member
without the necessity of finding the forces in the other members. It is
merely necessary that the cutting plane passes through the member whose
force is desired. It is essential, however, not to pass the plane so as to cut
more than three members whose internal forces are unknown, because the
method of sections is based on the equilibrium of non-concurrent forces for
which no more than three unknowns can be determined. An exception is
the case where the cutting plane may pass through the desired member as
well as several others, provided all these other members pass through a
common joint which is used as the center of moments. The method of
sections will be described more fully by the following illustrative problem.

ILLUSTRATIVE PROBLEM
416. Find the force acting in members BD, BE, and CE of the Bowstring truss
shown in Fig. 4-15, using the method of sections. In each case determine the force
by means of an equation which does not involve the other unknown forces.
Solution: The required forces in BD, BE, and CE can be found by passing the
section a-a through these members. Since the original truss is in equilibrium, the
two segments into which it is divided by a cutting plane will also be in equilibrium
96 ANALYSIS OF STRUCTURES [Chap. IV
Oe a ee

A, =18001b 1200 Ib 1200 lb 1200 lb H,,=1800 1b


Fic. 4-15.

if the loads carried by the cut members are replaced by external forces equal to these
loads. These external forces are shown in Fig. 4-16 acting on the FBD of the left
segment of the truss because it involves a smaller number of forces than the right
segment.

A,=1800lb 1200 1b BE
Fia. 4-16. — Free-body diagram of left section.

To determine the value of CE we use an equation of equilibrium which eliminates


BD and BE. Since BD and BE intersect at B, these forces have no moment about
B; hence we apply the equation
[D=IMz = 0] 8 CE — 12 X 1800 = 0 CE = 2700lbT Ans.
The member CE is in tension since the force pulls away from the pin C.
The value of BD is found by taking moments about # which thereby eliminates
BE and CE since their lines of action, if prolonged, intersect at H. As a rule, the
center of moments is chosen at the intersection of the two unknown forces to be
eliminated. Note that here the spatial position of the moment center E is de-
termined from the dimensions of the original truss.
The moment of BD about F is most easily found as the sum of the moments of its
Art. 4-4] Method of Sections 97

horizontal and vertical components. It is convenient to replace BD at D (located on


its line of action) by the horizontal and vertical components shown dashed in Fig.
4-16. This eliminates the vertical component BD, from the moment summation
about HE. We therefore obtain
[D=ZMz = 0] 12 BD; + 12(1200) — 24(1800) = 0 BD), = 2400 lb
whence, using the proportionality between the force and slope triangles, we have
BDIEPED. “BD, 24
Age BDA 1800 Ih anda D12590
Ib Gad:
1 3 3
To solve for BE independently of BD and CE, we follow the rule of taking mo-
ments about the intersection of the forces to be eliminated. In this instance, the
intersection of BD and CE is not at once evident, but it is easily located by observing
that the slope of BD is such that it drops one foot vertically in each three feet hori-
zontally. Hence the point O must lie three times as far to the left of C as B is above
C;i.e., 3 X 8 = 24 ft, which locates O as shown on Fig. 4-16. In taking moments
about O, replace BE by its horizontal and vertical components acting at H, thereby
eliminating the horizontal component BE; from the moment summation. Thus we
obtain
[D=Mo = 0) 36 BE, — 24(1200) + 12(1800) = 0 BE, = 200 lb
whence, from the proportionality between the force and slope triangles, we get
BE BE, BE, 200
VB = 3 = 5 am a oF BE; == 300lblk and BE = 3611bC Ans.
ns

We can now check the accuracy of our computations by taking horizontal and
vertical summations of the forces acting on the FBD of Fig. 4-16. Thus,
[2H = 0] CE — BE, — BD, =0 or 2700 — 300 — 2400 = 0 Check
[ZV = 0]
BE, — BD, + 1800 — 1200 = 0 or 200 — 800 + 1800 — 1200 = 0 Check

PROBLEMS
417. Using the method of sections, determine the force in members BD, CD,
and CE of the roof truss shown in Fig. P-417.
Ans. BD = 160 lb C; CD = 200 lb C; CE = 320 lb T

B D

C
cea"
360 Ib
Fic, P-417.
98 ANALYSIS OF STRUCTURES [Chap. IV

418. The Warren truss loaded as shown in Fig. P-418 is supported by a roller at C
anda hinge at @. By the method of sections, compute the force in the members BC,
DF, and CE. Ans. BC = 448 lb C; DF = 800 lb C; CH = 100 lb T
800 lb

400 lb 1000 lb
Fic. P-418.

419. Use the method of sections to determine the forces in members BD, CD,
and CE of the Warren truss described in Prob. 408 and Fig. P-408 on page 91.
420. Determine the force in the members DF, DG, and EG of the Howe truss
shown in Fig. P-420. Ans. DF = 2800lb C; DG = 1500lb C; HG = 4000 lb T
800 lb 1200 1b

2000 lb
4 4 Panels at 12’=48’
A,=1900 Ib H,=2100 lb
Fia. P-420.

421. Use the method of sections to compute the force in the members DF, EF,
and HG of the cantilever truss described in Prob. 411 and Fig. P-411 on page 92.
422. Refer to the truss described in Prob. 412 on page 92, and compute the
force in members BD, CD, and CE by the method of sections.
423. Use the method of sections to determine the force acting in members DF,
EF, and EG of the Howe truss described in Fig. P-409 on page 91.
424. For the truss shown in Fig. P-424, determine the force in BF by the method
of joints and then check this result using the method of sections. Hint: To apply
Art. 4-4] Method of Sections 99

the method of sections, first obtain the


value of BE by inspection.
Agrseuee He 2500) lb) G
425. In the Fink truss shown in Fig.
P-425, the web members BC and EF are
perpendicular to the inclined members at
their midpoints. Use the method of sec-
tions to determine the force in members
DF, DE, and CE.
Ans DE 5.62) kips ©; DEH = 199
kips T; CH = 4 kips T
426. Show that the method of joints
cannot determine the forces in all bars of
the Fan Fink truss in Fig. P-426. Then
use the method of sections to compute
the force in bars FH, GH, and EK.
Ans. FH = 1100 lb C; GH = 520 lb 2400 Ib 1200 lb
T; EK = 693 lb T Fig. P-424.
Qk

Fic. P-425.

200 lb

cS 60’
Fic. P-426.
100 ANALYSIS OF STRUCTURES [Chap. IV

427. Determine the force in bars BD, CD, and DE of the nacelle truss shown in
Fig. P-427.
6001lb 12001b 1200lb 1200\b

Hie. P=427,

428. Use the method of sections to determine the force in members DF, FG, and
GI of the triangular Howe truss shown in Fig. P-428. Hint: First determine by in-
spection the forces in the web members of the right side of the truss.
Ans. DF = 2.51 kips C; FG = 2.24 kips T; GI = 0.455 kips T
2 kips
F

= 6 panels at 10’= 60’ =


Fic. P-428.

429. For the cantilever truss shown in Fig.


P-429, determine the forces in members DF, FH b)
FI, GI, and FG.

200 lb 200 Ib 400 lb 400 lb


Fig. P-429.
Art. 4-4] Method of Sections 101
430. The loads on the Parker truss shown in Fig. P-430 are in kips. One kip
equals 1000 lb. Determine the forces in members BD, BE, CE, and DE.

ox
. 30* 30* 30% 30* 30* 30% 30%
8 panels at 25= 200’
Fic. P-430 and P-481.

431. Determine the force in the members DF, DG, and EG for the Parker truss
shown in Fig. P-430.
Ans. DF = 162 kips C; DG = 32.7 kips T; FG = 140.6 kips T
432. Use the method of sections to compute the force in members AB, AD, BC,
and BD of the truss shown in Fig. P-432.
Ans. AB = 5910 lb C; AD = 3000 lb T; BC = 6400 lb C; BD = 5000 lb T

2400lb 3600lb

Fig. P-432.

433. Compute the forces in bars AB, AC, DF, and DE of the scissors truss shown
in Fig. P-433.
Ans. AB = 70.8kipsC;AC = 48.0kipsT; DF = 42.5 kipsC; DE = 9.6 kipsC
434. Compute the force in bars GJ, GH, EH, and HI for the scissors truss shown
in Fig. P-433.
435. For the transmission tower shown in Fig. P-435, determine the force in
member CJ. Hint: First use section a-a to find the force in BC.
Ans. CJ = 250 lb C
102 ANALYSIS OF STRUCTURES [Chap. IV

[104 P=7801b

Fic. P-433 and P-434. Fic. P-435.

4-5. Redundant Members; Counter Diagonals


In trusses subjected to moving loads, such as railroad bridges, the
diagonal members may undergo a reversal of stress; that is, a diagonal
normally in tension may become subject to a compressive load. If the
diagonal is composed of eyebars or is otherwise so slender compared with
its length that it will buckle under compressive loads, an additional diagonal
sloping in the opposite direction must be provided in the truss panel to
prevent collapse of the structure.
Additional diagonals of this type might be called redundant, 1.e., unneces-
sary; but as only one of these diagonals acts at a time, the term redundant
does not apply in this case, although it is frequently used. These additional
diagonals are usually known as counter diagonals or, more briefly, counters.
They are generally represented by dashed lines on a truss. In considering
the action of counters, it is convenient to regard them as wires which can
support tension but will buckle instantly if subjected to compression.
As an example consider the truss shown in Fig. 4-17. It is required to
determine which of the tension diagonals AD or BC in the third panel is
acting under the given loading. This is accomplished by passing the section
a-a through the truss. Considering the part of the truss to the left of
section a-a, we see that the upward effect of R: must be balanced by a
downward component of the force in the diagonals.
In order to have a downward component, the forces in the diagonals act
as shown. If BC were acting, the arrow toward pin C indicates that the
member would be in compression and hence buckle. On the other hand,
Art. 4-5] Redundant Members; Counter Diagonals 103
SE a tan a a aga td

qj a
600 lb 600 Ib 600 Ib
6 panels at 20’=120’
R,=600 Ib R»=1200Ib
Fic. 4-17. — Counter diagonals.

the arrow away from the pin at A indicates AD to be under tension and AD
is the tension member acting. From the dimensions of the truss and by
equating a vertical summation of forces to zero, we find the force in member
AD to be
[ZY = 0] 600 — AD X 318 = 0 AD = 1000lbT Ans.
PROBLEMS

436. In Fig. P-420, page 98, assume that counter diagonals act from B to # and
from £ to F in addition to the counter diagonals CD and DG shown in the figure.
Assuming that these counter diagonals can support tension only, determine which
diagonals are acting and the force in each.

B D

Fic. P-437.

437. The center panel of the truss in Fig. P-437 contains two flexible cables.
What load P will cause a compressive force of 2000 lb in BD? Then determine
which tension diagonal BE or CD is acting and the force in it.
Ans. CD = 1414 lb T
438. The center diagonals of the truss in Fig. P-438 can support tension only.
Compute the force in each center diagonal and the force in BC, DH, and FG.
Ange BE = 10501b T; DG = 175 lb T
104 ANALYSIS OF STRUCTURES [Chap. IV

200 lb 200 lb 200 lb


Fic. P-438.

4—6. Definition and Characteristics of Three-Force Members


A three-force member is one subjected to three or more forces applied at
different positions. It is essentially subjected to bending loads; in fact,
according to the above definition, any beam is a three-force member.
Consider the beam shown in Fig.
Jy 4-18, which is supported by a hinge at
one end and a roller at the other. It
is easy to see that the reactions are
vertical, since the roller permits the
fay Lo» ends of the beam to approach each
Roller Hinge: other as the beam flexes. If both ends
| of the beam are hinged to rigid sup-
ae B, Ports as in Fig. 4-19, the flexing action
Te eee ee one mote of the bending load will be restrained,
and hinge. thereby causing the horizontal com-
ponents of the end reactions shown,
This effect occurs in all members which are hinged at both ends and
subjected to bending loads.
In Fig. 4-19 the total hinge force at A is shown by the dashed force A
inclined at an unknown angle 64 with the horizontal. A similar force and

Fic. 4-19. — Beam hinged to rigid supports.


Art. 4-7] Frames Containing Three-Force Members 105

angle exist at B. There are therefore four unknown elements. They may
be represented by the two unknown forces A and B plus the two unknown
angles 64 and 6, or by the unknown magnitudes of their components, i.e.,
A,, An, B,, and By. The three equations of equilibrium are not sufficient
to solve for all four of these unknowns.
It is important to note that in the characteristic action of a three-force
member, the end forces are not directed along the axis of the member as
Figs. 4-18 and 4-19 show. Consequently a section passed through a
three-force member will disclose that the member is subjected not only to
an axial tension or compression but also to a bending action whose effect
varies with the location of the section. Therefore it is not convenient to
analyze a structure containing three-foree members by passing a section
through the members as was done in Art. 4-4; instead we must disconnect
the members and draw a free-body diagram of each one.

4-7. Method of Members Applied to Frames Containing Three-Force


Members
if some or all the members of a pin-connected structure are subjected to
bending action, the structure is called a frame. Since it is not feasible to
pass a section through the members of such structures, they are analyzed
by considering the separate free-body diagram of each three-force member;
the procedure is called the method of members.
The essential principle involved in drawing the free-body diagrams of the
several members of a frame is that of action and reaction. When separating
the members for analysis, the forces (usually represented by their compo-
nents) exerted by the connecting pins must be consistently represented as
acting in opposite directions on the separated members. If either compo-
nent of a force is incorrectly assumed in direction, the solution will still
give its correct magnitude but with a negative sign. This would mean that
the component of the force on both separated members acts in the opposite
direction to that originally assumed. Further discussion of the method of
members is given in the following illustrative problem.

ILLUSTRATIVE PROBLEM
439. The A-frame shown in Fig. 4-20 supports a 600-lb load and has the given
dimensions. Neglecting the weights of the members, compute the values of the
forces acting on the pins B, C, and D.
Solution: The roller at A compels the reaction to be vertical. Since no horizontal
forces are acting, the reaction at HZ must also be vertical. These reactions are found
by means of the following equations of equilibrium:
(2M, = 0] 20 LH, — 600 K 12 = 0 E, = 360lb
[Mz = 0] 20 A, — 600 x 8 = 0 A, = 240lb
106 ANALYSIS OF STRUCTURES [Chap. IV

Fic. 4-20. — A-frame.

The free-body diagrams of the members of the A-frame are shown in Fig. 4-21.
From Art. 4-6, the forces acting on BD can be drawn as shown in part (b) of this
figure. Likewise the FBD of AC is shown in part (a). AC is subjected to the re-
action A, = 240 lb and also to the forces B, and B;, at B and to the forces C, and
C,at C. Note that B, and B; acting on AC are numerically equal but oppositely
directed to B, and B;, acting on BD. This is due to the fact that action and re-
action forces are equal but oppositely directed. Apply this principle carefully when
setting up the free-body diagrams.

A,=240 lb
(a) (b)
Fic. 4-21. — Free-body diagrams.

In a similar fashion the FBD of CE shown in part (c) is subjected to the re-
action H, = 360 lb and the forces Cy, Cr, D», and D;. Note that the forces C,
and C;, acting on CH are numerically equal but opposite in sense to the reactive
forces acting at C of member AC. Similarly, D, and D;, acting on CE are numerically
equal but opposite in sense to the reactive forces acting at D of member BD.
Each of the free-body diagrams represents a non-concurrent force system in
equilibrium. Hence three equations of equilibrium may be written for each of the
Art. 4-7] Frames Containing Three-Force Members 107
free-body diagrams. For members AC and BD, there is a total of six unknown
quantities for which six independent equations may be written. As a general rule,
it rs best to solve first for the unknown quantities common to both free-body diagrams,
in this case for B, and B;. Referring to BD (Fig. 4-21b), a moment summation
about D determines B,.
[ELM p = 0] 12 B, — 600 X 4=0 » = 200 lb Ans.
Inserting the known value of B, on the FBD of AC, amoment summation about C
of Fig. 4-21a determines B,.
[MIM = 0] 6 B, + 200 X 6 — 240 x 10 =0 Brn
= 200\lb Ans.
For each of the members BD and AC, there remain two equations of equilibrium
which may now be applied to determine the remaining unknowns. Thus for BD we
have
[2X — 10}; Dy, — B,, = (0) Dj, = Bi, == 2()0 Ib Ans.

[ZY = 0] D, + 200 — 600 = 0 D, = 400 lb Ans.

To the FBD of AC we apply


[2X = 0] B, —C, =0 C, = By, = 200 lb Ans.
[ZY = 0] C, — 200 + 240 = 0 C, = —40lb Ans.
The three equations of equilibrium that can be written for member CEH have not
been used so far. They may now be employed to check the above values. Thus
[2X = 0] (on = Dy; — 0 Gr = Dy, = 200. lb Check

Ee ya— QO] 360 — 400 — C, = 0 C, = —40 lb Check


Note that the minus sign of C,, indicates that the direction of C, is actually opposite
to that shown in both Figs. 4-21a and 4—21c.
It should be observed that any combination of two of the free-body diagrams of
AC, CE, or BD will yield six equations to be solved for the six unknowns. The third
free-body diagram may then be used for check purposes.
If desired, the resultant value of the hinge forces may be found from their com-
ponents, as follows:

[c = VC? + G | C =~+/ (40)? + (200)? C = 204lb Ans.

E =/BP+ B | B = /(200)? + (200) B = 2831b Ans.


[ =/D? + p:| D =~/(400)? + (200)? D = 4471b Ans.

PROBLEMS
440. For the frame loaded as shown in Fig. P-440, determine the horizontal and
vertical components of the pin pressure at B. Specify directions (up or down; left
or right) of the force as it acts upon member CD.
Ans. B, = 550 lb down; By, = 450 lb left
108 ANALYSIS OF STRUCTURES [Chap. IV

Fic. P-441.

441. The structure shown in Fig. P-441 is hinged at A and C. Find the hori-
zontal and vertical components of the hinge force at B, C, and A.
Ans. B, = 100 lb; B, = 1751b
442. Each member of the frame shown in Fig. P-348 on page 76 weighs 50 lb
per ft. Compute the horizontal and vertical components of the pin pressures at
CAD wander
443. The frame shown in Fig. P-443 is hinged to rigid supports at A and £.
Find the components of the hinge forces at A and H and the forces in members
BC and BD.
Ans. A, = EH, = 60 1b; A, = E, = 120 lb; BC = 100 lb T; BD = 2001bC

120 lb

Fia. P-443. Fic. P-444.


Art. 4-7] Frames Containing Three-Force Members 109

444. The frame shown in Fig. P-444 is sup-


ported by a hinge at A and aroller at EH. Com-
pute the horizontal and vertical components of
the hinge forces at B and C as they act upon
member AC.
Ans. By = 120 Ib up; B, = 40 Ib right;
C, = 96 lb down; C;, = 40 lb left
445. The frame shown in Fig. P-445 is sup-
ported by a hinge at # and a roller at D. Com-
pute the horizontal and vertical components
of the hinge force at C as it acts upon BD.
Ans. Cy, = 70 lb up; Ch = 280 lb right a %

jens
446. A three-hinged arch is composed of
8! =|
two trusses hinged together at D in Fig. P-446.
Fra. P-445.
Compute the components of the reaction at A
and then find the forces acting in bars AB and AC. Hint: First isolate each truss
as a free body.
Ans. A, = 420 lb; A, = 320 lb; AB = 700 lb C; AC = 240 lb T

240 lb 720|b

Fic. P-447.
110 ANALYSIS OF STRUCTURES [Chap. IV

447. Two trusses are joined as shown in Fig. P-447 to form a three-hinged arch,
Compute the horizontal and vertical components of the hinge force at B and then
determine the type and magnitude of force in bars BD and BE.
400 lb

Fig. P-449.

448. A beam carrying the loads


shown in Fig. P-448 is composed of
three segments. It is supported by
four vertical reactions and joined by
two frictionless hinges. Determine
the values of the reactions.
Ans, R, = 200 lb; R, = 2120 Ib;

Ib
ft
800
per 449. The bridge shown in Fig.
P-449 consists of two end sections,
each weighing 200 tons with center of
gravity at G, hinged to a uniform cen-
ter span weighing 120 tons. Compute
the reactions at A, B, E, and F.
450. A billboard BC weighing 1000
lb is subjected to a wind pressure of
300 lb per ft as shown in Fig. P-450.
Fic. P-450, Neglecting the weights of the support-
Art. 4-7] Frames Containing Three-Force Members 111

240 1b

bv
>——

Fig. P-451.
ing members, determine the components
of the hinge forces at A and F.
Ans Ay = 2250 Ibs Ay. = 4500) Ibs
» = 3250 lb; F, = 1500 lb
451. The frame shown in Fig. P-451
is hinged at # and roller supported at A.
Determine the horizontal and vertical
components of the hinge forces at B, C,
and D. Neglect the weights of the mem-
bers.
452. For the frame shown in Fig.
P-452, determine the horizontal and ver- Fia. P-452.

2400 Ib
Fig. P-453.
112 ANALYSIS OF STRUCTURES [Chap. IV

tical components of the hinge force at


B as it acts upon member AC.

453. For the frame shown in Fig.


P-453, determine the resultant hinge
forces at B, C, and EL.
Ans. B = 2530 iby C y= 800Ub;
E = 5060 lb

454. Determine the horizontal and


vertical components of the hinge force
at A for the structure shown in Fig.
P-454. Neglect the weights of the
members and of the pulleys.
Fic. P-454.

SUMMARY
Structures are composed of members which are commonly divided into
two groups: those subjected to axial tensile or compressive loads only and
those subjected to bending. A member of the first type is called a two-force
member; a member of the second type is called a beam or a three-force
member.
In two-force members (Art. 4-2), the forces are usually applied at the
ends of the member and tend either to lengthen or to shorten it, thereby
producing respectively tensile or compressive forces in the member. The
action of a tensile member on its end pins is to pull away from its end
pins; the effect of a member subjected to compression is to push back
against its end pins.
The forces in truss members are determined on the assumption that the
members are two-force members. Hence the forces acting on any pin form
a concurrent system. The conditions of equilibrium for concurrent force
systems form the basis of the method of joints (Art. 4-3). If a cutting
section is assumed to be passed through three members of a truss and the
cut members replaced by the internal loads being transmitted by the cut
members, a free-body diagram of part of the truss is obtained which is
acted upon by three unknown forces. These three forces are determined
by applying the conditions of equilibrium of non-concurrent forces; the
technique is known as the method of sections (Art. 4-4).
Structures containing three-foree members must be analyzed differently.
A three-force member (Art. 4-6) is defined as one which supports three or
more forces. The characteristic action in a three-force member is bending;
Summary 113

this means that the internal force in the member is not directed along its
axis. Therefore, because the direction of the internal force is unknown,
neither the method of joints nor the method of sections can be used.
At this stage in the study of mechanics, all that can be done is to deter-
mine the forces acting at the pins joining the three-foree members. The
procedure is to isolate each three-foree member by means of free-body
diagrams. On these diagrams the forces at the pins are resolved into
rectangular components. When drawing the free-body diagrams of mem-
bers which act mutually upon each other, these components should be
shown as equal but oppositely directed on each free-body diagram. The
simultaneous application of the conditions of equilibrium for non-con-
current forces to each free-body diagram will determine the unknown forces
at the pins (refer to Art. 4-7).
Chapter V.
Friction

5-1. Friction Defined


Friction may be defined as the contact resistance exerted by one body
upon a second body when the second body moves or tends to move past the
first body. From this definition, it should be observed that friction is a
retarding force always acting opposite to the motion or the tendency to
move. As we shall see, friction exists primarily because of the roughness of
the contact surfaces. If the contact surfaces are perfectly smooth, as as-
sumed in some earlier problems, friction can be neglected.
In machines, friction is both a liability and an asset. Where it causes
loss of power it is undesirable; but in certain types of friction drives or in
brakes it is very desirable. In this chapter we shall consider the application
of the principles of friction to engineering problems.

5-2. Theory of Friction


The following experiment is useful in discussing the principles of friction
as applied to dry unlubricated surfaces. Let a block of weight W rest on a
rough horizontal surface. Assume a horizontal force P to be applied to the
block, as shown in Fig. 5-1. When P is zero, the frictional resistance is
zero. When P is given increasing values,
Ww the frictional resistance also increases;
values are indicated on the graph in Fig.
5-2. This graph is not rigorously accu-
rate; it is intended only as a guide for
P interpreting results.
The graph indicates that up to impend-
ar ing motion, the frictional resistance F' is
numerically equal to the applied load P.
N After motion occurs, the frictional resis-
Fic. 5-1.— Frictional resistance. tance decreases rapidly to a kinetic value
which remains fairly constant.
From such experimental results it is observed that as long as the block
remains at rest the frictional resistance must equal the resultant force
tending to cause motion. This is true up to the instant at which the fric-
tional resistance can no longer balance the resultant of applied forces.
114
Art. 5-2] Theory of Friction 115
ee a ee ee

Max. available friction,


Motion impending
ok.
Kinetic friction,
Motion occuring

Friction
(F) Static friction,
No motion

Applied Force (P)


Fic. 5-2. — Variation of frictional resistance.

When this condition exists, motion occurs. Once motion takes place, the
frictional resistance drops to a value below that acting when motion starts.
One way of understanding these results is to examine a magnified view
of the contact surfaces. These are shown in Fig. 5-3. The surfaces are
assumed to be composed of irregular-
ities (which can be likened to hills
and vales) which mesh together. The
frictional resistance is developed by
the effort of P to pull the hills of the
block out of the meshing valleys of the
plane surface. In an ideal smooth
surface there are no hills and vales to
mesh together, and no frictional resis-
tance is possible. This is why we have Tes 2 Mitel vie on erntact
assumed the reaction to be normal surfaces.
to the surface when discussing ideal
smooth surfaces in preceding articles. Actually an ideal smooth surface
does not exist; it may be approximated by superfinishing ground surfaces.!
It is apparent that frictional resistance depends upon the degree of
wedging action between the hills and vales of the contact surfaces. The
measure of this wedging action depends upon the normal pressure N be-
tween the surfaces. As a result, the maximum frictional resistance is said
to be proportional to the normal pressure and is expressed symbolically as

FaN (a)
where the sign « is read as “‘is proportional to.”’ This may be reduced to
an equation by putting in a constant of proportionality, say f, which de-
1In some cases, such as carefully finished flat surfaces of gage blocks, the surfaces
adhere because of molecular adhesion. Where adhesion exists, it will be assumed to be
part of the frictional resistance and its effect to be included in the coefficient of friction.
116 FRICTION [Chap. V

pends upon the roughness of the contact surfaces. This constant is called
the coefficient of friction, and Eq. (a) may be rewritten as?
F =fN (5-1)
If the applied force exceeds the maximum frictional resistance, motion of
the block ensues. In motion there is less chante for the wedging action
described above to take place; i.e., the hills and vales are not as free to
mesh as when the block was at rest. This is the reason for a decrease in
frictional resistance when motion occurs.
In the case of moving bodies, the resistance is defined as the kinetic
frictional resistance. It always acts at maximum value. Note the contrast
with static friction, which adjusts itself to the value required to resist mo-
tion. (Refer to the graph discussed above.) The magnitude of the kinetic
friction is also expressed by Hq. (5-1) by inserting the coefficient for kinetic
friction f;, in place of the coefficient for static friction f;. In many instances
the subscripts & and s are omitted when the statement of the data makes
the meaning clear.
When motion first begins, the hills and vales of the contact surfaces mesh
with less frequency, causing the static friction to decrease gradually to the
kinetic friction value. For high speeds, the kinetic friction decreases still
more. When a body in motion is being brought to rest, the converse is also
true; the kinetic friction gradually increases at very low speeds up to a
maximum value equal to that of the static friction as the body finally
comes to rest. It is usually assumed, however, that the kinetic friction is
constant in value.
The foregoing analysis applies only to friction between dry unlubricated
surfaces. Ifa lubricant such as oil is used, the hills and vales of the contact
surfaces are separated by the lubricant, thereby materially reducing the
frictional resistance. Indeed, if the thickness of the lubricating film is
sufficient to separate the contact surfaces completely, the only frictional
resistance remaining will be the internal friction in the lubricant itself,
measured by its viscosity. The viscosity varies with the temperature, being
less at high than at low temperatures. For an extended discussion of the
theory of lubricated surfaces, which is beyond the scope of this book, the
reader is referred to a standard work upon this subject.

5-3. Angle of Friction


Figure 5-1 is redrawn in Fig. 5-4 to show that F and WN are really com-
ponents of the total reaction R exerted by the plane surface against the
* In using this relation, 7 is the maximum available frictional resistance. Refer to
Art. 5-3. Also note that Eq. (5-1) cannot be used to determine F unless motion is
impending. See Illus. Prob. 501.
Art. 5-3] Angle of Friction ule

block. The size of the angle between Ww


Rand N depends on the value of the
frictional resistance F. If F is zero,
this angle will be zero. As F increases, P
so does the angle. The particular
value of this angle when maximum
frictional resistance is acting is defined
as the angle of friction. It acts at its
maximum value of ¢ only when motion NT
as tmpending.
From Fig. 5-4, it is apparent that
Fic. 5-4. — Angle of friction.
the angle of friction may be defined by
the relation

tan @ = = (b)

Comparing Eq. (b) with Eq. (5-1), we see that

tang =f (5-2)
In other words, the tangent of the angle of friction is the coefficient of
friction. Hence, determining the angle of friction affords a means of ob-
taining the coefficient of friction.
A simple experiment for determining
the angle of friction is to place a block
of weight W upon an inclined plane for
‘ which the angle of inclination @ can be
gradually increased from zero to a maxi-
mum value at which the block is on the
verge of sliding down the incline. Figure
5-5 shows this condition. In order for
equilibrium to exist, it is necessary that
the weight W and the reaction R be
collinear (see Axiom 2). From the figure
Fic. 5-5.— At impending motion, it is evident that, when motion impends,
eozle ot pec ae angle of In- the angle of inclination 6 is equal to the
angle of friction @.
When considering coplanar forces, in order to have no motion the static
reaction must lie within the angle ABC in Fig. 5-6 (shown on the next
page), whereas for problems involving non-coplanar forces, it must be con-
tained within the cone generated by revolving line AB about the normal
BN. The cone so formed is called the cone of friction.
118 FRICTION [Chap. V

5-4. Laws of Friction


A summary of the principles discussed
in the preceding articles may be called the
laws of friction and may be stated as
follows:
1. If friction is neglected, the reactions
are always normal to the surfaces in con-
tact.
Fia. 5-6. — Cone of friction. 2. Friction always acts to oppose the
motion of the free body (or its tendency
to move). It is tangent to the surfaces in contact.
3. If static friction is acting, the value of the friction force may vary
from zero to the maximum available value, depending upon the resultant
force tending to cause motion.
4. The maximum available value of static friction (i.e., the limiting
friction when motion impends) is equal to f;N where f; is the coefficient
of static friction and N is the normal pressure.
5. If kinetic friction is acting, the friction force is constant at its limiting
value. (Actually, kinetic friction decreases somewhat at high velocities
and increases at very low speeds; see Art. 5-2.)
6. The kinetic friction is equal to f,N where f;, is the coefficient of kinetic
friction and N is the normal pressure.
7. The angle between the total reaction and its normal component, when
limiting friction is acting, is called the angle of friction. The tangent of
this angle is the coefficient of friction.

ILLUSTRATIVE PROBLEMS
501. A 200-lb block is in contact with a plane inclined at 30° to the horizontal.
A force P, parallel to and acting up the plane, is applied to the body. If the coeffi-
cient of static friction is 0.20, (a) find the value of P to just cause motion to impend
up the plane, and (b) find P to just prevent motion down the plane. (c) If P = 80
lb, determine the magnitude and direction of the friction force.
Solution: Part a. If the block has impending motion up the plane, the friction
force, acting at its limiting value, will act down the plane to resist motion. The
FBD of the block is shown in Fig. 5-7. Reference axes are selected so that the
X axis is parallel to and positive in the direction of impending motion.
To find NV, we apply the equation
[DY =.0] N — 200 cos 30° = 0 N = 173.2 lb
Since limiting friction acts, we may use the equation
[= FN] F = 0.2 X 173.2 = 34.641b
* These principles are based upon Coulomb’s experiments in 1781 on the friction of
plane dry surfaces and those performed by Morin in 1831.
Art. 5-4] Laws of Friction 119
i
To determine P, we apply the equation
[2X = 0] P — F — 200 sin 30° = 0
P = 34.64 + 100 P = 134.64 lb Ans.
Part b. When the block has impending motion down the plane, the friction force,
acting at its limiting value, will act up the plane to resist the motion. The FBD
is shown in Fig. 5-8. Note that the X axis is chosen parallel to and positive in the
200 lb 200 Ib

Fic. 5-7. — Impending motion up the Fic. 5-8. — Impending motion down
plane. the plane.

direction of impending motion. WN and the limiting value of F are the same as in
Part a. Hence to find P we need only apply the equation
[=X = 0] 200 sin 30° - P— F = 0
P = 100 — 34.64 P = 65.36 lb Ans.
Part c. The results obtained for Parts a and 6 indicate that the 200-lb block will
remain at rest for all values of P between 65.36 lb and 134.64 lb, i.e., for P between
200 sin 30° + F = 100 + 34.64. Hence when P = 80 lb, the block will tend to
slide down the plane, although limiting friction will not be developed. Since
equilibrium is present, from Fig. 5-8 we have
x — 0] 200 sin 30° —-P—F=0
F = 100 — 80 i= 20 bb Anis:
This result can be reached by another method of reasoning. Temporarily neglect-
ing friction, a summation of forces parallel to the plane indicates that the downplane
component of the block (i.e., 200 sin 30° = 100 Ib) could not be balanced by an up-
plane force P of 80 lb. For balance, a frictional resistance of F = 20 lb would be
required acting upplane as shown in Fig. 5-8. Since the available static friction is
34.64 lb, we conclude that limiting friction will not be developed.
502. A 200-lb block is at rest on a 30° incline. The coefficient of friction between
the block and the incline is 0.20. Compute the value of a horizontal force P that
will cause motion to impend up the incline.
Solution: The FBD of the block is shown in Fig. 5-9a. Since motion is impend-
ing up the incline, the maximum static friction F is directed down the incline. A
point diagram of the forces is formed by first selecting X and Y axes with the
120 FRICTION [Chap. V

X axis parallel to and positive in the direction of impending motion, and then
imagining the block squeezed to a point coincident with the origin of the axes. The
forces on the body are then applied to this point to form the concurrent system
shown in Fig. 5-9b. (Note: The point diagram is sometimes more convenient than
the FBD for computing components.)
200 Ib Y
a

30°
PB

F 30°
200 Ib
(a) (b)
Fic. 5-9.

The three unknowns JN, F, and P are found from Eq. (5-1) and the two equations
of equilibrium for concurrent forces. We now have
yes) N — 200 cos 30° — P sin 30° = 0
N = 173.2 +0.5P (a)
[Fi= jfN] Fi= 0:20173.2 > 0.57)
F = 34.64+0.1P (b)
[2X = 0] P cos 30° — 200 sin 30° — F = 0
Substituting the value of F from (6) we obtain
P= 16 lb Ans:
503. Resolve Prob. 502, using the angle of friction ¢@ and the total reaction of
the incline on the block instead of its components F and N.
200 1b

1b
200

(a) (b) (c)


Fie. 5-10.
Solution: Whenever the normal pressure N must be expressed in terms of an un-
known force, such as P in the preceding example, it will generally be simpler to use
the total reaction R instead of its components F and N.
Art. 5-4] Laws of Friction 121

Since motion is impending, F will make the angle ¢ with N as shown in Fig. 5-10a.
The value of ¢ is found from Eq. (5-2) to be

[tan ¢ = f] tan @ = 0.20 Orneon

The block is subjected to three forces in equilibrium. The point diagram of the
forces acting on the block is shown in Fig. 5-10b. This system will be recognized
as a concurrent force system in equilibrium and may be solved by the method
developed in Art. 3-3.
If the X axis is taken through R, a Y summation (Y axis not shown) will de-
termine P at once by eliminating R. We thereby obtain
[ZY = 0] P sin 48.7° — 200 sin 41.3° = 0 Peal OnlommeATes:
A preferred variation of this solution when only three forces are involved consists
of applying the sine law to the force polygon shown in Fig. 5-10c. Since equilibrium
exists, the force polygon will close. The 200-lb weight is represented by the vertical
vector shown. Through the tip of this vector, a horizontal line of indeterminate
length is drawn to represent the known direction of P. From Fig. 5-10a, the known
direction of R is 30° + @ = 41.3° with the vertical. A line representing R may be
drawn through the tail of the 200-lb vector to intersect P as shown.
Values may now be obtained graphically by scaling from the polygon, or analyt-
ically by applying the sine law. Using the latter, we have

1 200
sin 41.3° sin 48.7°
whence as before
Pe V76lbr Ans:

504. Determine the minimum value and the direction of a force P required to
cause motion of a 200-lb block to impend up a 30° incline. The coefficient of friction
is 0.20.

1b
200

(b)
Fic. 5-11.

Solution: The FBD of the block is shown in Fig. 5-11a. Since motion is impend-
ing up the incline, the frictional resistance is directed down the incline. The reac-
122 FRICTION [Chap. V

tion of the plane against the block is therefore at an angle ¢ with N as shown. The
value of ¢ is found from
[tan @ = f] tan @ = 0.20 a= Whe
and R is therefore inclined at 41.3° with the vertical.
If the polygon of forces is plotted as shown in Fig. 5-11b, the vector representing
R may be laid off through the tail of the 200-Ib weight. The vector representing P is
now drawn through the tip of the 200-Ib vector perpendicular to R. This obviously
will give the minimum length of P (and hence the minimum value) to intersect R.
From Fig. 5-11b, we now have
P= 200 sin 41.32 = 132 lb Ans:
The inclination of P with the horizontal is also 41.3°, whence the angle a is found
to be
41.3° = a+ 30° a= 1137 Ans:
Observe that this value of a is equal to the angle of friction @¢.

PROBLEMS

505. A block weighing W lb is placed upon a plane inclined at an angle @ with


the horizontal. Discuss what will happen if the angle of friction ¢ is (a) greater than
6, (b) equal to @, (c) less than 6.
506. A 400-lb block is resting on a rough horizontal surface for which the co-
efficient of friction is 0.40. Determine the force P required to cause motion to im-
pend if applied to the block (a) horizontally or (b) downward at 30° with the hori-
zontal. (c) What minimum force is required to start motion?
Ans. (a) 160 lb; (b) 240 Ib; (ce) 148.7 lb
507. The 500-lb block shown in Fig. P-507 is in contact with a 45° incline. The
coefficient of static friction is 0.25. Compute the value of the horizontal force P
necessary to (a) just start the block up the incline or (b) just prevent motion down
the incline. (c) If P = 400 lb, what is the amount and direction of the friction
force?

P , 400 Ib BD
°
45 aye

Fie. P-507. Fic. P-508.

508. The 200-lb block shown in Fig. P-508 has impending motion up the plane
caused by the horizontal force of 400 lb. Determine the coefficient of static friction
between the contact surfaces. Ans. f =.0.66
Art. 5-4] Laws of Friction 123

509. The blocks shown in Fig.


P-509 are connected by flexible, in-
extensible cords passing over friction-
less pulleys. At A the coefficients of
friction are fs = 0.30 and f, = 0.20
while at B they aref, = 0.40 andf, =
0.30. Compute the magnitude and di-
rection of the friction force acting on
each block.
ATs A — 45 b> Fs — 36 Ib
510. What weight W is necessary
to start the system of blocks shown
in Fig. P-510 moving to the right?
Fia. P-509.
The coefficient of friction is 0.10 and
the pulleys are assumed to be friction-
less. Ans. W = 295 lb

Fic. P-510.

511. Find the least value of P required to cause the system of blocks shown in
Fig. P-511 to have impending motion to the left. The coefficient of friction under
each block is 0.20. thas, IP = 2b ilee ey SS hla”
512. A homogeneous block of weight W rests upon the incline shown in Fig.
P-512. If the coefficient of friction is 0.30, determine the greatest height A at which
a force P parallel to the incline may be ap-
plied so that the block will slide up the in- tbs
cline without tipping over. 3 :
Anis Wp — 4,7 On. se)

513. In Fig. P-512, the homogeneous


block weighs 300 lb and the coefficient of

Fie. P-511. . Fra. P-512 and P-513,


124 FRICTION [Chap. V

friction is 0.40. If h = 5 in., determine the force P to cause motion to impend.


514. The 100-lb cylinder shown in Fig. P-514 is held at rest on the 30° incline by
a weight P suspended from a cord wrapped around the cylinder. If slipping im-
pends, determine P and the coefficient of friction.

Wy@ =100 Ib :

of
Fig. P-514. Fig. P-515 and P-516.

515. Block A in Fig. P-515 weighs 120 lb, block B weighs 200 lb, and the cord is
parallel to the incline. If the coefficient of friction for all surfaces in contact is 0.25,
determine the angle @ of the incline at which motion of B impends.
516. Referring to Prob. 515, if the coefficient of friction is 0.60 and 6 = 30°,
what force P applied to B acting down and parallel to the incline will start motion?
What is the tension in the cord attached to A?
AWS, (PS WAG los 2 = 1222! ihe

5-5. Further Problems in Friction


The problems in the preceding article were concerned, for the most part,
with only one body and were introductory in character. In this article we
shall discuss further applications of the principles of friction. Although
these problems are slightly more advanced, we shall see that no further
principles of friction are required; all that is necessary is the development
of the proper technique to solve the problems with the minimum of effort.
In some of the problems, it will be simpler to use the frictional and normal
components of a surface reaction; in others we should use the total surface
reaction. A general rule is this: when the normal pressure can be computed
directly in terms of known forces, use the frictional and normal components
of the surface reaction, but for other cases, use the total surface reaction.
Further, when only three forces act on a free body, usually apply the sine
law to the triangle of forces. When more than three forces are involved, of
which only two are unknown, it is suggested that force summations be
taken with respect to perpendicular axes one of which coincides with one
of the unknown forces. Finally for non-concurrent systems, always con-
Art. 5-5] Further Problems in Friction 125
sider the possibility of eliminating two of the unknown forces by taking a
moment summation about the intersection of their lines of action, or if this
is not convenient, consider force summations that will involve one, or at
most two, of the unknown forces. By applying these suggestions, it will
usually be possible to determine each unknown quantity independently of
the others.

ILLUSTRATIVE PROBLEMS
517. The blocks shown in Fig. 5-12 are separated by a solid strut which is at-
tached to the blocks with frictionless pins. If the coefficient of friction for all sur-
faces is 0.20, determine the value of the horizontal force P to cause motion to im-
pend to the right.

Fic. 5-12.

Solution: First draw the FBD of each block. Observe that the strut exerts an
equal but oppositely directed force C on each block. Since motion is impending,
the friction forces may be computed from the relation F = fN. In this problem,
however, it is best to use the technique of the total reaction at the contact surface
(described in Prob. 503) for the 200-lb block and to use the frictional and normal
components of the reaction (described in Prob. 502) for the 400-lb block.
We start with the 200-lb block on which the unknown forces are R; and C. When
force C is determined, the unknown forces on the 400-lb block will likewise reduce
to only two, viz., P and Rz. In Fig. 5-13b, the reaction FR, is inclined at the angle
of friction ¢ with the normal pressure. The value of ¢ is determined from Eq. (5-2).
Thus we obtain
[tan ¢ = f] tan @ = 0.20 w= dbl as
Since the three forces acting on the 200-lb block are in equilibrium, they form the
force triangle shown in Fig. 5-13c. Applying the sine law, we find the value of C
to be
C oa
or C = 186lb
sin 56.3° sin 63.7°
Consider now the 400-lb block (Fig. 5-13a) on which, since Cis now known, there
are only the unknown forces P and R. In this instance, it is easier to replace Ry
126 FRICTION [Chap. V

Fic. 5-138.

(not shown) by its components F, and N». The reason for this change in technique
is that N» can be expressed directly in terms of known forces. Thus we obtain

[ZY = 0] Nz — 400 — 186 sin 30° = 0 N = 493 lb


Since motion impends, the friction force is found from the relation

[F = fN] F = 0.20 X 4938 F = 98.6 lb


Finally P is determined from the condition

[2X = 0] P — F, — 186 cos 30° = 0


P = 98.6 + 161 = 259.6 lb Ans.
518. A plank 20 ft long is placed on rough inclined planes at an angle of 15° with
the horizontal as shown in Fig. 5-14a. The angle of friction is ¢ = 15° at both in-
clines. If the weight of the plank is W = 200 lb concentrated at 10 ft from end A,
compute the minimum distance x from B at which a load P = 100 lb can be placed
before slipping impends.
Solution: When P is placed too close to B, this end tends to slip down and end A
toslip up. To resist the impending motion, the reactions at A and B are inclined at
the angle of friction @ with their normal pressures so that their components along
the inclines oppose the tendency to slip.
The distance x can be determined by equating a moment summation about B to
Art. 5-5] Further Problems in Friction 127

W= 200 1b
P=1001b

1b
=300

Wee

(a)
Fig. 5-14.

zero. Since this summation involves the value of R4, we start by applying the sine
law to the polygon of forces shown in Fig. 5-14b. This gives
Ra 300
sin 30° sin 105° o Ra = 155 lb

In applying a moment summation about B, it is convenient to express the mo-


ment of R,4 in terms of components acting through A which are parallel and per-
pendicular to AB. Noting that R, is inclined at 60° with AB, we then obtain
[>Mp = 0) (R4 sin 60°)(20) — 200(10 cos 15°) — 100(a# cos 15°) = 0
whence substituting the value R4 = 155 lb, we find
e= 7.8 tt Ans:

PROBLEMS
519. In Fig. P-519, two blocks are connected by a solid strut attached to each
block with frictionless pins. If the coefficient of friction under each block is 0.25
and B weighs 270 lb, find the minimum weight of A to prevent motion.
Ans. Wa = 600 lb

ica font
Fic. P-519, P-520, and P-521.
128 FRICTION [Chap. V

520. Referring to Fig. P-519, block A weighs 400 lb and B weighs 300 lb. If
f = 0.20 under B, compute the minimum coefficient of friction under A to prevent
motion.
521. In Fig. P-519, if f= 0.30 under both blocks and A weighs 400 Ib, find the
maximum weight of B that can be started up the incline by applying to A a right-
ward horizontal force P of 500 lb. Ans. We = 263 lb
522. Repeat Illus. Prob. 517, assuming that the strut is a uniform rod weighing
300 lb. Hint: First isolate the strut as a free body, resolving its end forces into com-
ponents acting along and perpendicular to the strut. Ans. P = 424 |b
523. A force of 400 lb is applied to the pulley shown in Fig. P-523. The pulley
is prevented from rotating by a force P applied to the end of the brake lever. If
the coefficient of friction at the brake surface is 0.20, determine the value of P.
Ans, P= 300 lb

Erg: P=523) Fig. P-524.

524. A horizontal arm having a bushing 2 in. long is slipped over a 2-in. diameter
vertical rod, as shown in Fig. P-524. The coefficient of friction between the bushing
and the rod is 0.20. Compute the minimum length ZL at which a weight W can
be placed to prevent the arm from slipping down the rod. Neglect the weight of the
arm. As, dy = Ham.
525. A uniform ladder 16 ft long and weighing W lb is placed with one end on the
ground and the other against a vertical wall. The angle of friction at all contact
surfaces is 20°. Find the minimum value of the angle @ at which the ladder can be
inclined with the horizontal before slipping occurs. Aas, 0 = iP
526. A ladder 20 ft long weighs 40 lb and its center of gravity is 8 ft from the
bottom. The ladder is placed against a vertical wall so that it makes an angle of 60°
with the ground. How far up the ladder can a 160-lb man climb before the ladder is
on the verge of slipping? The angle of friction at all contact surfaces is 15°.
Art. 5-5] Further Problems in Friction 129

527. A homogeneous cylinder 3 ft in diameter and weighing 300 lb is resting on


two inclined planes as shown in Fig. P-527. If the angle of friction is 15° for all con-
tact surfaces, compute the magnitude of the couple required to start the cylinder
rotating counterclockwise. ENS, (Cl = 8) Most
528. Instead of a couple, determine the minimum horizontal force P applied
tangentially to the left at the top of the cylinder described in Prob. 527 to start the
cylinder rotating counterclockwise.

60°

Fic. P-527 and P-528. Fic. P-529.

529. Asshown in Fig. P-529, a homogeneous cylinder 2 ft in diameter and weigh-


ing 120 lb is acted upon by a vertical force P. Determine the magnitude of P
necessary to start the cylinder turning. Assume that f = 0.30.
530. A plank 10 ft long is placed in a horizontal position with its ends resting
on two inclined planes, as shown in Fig. P-530. The angle of friction is 20°. Deter-
mine how close the load P can be placed to each end before slipping impends.
Ans. 0.76 ft; 2.81 ft

Fic. P-530. Fig. P-531.

531. A uniform plank of weight W and total length 2 Z is placed as shown


in Fig. P-531 with its ends in contact with the inclined planes. The angle of friction
is 15°. Determine the maximum value of the angle a at which slipping impends.
Ans. a = 36.2°

532. In Fig. P-532, two blocks each weighing 150 lb are connected by a uniform
horizontal bar which weighs 100 lb. If the angle of friction is 15° under each block,
130 FRICTION [Chap. V

find P directed parallel to the 45° incline that will cause impending motion to the
left. Ans. P = 75.8 lb

of See
Fig. P-532. Fic. P-533.

533. A uniform bar AB, weighing 424 lb, is fastened by a frictionless pin to a
block weighing 200 Ib as shown in Fig. P-533. At the vertical wall, f = 0.268 while
under the block, f = 0.20. Determine the force P needed to start motion to the
right. Ans. P = 430 lb

5-6. Wedges
The principles involved in dealing with wedges are no different from
those previously described. Actually, the wedge problem shown in Fig.
5-15 is very similar to Illus. Prob. 517 on page 125. The length of the strut
in that type of problem is here reduced to zero so that the blocks make
direct contact; also the shape of the blocks is changed so that they make
contact along a common surface. The contact reactions between the blocks
at this common surface are not only equal and oppositely directed on the
free-body diagram of each block; they also act so that their tangential or
frictional components along the common contact surface oppose the im-
pending motion of each block.

ILLUSTRATIVE PROBLEM
534. The block A in Fig. 5-15 supports a load W = 1000 lb and is to be raised
by forcing the wedge B under it. The angle of friction for all surfaces in contact
is 6 = 15°. Determine the force P which is necessary to start the wedge under
the block.
Solution: The original system of block and wedge is acted upon by the three
unknown forces P, Ri, and Ry. The reactions R; and Ry make the angle ¢ with their
normals and are directed as shown so as to oppose the motion. Since the position
of the forces is unknown and dimensions are unspecified, the equation of equilibrium,
2M = 0, cannot be applied. It will be necessary to consider the FBD of each body
shown in Fig. 5-16.
Art. 5-6] Wedges 131
W=1000 lb

Fic. 5-15. — Block raised by wedge.


W=10001b

(b)
Fic. 5-16. — Free-body diagrams of block and wedge.
132 FRICTION [Chap. V

Acting on the FBD of block A in Fig. 5-16a, in addition to W and Ri, is the
reaction R; exerted by the wedge B upon the block. These reaction forces are in-
clined to the normals of the contact surfaces so that their projections upon the
contact surfaces oppose the impending motion. R; is directed at an angle ¢ = 15°
with the normal or, as shown, at an angle of 35° with the vertical. Adding these
forces tip-to-tail gives the force polygon shown.
The FBD of wedge B in Fig. 5-16b, in addition toP and Rz, also has the force Rs
exerted upon it by the block A. Note that R; acting on the wedge is numerically
equal to Rs acting on the block. This follows from Axiom 4 relating to action and
reaction forces. The force polygon for the forces acting on the wedge is also shown.
By applying the sine law to the force polygon shown in part (a), we obtain
R3 1000
or R; = 1503 lb
sin 105° sin 40°
Using this value of R; in the force polygon of part (b), the sine law gives
P1508 or P= 1192lb Ans.
sin 50° sin 75°
If a graphical solution is desired, the above values could be scaled directly from
the force polygons.

PROBLEMS
535. A wedge is used to split logs. If dis the angle of friction between the wedge
and the log, determine the maximum angle a of the wedge so that it will remain em-
bedded in the log. Ans. a = 26
536. In Fig. P-536, determine the minimum weight of block B that will keep it at
rest while a force P starts block A up the inclined surface of B. The weight of A is
100 lb and the angle of friction for all surfaces in contact is 15°.

Fic. P-536. Fie. P-537 and P-538.

537. In Fig. P-537, determine the value of P just sufficient to start the 10°
wedge under the 400-Ib block. The angle of friction is 20° for all contact surfaces.
538. In Prob. 537, determine the value of P acting to the left that is required to
pull the wedge out from under the 400-Ib block. Ans. P = 203 lb
Art. 5-6] Wedges 133
539. If the wedge described in Illus. Prob. 534 had a weight of 400 Ib, what
value of P would be required (a) to start the wedge under the block and (b) to pull
the wedge out from under the block? Ans. (a) P = 1299 lbs (by) P9284 |b
540. As shown in Fig. P-540, two blocks, each weighing 200 lb and resting on a
horizontal surface, are to be pushed apart by a 30° wedge. The angle of friction is
15° for all contact surfaces. What value of P is required to start movement of the
blocks? How would this answer be changed if the weight of one of the blocks were
increased to 300 lb? Ans. P = 78.3 lb; no change

2000 1b

Fic. P-540. Fig. P-541.

541. Determine the force P required to start the wedge shown in Fig. P-541.
The angle of friction for all surfaces in contact is 15°. Ans. P = 944 lb
542. What force P must be applied to the
wedges shown in Fig. P-542 to start them under 2000 1b
the block? The angle of friction for all contact
surfaces is 10°.

W=1000Ib

Fic. P-542. Fic. P-543.

543. To adjust the vertical position of a column supporting a 2000-lb load,


two 5° wedges are used as shown in Fig. P-543. Determine the force P necessary to
start the wedges if the angle of friction at all surfaces is 25°. Neglect friction at the
rollers. Ans. P = 2085 lb
544. In Illus. Prob. 534, if the angle of friction is 10° at all surfaces in contact,
determine the maximum wedge angle a that will give the wedge a mechanical ad-
vantage; i.e., make P less than the weight W of the block. Ans. a = 25°
134 FRICTION [Chap. V

5-7. Square-Threaded Screws


A square-threaded screw is essentially an inclined plane wrapped around
a cylinder. Thus in Fig. 5-17 the inclined plane of height L and base b,
if wrapped around the cylinder, would create the helix shown. An actual
square-threaded screw results when the inclined, plane has a certain width
and constant thickness as it is wrapped around the cylinder. The result is
pictured in Fig. 5-18. As before, the height of the equivalent inclined plane
is the distance L, which is called the lead
of the screw. This is the distance that
a nut will advance along the screw in
one revolution. In the case of a single-
threaded screw, the lead is synonymous
with the pitch, which is the distance

3p

1 po be r—>

: Fic. 5-18. — Square-threaded


Fic. 5-17. — Helix formed by inclined plane. screw.

between similar points on adjacent threads. In a multiple-threaded


screw, the lead is mp, where m denotes the multiplicity of thread-

(a) (b) P
Fic. 5-19. — Forces on a square-threaded screw.
Art. 5-7] Square-Threaded Screws 135

ing. The base length of the equivalent inclined plane is taken as the
circumference of the mean radius of the thread and is expressed as b = 2 ar.
The mean radius is equal to one-half the sum of the outer radius and the
root radius of the thread. The pitch angle @ of the equivalent inclined plane
is determined from the relation tan 6 = 5 .
Tv

When the screw is used to lift a weight, as in a jackscrew, the weight may
be assumed to be concentrated on one small element of the thread, as shown
in Fig. 5-19. The force Q, acting in a plane perpendicular to the axis of the
thread and at the mean radius of the thread, is determined from the free-
body diagram of the weight shown on the equivalent inclined plane in
Fig. 5-20a.

Q
(b)
Fic. 5-20. — Motion impending up equivalent incline.

With motion impending up the incline, the value of Q is obtained from


the force triangle (Fig. 5—20b) to be
Q = W tan (¢ + @) (a)

With motion impending down the incline, the free-body diagram and the
force diagram are as shown in Fig. 5-21, from which we obtain
Q = W tan (¢ — @) (b)

(a) (b)
Fic. 5-21. — Motion impending down equivalent incline.
136 FRICTION [Chap. V
RON ES
It is evident that if the screw is to be self-locking, the angle of friction @
must be larger than the pitch angle 0.
The force P exerted at the end of a lever arm of length a (see Fig. 5-19) is
determined from the principle that the moment of P with respect to the
axis of the screw must equal the moment effect of Q. We obtain Pa =
Qr or, >

pH a
UE ret cu)
a
(5-3)

PROBLEMS
545. A single-threaded jackscrew has a pitch of 0.5 in. and a mean radius of
1.75in. The coefficient of static friction is 0.15, and of kinetic friction, 0.10. (a) De-
termine the force P applied at the end of a lever 2 ft long which will start lifting
a weight of 2 tons. (b) What value of P will keep the jackscrew turning?
Ans. (a) P = 57 1b; (b) P = 42.3 Ib
546. The distance between adjacent threads on a triple-threaded jackscrew is
2in. The mean radius is 2 in. The coefficient of friction is 0.10. What load can
be raised by exerting a moment of 2000 lb-ft?
547. As shown in Fig. P-547, a
square-threaded screw is used in a
vise to exert a pressure of 2 tons. If
the screw is double-threaded and has
a pitch of 0.25 in. and a mean radius
of 1.5 in., determine the torque that
must be applied at B to create this
pressure. Assume the coefficient of
Fig. P-547. — Vise. friction to be 0.15.
Ans. M = 102.5 lb-ft
548. <A single-threaded square screw has 23 threads per inch. The root diameter
is 2.6 in. and the outside diameter is 3 in. The coefficient of friction is 0.10. Deter-
mine the moment necessary to start lifting a vertical axial load of 40,000 lb. What
moment is necessary to start lowering the load?

5-8. Belt Friction


The transmission of power by means of belt or rope drives or the braking
of large loads by means of band brakes depends upon the frictional resist-
ance developed between the belt and the driving or resisting surface with
which it is in contact. If a driving pulley is perfectly smooth, no driving
torque is developed because no frictional resistance exists, and consequently
the tension throughout the belt will be constant and will have the same
value on both sides of the pulley. If the surface of the pulley is rough, how-
ever, the tension in the belt will vary throughout the length of contact, the
Art. 5-8] Belt Friction LGW,
an A a aol
difference in the belt tensions being caused by the frictional resistance.
In this article we shall discuss the limiting case of belt friction when slipping
impends.
Figure 5-22 represents a pulley of radius r driven in a clockwise direction
by means of the friction developed between its surface and the belt. The
belt is in contact with the pulley throughout an angle 6 determined by radii
drawn to the points of tangency of the belt. It is evident that the driving
torque is caused by the difference in tension between 7’, in the tight side

:
Fic. 5-22. — Belt friction; clockwise
V
Fic. 5-23. — Free-body diagram of an
rotation. element of the belt.

of the belt and T in the slack side. The difference in the tensions is gradu-
ally taken up in the friction between the belt and the surface of the pulley.
We shall examine this difference in tensions by considering a free-body
diagram of a small element of the belt located at an angle @ from the tangent
point of the slack side of the belt.
We now apply the conditions of equilibrium to Fig. 5-23. If the thickness
of the belt is neglected, a summation of moments about the center of the
pulley gives
[2M = 0] (T+ dT)r —Tr—r-dF=0
whence
dF = dT (a)
Projecting the forces upon the Y axis gives
. do
[zY = 0] aN — (T + dT) sin — T sin = 0
or

aK ar sin He aT sin; 5
6
138 FRICTION [Chap. V

whence, neglecting the product of differentials as being negligibly small and


noting that for smail angles the sine is practically equal to the angle ex-
pressed in radians, we have

dN =2T- 7 = Td (b)

When slipping impends, the friction relation gives


[F = fN] dF = fT do (c)
Eliminating dF between Eqs. (a) and (c) we obtain
dT =i (eae (d)
whence, separating the variables and integrating over the line of contact
of the belt, we have

whence i

log.7!= 18 (©)
or a
a = efB (5-4)

Note that the angle of contact 8 must be expressed in radians.


The above equation is easily solved by using common logarithms:

it
logo 7 = logio T: — logy T. = 0.434 fB (5-5)

ILLUSTRATIVE PROBLEM
549. A differential band brake is used to measure the torque output of an
engine. The dimensions are shown in Fig. 5-24. Determine the torque M on the
brake when P = 10 lb. Assume the coefficient of kinetic friction to be 0.20.
Solution: The tension 7; must be greater than 7. in order for the frictional
moment (7; — T2)r to resist M. From the FBD of the bell crank (Fig. 5-24b), a
summation of moments about the hinge A gives
[2M, = 0] 20 X 10+3% =2T, (a)
Another relation between 7; and 7, is found by applying Eq. (5-5). This gives

ip i
logio—T> = 0.434f6 logio—

= 0.434 X 0.2 X 37 = 0.409
from which

—*fh — 2.565 (b)


Art. 5-8] Belt Friction 139

(a) (b)
Fig. 5-24. — Differential band brake.

Solving Eqs. (a) and (6) simultaneously, we obtain


T, = 713 lb and T. = 278 lb
Then the torque exerted upon the brake is
[M = (T, — T;)r] M = (713 — 278) X 10 M = 4350 lb-in. Ans.

PROBLEMS
550. A rope making 1; turns around a stationary horizontal drum is used to
support a heavy weight. If the coefficient of friction is 0.4, what weight can be
supported by exerting a 50-lb force at the other end of the rope?
551. A rope wrapped twice around a post will support a weight of 4000 lb
when a force of 50 lb is exerted at the other end. Determine the coefficient of
friction. Ans. f = 0.349
552. A boat exerts a pull of 4000 lb on its hawser which is wrapped about a
capstan on the dock. If the coefficient of friction is 0.3, how many turns must the

Fic. P-553. Fig. P-554.


140 FRICTION [Chap. V
isle eee cence a ise et
hawser make around the capstan so that the pull at the other end does not exceed
50 Ib?
553. A torque of 240 lb-ft acts on the brake drum shown in Fig. P-553. If the
brake band is in contact with the brake drum through 250° and the coefficient of
friction is 0.3, determine the force P at the end of the brake lever.
Ans. P = 61.7 lb
554. In Fig. P-554, the coefficient
of friction is 0.20 between the rope and~
the fixed drum and between all surfaces
in contact. Determine the minimum
weight W to prevent downplane motion
of the 1000-lb body.
Ans. W = 253 lb
555. In Fig. P-555, a flexible belt
runs from A over the compound pulley
P, around the floating pulley B and
back over P to a 200-lb weight. The
coefficient of friction is L between the
belt and the compound pulley P. Find
the maximum weight W that can be
supported without rotating the pulley
Fic, P-555. P or slipping the belt on the pulley P.
Ans. W = 704 \b

5-9. Rolling Resistance

When wheels, cylinders, or similar objects roll without slipping along


flat surfaces, the frictional resistances are generally assumed to be static.
As is shown in Art. 13-6, the unbalanced moment causing motion is deter-
mined in these cases by taking a moment summation about the line of
contact with the ground.
For example, in Fig. 5-25, if the ground is rigid and unyielding, a moment
summation about C becomes zero because the weight W passes through C.
Thus, theoretically, a wheel once set in motion on a horizontal surface
would roll forever, because no retarding forces are set up. Actually we
know this to be false. The resistance to the motion of a wheel in such cases
is called rolling resistance. This resistance is the result of the wheel rolling
against a yielding surface, as in Fig. 5-26. A similar effect would be caused
when a yielding wheel rolls upon a rigid surface or, as is usually the case,
when both the wheel and the surface yield. The ground in front of the
wheel is depressed, causing the normal pressure N to act ahead of the line
of action of the weight W. When we take moments about the line of con-
tact C, we find that a resisting moment of magnitude Wb is created which
Art. 5-9] Rolling Resistance 141
a
Direction
of rolling

N
Fic. 5-25. — Wheel on a rigid surface. Fie. 5-26. — Wheel on a yielding surface.

resists the forward rolling of the wheel. The small distance b is known as
the coefficient of rolling resistance. Since the dimension of this coefficient
is expressed in inches, this coefficient is not synonymous with the coefficient
of friction, which was defined in Art. 5-2 as a ratio between the frictional
resistance and the normal pressure. Moreover, it differs from the usual.
definition of the coefficient of friction in that it is not constant. Although
the coefficient of rolling resistance is usually assumed to be constant, ex-
periment shows that it varies with the radius of the wheel. In the case of
ordinary steel railroad wheels on steel rails, it is about 0.02 in.

SUMMARY

Friction is defined as the contact resistance exerted by one body upon a


second body when the second body moves or tends to move past the first
body. In Art. 5-2, the relation between the frictional resistance F, the
coefficient of friction f, and the normal pressure N at the surface of contact
was expressed by
F =fN (5-1)

Note carefully that Eq. (5-1) determines only the maximum resistance
which can exist. When motion is not impending, the friction may have
any value from zero up to F, depending on the resultant force tending to
cause motion.
It is frequently desirable to consider the contact reaction R rather than
its components F and N (see Art. 5-3). When limiting friction is acting,
this contact reaction will be inclined at the angle of friction ¢ with the
normal pressure. The angle of friction is related to the coefficient of friction
by the equation
tan $ = f (5-2)
142 FRICTION [Chap. V

The torque required to turn a square-threaded screw (Art. 5-7) against a


load W is expressed in terms of the pitch angle 6, the angle of friction ¢,
and the mean radius r by the equation
M = Wrtan (¢ + 8) (5-3)
The plus sign is used when W resists the torque; the minus sign, when W
aids the torque.
The difference in belt tensions 7; and JT, caused by wrapping a belt
through a contact angle of 8 radians results in the following relation:

Toe
— = efB (5-4)
or
Ty
logio T. = 0.434 FB (5-5)
2

where f is the coefficient of friction (see Art. 5-8).


Chapter VI.
Force Systems in Space

6-1. Introduction
In the preceding chapters on coplanar force systems, we have seen how
two fundamental concepts, (1) that which relates a force to its components
and (2) the moment effect of a force, were applied. When we consider force
systems in space, these same basic concepts are all that are necessary, only
they must be extended to include the more general case of space forces.
We shall see that the magnitude and direction of the resultant is specified
by the summations of the components of the forces comprising the system.
The position of the resultant may be determined from the moment effect
of the system. However, we are not interested so much in the determi-
nation of the resultant as in the application of the conditions under which
the resultant is zero, i.e., when equilibrium exists.

6-2. The Three Mutually Perpendicular Components of a Force


Assume three concurrent forces F, F,, and F, to act along the reference
axes shown in Fig. 6-1. They are the mutually perpendicular components

A
ue E, B

Fic. 6-1. — Resultant of three mutually perpendicular forces.


143
144 FORCE SYSTEMS IN SPACE [Chap. VI

of a force F whose value is determined by a tip-to-tail addition of their


free vectors. To the tip of Ff, we place the tail of F,, then add F, to the
tip of F, as shown in the figure. Observe carefully that the order of vector
addition has no effect on the resultant; this is true for either space forces or
coplanar forces.
The resultant F extends from the tail of the first vector at O to the tip
of the last vector at C. A glance at Fig. 6-1 shows that the tip-to-tail ad-
dition of the vectors determines the sides of a box of which F is the body
diagonal. Observe that OB is the resultant of F, and F, and may be de-
fined by the relation
(OB) =F, li) (a)
Observe also that OB and BC (which has the value F,,) form a right triangle
of which F is the hypotenuse, so that we have
F? = OB? + BC? = F2+4+ F2+4+ F,? (b)
from which we finally obtain
F=V/F2 4+ F? + F2 (6-1)

The inclination of a force in space is defined by the angles included be-


tween the reference axes and the force. It is generally denoted by @ with
a suitable subscript. For example, the angle between F and the X axis is
defined by 6,; that between F and the Y axis by 6,; and that between F
and the Z axis by 0... These angles are shown in Fig. 6-1, but are some-
what distorted because they are angles in space. The angle shown clearest
in the figure is 6, included between F and its Y projection F,. From the
trigonometry of the right triangle OHC, it is apparent that F, = F cos 6,.
Also from the right triangle OFC we have F, = F cos 6,, and from the
right triangle OAC, F, = F cos 6,. The similarity of these expressions is
seen in the following summary:

F, = F cos0, cos 0, = F
. F

, =sF cos 6, or Fy,


cos 0, = Ff
F (6-2)

F; = F cos 0, fs
cos 6, = Fs

Note again that 6, lies in the plane determined by F and the X axis; 6, lies
in the plane determined by F and the Y axis; and 9@, lies in the plane de-
termined by F and the Z axis. The cosines of the angles 6,, 6,, and 6, are
defined as the direction cosines of the force.
Very often a force is specified in direction by the coordinates of the points
Art. 6-2] The Three Mutually Perpendicular Components of a Force 145

through which its line of action passes. The distance d separating such
points is the body diagonal of a box whose sides are the mutually perpen-
dicular components of this distance. This box is geometrically similar to
the rectangular parallelepiped formed by the components of a force; the
force being the body diagonal of this parallelepiped. Consequently the
components of a force are directly proportional to the components of the
distance d separating two points on the line of action of the force. This
proportionality is an extension of the relations between the sides of a force
triangle and its corresponding slope triangle that was discussed in Illus.
Prob. 202 (page 17). It is expressed by

ea (6-3)
As an example, let us determine the components of a force F = 300 Ib
whose line of action coincides with the line joining points A and B in Fig.
6-2. From this figure, the components of the distance d between A and

Fic. 6-2. — The components of a force as determined by the coordinates of two points on
its line of action.

B are x = 5, y = 4, and z = 6 xo that, applying Eq. (6-1), the length of


d is found to be
[d=Vetyt+| d = V5) + (4 + 6)? d = 8.78
146 Force SYSTEMS IN SPACE [Chap. VI

Then from Eq. (6-3), the desired components of F are determined from
jel Me ie SU
Fee a Oem
which gives
FF, = 17 lb: 2, = 13/ los f= — 20> lb ais:
The signs of the components correspond with the positive directions of
the reference axes. Note that the components of the force F which here is
directed along AB from A toward B, will point in the directions correspond-
ing to the paths taken in moving from A to B along directions parallel to
the coordinate axes. The components, which must intersect on the line of
action of the force, may be shown as acting at A or B, or at any other con-
venient point on the action line of the force.

6-3. Resultant of Concurrent Force Systems in Space


The resultant of a system of concurrent space forces is found in a fashion
similar to that used for coplanar concurrent forces. The X, Y, and Z com-
ponents of the resultant are equal to the algebraic summations of the X,
Y, and Z components of the forces composing the system. Having deter-
mined the components of the resultant, we may apply the methods outlined
in Art. 6-2 to determine the resultant itself. In determining the resultant,
it is necessary to give the magnitude of the resultant, its direction cosines,
and its pointing in space. For example, the pointing of a force that passes
through the origin and the point (8, —4, 5) may be described as forward
and down to the right.

ILLUSTRATIVE PROBLEM
601. Determine the resultant of the system of concurrent forces having the fol-
lowing magnitudes and passing through the origin and the indicated points: P =
200 lb (+4, +8, +5); Q = 400 lb (+6, —3, —5); F = 300 lb (—3, 6, —4).
Solution: Figure 6-3 shows the forces as they act in space and as they appear in
the top and front views. It is convenient to tabulate the components of distances
and of forces as follows:

Components of Distance : Forces


Force Distance
w y 2 d X Comp. |Y Comp. |Z Comp.

P = 200 +4 +3 +0 2.07 fi a TS.3 Wt Aeterna


Q = 400 +6 =o —5 8.37 | +287 | —143.4 | —239
F = 300 —3 +6 —4 7.81 =115.2 | +230:5 | —153.7

Totals +285.1 | +171.9 | —251.3


Art. 6-3] Resultant of Concurrent Force Systems in Space 147
a
a a a
The table is almost self-explanatory. The terms in the column headed Distance
are found from

la-ve Fyre | For P: d=~/42+


32+4+
52 = 7.07

F=300lb

400 lb
300 lb

Top view | 2001b


tYZ
300 Ib

Fic. 6-3.

The signs of the components of the forces are evident from their directions in
Fig. 6-3b, or may be deduced from the directions followed in moving along paths
parallel to the coordinate axes from O to the points A, B, and C in Fig. 6-8a. The
magnitudes of the components are determined from

eM.) FF ; ad
ane eokagr Nh
je o-2 5) Hae Einwal space sepia ir Sg7
whence
P,= 1133 lb; FP, =S8681b2.P,=1414 Ib
From the totals of the columns, we have
ZX = +285.1 lb
ZY = +171.9 lb
ZZ = —251.3 lb
By visualizing a vector addition of these terms, we see that the resultant points
to the right, up, and backward. The magnitude and the direction cosines of the
resultant are determined from the following equations:

[z = o/(2X)? + (ZY)? + Ezy | R= +/ (285.1)? + aes one


= ns.
148 Force SYSTEMS IN SPACE [Chap. VI
ee
SSO) ee e
} 285.1 ; Ans.
* cos Ox = He = 0.686 hs = 46,7
co 0, =

3 171.9 ;
E 0, = = cos by = Tie 0.413 6, = 65.6° Ans.

2 251.3 juga
6, eS 52.8 Ans.
E (i). = Pon cos i. = “416 => 0.604 |

Note that no signs are used in the direction cosines; the visualization of the point-
ing of the force describes the angles completely.

PROBLEMS
602. Determine the magnitude of the resultant, its pointing, and its direction
cosines for the following system of non-coplanar, concurrent forces. 300 lb (+3,
—4, +6); 400 lb (—2, +4, —5); 200 Ib (—4, +5, —3).
Ans. R = 297 lb pointing backward, up, and to the left; cos 6, = 0.395; cos 6, =
0.763; cos 6, = 0.511
603. Determine the magnitude of the resultant, its pointing, and its direction
cosines for the following system of non-coplanar, concurrent forces. 100 Ib (+2,
+3, +4); 300 lb (—3, —4, +5); 200 lb (0, 0, +4).
Ans. R = 508 lb pointing forward, down, and to the left; cos 6, = 0.178;
cos 6, = 0.225; cos 0, = 0.959
604. Determine the magnitude of the resultant, its pointing, and its direction
cosines for the following system of non-coplanar, concurrent forces. 200 lb (+4,
+15 —8)-4001b (=6,-64). — 5); 300 Ib 249283).
605. Three concurrent forces P, Q, and F have a resultant of 5 lb directed
forward and up to the right at 6, = 60°, 6, = 60°, 6, = 45°. P equals 20 lb and
passes through the origin and the point (2, 1, 4). The value of Q is also 20 lb and
it passes through the point (5, 2, 3). Determine the magnitude of the third force F
and the angles it makes with the reference axes.
Ans. F = 33.7 lb pointing down, back, and to the left. 6, = 48.4°, 0, = 75.7°,
6, = 45.6°

6-4. Moment of a Force about an Axis


By definition, the moment of a force about an axis is a measure of its
rotational effect about the axis. The idea of a moment arm of a force,
developed in Art. 2-4, is not of much use when dealing with forces in space;
however, the principle that the moment of a force is equal to the moment
sum of its components (Art. 2-5) is of great value.
To understand the physical significance of the moment of any force about
any axis, let us imagine a device like that shown in Fig. 64a. Assume the
axis (the X axis in this instance) to be a taut wire on which is mounted the
structure shown. Through the structure a hole is drilled which is slightly
larger than the diameter of the wire. The dimensions of the structure are
Art. 6-4] Moment of a Force about an Axis 149
re r ee e ee ee
such that the end of the arm contains point A through which a force F
passes. (The force is represented by means of its X, Y, and Z components. )
The component F, which is parallel to the axis (i.e., the wire) can only
translate the structure along the wire; it is evidently impossible for F', to

End View
(a) (b)
Fia. 6-4. — Moment of a force about the X axis.

rotate the structure about the axis. From another point of view we could
say that F, intersects the X axis at infinity and therefore has no moment
arm about the axis. In any event, we conclude that a force has no moment
about a parallel axis.
A different situation exists, however, with respect to both F, and F,.
Looking at the end view of the structure (Fig. 6-4b), we recognize a condi-
tion similar to that for moments of coplanar forces. In fact, F, and F, are
coplanar, and the X axis appears as a point which is the center of moments.
If the counterclockwise sense of rotation be assumed as positive, then as
the moment of F about the X axis we obtain
M, = Fy-2:— F.-Y
A similar analysis may be made to determine the moment M, of F about
the Y axis or the moment M, of F about the Z axis. In general, the moment
of a force about any axis is due to the components of the force lying vn the plane
perpendicular to the axis of moments. ‘Thus, as developed above, the moment
of a force about the X axis is due solely to the Y and Z components of the
force. It is helpful to observe that there is always one X term, one Y term,
and one Z term involved in the moment of a force component about a co-
ordinate axis; e.g., the moment arm of the Y component of a force about
an X axis is its Z coordinate.
The moment of a force about an axis can be represented geometrically
by a vector. By convention, the vector is directed along the axis in such a
manner that it points in the direction of the extended thumb of the right
150 Forcr SYSTEMS IN SPACE [Chap. VI
a Aa rian LI RnR I Ee eS A ana es ete
hand when the fingers of that hand are curled about the axis of moments
in the sense of the moment. This is known as the right-hand rule. The
right-hand rule is also convenient to determine the sign of a moment. Ex-
tend the thumb in the direction from the origin along the positive sense
of the coordinate axis. The positive sense of moment about the axis may
then be taken to correspond with the curling of the fingers of that hand.

Fic. 6-5. — Vector representation of a moment and its components.

For example, in Fig. 6-5 the moment of a force about line N is represented
by the vector M. The components of the moment vector are M,, M,, and
M, directed as shown. The sense of the moment is indicated in each case
by the small curved arrows. The moment / acts in the plane ABC which
is perpendicular to line N. The direction of the plane ABC is defined by the
angles @,, 6, and 6, (not shown) which its normal N makes with the coordi-
nate axes.

ILLUSTRATIVE PROBLEM

606. As shown in Fig. 6-6, a 200-lb force F passes through point A to point B.
Compute the moment of force F about each coordinate axis.
Solution: It is convenient to consider the moment of F as equivalent to the sum
of the moments of its components. We begin by resolving F into its X, Y, and Z
Art. 6-4] Moment of a Force about an Axis 151
e r
|We
ey

Fic. 6-6.

components acting at A. Proceeding as in Art. 6-3, we obtain the components


of F from the equations

[a-ve Fyre | d= AB=/PEP


TE = 64
Hoes, Fs FF F, Fy,FF, 200
si y z d Raye GA
whence the magnitudes of the components are
P, = 125 Ib} Fy = 93.7 Ib; F, = 125 Ib
To determine the moment /, about the X axis, we observe that F, is parallel
to the X axis and hence has no moment about it; that F, intersects the X axis and
therefore also does not contribute to M,. If we assume counterclockwise moments
to be positive when looking in the direction of the positive end of the axis toward
the origin (another way of stating the right-hand rule), we obtain
M, = —F, X 4 = —93.7 X 4 = —374.8 lb-ft Ans.
To determine the moment M, about the Y axis, we assume that counterclockwise
moments are positive when viewed from the positive end of the axis toward the
origin. Since F, is parallel to the Y axis it produces zero moment, whence
M, = F,-6 — F,-4 = 125 X 6 — 125 XK4 = 250|b-ft Ans.
To determine M,, the moment of F about the Z axis, we observe that the moment
is due only to F, and F,. In this case Ff intersects the Z axis and produces zero
moment. If we again assume the counterclockwise sense to be positive when view-
ing the Z axis from its positive end toward the origin, we obtain
M, = F,-6 = 93.7 X 6 = 562.2 lb-ft Ans.
The positive sense of moments used in this problem corresponds to the right-hand
rule previously discussed. However, we may choose any convenient sense of
moment as positive provided that we are consistent in this choice.
As an exercise, you should check these results by resolving F into its components
152 Force SYSTEMS IN SPACE [Chap. VI
Ee
acting at B. Is it more convenient to find the moment effects by resolving the force
into components acting at A or at B?

PROBLEMS
607. A force of 100 lb is directed from A toward B in the cube shown in Fig.
P-607. Determine the moment of the force about each of the coordinate axes.
Ans. M, = —62.4 lb-ft; My = 250 lb-ft; Mz = —250 lb-ft

Fie. P-607.

608. A force of 200 lb is directed from B toward C in the cube shown in Fig.
P-607. Determine the moment of the force about each of the coordinate axes.
609. A force of 300 lb is directed from B toward D in the cube shown in Fig.
P-607. Determine the moment of the force about each of the coordinate axes.
610. A force of 400 lb is directed from C toward # in the cube shown in Fig.
P-607. Determine the moment of the force about each of the coordinate axes.
Ans. M, = 653 lb-ft; M, = 979 lb-ft; M, = —1306 lb-ft
611. A force P, directed from F toward B in the cube shown in Fig. P-607,
causes amoment M, = 1600 lb-ft. Determine P and its moment about the X and Z
axes.
612. A force P is directed from a point A (4, 1, 4) toward a point B (—8, 4, —1).
If it causes a moment M, = 1900 lb-ft, determine the moment of P about the X
and Y axes. Ans. M, = —1700 lb-ft; M, = —800 lb-ft

6-5. Equilibrium of Concurrent Space Forces


As stated in Art. 3-1, equilibrium is defined as that condition when the
resultant of a system of forces is zero. Therefore the conditions of equi-
librium of concurrent space forces are expressed by those equations which
Art: 6-5] Equilibrium of Concurrent Space Forces 153
insure that the resultant will be zero. In Art. 6-3, the resultant of con-
current space forces was determined in magnitude by the equation

it av
db Oe Oyoe OT AL
Obviously F will be zero, and hence equilibrium will exist, only under
the following conditions:
Sxa—0
=Y=0 (6-4)
WA Sale

These three independent conditions of equilibrium may be used to deter-


mine a maximum of three unknown quantities to maintain equilibrium of a
concurrent space system of forces.
It is also possible to express the conditions for equilibrium in terms of
moment equations. If moments are taken about some axis which does not
intersect the line of action of the resultant force, it is evident that M =
R - d will equal zero only when R = 0. The moment WM of the resultant
has three components, consisting of the moment summations 2M,, =M,,
and ~M., for the forces composing the system with respect to coordinate
X, Y, and Z axes. Therefore, R will also be zero when
EMS =0
= M7 0 (6-5)
=Mii=0
Either Eqs. (6—4) or (6-5) or some combination of them may be used to
determine the three unknown quantities necessary to maintain equilibrium
of concurrent space forces. The choice depends upon the particular prob-
lem to be solved.

ILLUSTRATIVE PROBLEM
613. The framework shown in Fig. 6-7 is composed of three members, AB, AC,
and AD. Points B and C are in the same horizontal plane. Points C and D are in
the same vertical plane. At A a 1000-lb force acts in a direction parallel to the X
axis. Neglecting the weights of the members, determine the forces in AB, AC,
and AD.
Solution: Since the weights of the members are neglected, each member is acted
upon by only two forces: those applied at the ends of the member. Accordingly, as
shown in Art. 4-2, the force in each member must be axial. It will be convenient
to denote the internal forces by the forces acting at the ends B, C, and D of the
separate members. These forces have been resolved into their X, Y, and Z com-
ponents as shown in the figure. Because the direction of the force is known to
be along the member, the determination of any component, such as D,, will de-
termine the corresponding force D and its other components from the relations
154 SPACE
Force SYSTEMS IN Se [Chap. VI
cl a
iraeihn, Saltese ei d ee

IY Dz = Dy = Le = a The values of 2,
ey y z
y, 2, and d are determined by the com-
1000lb ponents of the distance d separating
points A and D. Note that the com-
ponents of a force, such as that at D,
which is directed along AD from A
toward D, will point in the directions
corresponding to the path taken in
moving from A to D along coordinate
axes.
Denote by Yc a line through C par-
D allel to the Y axis. (Line Ye is not
Sa re shown on the figure.) A moment sum-
AN pe _X mation about Yc equated to zero will
J bv ox determine D, because the forces at C
Da Ne ° ge | and the force D, intersect the axis of
Z 1B, moments, and because B, and D,, being
Fic. 6-7. parallel to Yc, have no effect on the
moment summation. Thus we get
[2My, = 0] 15D, 1000 X 10.0 D, = 667 |b
Applying Eq. (6-2), we find the other components and magnitude of D to be
pe 667MED, Dp D
Dog) a @ 10Mie14
95 ~~ 4/321
from which
Dy = OBS ling Dy = SEB Mop ld) =] WIP Myi Alas,

A moment summation about Yp (not shown in the figure) equated to zero will
similarly eliminate all unknown quantities except C,. Hence we obtain
[=My, = 0] 15 C, — 1000X 5 =0 Cz = 333 lb
and

[eee a33 Cy _Ce_ C


x y eat 1020 LelOme 47.000
from which
Cy, = 667 lb; C. = 333 lb; C = 8151bT Ans.
To determine B,, set moments about Zc to zero, which eliminates all unknowns
except B, and D,. But the value for D, has already been determined, whence
[2M z, = 0] 10 By, -- 6D, — 1000 x 20 = 0
10 B, + 6 X 667 — 20,000 = 0 B, = 1600 lbC Ans.
If desired, the value of B, could also have been determined from the condition of
equilibrium ZY = 0 since the values of C, and D, have previously been found.
Thus we obtain
[ZY = 0] B, — 667 — 933 = 0 B, = 1600 lb C Check
Art. 6-5] Equilibrium of Concurrent Space Forces 155

Still another acceptable procedure would be, after having found D, from
2My, = 0, to determine C, from 2X = 0 or C, from >Z=0. The Y com-
ponents of D and C having been found as discussed previously, the condition
ZY = 0 can be used to determine B,. The condition >Mz, = 0 could then be
used as a check.

PROBLEMS
614. The shear-leg derrick shown in Fig. P-614 supports a vertical load of 2000 Ib
applied at A. Points B, C, and D are in the same horizontal plane and A, O, and D
are in the XY plane. Determine the force in each member of the derrick.

IY
PSP ogee a hee came ba geag

15’

Fig. P-614.

615. The framework shown in Fig. P-615 consists of three members AB, AC, and
AD whose lower ends are in the same horizontal plane. A horizontal force of 1000
lb acting parallel to the X axis is applied at A. Determine the force in each mem-
ber. Ans. AB = 1118 lb C; AC = 766 lb T; AD = 523 lbT

1000 Ib

Fic. P-615 and P-616.


156 Force SYSTEMS IN SPACE [Chap. VI
Be ee

616. Referring to Fig. P-615, replace the 1000-lb force by a vertical downward
load of 2000 Ib. Determine the force in each member under this revised loading.
617. The points B, C, and D of the cantilever framework shown in Fig. P-617 are
attached to a vertical wall. The 400-lb load is parallel to the Z axis, and the 1200-lb
load is vertical. Compute the force in each member.
Ans. AB = 1615 lb C; AC = 588 lb C; AD = 2320 lb T

SA 1200 Ib
Fie. P-617.

618. The unsymmetrical cantilever framework shown in Fig. P-618 supports a


vertical load of 1700 lb at A. Points C and D are in the same vertical plane while B
is 3 ft in front of this plane. Compute
the force in each member.
Ans. vA Be lion lbiC vA C= s1260
lb T; AD = 183 lb T
619. Solve Prob. 618 if the 1700-lb
load in Fig. P-618 acts horizontally
A outward from A in the direction from
E toward A.

620. The framework shown in Fig.


17001h P-620 supports a vertical load of 2000
lb. Points B, C, and D are in the same
horizontal plane. Determine the force
in each member.
Fia. P-618 and P-619.
Ans. AD = 6401bC¢
621. A vertical load P = 800 lb applied to the tripod shown in Fig. P-621 causes
a compressive force of 256 lb in leg AB and a compressive force of 283 lb in leg AC.
Determine the force in leg AD and the coordinates xp and zp of its lower end D.
Ans. AD = 433 lb C; gp = 4 ft; zp = 1 ft
Art. 6-6] Equilibrium for Non-Concurrent Space Forces Nays
Sa ee
622. In Fig. P-621, if P = 1200 lb and 2000 lb
the coordinates of D are xp = 5 ft and
zp = 2 ft, compute the force in each leg of
the tripod.
Ans. AB = 471. lb C; AC = 453 lb C:
ANDY = RSP? Noy (C.
623. Determine the maximum safe ver-
tical load W that can be supported by the
tripod shown in Fig. P-623 without ex-
ceeding a compressive load of 2400 lb in
any member. Ans. W = 5200 lb

Fic. P-621 and P-622. Fie. P-623.

6-6. Equilibrium for Non-Concurrent Space Forces


Equilibrium is defined as a condition in which the resultant of a system
of forces is equal to zero. The equations of equilibrium for a non-concurrent
system of forces in space will therefore be expressed by the conditions neces-
sary to reduce the resultant of such a system to zero.
Using the procedure discussed in Art. 2-8 (page 39), each of the forces
of a non-concurrent space system may be replaced by an equal force acting
through the origin of a set of reference axes and a couple. Hach force may
then be resolved into its components F,, F,, and F., while the moment M
of each couple may be resolved into its components M,, M,, and M;. The
magnitude of the resultant is determined by the force summations =X,
158 ForcE SYSTEMS IN SPACE [Chap. VI

>Y,and DZ. The magnitude of the resultant couple is similarly determined


by the moment summations 2M,, 2M,, and 2M,.
If all the force summations are zero, the resultant cannot be a single force
but may be a couple. The possibility of the resultant being a couple is
eliminated if all the moment summations are also zero. Hence the resultant
will be zero and equilibrium exist only when all.of the above summations
are simultaneously equal to zero. The conditions of equilibrium therefore
are
rX =0
rY=0
=Z=0 (6-6)
=M, = 0[{
=M, =0
=M,= 0
These equations represent a maximum of six independent conditions
which may be used to determine a maximum of six unknown quantities that
may be necessary to maintain equilibrium for a non-concurrent system of
forces in space. The illustrative problems which follow indicate how these
equations of equilibrium are applied.
ILLUSTRATIVE PROBLEMS
624. The stiff-leg derrick shown in Fig. 6-8 supports a vertical load of 2000 Ib
applied at D. Points A and B are in the same horizontal plane. Points B and C are
Art. 6-6] Equilibrium for Non-Concurrent Space Forces 159

in the same vertical plane. The mast AZ is supported in a socket at A, and the
boom can rotate about the mast. In the position shown, the boom has been ro-
tated forward through a = 30°. Determine the components of the bearing reaction
at A and the forces in BE and CE.
Solution: Members BH and CE are two-force members in which the load is
axial. The directions of the forces B and C at the ends of these members are there-
fore known, and the forces in these members can be determined from any of their
components. The mast AF, however, is a three-force member because of the boom.
The direction of the reaction at A is hence not known,! and each of the components
at A must be determined since the relations between them are not known. Thus
there are five unknown quantities to be determined in the problem.
We begin by taking a moment summation about Xz. This eliminates the forces C
and B, which, being directed along CE and BE respectively, intersect the axis of
moments at H. Also eliminated are A,, which intersects Xz, and Az, which is
parallel to Xz. Thus the only unknown quantity left in the moment summation
about Xz is Az. We get therefore
[2Mx, = 0] 20 A, — 2000 X (DF sin 30°) = 0
20 A, — 2000 X 10 sin 30° = 0 A, = 500 lb Ans.
In a similar manner, a moment summation about Z x will eliminate all unknowns
except Az, giving
[2Mz, = 0] 20 A, — 2000 X (DF cos 30°) = 0
20 A, — 2000 X 10 X 0.866 = 0 A, = 866 lb Ans.
A moment summation about Yc eliminates the components of C as well as Ay,
B,, and B,. The value of B, can then be computed in terms of Az and A; for which
values have been previously determined. Thus
[2My, = 0] 20 B, —10 A, —10 A, = 0
20 B. — 10 X 866 — 10 X 500 = 0 B, = 683lb
To determine the force at B as well as its other components, we apply the equation

B, By, B, _ 8B O83—Bp,
Zip 2g 10
© 2010 7600
from which
B, = 1366 lb; B, = 683 lb; B = 1672 1b T Ans.
The condition of equilibrium, 2X = 0, may now be applied to determine C, from
which the force at C may be computed. We obtain
[ZX = 0] Ae te, ae
866 — 683 — C, = 0 C,= 183 Ib
Oh LO ane LS eal pan cspam
ee TE 10 14 10 /396
Cy, = 256 lb; C, = 183 lb; C = 364lb T Ans.
1The reactions at the hinges of three-foree members are not directed along the axis
of the member. See Art. 4-6.
160 Forcr SYSTEMS IN SPACE [Chap. VI

Finally, the value of A, can be obtained by a moment summation about Zz.


Note that the moment arm of the 2000-lb force about this axis is the sum of AG
and the X component of DF. Thus we obtain
[=Mz, = 0] 104, +6 Cz — 2000 (10 + 10 cos 30°) = 0
10 A, + 6 X 183 — 2000 X 18.66 =0 A, = 36221b Ans.
The condition SY = 0 can be used asa check. In applying it, we obtain
[ZY = 0] AP=B, = C2000
A, — 1366 — 256 — 2000 = 0 A, = 3622 lb Check
625. <A bar AC rests in a socket at C (Fig. 6-9) and is held in equilibrium by
members AH and BD. At A the 400-lb force is parallel to the Z axis, and the
1000-lb load is parallel to the Y axis. Points C, D, and E are in the same vertical
plane, and points A and # are in the same horizontal plane. Determine the forces
in members AF and BD and the components of the bearing reaction at C.

Y
AD, rr as
D I 400 1b
ks om ‘e Se

OF a tig m0 eee 2.
ie
0S,

‘ 1000 Ib
ZO

Fic. 6-9.

Solution: Members AF and BD are two-force members in each of which the force
is directed along the member. Since the direction of the force is known, only one
of the components need be computed in order to determine the force in the member
and the magnitude of its other components. However, AC is a three-force member
in which the end loads are not directed along the member. The direction of the re-
action at C’ is therefore not known, and each of the components at C must be com-
puted. Thus there are five unknown quantities in the problem.
The relations between forces and components at D and E are given by

E Eee ll Deed, CaDe wae


Art. 6-6] Equilibrium for Non-Concurrent Space Forces 161
eae ee ee a
Two equations involving only D; and E, are obtained by taking moments about
Yo and Ze. Thus

[2My, = 0] 6D, — 6 E, — 400 X 12 = 0


[2Mz, = 0] 13 D, + 9 H, — 1000 X 12 = 0
Solving, we find D, = 873 lb and EH, = 73 lb. Using the relations between the
forces and their components given above, we determine D = BD = 1334 lb T,
and #j— A= 81 7 1b/T.
Using the values of D, and EZ, we determine C, by applying the equation
x = 0] C, — D, — E, = 0
C, — 873 — 73 = 0 Cz = 946 lb Ans.
The relations between forces and components for D and E given above must be
used for determining C, and C,. We have
a4 = 0] C,+ D, — E, — 400 = 0
873 73
C+ 6\——— = Pla — 400 = 0 Cz= —219.5 lb Ans.

[ZY = 0] G,
4+ De = 100010
Li Sia

PROBLEMS
626. The plate shown in Fig. P-626 carries a load of 1000 lb applied at # and is
supported in a horizontal position by three vertical cables attached at A, B, and C.
Compute the tension in each cable. Ans. A = 200 lb; B = 400 lb; C = 400 lb

Fic. P-626 and P-627.

627. Solve Prob. 626 if, in addition to the 1000-lb load, the plate weighs 1200 lb.
628. An airplane with a total wingspread of 40 ft is equipped with a tricycle
landing gear. The two rear wheels of the landing gear have a 10-ft tread and are
12 ft behind the forward wheel. The airplane’s weight of 5700 lb acts 1 ft in front of
the rear wheels. (a) Determine the downward force applied at one wingtip along a
line 2 ft ahead of the rear wheels that will tilt the airplane when it is standing on the
runway. (b) What upward force at the same place on the wingtip will tilt the air-
plane? Ans. (a) 1650 Ib
162 Forcr SYSTEMS IN SPACE [Chap. VI
629. Refer to the unsymmetrical
cantilever framework described in
Prob. 618 on page 156 and repeated
here as Fig. P-629. If the vertical load
of 1700 lb is shifted to act at the mid-
point of member AB, compute the
components of the reaction at B and
the forces in the bars AC and AD.
630. The boom BE of the stiff-leg
1700 1b
derrick shown in Fig. P-630 is rotated
forward 30° measured in a horizontal
plane. The mast AB is vertical and
is supported in a socket at A. The
Fig. P-629.
points A, C, and D are in the same
horizontal plane. Determine the forces
in the legs BC and BD and the com-
ponents of the bearing reaction at A.
Ans. Az = 289 lb; Az = 167 lb;
Aly = ISO Noe IAC = 055 Milo
AUS 13D) = OP Mloy 0
631. The boom BC of the stiff-leg
derrick shown in Fig. P-631 is con-
tained in the XY plane. The mast AB
is vertical and rests in a socket at A.
Points A and D are in the same hori-
zontal plane. Points D and # are in
the same vertical plane. Determine
the forces in the legs BE and BD
and the components of the bearing
reaction at A.
Fic. P-630.

Ans. A, = 500 lb; A, = 1875 lb;


A, = 0; BE = 515 lb T;
BD = 6121bT
1000 Ib
632. The boom of the derrick
shown in Fig. P-632 is rotated back
30° from the XY plane. Determine
the forces in BC and BD and the com-
Eo ponents of the bearing reaction at the
MAES Mes oe S Seay
Be = socket A of the mast AB.
ie aa 10 at ag A.
Ans. BD = 1630 lb T; BC = 815
tao» wes
eZ lbT;A, = 866 lb;A, = 3732
Bie. P-6381. lb; Az = 500 1b
Art. 6-6] Equilibrium for Non-Concurrent Space Forces 163
e n ee
B

oe as A
eee Fic. P-632 and P-633.

Fic. P-634.

633. In Prob. 632, compute the posi-


tion of the boom which will cause maxi-
mum loadin BC. What is this maximum
load?
Ans. The boom must be rotated 30°
forward from the XY plane;
BOC 1630 lb oc

634. A uniform bar 10 ft long and


weighing W lb swivels about a frictionless
ball and socket joint A at its lower end.
Its upper end B rests against a rough
vertical wall as shown in Fig. P-634.
Determine the relation between the co-
efficient of friction f at the vertical wall
and the limiting equilibrium position
angle 6 of the vertical projection of the
bar as shown in the left side view.
Ans. f = }tan 0
635. A schematic diagram of a single-
cylinder engine is shown in Fig. P-635.
In the given position, a force of 18 lb is
required to turn the crankshaft against
the compression P in the engine. Com-
pute the forces on the bearings A and B. 181b Fic. P-635.
164 Force SYSTEMS IN SPACE [Chap. VI

Detail of tooth contact

Fic. P-636 and P-637.

636. A torque of 1200 in.-lb is applied to the driving pinion of the 4 to 1 gear re-
duction drive shown in Fig. P-636. The pressure angle for both pairs of gears is
143° as shown in the detail of tooth contact. Compute the total bearing reactions
at A and B.
637. Repeat Prob. 636 if the input torque is 900 in.-lb and the gears are of the
stub tooth type for which the pressure angle is 20°.
638. In Fig. P-638, the boom AC is supported in a ball and socket joint at C and
by the cables BD and AH. Compute the forces in the cables and the components of
the reaction at C.

Fig. P-638 and P-639.


Art. 6-6] Equilibrium for Non-Concurrent Space Forces 165

639. Solve Prob. 638 if the cable AE is replaced by one extending from A to a
point F 4 ft vertically above Z.
Ans. Cz = 2240 lb; C, = 1200 lb; C, = 320 lb; AF = 1198 lb T; BD =
1670 lb T

W=100lb
Fic. P-640.

640. A bar AB, 8 ft long and weigh-


ing 100 lb, is supported by three cables
having tensions P, Q, and T as shown
in Fig. P-640. Qis in the YZ plane
30° from the Y axis. P is in space
making 45° with the Y axis and so
placed that its projection on the XZ
plane makes an angle of 30° with the
Z axis. The magnitude and direction
of T are unknown. Determine the
values of P, Q, and 7, and the direc-
tion angles of 7’.
Ansa, P= 28:3 Ib; Q = 34.6 Ib;
fe blelbs 0, — (3,0 40, =
11.4°; 6, = 90°
641. Figure P-641 represents a
gate weighing 1200 lb which is sup- We
ported by a hinge at B and a ball 1200 lb
and socket joint at A. The hinge at
B provides only horizontal support.
A 270-lb force directed parallel to the 270 Ib
Z axis acts on the gate at H. The gate Fig. P-641 and P-642.
166 Forcr SYSTEMS IN SPACE (Chap. VI
a

is prevented from turning by a cable CD. Determine the components of the re-
actions at A and B and the tension in CD.
Ans. CD = 485 lb T; Az = 560 Ib; A, = 1020 lb; A, = 180 lb; Bz =) 200 1b;
B, = 180 lb

642. Solve Prob. 641 if the cable CD is replaced by one extending from C to a
point F 6 ft vertically below D, all other data remaining unchanged.

SUMMARY

The resultant of three mutually perpendicular forces (Art. 6-2) is given


by the equation
F=VF24+ FPF? + Fe (6-1)

and its direction cosines by


cos jee
0; = =

F
cos 0, = -F (6-2)

cos 8, = z

When the direction of a force is specified by the coordinates of two points


through which its line of action passes, the most convenient relation between
the force and its components is expressed by

Ce lees (6-3)
where x, y, and z are the coordinate distances between the two points on
the line of action of the force and d = V2? + y? + 22.
The resultant of a concurrent force system in space (Art. 6-3) is deter-
mined from the summations of the XY, Y, and Z components of the forces
composing the system. Equations similar to Eqs. (6-1) and (6-2) may be
used to determine the magnitude and direction cosines of the resultant.
The moment of a force about any axis (Art. 6-4) is determined as the
sum of the moment effects of the components of the force lying in the plane
perpendicular to the axis of moments. For example, if the X axis is chosen
as the moment axis, the moment of a force will be the sum of the moment
effects of its Y and Z components.
Equilibrium of concurrent space forces (Art. 6-5) is determined by apply-
ing either set of the following equations or a combination of them. How-
ever, since there are only three independent conditions of equilibrium, no
more than three unknown quantities may be determined.
Summary 167

EX =0
ZY =0 (6-4)
=Z=0
=M, = 0
=M, =0 (6-5)
=M,. =0
To create equilibrium of non-concurrent space forces (Art. 6-6), both
sets of the above equations must be satisfied. Hence a problem in the
equilibrium of non-concurrent space forces may be solved for as many as,
but no more than, six unknown quantities.
Chapter VII.
Centroids and Centers of Gravity

7-1. Introduction
A body of weight W is supported by a string attached at A, as shown
in Fig. 7-1. The only external forces acting on the body are its weight and
the reaction exerted by the string. Equilibrium
of the body can exist only if these two forces
are equal, opposite, and collinear. The line of
action of the weight W can be determined,
therefore, by the line of action of the support.
Let the body be supported in a new position
by the string now attached to B. The body
will shift its position so that the line of action of
the weight is again collinear with the string.
Thus two positions of the line of action of the
weight are determined experimentally. The
intersection of these positions of the line of
Fie. 7-1. action determines a point which is defined as
the center of gravity of the body; this is the
point through which the action line of the weight always passes.
From the above discussion it is apparent that the problem of locating
the center of gravity of a body reduces to determining the point through
which the resultant weight of the body acts.

7-2. Center of Gravity of a Flat Plate


The analytical location of the center of gravity is simply a variation of
the principle of moments; that is, the moment of the resultant is equal to
the moment sum of its parts. As an example, consider the flat plate of
irregular section shown in Fig. 7-2. <A pictorial as well as front and side
views is shown. The network shown divides the plate into small elements
having weights w), we, etc., which act at the center of each element. These
gravity forces form a parallel force system, the resultant of which is the
total weight W of the plate.
Let the coordinates of each elemental weight be (a1, y1), (v2, yz), ete., and
the coordinates of the resultant weight be (@, 7), as shown in Fig. 7-2.
Note the use of the bar sign. In this book the coordinates of a resultant —
168
Art. 7-3] Centroids of Areas and Lines 169

Front view Side view


Fic. 7-2. — Coordinates of the center of gravity.

be it force, weight, or area — are always distinguished by a bar sign. The


coordinates are read as “bar x,” etc. Taking moments of the weights about
the Y axis, we get

We = witi + woe + +++ = Lr (a)


With respect to the X axis, we have

WY = wii + Wye +++ + = Dwy (b)


These equations merely state that the moment of a weight W about an axis
is equal to the moment sum of its elemental weights.

7-3. Centroids of Areas and Lines


If the material of the plate in Fig. 7-2 is homogeneous, the weight W
may be expressed as the product of its density y (i.e., weight per unit
volume) multiplied by tA, where ¢ is the thickness of the plate and A is its
area. Similarly the weight w of an element is given by yta, where a is the
cross-sectional area of the element. Substituting these values in Eq. (a)
in Art. 7—2 results in
ytAz = ytayx, + ylaexe +. ae) ts oe ytZax
170 CENTROIDS AND CENTERS OF GRAVITY [Chap. VIT

whence, canceling the constant terms y and ¢, we get


Ax = oat
" (7-1)
and similarly Ay = Xay
By analogy with Eqs. (a) and (6) in Art. 7-3, the expression AZ, as well
as AY, is called the moment of area. It is equiyalent to the sum of the
moments of the elemental areas composing the total area. Note that the
moment of area is therefore defined as the product of the area multiplied by
the perpendicular distance from the center of area to the axis of moments.
If Eq. (7-1) is rewritten in the following form:

zg — 2aA
tos Yay | (7-1a)

7 TE
this gives a method of locating a point called the centroid of area. The cen-
troid of area is defined as the point corresponding to the center of gravity of
a plate of infinitesimal thickness. The term “centroid” rather than “center
of gravity” is used when referring to areas (as well as to lines and volumes)
because such figures do not have weight. The term “center of gravity”’ is
widely used, although it is a misnomer. Strictly speaking, it should refer
to the center of weight of actual
Y bodies.
& When referring to lines, the cen-
+ | troid may be determined by similar
means. A line may be assumed to
be the axis of a homogeneous slender
eu wire. Thus Fig. 7-3 represents the
center line of a homogeneous wire of
¥, length L and constant cross-sectional
5 : 5 _ x area a lying in the XY plane. The
weight W is given by the equation
Fic. 7-3. — Homogeneous slender wire. W = yal and the weight w of an
elemental length 1 by w = yal.
Substituting these values in Eqs. (a) and (b) in Art. 7-2, we have
yaLé = yale, + yal, +--+ = yadle
and
yaLg = yaly: + yaley, +--+ = yadly
whence, canceling the constant terms y and a, we get
Lx = oH,
Ly = Xly oe)
Art. 7-5] Centroids Determined by Integration 171

7-4. Importance of Centroids and Moments of Area


In subsequent work on strength of materials, the student will find the
location of the centroid of an area of great importance. For example, he
will learn that in order to produce uniform stress distribution, the loads
must be placed so that the line of action of their resultant coincides with
the centroid of the cross section of the member. The position of the centroid
of an area is also important for determining the location of the neutral axis
in the bending of beams, for in strength of materials it is shown that the
neutral axis (line of zero stress) passes through the centroid of the cross
section of the beam.
An axis passing through the centroid of an area is known as a centroidal
axis. The next chapter, which deals with moments of inertia, will make
clear the great importance of the position of centroidal axes of areas.
Many other instances of their importance will come to the student’s at-
tention in his engineering studies.
Of equal importance to the position of a centroid is the moment of an
area. We recall that the moment of an area with respect to an axis was
defined as the product of the area multiplied by the perpendicular distance
from its centroid to the axis. In dynamics, the moment of area is used to
determine the displacement of a body subjected to variable forces (see Art.
15-5). In strength of materials, it is used to determine shearing stresses
in beams. In addition, the moment of an area is extensively used for de-
termining the deflection of beams by the area-moment method. These
instances, as well as many others that the student will encounter, should
indicate the importance of a permanent, not a temporary, knowledge of
centroids and moments of area. :

7-5. Centroids Determined by Integration


We recall that integration is the process of summing up infinitesimal
quantities. Except for a change in symbols and procedure, integration is
equivalent to a finite summation. In the preceding articles, for example,
if the area of an element had been expressed as the differential dA (1.e., a
small part of the total area A), the equations for determining the centroid
of an area would become ;
Ax = [xdA (1-1b)
Ay = fydA

and for determining the centroid of a line, we could use


Lie = fx eh (1-28)
Ly = fydl
172 CENTROIDS AND CENTERS OF GRAVITY [Chap. VII

In determining the centroid by integration, the figure is divided into


differential elements so that:
1. All points of the element are located the same distance from the axis
of moments, or
2. The position of the centroid of the element is known so that the
moment of the element about the axis of moments is the product of the
element and the distance of its centroid from the axis.
If a plane figure has a line of symmetry, its centroid is located on that
line. This statement may be demonstrated by balancing a plate on its line
of symmetry, whence the moments of the weights (also areas if the plate
has constant thickness) on either side of the line of symmetry must be
numerically equal and of opposite sign. If a plane figure has two lines of
symmetry, the centroid is located at the point of intersection of the lines.

ILLUSTRATIVE PROBLEMS

701. Centroid of the Arc of a Circle. Determine the centroid of the line which
is an arc of a circle, as shown in Fig. 7-4.

Ww

Fig. 7-4. Fig. 7-5.

Solution: Let the axis of symmetry be chosen as the X axis. Then 7 = 0. If


the radius of the arc is denoted by r and the subtended angle by 2 a, the element of
arc dL and its distance from the Y axis are dL = rd@ and x =r cos 6. Applying
Eq. (7-2a), we have
+a a
aoa = Saab] an) 2 =f ros rd = rf cos
6 dé
=

= 27? sin a
Finally,
ae 2rsnma rsina
2ar a
Ake. FO Centroids Determined by Integration 173
TT
If the arc is a semicircle, as in Fig. 7-5, = 90° = — radians and sina = 1. Sub-
2
stituting these values in the above result gives

% 2
ee ee
T

702. Centroid of the Area of a Triangle. Y


The triangle shown in Fig. 7-6 has a base
6 and an altitude h. Locate the centroid h-y
of the triangular area with respect to the bot
base.
Solution: In accordance with Rule 1 eeu
given earlier, select strips parallel to the y
base as the differential elements of area. |
The area of any differential element is
then dA = xdy. Applying Eq. (7-1b), we Sanaa!
obtain, Fig. 7-6.

h
[JA+9 = fy dA] ($ bh) -9 = J vy dy (a)

From similar triangles, z = ;(h — y), so that Eq. (a) becomes

1 alli Sie

yY=th Ans.

Observe that for any triangle the distance from the centroid to any side is equal
to one-third of the altitude measured from that side. Furthermore, the centroid
of a triangle is located on a median because the median to any side contains the
centroids of all strips drawn parallel to that side. Therefore the centroid is at the
intersection of the medians.
703. Centroid of the Area of a Circular Sector. Determine the location of the
centroid of the area of the sector of the
circle shown in Fig. 7-7. Let the radius of
the circle be r and the subtended angle be
2a.
Solution: Let the axis of symmetry be
taken as the X axis; then 7 = 0. Select
as the element of area the shaded triangle
the position of whose centroid is known
from the answer to Prob. 702 to be « =
27 cos 6. The area of the element is
dA =sr-rd@=77r'd0. Applying Eq.
Fic. 7-7. (7-1b), we obtain
174 CENTROIDS AND CENTERS OF GRAVITY (Chap. VIL

+a +a

[Arf adsl 2 | preao= f 2rcosé-g7r7dd


+a

2 (2a) = 47) cos 0d = Zr>sina


—a

_ B® Pein@
bees Ans
3
If the sector is a semicircular area as in Fig. 7-8,a =
x Oe wae
ee ON = pTadians, whence by substituting in the last equa-

tion above we find the distance of the centroid from


the diameter to be

¢ er ia
OoAP

704. Centroid of the Area of a Parabolic Segment. In Fig. 7-9 is shown a para~
bolic segment bounded by the X axis, the line x = a, and the parabola y = ka’.
Determine the coordinates of the centroids.

Ne

Solution: Select the element of area as the shaded strip parallel to the Y axis.
All points in this element are the same distance from the Y axis. The area of the
element is dA = ydz. The area A of the entire parabolic segment is found from

[A = f dA] Ae |oyon=s| eer


0 0

To determine Z, we apply Eq. (7-1b) as follows:

aloe Sey fue olAl] Wor ie


0 0

(4 ka*) -¢ =+ka!
Art. (ol Centroids Determined by Integration 175

To determine 7, we use the same elementary strip; but since each point of the
element is not the same distance from the X axis, we must use Rule 2 above; i.e.,
the moment of the differential element about the X axis is the product of its cen-
troidal coordinate, $ y, multiplied by the area y dx. Applying Kq. (7-1b) again, we
obtain

[A- 9 = Sy dA] @tot)-g- | by-yde- ae] ode


0 0
1 k2q5
Ss a Rr, ee
(540°) ih
¥ = zo ka? = Bob Ans.

PROBLEMS

705. Determine the centroid of the shaded area shown in Fig. P-705, which is
bounded by the X axis, the line x = a and the parabola y? = kz.
Ans. pS) = 20 9 =. 6

|
ald 4

pee
Fic. P-705. Fig. P-706.

706. Determine the centroid of the quarter circle shown in Fig. P-706 whose
radius is r. 4r
3
707. Determine the centroid of the quadrant of the ellipse shown in Fig. P-707.
2 ie HIE bie de 2D
The equation of the ellipse is at E == Ail Ans.== rape ites oe

Fic. P-707. Fia. P-708 and P-709.


176 CENTROIDS AND CENTERS OF GRAVITY [Chap. VII

708. Compute the area of the spandrel in Fig. P-708 bounded by the X axis, the
line x = b, and the curve y = kx" where n = 0. What is the location of its centroid
from the line x = b? Prepare a table of areas and location of centroid for values of
7, = WO), ih, 2 ehaval 3,
1 b
Ans. } A = ——
Figs) -bh;
bh; & = ——;
rep ref Table
able X-1 on p. 265

709. Determine the y coordinate of the spandrel described in Prob. 708.


eee ae +1
Ans. Y= ea)

710. Locate the centroid of the area bounded by the X axis and the sine curve

y =
me
asin | from = 0 tox = Lb. Ans. ae
Ge nLee a)

7-6. Centroids of Composite Figures


Many of the figures used in engineering are composed of combinations
of the geometrical shapes discussed in the last article. Still other figures
are composed of structural shapes. The location of the centroids for
structural elements is given in handbooks.
When the given figure can be divided into the finite elements discussed
above, these elements can be treated in the same manner as were the in-
finitesimal elements. When this is done, the process is called finite summa-
tion, as contrasted to integration, or the summation of infinitesimal ele-
ments.
If a given area can be divided into parts, each centroid of which is known,
the moment of the total area will be the sum of the moments of area of its
parts. This statement is really an extension of Varignon’s theorem (Art.
2-5).
The centroid of the composite figure is determined by applying the
following equations, which were developed in Art. 7-38. In these equations
the elemental areas become the areas of the geometrical shapes into which
the entire area has been divided.
Ax = =
Ay = Xay (7-1)

A similar process may be applied to lines. The given line may be di-
vided into finite segments whose centroids are known, and the following
equations may be used:
Lx = Xlx
Ly = Xly
} (1-2)
Before these equations are applied to illustrative problems, it will be
convenient to summarize the location of centroids for common geometrical
Art..7-6| Centroids of Composite Figures IRAE

shapes (determined in preceding problems) given in Table VII-1 on page


178.
In addition to the geometric shapes shown in Table VII-1, other sections
commonly used are rolled structural sections such as angles and channels.
The areas and the location of the centroids of such sections are listed in
handbooks. A few typical sections and their values are shown in Table
VII-2. If any of these sections are given in the problems which follow, the
data from this table should be used. This makes it possible to use a struc-
tural shape in the same manner as any other geometrical shape.
Signs. The following discussion indicates the method of determining
which sign to give to an area or coordinate. Consider a plate balanced
on a knife edge denoted by Y—Y in Fig. 7-10. It is evident that the mo-
ment of W, about Y—Y is opposite in effect to the moment of W2. Since
the directions of W; and W, are both
down, the difference in moment effect
is caused by W, and W, being located
on opposite sides of the axis. Or if 2
is considered to be positive in terms of
coordinates with respect to the X—-Y
axes, 22 must be considered to be neg-
ative. Proceeding to replace weight in
terms of area, we may eliminate the
items y and ¢ from the product W =
ytA, whence we observe that the sign Fic. 7-10.
of the moment of area similarly de-
pends on the position of the centroid of the area relative to the coordinate
axes.
In the case of a plate with a hole cut in it as in Fig. 7-11, the resultant
weight W may be considered equivalent to the weight W, of the solid plate
minus the weight W» of the cut-out portion. In this instance, the direc-
tions of W; and W: are opposite, so their moments with respect to the

Fie, 7-11.
178 CENTROIDS AND CENTERS OF GRAVITY [Chap. VII

Tasie VII-1. CrnrROIDS FOR COMMON GEOMETRIC SHAPES

Y|
fe eel

Segment
of arc
Art. 7-6] Centroids of Composite Figures 179
TABLE VII-2. Properties or ANGLES AND CHANNELS

Size Area g y

Units Sq In

5 X 3 X $ angle 3.75 is OM75


6 X 4 X 1 angle 9.00 2.17 117) ——_
6 X 6 X $ angle 5.75 1.68 1.68 Web Thick-
8 X 6 X 1 angle 13.00 2.65 1.65 ness¢

10’’-15.3 lb channel 4.47 0.64 0 0.240


12’’-20.7 lb channel 6.03 0.70 0 0.280

Y axis are of opposite sign. If, as before, these weights are expressed in
terms of equivalent areas, the moment effect of the area of the cut-out is
opposite to the moment effect of the area of the original plate, even though
the centroid of each area is now on the same side of the axis.
Hence the sign of the moment of an area, expressed as ax for example,
depends on the signs of a and of z, the positive sign for area being associated
with area that adds to the net area of the figure, and the negative sign to
area that reduces the net area. More- Y
over, the sign of the coordinate of the |
centroid of an area may be plus or
minus, depending on the location of
the centroid with respect to the axis
of moments.

ILLUSTRATIVE PROBLEMS

711. A slender homogeneous wire of


uniform cross section is bent into the
form shown in Fig. 7-12. Determine the
position of the centroid of the wire with
respect to the given axes.
180° CENTERS OF GRAVITY
CENTROIDS ANDDe ee [Chap. VII
ih a elegy aE
Solution: The wire is considered equivalent to two line segments, one a semicircle
of 4-in. radius and the other a straight line 8 in. long. In Prob. 701 the centroid of
: 2r ;
the semicircular line was shown to be located a distance = — from the Y axis. By
a
applying Eq. (7-2) and paying careful attention to signs, we obtain

Pla = 22h r+ ae-n(-2X 4 48x

20.56 = —32 + 32 =0
oe = 0)
[Ly = ly] (4 + 8)9 = (47) X 0 + (8)(—4)
20.56 7 = —32
gq = —1.558 in.

The centroid of the wire lies on the Y axis 1.558 in. below the X axis. Ans.

712. Determine the position of the centroid of the shaded area shown in Fig.
7-13.
Solution: As indicated by the dashed lines, the net shaded area may be considered
as the sum of a quarter circle of 5 in. radius, a rectangle 5 in. by 4 in., and a rectangle
6 in. by 9 in., minus a triangle 7.5 in. by 9in. The location of the centroid for each
of these subdivisions is known from Table VII-1. It is immaterial that the tri-
angle cuts into both rectangles — the important concept is to subdivide the given
figure into such of the geometric shapes
listed in Table VII-1 that their net sum
equals that of the given figure.
Select reference axes as shown. When
there are a number of elements to
be considered, it is convenient to apply
Eq. (7-1) in the form of the tabulation
shown below. This simplifies the sum-
mations of the various terms. Before
the quantities are entered in this tabu-
lation, the proper sign should be put
before each term. An area is consid-
ered positive or negative according to
whether it adds to or subtracts from the
net area of the figure. The signs of
the coordinates of the centroids follow
the usual mathematical convention; i.e., x coordinates are positive to the right of
the Y axis and y coordinates are positive above the X axis.
For a specific example of how the tabulation is made, consider the triangle. Its
area is ¢ X 9 X 7.5 = 33.75 sq in. which is entered as a negative value because it
reduces the net area of the figure. Its centroid is 4+ X 7.5 = 2.5 in. from its left
edge and 3 X 9 = 3 in. above its base. Hence with respect to the selected reference
axes, its x coordinate is 6 — 2.5 = 3.5in. to the left of the Y axis and therefore nega-
Art. 7-6] Centroids of Composite Figures 181
tive, while its y coordinate is 4 — 3 = 1 in. below the X axis and hence also negative.
Its moments of area, ax and ay, are both positive because they are the product of
two negative quantities. The student is urged to pay particular attention to the
signs because incorrect signs are the greatest source of error.

CompuTATION OF RESULTS

Area (in.?) | 2 (in.) ax (in.*) y (in.) ay (in.*)

Quarter circle +19.65 +2.12 + 41.7 +2.12 +41.7


Rectangle 5 X 4 +20.00 +2.5 + 50.0 —2 —40.0
Rectangle 6 X 9 +54.00 —3 — 162.0 +0.5 +27.0
Triangle — 33.75 =e) |) eI —1 +33.8

Totals +59.90 + 47.9 + 62.5

Taking the sums from the tabulation, Eq. (7-1a) gives the following results:

E=
ae
Zax
B=
iy
GG
47.9 =
0.80 in.
}
;

_ _ zay bp lab2c5 : tee


E = A MSS 59.9 == OES |

713. Determine the moment of area Y


about the Y axis of the shaded area in
Fig. 7-14 contained between the line
OA and the curve y? = kz’.
Solution: This problem may be solved
by direct integration, but a simpler solu-
tion is to use the method of composite
parts. The shaded area may be resolved
into the triangle OAB from which is sub-
tracted the area under the curve y? = kz’.
The equation of the curve may be rewrit-
ten as y = Vk «? from which it is evident
that its area is that of the spandrel listed
in Table VII-1 with n = % and equal to

:.: zh = peril! = =bh. Its centroid is —5) = a = 50 = = from AB

and hence at 2 6 from the Y axis.


Applying Eq. (7-1) which states that the moment of area is equal to the sum of the
moments of area of its parts, we obtain
[Az = Laz] Az = (2 X 18 X 21)(% X 21) — (F X 21 X 18)(F X 21)
Az = 2646 — 2268 = 378 in? Ans.
182 CENTROIDS AND CENTERS OF GRAVITY [Chap. VII

PROBLEMS
714. The dimensions of the T-section of a cast-iron beam are shown in Fig.
P-714. How far is the centroid of the area above the base? Ans. gy = 3.07 in.

ae ‘

\S
WEL.
ey ee
Fig. P-714. Fie. P-715.

715. Determine the coordinates of the centroid of the area shown in Fig. P-715
with respect to the given axes.
716. Aslender homogeneous wire of uniform cross section is bent into the shape
shown in Fig. P-716. Determine the coordinates of its centroid.
Anse te—we34 ins 1) sch one

Fig. P-716. Fia. P-717.


he 717. Locate the centroid of the bent
wire shown in Fig. P-717. The wire is
| homogeneous and of uniform cross section.

718. Locate the centroid of the shaded


area shown in Fig. P-718.
719. Determine the centroid of the
lines that form the boundary of the shaded
area in Fig. P-718.
720. The centroid of the shaded area in
x Fig. P-720 is required to lie on the Y axis.
6" 6! Determine the distance 6 that will fulfill
Fic. P-718 and P-719. this requirement. ANS, (0 = Osim,
Art. 7-6] Centroids of Composite Figures 183

721. Refer to the T-section of Prob.


714 shown in Fig. P-714. To what value
should the 6-in. width of the flange be
changed so that the centroid of the area
is 2.5 in. above the base? 4”
=
for)

722. Locate the centroid of the shaded xX


area in Fig. P-722 created by cutting a 8" b’ yee
semicircle of diameter r from a quarter
circle of radius r. hi en
Ans. & = 0.6367; 9 = 0.348 r

x ¥i
Ol. 45% ol
Fig. P-723.

723. Locate the centroid of the shaded


| IK4" area in Fig. P-723.
aoe NI i 724. Find the coordinates of the centroid
\ ig \N\ S of the shaded area shown in Fig. P-724.
Ans. £= 7.73 in.» 9 = 5.07 in.
12" SS 725. Repeat Prob. 239 on page 35.
Avoid integration by using Table VII-1.
726. Locate the centroid of the shaded
N x area enclosed by the curve y? = az and
oO F the straight line shownin Fig. P-726. Hint:
a Observe that the curve y? = ax relative
Fic. P-724.

Teas
12" i Pall 2 tee
Fia. P-726. Fig. P-727.
184 CENTROIDS AND CENTERSierOF GRAVITY [Chap. VII
ciel i SERPS rey Foes i
to the Y axis is of the same form as y = ka? with respect to the X axis.
Ans. = 48 ft; 9 = 3 ft

727. Locate the centroid of the culvert shown in Fig. P-727. Hint: Integration
is unnecessary if the area is subdivided into the elements to be found in Table
VII-1. Ans. ¥ = 3.46 ft

728. Without using integration, compute 7 for the area bounded by the X axis,
2 3

the line « = 12, and the curve y = 4a ae — re Anse —"eoo

729. A rectangle is divided into two parts by the curve y = kx” as shown in
Fig. P-729. Using the known location of the centroid of the lower part A as given in
Table VII-1, show that the centroid of the upper part B is located by zg = $24
and jp = 2 Ya.

Fig. P-729. Fic. P-730.

730. A beam has the cross section shown in Fig. P-730. Compute the moment
of area of the shaded portion about the horizontal centroidal axis X, of the entire
section. (Note: It is shown in strength of materials that this result is used in com-
puting the maximum shearing stress.) IMDS T= Dela? OO = Lia

731. Two 10 in.-15.3 lb channels are welded together


as shown in Fig. P-731. Find the moment of area of
the upper channel about the horizontal centroidal axis
X, of the entire section.

732. A chord in a bridge truss is composed of the ele-


ments shown in Fig. P-732. Refer to Table VII-2 for the
properties of the angles and locate the centroid of the
built-up section. Ans. ¥Y = 10.54 in. from base

733. Locate the centroid of the built-up section


shown in Fig. P-733. Refer to Table VII-2 for the
properties of the elements.
Fic. P-731. Ans. Y = 9.45 in. from base
Art. 7-7] Theorems of Pappus 185

18"x 1"

". Wt y"
is 77 Ww 1”
5 x3 X5 5x3 Xo

" “ 1!
5x 3X5
% py" F "
6'«6’x1 6x65

12"- 20.71b Channel


Brembo 732: Fig. P-733.

7-7. Theorems of Pappus!


Pappus developed two simple theorems for determining the surface area
or the volume generated by revolving a plane curve or a plane area about'a
non-intersecting axis lying in the plane. The first theorem states that the
surface area is the product of the
length of the generating curve mul- Y
tiplied by the distance traveled by (eee ee A
its centroid. Thus let the curve /’ af
AB of length L in Fig. 7-15 be re- t a ers ¥
volved about OY through an angle a
of @ radians equal to or less !
than 2 7 radians. The differential / pnt)
length dL sweeps through the dis- or Some gree ae B
tance #6 generating a hoop whose Oe a ic
area is@z-dL. The total area gen- TS waren be .
erated by AB is the area of all ee Oe
such hoops or
A= f0xs- dl =0fx%7dL = 0xL (7-3)
If the generating line L is composed of several segments, the centroid ¢ of
that line need not be found since the product LZ is equivalent to and may
be replaced by the sum of the moments of length (i.e., 2lx) of those seg-
ments.
When the centroid of an area is known, the volume generated by rotating
the area about a non-intersecting axis can be found by applying the second
1 According to W. W. Rouse Ball’s A Short History of Mathematics, the theorems were
discovered by the Greek geometrician Pappus about a.p. 300 and restated by the Swiss
mathematician Paul Guldin (1577-1643) whose name is sometimes also associated with
the theorems.
186 CENTROIDS AND CENTERS OF GRAVITY [Chap. VI

theorem. This theorem states that the volume is the product of the area
of the figure multiplied by the length of the path described by the centroid
of the area.

O
Fic. 7-16. — Second Theorem of Pappus.

Let the area A in Fig. 7-16 be rotated about the axis OY through an
angle of @ radians equal to or less than 2 7 radians. The differential area
dA sweeps through the distance 6x generating a ring whose volume is
6x-dA. Therefore the total volume is
V=f0x-dA=0fx"dA = 0xA (7-4)
If the generating area A is composed of several parts, the centroid ¢ of that
area need not be found since the product AZ is equivalent to the sum of the
moments of area (i.e., Zax) of the several parts.
ILLUSTRATIVE PROBLEMS
734. Compute the surface area of the cone generated by revolving the line in
Fig. 7-17 about the Y axis.
Solution: Two cones are generated by the line:
one by the 4-in. length above A and the other by
the 6-in. length below A. The x coordinate of the
centroid for each segment of the line is given by
& = 32 sin 30°; hence # = 1 im. and # = 1.5 in.
Applying Eq. (7-3), we obtain
[A = 2 rzL] 4-in. segment: 41 =27 X 1X4
= 25.1 in?
6-in. segment: Az = 27 X 1.5 x
6 = 56.5 in.?
Total surface area = A; + Ay =
25.1-+ 56.5 =
81.6in.2 Ans.
D 735. The shaded area in Fig. 7-18 is composed of
Fig. 7-17. a second degree parabola and a semicircle. Deter-
Art. 7-7] Theorems of Pappus 187
eee ee eee ee ee
mine the volume generated by rotating the area through one revolution about
the Y, axis.

Y,

Fia. 7-18.

Solution: Using Table VII-1, the x coordinate of the centroid for each part of the
total area is
aol
Il 2+72X8=8in.
fo = 10 + 0.424 X 3 = 11.27 in.
Applying Eq. (7-4) we now find the volume to be

[V =0- Az = 62az] V=2 | AX) (8) + (22) a120 |


V = 2 7[128 + 159] = 1805in.* Ans.

PROBLEMS

736. A right triangle of sides 6 and h is rotated about an axis coinciding with
side / to generate a right circular cone. Compute the volume.
Ans. V = 3 1b
737. Derive the expressions for the surface area and volume generated by
rotating a semicircle of radius 7 about its diameter. Ans. A=4a9r;V =47r

738. Show that the volume of a paraboloid of revolution is equal to one-half the
circumscribing cylinder.
739. Determine the volume of an ellipsoid of revolution generated by rotating
an ellipse about (a) its major axis (prolate ellipsoid) and (b) its minor axis (oblate
ellipsoid). Take the larger semi-axis as a and the smaller semi-axis as 6.
Ans. (a) V = $ mab’; (b) V = § ra’

740. A circle of radius r lies in the XY plane with its center at a distance a above
the X axis. Revolving it about the X axis will generate a doughnut-shaped ring
called a torus, provided a is greater thanr. Compute the surface area and volume of
the torus.
188 CENTROIDS AND CENTERS OF GRAVITY [Chap. VII
eee
ee ete tn Le ee ee eS

741. A 60° pipe elbow has an internal diameter of 4in. The radius of curvature
of the pipe’s centerline is 6 in. Find the internal volume of the elbow.
Ans. V = 78.9 in.’
742. Find the volume of the spherical wedge formed by rotating through an angle
of 45° a semicircle of radius r about its base diameter. What is its total surface area?
ae Oe
Ans. V = ae A= 5m

743. Compute the surface area and volume generated by rotating the area shown
in Fig. P-743 through one revolution about the X axis.
Ans. A = 1002 in.2; V = 1668 in.?

Fig. P-743. Fig. P-744.

744. The rim ofa pulley has the cross section shown in Fig. P-744. If the rim is
made of steel weighing 490 lb per cu ft, determine the weight of the rim.
Ans. W = 238 lb
745. The area contained between two concentric semicircles of radii 1.5 in. and
3 in. is rotated about an axis 4 in. away and parallel to the base diameters of the
semicircles. Compute the surface area and volume generated by a complete revolu-
tion. GS, Al = HIS na? 7 = BOSH ams
746. Determine the surface area and volume generated by a complete revolution
about the X axis of the shaded area of Prob. 718 on page 182.
747. Compute the surface area and volume generated by a complete revolution
about the X axis of the shaded area of Prob. 723 on page 183.

7-8. Center of Gravity of Bodies. Centroids of Volumes


The principle of moments, by which the centroids of areas and lines
was determined in the preceding articles, is readily extended to locate the
center of gravity of bodies or the centroids of volumes. Thus consider the
Art. 7-8] _ Center of Gravity of Bodies. Centroids of Volumes
a a a ees ee Are ee ee ee MR ey ee 189
EY
body shown in Fig. 7-19. The total weight IW of the body is the sum of its
differential elements dW or W = fdW. Also, the moment of the resultant

Fig. 7-19.

weight W is equal to the moment sum of its elements. Hence with respect
to the Y and X axes respectively, we obtain

Wi = frdw (a)
and
Wy = Sy dw (b)

By rotating the body and its reference frame through 90° (either about
the X or Y axis) so that the Z axis becomes horizontal, a moment sum-
mation about the Z axis gives

Wz = fzdW (c)
If the body in Fig. 7-19 is composed of the same material throughout, its
weight W is the product of its density y (.e., weight per unit volume)
multiplied by its volume V. Similarly the weight dW of an elemental
prism is the product of the density y and its elemental volume dV. Sub-
stituting these values in Eq. (a) results in

[W-t= fxdw] y¥-V-£=ySfzrdV

whence, by canceling 7, we obtain

V-g= fray (d)


and similarly,
Va Say (e)
Vee z= f2dVv (f)
190 CENTROIDS AND CENTERS OF GRAVITY [Chap. VIE

Note that Eqs. (d), (e), and (f) do not apply to bodies composed of more
than one material, or where the material varies in density throughout the
body.
When dealing with composite bodies or volumes, a process of finite
summation, similar to that discussed in Art. 7-6, is used. The above equa-
tions then become =
Wx = <wx)
Wy = <wy (7-5)
Wz = <wz
and
oe = OKs
Vy = Xvy (7-6)
Vz = vz

If a body has a plane of symmetry, the center of gravity lies in that


plane. If it has two planes of symmetry, the center of gravity is on the
line of intersection of the planes. If it has three planes of symmetry, the
center of gravity is at the point of intersection of the three planes.

ILLUSTRATIVE PROBLEMS
748. Center of Gravity of the Volume of a Right Circular Cone. The radius of the
base of the right circular cone shown in Fig. 7-20 is denoted by r and its altitude
by h. The axis of the cone is taken as the X axis. Determine the location of the
centroid of the volume of the cone.
Y

ee
ie h

( x
!

SSN
/
Zz
Fig. 7-20.

Solution: From symmetry, 7 = 0and2z = 0. To determine f, select a thin plate


parallel to the base of the cone as the differential volume dV. The centroid of the
thin plate is at its center and at a distance x from the YZ plane, as shown in the
figure. Applying Eq. (d), we obtain
h
[Viet =" fadV| (4 mrh) - = = f amy? dx (a)
Art. 7-8] Center of Gravity of Bodies. Centroids of Volumes 191
r
From similar triangles, ar y= ae hence Eq. (a) becomes
_

749. The wooden block shown in Fig. 7-21 is 12 X 12 X 9in. A hole 4 in. in
diameter and 6 in. deep is drilled in it. The hole is filled by a metal pin weighing
50 lb. If wood weighs 40 lb per cu ft, locate the center of gravity of the body.

Fic. 7-21.

Solution: Assume the body to be composed of the following parts: (1) the full-
size block, less (2) a wooden pin 4 in. in diameter and 6 in. long, plus (3) the metal
pin which replaces the wooden pin. The weights of the wooden block and pin are

40)
[W = yAL] Block: Wi = 1728 xX 12 X 12 X 9 = 301b

. 40 mw X (4)?
Pin: ; We = 1728 x 4 x 6 = 1,7575 lb

The following tabulation is now made:

Item W ti Wx y Wy Z Wz

Wood block 30 6 180 4.5 135 6 180


Wood pin —1.75 4 —7 3 —5.25 9 | —15.75
Steel pin 50 4 200 3 150 9 450

Totals 78.25 373 279.75 614.25


192 CENTROIDS AND CENTERS OF GRAVITY [Chap. VII

From the sums in the table, we obtain

re We phe
= ay,
E = 77 + = 78.25 os
| = Sa
SWy U =
_ 279.75
E95
_ al
ae
We _ 614.25
E55 | ae = ape = re 77.85
PROBLEMS
750. Determine the centroid of the surface of a right circular cone of altitude h.
Ans. %h from apex on the axis of the cone
751. Determine the centroid of a hemisphere of radius r, taking the axis of
symmetry as the Z axis. Ans. 2 = $r from the base
752. A uniform wire is bent into the shape shown in Fig. P-752. The straight
segments lie in the X-Z plane, and the 8-in. length makes an angle of 30° with the
X axis. The semicircular segment isin the X-Y plane. Locate the center of gravity
of the wire. Nis, eh = ye 1s O) = Oe eS eS i,

>=

Fic. P-752. Fic. P-753.

753. A thin plate of sheet metal is composed of two rectangles and a triangle
bent as shown in Fig. P-753. A hole whose area is 12 sq in. has been cut in the plate.
Determine the coordinates of the center of gravity.
754. Locate the center of gravity of a steel rivet having a cylindrical body 1 in. in
diameter and 2 in. long with a hemispherical head of 1 in. radius. Use the result of
Prob. 751.
755. <A body consists of a right circular cone whose base is 12 in. and whose
altitude is 16 in. A hole 8 in. in diameter and 4 in. deep has been drilled from the
base. The axis of the hole coincides with the axis of the cone. Locate the centroid
of the net volume. Use the result of Prob. 748. Ans. 5 in. from base
756. Determine the height 4 of the cylinder mounted on the hemispherical base
shown in Fig. P-756 so that the composite body will be in stable equilibrium on its
Summary 193
base. Hint: As long as the center of gravity does not lie above the XX plane,
there will exist a restoring couple when the body is tipped. Ans. h = 1.414 ft
757. Repeat Prob. 756 if the cylindrical por-
tion of the body in Fig. P-756 is replaced by a
right conical portion with a 2-ft radius base and
altitude h. Ans. h = 3.464 ft

any:
a
Fie. P-756 and P-757. Fic. P-758.

758. A steel ball is mounted on top of a timber cylinder as shown in Fig. P-758.
Steel weighs 490 lb per cu ft and timber weighs 100 lb per cu ft. Determine the
position of the center of gravity.

SUMMARY

The center of gravity of a body is the point through which the line of
action of the weight always passes. The term centroid is used when refer-
ring to the center of gravity of a weightless figure such as a line, an area,
or a volume. Centroids and centers of gravity are determined from the
following equations:
Line J SAL = Xlx|
(7-2)
Ly = fydL zly J
At = fxdA ea
(7-1)
Ay = fydA Lay
Vt = fxdV Lux
Vg = fydV LXvy ; (7-6)
Vz = fzdV Lvz
Wi = fxdW Lwx
Wy = fy dW xwy 7 (7-5)
Wz= fzdw Lwz
194 CENTROIDS AND CENTERS OF GRAVITY [Chap. VII

In applying these equations to composite figures (Arts. 7-6 and 7-8),


the figure is resolved into its component geometric parts and Tables VII-1
and VII-2 are used to locate the centroids of these parts.
Particular attention is called to the form AZ, which is called moment of
area. When the moment of area is desired, as in many problems in dy-
namics and strength of materials, it is found as the sum of the moments of
area of the parts composing the figure. ;
The theorems of Pappus (Art. 7-7) provide a means of determining the
surface area generated by revolving a line about a non-intersecting axis,
and also the volume generated by rotating an area about a non-intersecting
axis of revolution. The theorems are expressed by the following equations:
A = 0xL = 621x (7-3)
V = 0xA = 0Zax (7-4)
in which the angle 6 is equal to or less than 2 7 radians.
Chapter VIII.
Moments of Inertia

8-1. Definition of Moment of Inertia


Many engineering formulas, such as those relating to strength of beams,
columns, deflection of beams, involve the use of a mathematical expression
of the form fp? dA, where p is the perpendicular distance from dA to the
axis of inertia. This integral appears so frequently that it has been named
moment of inertia.!_ Moment of inertia applied to areas has no real meaning
when examined by itself; it is merely a mathematical expression usually
denoted by the symbol J. How-
ever, when used in combination
with other terms, as in the flexure
formula for beam stresses, S =
* it begins to have significance.
The mathematical definition of
moment of inertia, J = fp? dA,
indicates that an area is divided
into small parts such as dA, and
each area is multiplied by the
square of its moment arm about
the reference axis. Thus, asshown Fic. 8-1.
in Fig. 8-1, if the coordinates of
the center of the differential area dA are (x, y), the moment of inertia
about the X axis is the summation of the product of each area dA by the
square of its moment arm y. This gives
I,= fy dA (8-1)
Similarly, the moment of inertia about the Y axis is given by
L= {x da (8-2)
1The term moment of inertia is derived as follows: Force is related to the mass (..e.,
inertia) of a body and its acceleration by the equation F = Ma. The equation relating
applied forces to the angular acceleration a of rotating bodies is -d = [|fp?dM]-a.
If the first equation is stated as force equals inertia times acceleration, then by analogy
the second equation may be stated as moment of force equals moment of inertia times
acceleration. By comparison of the above statements, the expression fp? dM is termed
moment of inertia. Similarly, for areas, the expression fp? dA is known as the moment
of inertia.
195
196 Moments oF INERTIA [Chap. VIII

The moment of inertia (of area) is sometimes called the second moment
of area because each differential area multiplied by its moment arm gives
the moment of area; when multiplied a second time by its moment arm it
gives the moment of inertia. The term second moment of area is preferable
to the expression moment of inertia; the latter is confusing when applied
to an area having no inertia. The term moment of inertia, however, is
long established and is not likely to be superseded by the other.
Units and Signs. Examination of the integral fp? dA shows it to be a
fourth-dimensional term because it is composed of a distance squared
multiplied by an area. Thus if L is the unit of distance, the unit of J is
(L)4. A convenient unit of D is inches or feet; this gives quartic inches
(in.*) or quartic feet (ft*) as the dimensional unit of J.
The sign of J is obviously independent of the sign of the moment arm p
(since if p is minus, squaring it makes it plus); it depends entirely on the
sign of the area. We shall define a positive area as one which adds to the
area of a figure, and a negative area as one which reduces the area of
the figure. For a net area, the moment of inertia must always be positive.

8-2. Polar Moment of Inertia


The moment of inertia for an area relative to a line or axis perpendicular
to the plane of the area is called the polar moment of inertia and is denoted

Ps Fia. 8-2.

by the symbol J. In Fig. 8-2 the moment of inertia of an area in the XY


plane with respect to the Z axis is
= fp? dA] Jz= frdA = f(2?+ y*)dA
= fve’daA+ fydA
whence from Eqs. (8-1) and (8-2) we finally obtain
L=L+1 (8-3)
Art. 8-4] Transfer Formula for Moment of Inertia 197

Expressed in words, this equation states that the polar moment of in-
ertia for an area with respect to an axis perpendicular to its plane is equal
to the sum of the moments of inertia about any two mutually perpendicular
axes in its plane which intersect on the polar axis.

8-3. Radius of Gyration


The term radius of gyration is used to describe another mathematical
expression and appears most frequently in column formulas. Radius of
gyration is usually denoted by the symbol k (sometimes by r) and is defined
by the relation
Be Ae or I= AR’ (8-4)

where I is the moment of inertia and A the cross-sectional area.


The following is a geometrical interpretation of this relation. Assume
the area of Fig. 8-1 to be squeezed into a long narrow strip as shown in

eure squeezed into a strip

k
x 5 SRE PRS EF cer x
Fic. 8-3. — Concept of radius of gyration.

Fig. 8-3. Each differential element of area dA will then be the same
distance k from the axis of inertia. The moment of inertia is given by
l=f{pdA=khfdA = Ar
because each differential element has the same moment arm. The strip
may be placed on either side of the reference axis, since if k is minus,
squaring it will automatically make it plus. Or part of the strip may be
at a distance k from one side of the reference axis and the remainder of the
strip at an equal distance k from the other side of the axis.
In view of this discussion, the radius of gyration is frequently considered
to be the uniform distance from the reference axis at which the entire area
may be assumed to be distributed. For an area whose dimensions per-
pendicular to a reference axis are negligibly small compared with its distance
from that axis, radius of gyration is practically equivalent to the centroidal
location of the area.

8-4. Transfer Formula for Moment of Inertia

It is often necessary to transfer the moment of inertia from one axis to


another parallel axis. The transfer formula affords a method of doing this
198 OF INERTIA
Moments aR [Chap. VIIT
iii an cet 5S Die tee ae
without further integration. For
example, in Fig. 8-4, the moment
of inertia with respect to a centroi-
dal X axis (X,) is given by the ex-
pression, = fy? dA. The moment
of inertia for the same area with re-
Fia. 8-4. — Moments of inertia between spect to a parallel axis (X) located
parallel axes. a distance d from the centroidal axis
is given by the equation

UT = fp? dA] I, = S(y+d)dA


= fydA+2dfydA+ @f dA (a)

The d is written outside the integral sign because it is a constant that


represents the distance separating the axes. The second of the right-hand
terms in Eq. (a) becomes zero because fy dA = A - %, where ¥ represents
the distance from the reference axis X, to the centroid. In this instance 7
has the value of zero because X, passes through the centroid. We obtain
finally,
i = ik ar Ad? (8-5)

Put into words, this equation states that for any area the moment of
inertia with respect to any axis in the plane of the area is equal to the
moment of inertia with respect to a parallel centroidal axis plus a transfer
term composed of the product of the area multiplied by the square of the
distance between the axes. Evidently the least moment of inertia for any
given direction of an axis is the centroidal moment of inertia. Note care-
fully that the centroidal axis involved in the transfer formula is always the
centroidal axis of the area used in the transfer term Ad?.
A similar relation exists between the radii of gyration with respect to
parallel axes, one of which is a centroidal axis. Replacing I, by Ak,2 and
I, by Ak,2 in Eq. (8-5), we obtain

Ak? = Ak? + Ad?


whence
ke ke ate Et (8-6)
In like fashion, for polar moments of inertia and polar radii of gyration
we obtain the following analogous relations between any axis and a parallel
centroidal axis:
J=J+Ad*
k i+ @
R= \ (8-7)
Art./8-5] Moments of Inertia by Integration 199
i
8-5. Moments of Inertia by Integration
In determining the moment of inertia by integration, it is desirable to
choose the differential area so that either
1. All parts of the differential area are at the same distance from the
reference axis,” or
2. The moment of inertia of the differential area with respect to the
reference axis is known. The moment of inertia of the area is then the
summation of the moments of inertia of its elements.
As in the case of centroids, the moment of inertia of a composite figure
may be found by combining the moments of inertia of its parts. When
the evaluation of J for a particular part is known, the transfer formula
(Art. 8-4) is used to refer the moments of inertia of the various parts of the
figure to a common reference axis.

ILLUSTRATIVE PROBLEMS
801. Determine the moment of inertia for a rectangle of base b and depth A with
respect to (a) a centroidal axis parallel to the base and (b) an axis coinciding with
the base.
----A

Fic. 8-5. Fig. 8-6.

Solution: Centroidal Axis. Select the differential element as shown in Fig. 8-5.
All parts of the element are at the same distance from the centroidal X, axis. Ap-
plying Eq. (8-1), we find the centroidal moment of inertia to be
Ale gahle bh’
E * Jv iA| te ine ei Raa E ioe ibe aka
= i = 2 = Y = -——_— A .

Axis Coinciding with the Base. The preceding result can be transferred through
the distance h/2 to the parallel base axis by applying the transfer formula as follows:
; bh’ iy bh’
{lf fas,
=I1+ Ad2 If:=—ig 7 (bh)\-])
@ (4 =—3 Ans.

2 When all parts of an element are at the same distance from an axis, this distance
is really the radius of gyration for the element. See Fig. 8-3.
200 Moments OF INERTIA [Chap. VIII

The moment of inertia of the parallelogram in Fig. 8-6 has the same values as for a
rectangle because the clemental strips composing the parallelogram have merely
shifted their position laterally from the dashed rectangle of corresponding dimen-
sions, but have not altered their distances from the corresponding axes of inertia.
802. Determine the moment of inertia for a triangle of base 6 and altitude h
with respect to (a) an axis coinciding with its base and (b) a centroidal axis parallel
to its base. y
Solution: Axis Coinciding with the Base. Select the differential element, as shown
in Fig. 8-7. From similar triangles, the length « = ;(h — y). The moment of in-

ertia with respect to the X axis is obtained from


h h

U2 = Sy’ dA] nef ady= fy Th yay

=F] ET fray -fv hy? dy dy ne Sere


bh
Ip = 75 An

|
| 2
gh
h | h
sah Eee
Y x
ghVEY, xX
ee, eee
Fic. 8-8.

Centroidal Axis. To obtain the centroidal moment of inertia /,, we transfer the
known value of J, from the base axis X to the parallel centroidal axis X,. Since the
transfer distance is h/3 as shown in Fig. 8-8, we obtain

fi ieee= meee Bhs


2d ae (“)(2)
OR BY _ bh
12 “y PAYING 36
803. Determine the moment of inertia of a circular area of radius r with respect
to a diametral axis.
Solution: Using polar coordinates, select the differential element, as shown in
Fig. 8-9. From the figure, y = p sin #6. The moment of inertia with respect to the
diameter is

U2 = Sy dA] re Pf p” sin? 6 p dé dp
0 0
r 2a rf 23 r4

{ ptdp-sint@ a9 =" f sin? 6d0 = —-r


0 Jo 4/0 4
ar
|)
LaelI
2=— Ans.
4
Art. 8-5] Moments of Inertia by Integration 201

Fic. 8-9. Fig. 8-10.

An alternate and simpler solution is to use Fig. 8-10 in which the differential ele-
ment is taken as the shaded ring of area dA = (2 rp)(dp). The polar moment of
inertia is
2 us 4
J = Sp dA) ll Spears
0
The rectangular moments of inertia J, and J, are obviously equal because of sym-
metry so that applying Eq. (8-3) yields

PROBLEMS
804. Determine the moment of inertia of a triangle of base b and altitude h
with respect to an axis through the apex parallel to the base. Use the transfer
bhi
formula and the results of Illus. Prob. 802. Ans. I= ue

805. Determine the moment of inertia of the quarter circle shown in Fig. P-805
; ae
with respect to the given axes. Ans. I, =I, = 76

Fic. P-805. Fia. P-806.


202 Moments oF INERTIA [Chap. VIII

806. Determine the moment of inertia of the semicircle shown in Fig. P-806 with
= ar’
respect to the given axes. Ans Tae I= Z

807. Show that the moment of inertia of a semicircle of radius r is 0.11 r* with
respect to a centroidal axis parallel to the diameter.
808. Determine the moment of inertia for the quarter circle shown in Fig. P-805
with respect to a centroidal X axis. Ans. I, = 0.055 r4
809. Determine the moment of inertia with respect to the X axis for the area
2 a2
enclosed by the ellipse whose equation is = + _ =1. Also determine the radius
a
: = bir b
of gyration. INS, dln = 4 jhe = 35

810. Determine the moment of inertia and the radius of gyration, with respect
to the Y axis, of the area cut from the first quadrant by the curve y = 4 — 2?
where x and y are in inches.
i Ans, “1, = {6 ms; ky == mm.
Bt 811. Determine the moment of inertia
with respect to the X axis of the shaded
b parabolic area shown in Fig. P-811.
Ans. I, = 7% ab?
812. Determine /, for the shaded para-
bolic area of Fig. P-811.
Fic. P-811 and P-812. Ans. Iy = Fad

8-6. Moments of Inertia for Composite Areas


When a composite area can be divided into geometric elements (rec-
tangles, triangles, etc.) for which the moments of inertia are known, the
moment of inertia for the composite area is the sum of the moments of
inertia for the separate elements. Before the moments of inertia of the
elements can be added, however, they must all be found with respect to
the same axis.
In the problems which follow, the values of the moments of inertia for
geometric elements can be taken from the results of the problems in Art.
8-5; these are summarized in Table VIII-1. When the geometric shape is
the cross section of a structural element such as an angle or channel, the
values in Table VIII-2 can be used.

ILLUSTRATIVE PROBLEM
813. Determine the moments of inertia with respect to the centroidal X and Y
axes of the wide flange beam section shown in Fig. 8-11.
Solution: The moment of inertia of a composite area is the sum of the moments of
inertia of the various parts of the area, all the moments of inertia being referred to
Art. 8-6] Moments of Inertia for Composite Areas 203
TaBLe VIII-1. Moments or INERTIA FOR GEOMETRIC SHAPES

Shape Moment of Inertia Radius of Gyration


Rectangle

> _ bh
Ihe = 42
7 ecules
ke /12

Ihe — bh kz = ae
3 VJ/3

7.
12> 36
bi pee
Vi18
‘e = a kz = ios
12 /6

= art aprel
i= Fi ie = 9

Ji mrt
9 ke a
va

= ar4 : r
ssi elias ke = ky = 5
I, = 0.1174 ke, = 0.264r
eee
Quarter circle

i |. ur , 1 oeys
Te = 1y= 76 eats
_ Xo 1, = I, = 0.055r4 ke = ky = 0.264r
Lier, €
Lr
Ellipse
7 mab? k pagh
Mes = 4 2 5

= aha? Fen]
l, = oe ky = 5
204 Moments oF INERTIA [Chap. VIII

TABLE VIII-2. PRopERTIES OF STRUCTURAL SECTIONS

Angle Channel

Area E, i -
Size Por I, Iy Ly 7
Section

in in.! in.* in. In.

5x3x4 3.75 2.6 9.5 1.75 0.75


Ee ee 4.75 6.3 17.4 1.99 0.99
a 18x4x1 11.00 11.6 69.6 3.05 1.05
= Vso 13.00 38.8 80.8 2.65 1.65
S— | 10in.-15.3 lb 4.47 66.9 2.3 0.64 0
& | 12in-20.7 Ib 6.03 128.1 3.9 0.70 0

the same axis of inertia before the addition is


made.
With respect to the X, axis, the simplest subdi-
vision of the given area is to resolve it into a large
rectangle 8 X 12 in. from which two smaller rec-
tangles, each 3.5 in. X 10 in., are subtracted. The
centroidal axis for each of these parts coincides
° with the X, axis of the figure; hence the transfer
formula need not be used. Using the result listed
in Table VIII-1, we therefore obtain
3 3
1. = oe 8 X 12 rectangle:

7 8 X (12)
12 = iS tan,
in.4
Fig. 8-11.

= ie 3
Two 3.5 X 10 rectangles: J, = 2 E Beet) |= 583 in.
12

Hence for shaded area: I, = 1152 — 583 = 569in.4 Ans.

With respect to the Y, axis, assume the figure to be composed of a 1 X 10 in.


rectangle and two 1 X 8 in. rectangles. The Y, axis is also the centroidal axis for
Art. 8-6] Moments of Inertia for Composite Areas 205

each of these rectangles so that this subdivision of the area again eliminates the need
for using the transfer formula. Referring again to Table VIII-1, we have
- _ hb
[7 = | 7 _ 10
Xx (2)
12 1 X 10 in. rectangle: ic nae =
0.833 in.
} 4

é ; 1 : ‘
Two 1 X 8 in. rectangles: Jy = 2 x#) = 85.333 in.4

Net area: I, = 0.833 + 85.333 = 86.166in.4 Ans.


Such numerical accuracy is unnecessary; the answer would usually be given as
86.2 in.4
814. Compute the moment of inertia for the composite area shown in Fig. 8-12
with respect to the indicated X axis.
Solution: The area is composed of a semicircle (S) whose radius is 3 in.,a6 X Qin.
rectangle (R), anda 4 X Qin. triangle (7). With respect to the X axis, the moment
of inertia for the area is the sum of the
moments of inertia of these elements,
each moment of inertia being referred to
the X axis before addition:
I; =Ir+Is+Ir (a)
Expressing the moment of inertia of
each element in terms of its centroidal
moment of inertia plus a transfer term,
we obtain
Ip = Ir + (Ad’)r
Iz =I + (Ad?)s
Ip =I7 + (Ad@’)r
Adding the left- and right-hand mem-
bers of these equations results in
I, = 21+ 2Ae (b)
Equation (6) indicates that the moment of inertia of a composite figure is the sum-
mation of the centroidal moments of inertia of the elements plus the summation of
the transfer terms for these elements. This equation is readily adapted to tabular
computation, as shown below. From Table VIII-1, the values of / for each of these
= ii = bh? rere: :
elements are Jp = 72’ Is = 0.11 r4, and [7 = —~- The transfer distances are in-
36
dicated on Fig. 8-12.

Item Ha Area d a? Ad

Rectangle 365 | 4 DED 6.25 BOUAD


Semicircle 8.91 14.14 Bod 10.7 Toles
Triangle 81 18 4.0 16 288

Totals 454.91 776.8


206 Moments OF INERTIA [Chap. VIII

Taking the summations from the table and sub-


stituting in Eq. (b), we obtain
I, = DI + DAd? T, = 454.91 + 776.8
Ih == BY Alas.

815. A girder is composed of four 6 X 6 X % in.


angles connected to a web plate 23.5 in. X 1 in.,
plus two flange plates each 18 in. X 1 in. as shown
in Fig. 8-13. The properties of the angle are
I, =I, = 19.9 in.4, the area = 5.75 in, and
& = 7 = 1.68 in. Compute the moment of inertia
with respect to the centroidal X, axis.
Solution: The tabular computation used in Prob.
814 is well suited for cases in which there are many
elements. In the present problem, the elements
are symmetrically placed so that a table is hardly
Fie. 8-18. Nos
justified. "
Direct application of the transfer formula J = J + Ad? to each element gives
ib 23:5)° t
For web plate: i eae + (23.5 X 1)(0)? = 1080 in.4

1 IDye :
For two flange plates: I = 2 kzecn + (18 x 1012.5" |-5620 in.4

For four angles: I =4[19.9 + 5.75(10.32)?] = 2530 in.4


For entire figure: I, = 1080 + 5620 + 2530 = 9230in.4 Ans.

PROBLEMS

816. Arectangleis3in. by 6in. Determine the polar moment of inertia and the
radius of gyration with respect to a polar axis through one corner.
INOS. di = HU hag? (hy, = BERS ii,
817. Determine the moment of inertia and radius of gyration with respect to a
polar centroidal axis of the cross section of a hollow tube whose outside diameter is
6 in. and inside diameter is 4 in.
818. A hollow square cross section consists of an 8 in. by 8 in. square from which
is subtracted a concentrically placed square 4 in. by 4in. Find the polar moment of
inertia and the polar radius of gyration with respect to a Z axis passing through one
of the outside corners. Ans. J = 2176 in.*; & = 6.74 in.
819. Determine the moment of inertia of the T-section shown in Fig. P-819 with
respect to its centroidal X, axis. Ans. 9 = 3.5 in.; I, = 291 in.!
820. Determine the moment of inertia of the area shown in Fig. P-820 with
respect to its centroidal axes. Anse) 9 = 5.7 mgd g)= 856 in? J) 163 in
821. Find the moment of inertia about the indicated X axis for the shaded area
shown in Fig. P-821. Ans. I, = 908 in.
Art. 8-6] Moments of Inertia for Composite Areas 207

——_—_
3 —_+
Fic. P-819.

os

10”

__yXo

1g” 3) 7
| X
Fic. P-821. Fic. P-822.

822. Find the centroidal moments of inertia of the trapezoid shown in Fig.
P-822.
823. An equilateral triangle has its base b horizontal. Show that the centroidal
moments of inertia with respect to horizontal and vertical axes are equal.
824. Compute the moment of inertia with respect to an axis passing through
two opposite apexes of a regular hexagon
of side a.

825. Compute the moment of inertia


of the 10 in. by 15 in. rectangle shown in
Fig. P-825 about the X axis to which it
is inclined at an angle 9 = sin-! 4. Hint:
Resolve the figure into parts A, B, and C.
Ans. I, = 3600 in.*

826. The cross section shown in Fig.


Fic. P-825. P-826 is that of a structural member known
208 Moments oF INERTIA [Chap. VIII

as a Z section. Determine the values of I, and Iy.


Ans. I, = 42.1 in.4; I, = 15.4 in.4; Area = 8.63 in.
827. The built-up section shown in Fig. P-827 is composed of two 8 X 6 X Jin.

8x 6x 1"

|g1”

Fic. P-826. Fig. P-827.

angles riveted to a 12 X 1 in. web plate. Determine the moment of inertia with
respect to the centroidal X axis.
828. Two 12-in. 20.7-lb channels are latticed together to form the section shown
in Fig. P-828. Determine how far apart the channels should be placed so as to
make /, equal to /, for the section. (Neglect the lattice bars which are indicated by
the dashed lines.) Ans. d = 7.69 in.

0G

zl Es
d

Fic. P-828. Fig. P-829.

829. Determine the distance d at which the two 3 in. by 8 in. rectangles shown in
Fig. P-829 should be spaced so that J, = I,.
830. The short legs of four 6 by 4 by 3 in. angles are connected to a web plate
235 in. by Tein. to form the plate and angle girder shown in Fig. P-830. Compute
the value of /,.
831. A plate and angle column is composed of four 8 by 4 by 1 in. angles with the
short legs connected to a web plate 14 in. by 1 in. plus two flange plates each 18 in.
by 24 in. as shown in Fig. P-831. Determine the values of /, and 1,.
Ans. JI, = 7682 10.4; 7, = 3021 in#*
Art. 8-6] Moments of Inertia for Composite Areas 209

Fie. P-830. Fie. P-831.

832. Determine the centroidal moments of inertia of the built-up column sec-
tion shown in Fig. P-832. It is composed of two 16 by 1 in. plates riveted to two
12-in. 20.7-lb channels.

2Cs
12”- 20.7 lb

Fic. P-832. Fig, P-833.

833. Four Z bars, each having the size and properties determined in Prob. 826,
are riveted to a 12 by 1 in. plate to form the section shown in Fig. P-833. Determine
the centroidal moments of inertia. Ams, 1-= 1300 in,*2 1, = 592 inet
834. A 10-in. 15.3-lb channel is welded to the top of a 14 WF
34 beam as shown in Fig. P-834. The wide flange beam has an
overall height of 14.00 in., an area of 10.00 in.?, and J, of 339.2
in. Compute 7 and the moment of inertia about the cen-
troidal X axis.
835. Two 10-in. 15.3-lb channels are welded together as
shown in Fig. P-835. Compute the values of /, for arrange-
ments (a) and (b). Each channel web is 0.24 in. thick.
Ans. (a) 9 = 7.30 in.; J, = 116.5 in.4; (b) 7 = 7.82
Melee — 1403 1n
210 Moments OF INERTIA [Chap. VIII

Fig. P-835.

8-7. Product of Inertia


The product of inertia is a mathematical expression of the form f xy dA
and is denoted by the symbol P. The product of inertia is not used as
often as the moment of inertia but is needed in such problems as determin-
ing maximum and minimum moments of inertia, unsymmetrical bending of |
beams, and structural analysis of indeterminate frames.
Units and Signs. The unit of the product of inertia is of the same form
as that for the moment of inertia, namely, (length)*. Unlike the moment
of inertia, however, the sign for the product of inertia depends upon the
location of the area relative to the axes, being positive if the area lies
principally in the first or third quadrants and negative if the area lies
principally in the second or fourth quadrants. For example, the area in

Fig. 8-14. Fig. 8-15.

Fig. 8-14 lies in the first quadrant of the XY axes and P,, = f zy dA is
positive because all x and y coordinates of each differential area are positive.
However, with respect to a new set of axes, marked as X’ and Y’ and rotated
Art. 8-9] Transfer Formula for Product of Inertia 211

90° counterclockwise from the original set of axes, the area is in the fourth
quadrant. The new coordinates of dA are 2’ = y and y’ = —z so that
with respect to the new axes, the product of inertia is
= JS y(—2) dA = —SfaydA = = Pa
Pa —— S x'y' dA

Not only does this result confirm the rule of sign stated previously, but
it indicates that during the rotation of the axes, there will occur one critical
position at which the product of inertia changes sign and will have a zero
value. When in this position, the axes are known as the principal axes of
the area. Their application is discussed in Art. 8-12.

8-8. Product of Inertia Is Zero with Respect to Axes of Symmetry


If an area has an axis of symmetry, this axis together with any axis per-
pendicular to it will form a set of axes for which the product of inertia is
zero. Consideration of the symmetrical T section shown in Fig. 8-15 will
disclose that for any differential area like that at A, there is a symmetrically
placed equal differential area at B. With respect to the Y axis of symmetry,
the x coordinates of A and B are equal but of opposite sign, whereas their y
coordinates are equal and of the same sign regardless of the position of the
X axis. Hence the sum of the products zy dA for each such pair of sym-
metrically placed elements as A and B will be zero. It follows, therefore,
that the value of f§ xy dA for the entire area will be zero if either or both
reference axes are axes of symmetry.

8-9. Transfer Formula for Product of Inertia


Consider any irregular area, such as that in Fig. 8-16, whose cross-
sectional area is A and whose product of inertia relative to the centroidal
axes is denoted by P,,. Let a parallel set of axes X and Y be located so
that the coordinates of the centroid of the given irregular area are and 7
relative to these axes.

Lo
Fic. 8-16. — Products of inertia between parallel sets of axes.
212 Moments OF INERTIA [Chap. VIII

From the fundamental definition of product of inertia we have, with


respect to centroidal axes
Py = fry dA (a)
and with respect to any parallel set of X and Y axes

Pa, = S(t + Hy +944 (0)


Expanding Eq. (6) gives
P,=J2y dA -afy dA +4 fa dA ty J dA (c)
Note that the two middle terms represent the moment of area relative to
the centroidal axes multiplied respectively by the constants and gy. Since
the moment of area relative to centroidal axes is zero, Eq. (c) finally reduces
to
Pr, = Pry + AXG (8-8)
This equation, which is known as the transfer formula for products of
inertia, forms the basis of the method of computing products of inertia for
areas composed of simple geometrical shapes. The signs of ¢ and @ in this
equation may be taken either as the coordinates of c.g. relative to the
X-Y axes or as the coordinates of O with respect to the centroidal X,-Y,
axes. If the former, ¢ and @ in Fig. 8-16 are both plus; if the latter, and g
are both minus; in either case their product is the same.

ILLUSTRATIVE PROBLEMS
836. Determine the product of inertia of the right triangle shown in Fig. 8-17
with respect to the X and Y axes.
Solution: In applying the definition of
product of inertia, P = f(xy dA, observe
that « and y represent the coordinates of
the centroid of the differential area dA.
For the right triangle illustrated, select
the differential area as the shaded strip
parallel to the base. The area of this strip
is dA = x dy, and the coordinates of its
centroid are $ x and y.
From consideration of similar triangles,
aa : b
Fic. 8-17. it is evident that z= h (h — y). Hence

b : ee :
dA =axdy = i (h — y) dy. Applying the definition of product of inertia, we obtain

hates b
[P = f zy -dA] pa = J [Rea-v]-y-[Pa-nav|
Art. 8-9] Transfer Formula for Product of Inertia 213

be ("
op, ey — 2h# +9") dy
= b2 hy? 2hy? _ ys la
2h} 2 3 7 0

Pay = +

837. Determine the product of inertia of the 8 X 6 X 1 in. angle section shown
in Fig. 8-18 with respect to the indicated X and Y axes.
Solution: The angle section can be considered composed of a 5 X 1 in. rectangle
plus an 8 X 1 in. rectangle. For the first rectangle, the centroidal axes parallel to
the X and Y axes are axes of symmetry; hence, from Art. 8-8, P,, for this rectangle
equals zero. The situation is similar for the other rectangle. Hence for the com-
posite area, we obtain
[Poy = Poy + Azy] 5X1 im rectangle: P., = (5 X 1) X.0.5 X 3.5 = 8.78 in.*
Sol invrectanvle. Fan= (8 < 1) 4.0.5 = 16,0 in.4
For the composite area: Pz, = 8.78 + 16 = 24.78 in. Ans.

>
[Y
q" = 1
Zs

I n”

6" 5

ari fue de

8”
| rs ee eee
x
© a

Fic. 8-18. Fic. 8-19.

If the angle is rotated 90° counterclockwise to


the position shown in Fig. 8-19, the same value
of P., but with opposite sign will be obtained;
ie., Pz, = —24.78 in.! This result is equivalent |
to a 90° rotation of the axes in Fig. 8-18, there- 7
by showing that Pyz = —Pz.,. The negative
sign resulting from the interchange of subscripts
is caused by choosing the Y axis as the first axis;
then the second axis is the negative part of the X ,
axis lying 90° counterclockwise from Y. This x
change of sign was discussed in connection with 2
Fig. 8-14 on page 210.
838. From the answer to Prob. 836, use the
transfer formula to obtain the product of inertia ——
of the right triangle shown in Fig. 8-20 with re- |«_—_ s +1
spect to the indicated centroidal axes. Fig. 8-20.
214 Moments oF INERTIA [Chap. VIII

bh? (eu.
Solution: From the answer to Prob. 836 we have Pz, = ——: Applying the trans-
fer formula, we obtain 24

5 ets Bh = 6 O 10
[Poy = Poy + AZy] og ~ Pu ta Xerxes

= bh?

ane es
a b2h2 b2h2 b2p2
zy 24 ca 18 i 79 ns.

Note the minus sign carefully. It confirms the rule of sign stated on page 210 be-
cause here most of the area lies in the second and fourth quadrants of the centroidal
axes. If the triangle is rotated through 90° from the position shown, the sign of Pry
22
will change to plus, but its magnitude will still be =

PROBLEMS
839. Determine the product of inertia of an 8 by 6 by 1 in. angle section with
respect to centroidal axes parallel to the legs. Explain the significance of the plus or
minus sign of the answer. Refer to Table VIII-2 for the location of the centroid.
Ans. P,, = 2232.31 in.
840. Repeat Prob. 839 using an 8 by 4 by 1 in. angle section
841. Determine the product of inertia of the Z section shown in Fig. P-841 with
respect to the centroidal X and Y axes. Ans. Px = —18.9 in.*

31!

Fig. P-841. Fic. P-842.

842. Compute the product of inertia of the shaded area described in Fig. P-842
with respect to the specified X and Y axes. Ans. Pz, = 800 in.*
843. Compute the product of inertia of the triangular area shown in Fig. P-843
with respect to the X and Y axes. Ans} Psy = 455.6 iné
Art. 8-10] Moments of Inertia with Respect to Inclined Axes 215
Se a a re ee a
844. Compute the product of inertia of the 4
triangular area shown in Fig. P-843 with respect er
to centroidal axes parallel to the given X and Y
axes.
Ans. Py = —30.4 in.!
845. Determine the product of inertia of the
quarter circular area shown in Fig. P-845 with 9”
respect to the given X and Y axes.
4
ATS eee E
8
846. Use the result of Prob. 845 to determine +5 aces
the product of inertia of the shaded area described a ee
in Fig. P-846 with respect to the given X and Y Fig. P-843 and P-844.
axes. i
Ans. Ps, = —
12

Fig. P-845. Fic. P-846.

8-10. Moments of Inertia with Respect to Inclined Axes


In some cases, especially in strength of materials, it is necessary to de-
termine the moment of inertia with respect to axes which are inclined to
the usual axes. The moment of inertia in such cases can be obtained by
formal integration, but a general formula is usually easier to use.
The problem may be stated as follows: Assuming the values of J,, I,,
and P,,, with respect to the X and Y axes to be known, determine the values
of I., I., and Pu, with respect to the U and V axes inclined at an angle a
with the X and Y axes, as shown in Fig. 8-21.
The coordinates for a typical differential area dA are given by x and y
with respect to the X and Y axes, and by wu and v relative to the U and V
axes. The relations between these coordinates can be obtained by project-
ing x and y upon the U and V axes. This gives
v SA tirana, (a)
u=ysna+zcosa
216 Moments OF INERTIA [Chap. VIII

Fic. 8-21. — Moments of inertia with respect to inclined axes.

By definition (I = fp? dA), the values of J. and I, are


Lo=sfa7 dA (b)
Ua BO EORE (c)

Replacing v in Eq. (6) by its value from Eq. (a), we obtain


I, = Sy? cos?a — 2 xy sina cosa + x sin? a) dA
Since
La fydA, 1,—] fda, and Pz = fae.

this reduces to
I, = 12008 o + 1, sin? a — Pe sin. 2a (d)
If the relations

cova = Manele
wees
ae and sin?a = ioe
= = (e)

are substituted in Eq. (d), the result is

a BE BV
I, eos 20 — Pay sin20 (8-9)

Similarly, replacing wu in Eq. (c) by its value from Eq. (a) gives
I, = Sly’ sin?a + 2 zy sina cosa + 2? cos?a) dA

This reduces to
I, = I, sin? a + I, cos? a + Px sin 2a (f)

The relations in Kq. (e) transform Eq. (f) ito

I, +1 I, — :
Ip = At — eos 20 + Py sin20 (8-10)
Art. 8-11] Mohr’s Circle for Moments of Inertia 217
When values of J,, Z,, and P,, are known, Eqs. (8-9) and (8-10) permit
the values of J, and J,, with respect to the U and V axes inclined at an
angle a to the X and Y axes, to be determined without further integration.
In a sense, these equations do for inclined axes what the transfer formula
does for parallel axes.
Adding Eqs. (8-9) and (8-10) gives the relation
I u ar re. vfx ar ysy

which shows that the sum of the moments of inertia with respect to any
set of rectangular axes through the same p oint is a constant quantity. This
conclusion could also have been obtained from Art. 8-2, which shows that
the polar moment of inertia J. is the sum of the moments of inertia with
respect to rectangular axes passing through the polar axis. Hence, since
J, is a constant, we obtain as before
‘PN Pe ew ee Se
To determine the product of inertia relative to the U and V axes, we
note that P.. is defined as
Pus = fudA (9)
Substituting the values of uw and v given in Eq. (a), we have
Pur = JS (y*? sina cosa + zy cos? a — zy sin?a — x sine cosa) dA
= sin 2a + Px, costa — Pay sin'a — Hsin 2a (h)

whence by using the relation cos? a — sin?a = cos 2a, we obtain

P., = ao sin2a+ P,, cos 2a (8-11)


The angles defining maximum and minimum moments of inertia may be
found by differentiating Eq. (8-9) with respect to a and setting the de-
rivative equal to zero. For these values of a, it will be found that the
product of inertia is zero while the maximum and minimum moments of
inertia are:

Trays = EE
min
“iee)
I, — I,\?
CRA (8-9a)
8-11. Mohr’s Circle for Moments of Inertia
The equations developed in the preceding article need not be memorized
in order to apply them. A visual interpretation of them, devised by the
German engineer Otto Mohr in 1882, uses a circle; accordingly, the con-
struction is known as Mohr’s circle.’ This circle represents all the possible
3 Mohr’s circle for moments of inertia is similar to Mohr’s circle for combined stresses
used in strength of materials. With a suitable change of symbols, one can be understood
from the other.
218 Moments OF INERTIA [Chap. VIII
e
O38 AE e eee
values of I and P with respect to all axes passing through a specified point
in an area. If the circle is plotted to scale, the results can be obtained
graphically; usually, however, only a rough sketch of the circle need be
drawn, analytical results being obtained from it by following the rules
given below.
At first it may seem more difficult for you to remember the rules for
Mohr’s circle than the equations it replaces. After you have used them,
however, you will find the rules much easier to apply (and remember) than
the equations. Mohr’s circle has the further advantage of presenting a
picture of the equations that the average student interprets more readily
than the equations themselves.

RULES FOR APPLYING MOHR’S CIRCLE


1. On a set of rectangular coordinate axes, choose one axis on which to
plot numerical values of moments of inertia, and the other on which to
plot products of inertia; call these the J and P axes. Plot points having the
coordinates (Iz, Pz) and (,, —P.,). The values for J,, Jy, and P., are
assumed to be already known. Note carefully that the value for P., with
its real sign is associated with the value of J,, and that the value of P.,
with the opposite sign is associated with 7,. Actually P,,- should be paired
with I,, but Py. = —P., as we discussed in Illus. Prob. 837 on page 213.
2. Join the points just plotted by a straight line. This line is the di-
ameter of Mohr’s circle having its center on the J coordinate axis. Draw
the circle.
3. As different axes are passed through the selected point in the given
area, the values of J and P relative to these axes are represented by the
coordinates of points whose position shifts around the circumference of
Mohr’s circle.
4. The radius of the circle to any point on its circumference represents
the axis of inertia corresponding to the J coordinate of that point.
5. The angle between any two radii on Mohr’s circle is double the actual
angle between the two axes of inertia represented by these two radii. The
rotational sense of this angle corresponds to the rotational sense of the
actual angle between the axes; i.e., if the U axis of inertia is located at a
counterclockwise angle a relative to the X axis of inertia, then on Mohr’s
circle, the U radius is laid off as a counterclockwise angle 2 a from the X
radius.

ILLUSTRATIVE PROBLEM
847. For the rectangle shown in Fig. 8-22, compute the values of I upel 5, A, Pe
with respect to the U and V axes. These axes are inclined 30° counterclockwise to
the X and Y axes.
Art. 8-11] Mohr’s Circle for Moments of Inertia 219

Solution: The moments of inertia and the product


of inertia are first found with respect to the X and Y
axes, as follows:

EE
_ bhi
12 UEBo
_ ear
6 X (12)?
am gaa hisSU

hobs 12 < (6)3 '


ES =] 1, = PO" =or ins

Pz, = 0 because X and Y are


axes of symmetry
Following the rules given above, draw a set of rec-
tangular coordinate axes and label them J and P, as Fic. 8-22.
shown in Fig. 8-23. Using the values of J, I,, and
P,,, plot the points A and B whose coordinates are (864, 0) and (216, 0).
According to Rule 2, the diameter of Mohr’s circleisAB. Its center C is midway
between A and B. The / coordinate of C is 540 in.‘ The radius of the circle is the
distance CA = 864 — 540 = 324 in4
if

oc =864 gs
=540

Fic. 8-23. — Application of Mohr’s circle.

From Rule 4, the radius C'A represents the axis of inertia corresponding to the
I coordinate of A; in this case, the X axis. Applying Rule 5, we find that the U axis
of inertia is represented by the radius CD laid off 60° counterclockwise from the
X axis (CA). Also, since V is actually 90° from U, the V axis (CE) is laid off 180°
(i.e., double scale) from the U axis. D, C, and E form a straight line.
From Rule 3, the coordinates of D represent I,, and P,,,; the coordinates of #
represent J, and P,,, with the opposite sign. Accordingly, from the diagram we
obtain
lf, = OC + CD cos 60°] I, I= 540 + 324 cos 60° = 540 + 162
L, = 702in.4 Ans.
[I, = OC — CE cos 60°] dir ll 540 — 324 cos 60° io = ooo." Ans,
[Puo = CD sin 60°] Pad 324 sin 60° Pay = 281 int Ans.
220 Moments OF INERTIA [Chap. VIII

8-12. Maximum and Minimum Moments of Inertia. Principal Axes


An inspection of Mohr’s circle will show that the points whose coordinates
indicate maximum and minimum moments of inertia are located on the
T axis and have a zero product of inertia. Conversely, axes which have a
zero product of inertia must be axes of maximum or minimum inertia.
Such axes are called principal axes.
As we have already seen (Art. 8-8), the products of inertia relative to
axes of symmetry are zero. Hence we conclude that axes of symmetry
must be principal axes because they always yield values of maximum and
minimum moments of inertia. But many figures do not have axes of sym-
metry, although they do have principal axes with respect to which the
product of inertia is zero. Axes of symmetry are always principal axes, but
the converse is not necessarily true.

ILLUSTRATIVE PROBLEM
848. A certain area is found to have the following values with respect to the
X and Y axes: J, = 100 in.', J, = 60 in.’, and Pz, = 15in.4- Determine the maxi-
mum and minimum moments of inertia and illustrate the position of the principal
axes relative to the X and Y axes.
Solution: On a set of J and P axes, as shown in Fig. 8-24, plot points having the
following coordinates.
(I, = ee ( I,= 60
Pens —P,, = —15
Note that the given value of P,, is associated with J, and that the value of P,,, with
the opposite sign is associated with Jy. If Pz, had been negative originally, this
value would have been associated with J,, and the positive value of Pz, with Ty.
Plotting these points gives two points on Mohr’s circle. Joining them gives the
diameter of the circle shown in Fig. 8-24. Obviously the radius of the circle is

Fic. 8-24. — Maximum and minimum moments of inertia.


Art. 8-12] Maximum and Minimum Moments of Inertia 221
ee ey
CA = V (20)? + (15)? = 25. The maximum and minimum moments of inertia are
located at B and D; hence
[Max.J = OC + CB] Max. J = 80+ 25 = 105in.‘ Ans.
{[Min. J = OC — CD] Min. J = 80-25 = 55in.A Ans.
To go from the X axis to the axis of maximum inertia CB, we must rotate clock-
wise through an angle 2a. From the diagram

_ AE 15
|tan 2 = 7 | tan 2a = — = 0.75
20
2a = 36.9° anda = 18.45° Ans.
Angle a, which locates the axis of the
maximum moment of inertia (the U axis),
is also rotated clockwise on the original
reference axes; this gives the position
shown in Fig. 8-25. The axis of mini-
mum moment of inertia (i.e., the V axis)
is at 90° to the U axis.

PROBLEMS

849. For a certain area it is known


that J, = 60 in.*, J, = 20 in.*, and P,, =
0. Find the moment of inertia of this
area with respect to a U axis which is ro- yg. 8-95 — Location of U and V axesof
tated 30° counterclockwise from the X maximum and minimum moments of
axis. Ans. I, = 50 in. inertia.
850. A certain area has the following properties: 7, = 40 in.*, J, = 100 in.‘,
P, = 40 in‘. Determine the values of the maximum and minimum moments of
inertia, and also the angle that the axis of maximum inertia makes with the X axis.
Illustrate by a diagram. Ans. Max, 7 = 120 in. min. J = 20in.*76 = 63.4°
851. Refer to Prob. 850 and determine the moments of inertia and the product
of inertia with respect to U and V axes inclined 45° clockwise to X and Y re-
spectively.
852. A right triangle has a base of 8 in. and an altitude of 9 in. Determine the
maximum and minimum moments of inertia with respect to the principal axes pass-
ing through the centroid. Arise ia 2S Onnees tate 7: ein,
853. For an 8 by 6 by 1 in. angle, it is known that J, = 38.8 in.‘, J, = 80.8 in.4,
and Pz, = —32.3 in.t Compute the minimum radius of gyration.
854. Determine the maximum and minimum moments of inertia of the Z section
described in Fig. P-826 on page 208 with respect to the principal axes passing
through the centroid. From Prob. 841 it is known that P», = —18.9 in.!
Ans. Max. I = 52.0 in.*; min. J = 5.61 in.‘
855. Show that the moment of inertia for the area of any regular polygon is a
222 Moments OF INERTIA [Chap. VIII

constant with respect to all axes in the plane of the area which pass through its
centroid.
856. Show that the moment of inertia for the area of a quarter circle with respect
4
to its axis of symmetry is (7 — 2) r where r is the radius.

857. The figure for Prob. 825 is redrawn as shown in Fig. P-857. Check the
answer to Prob. 825 (i.e., find Jz) by first computing /,, and J,, then using Mohr’s
cirele to find /,, and finally transferring this value to the X axis.

Fig. P-857. Fic. P-858.

858. Use the idea discussed in Prob. 857 to compute the value of J, for the area
shown in Fig. P-858. Ans. I, = 2660 in4

8-13. Moments of Inertia of Bodies


Problems concerned with the rotation of solid bodies involve the mathe-
, : dw Te
matical expression Jp” 73 which is known as the moment of inertia of a
body. It is also frequently called the mass moment of inertia because the
Pt Re at :
ratio — is widely known as the mass of a body. It will be convenient to

use the symbol (equivalent to sy to denote the mass of a body, and


ry

the symbol dM (equivalent to )to denote the mass of a differential


element of the body. The symbol J which has been used to denote the
moment of inertia of area will also be used for the mass moment of inertia.
In cases where confusion may arise, the subscripts M and A will be used
for moment of inertia of mass and area respectively, viz., In and I 4.
No physical interpretation of the mass moment of inertia will be given.
The expression merely indicates a mathematical operation consisting of
Art. 8-15] Transfer Formula for Mass Moments of Inertia 223
a A a a Ne cd
summing up the product of each differential mass multiplied by the square
of its distance from the axis of inertia. In a sense, however, when used in
connection with rotating bodies, the expression may be regarded as a
measure of the resistance of the body to angular acceleration.
Units. Examination of the integral /p?dM dimensionally shows that
the unit is composed of a distance squared multiplied by a mass. Noting
x

that mass is - and substituting dimensionally, we have (using the foot-


pound-second system of units)
W : : Ib
M X (L) Oy Seei XL es =e
(dimensionally) ft/sec! i etre beana2
ft-lb-see

No name has been generally accepted for this combination. Among the
names suggested are “‘slug-feet squared,” “‘engineer’s unit,’ or simply
“unit.”’ In this book we shall speak of a body possessing J units of inertia,
the unit being the combination ft-lb-sec’.

8-14. Radius of Gyration


Mass moment of inertia is sometimes expressed in terms of the product
of its mass and the square of a distance k, according to the equation

I= MR or k= Ve (8-12)

In this equation k is called the radius of gyration. It may be regarded as


the distance from the axis of inertia to the point at which the mass of the
body may be assumed to be concentrated and still have the same moment
of inertia as does the actual distributed mass of the body.

8-15. Transfer Formula for Mass Moments of Inertia


Frequently the mass moment of inertia about axes must be determined
when it is inconvenient to apply the
integration of f/p?dM. In such cases
the transfer formula is a means of
transferring a known value of moment
of inertia from one axis to a parallel
axis without further integration.
Figure 8—26 represents the cross sec-
tion of a body whose mass moment of
inertia with respect to a centroidal
axis perpendicular to the section is
denoted by I. The value of J is given
by the equation
T= frdM = f(22+y)dM (a) Fic. 8-26.
224 Moments oF INERTIA [Chap. VIII

Let B be the point at which any parallel axis intersects the section. The
mass moment of inertia with respect to this parallel axis through B is de-
noted by J, and its value is given by the equation
Lf pa (b)
However, from the figure
e=(e+ar?+tiy—-be=xP+2ar4+7+y-2by4+0?
Since d? = a? + 6? and r? = x? + y?, this becomes
e=r+td+ 2ax — 2 by (c)
Substituting this value of p? in Eq. (6), we obtain
T= frdaM+@sdM+2afx2dM —2bfydM (d)
Note that fxrdM = M-& and fydM =M -@; also that @ and 7
represent the distance from the center of mass to the reference axes (zero
in this case because the reference axes contain the center of mass). Hence
the third and fourth terms in Eq. (d) become zero, and finally
t=1- Me (8-13)
The equation states that for any body the moment of inertia with re-
spect to any axis is equal to the sum of the moment of inertia with respect
to a parallel centroidal axis and a transfer term composed of the product
of the mass multiplied by the square of the distance between the axes.
Note carefully that the centroidal axis involved in the transfer formula is
always the centroidal axis of the mass used in the transfer term Ma?.
A similar relation exists between the radii of gyration with respect to
parallel axes, one of which is a centroidal axis. Replacing J by Mk? and
I by ME? in Eq. (8-13), we have
Mk? = Mk? + Ma?
whence _
R= k? + @ (8-14)

8-16. Mass Moments of Inertia by Integration


When the fundamental definition of mass moment of inertia, 7 = {p? dM,
is applied to bodies, the differential element of mass used in the integration
should be selected so that either
1. All points in the element are at the same distance from the axis,‘ or
2. The mass moment of inertia of the selected element with respect to
the reference axis is already known. The mass moment of inertia of the
body is then found by summing up the moments of inertia of all the
elements.
4This distance in effect is the radius of gyration of the element.
Art. 8-16] Mass Moments of Inertia by Integration 225
ee A ee

ILLUSTRATIVE PROBLEMS

859. Show that for any homogeneous right prism, the mass moment of inertia
with respect to any axis perpendicular to the base is equal to the product of its
mass and the square of the polar radius of
gyration of the area of its base with respect to
the specified axis.
Solution: As in Fig. 8-27, take the Z axis per-
pendicular to the base of the right prism. Ap-
plying Rule 1, select the differential mass as the
elemental prism whose base is dA and whose
height is h. All points in this prism are at the
same distance p from the Z axis. Denoting the
mass per unit of volume by y, we have dM =
yh dA. According to the definition of mass mo-
ment of inertia

YZ = fp’ dM] I = yhfp?dd = yhJa


because /p?dA is Ja, the polar moment of
inertia of area of the base. Replacing J4 by Fig. 8-27.
Ak4?, where A is the area of the base of the right
prism and k, is the polar radius of gyration for this area, we have

= yhAka’
whence, since mass M equals yhA, we finally obtain

iE =< Mk?

This makes it easy to compute the mass moment of inertia of any right prism.
All that is necessary is to use the methods of Art. 8-6 to determine the polar moment
of inertia for the area of the base with respect to the specified axis, divide this value
of J4 by the area of the base to obtain k4, and multiply the result by the mass of the
prism.
860. Use the result obtained in Prob. 859 to determine the moment of inertia
with respect to its geometric axis of (1) a solid right circular cylinder of radius r
and (2) a hollow right circular cylinder of outer radius FR and inner radius r.
Solution: Part 1. The centroidal polar moment of inertia of a circle, from Prob.
803, is J = 477‘, and the area of a circle is rr’. Hence the square of the polar
radius of gyration of the base is

df eeroT, als

aa ed rea A

Applying the result of Prob. 859, we obtain


(I = Mk.’ I=4Mr Ans.
226 Moments OF INERTIA [Chap. VIII

Part 2. The base of a hollow cylinder is shown in


Fig. 8-28. The polar moment of inertia with respect
to a centroidal axis is
J=4n(Rt—1r') = 3 7(R? + 2°)(R? — 2’)
The area of the base is A = 7(R? — r?) and therefore

E A see % w(R? + r?)(R?2 — r?)


Ae a(R? — r?)
1
= Spee 2
Fra. 8-28. 5 ene)
Using the result of Prob. 859, we have
eS Were [=4+M(R?+ 7°) Ans.

861. Determine the mass moment of inertia of the homogeneous rectangular


parallelepiped shown in Fig. 8-29 with respect to a centroidal axis parallel to any
edge.

Fria. 8-29.

Solution: Let us determine the mass moment of inertia with respect to the Z axis.
The Z axis is the centroidal polar axis of inertia for the face whose lengths are a
and 6. The polar moment of inertia for this area is

=
apes
b? aba
gregae
le=1e+ 1,
f de eo? eae
Therefore, since the area A of the Z face is ab

i] L
[w= F k= h@ +b)
Applying the result of Prob. 859 to this, we have

I = Mk] I,=7,M(@e@+) Ans.


Art. 8-16] Mass Moments of Inertia by Integration 227

Similarly for the X and Y axes we obtain for the mass moments of inertia

I,= ty MQ + &)
I, = ty M(@ +e)
862. Determine the mass moment of inertia of
a homogeneous sphere of radius r with respect to a
diameter.
Solution: Fig. 8-30 shows the cross section of
the sphere in the XY plane. Select the differential
mass as the thin circular plate whose edge is
shaded in the figure. This plate has a radius zx
and a height dy. The mass dM of the elemental
eylinder is yrx? dy, and from Prob. 860, its moment
of inertia relative to the Y axis is 3 yra‘ dy. From Fic. 8-30.
Rule 2, the moment of inertia of the sphere is
the sum of the moments of inertia of all such elemental circular plates. Thus, we
obtain
+7 =i

ox eeal et eeah a i i ileago ane (a)

The volume of a sphere is $ zr’, and its mass is therefore $ yar’. Factoring this
expression from (a), we have

i= adam
whence
I=2Mr’ Ans.
863. Determine the moment of inertia of a slender
rod of length LZ and mass M with respect to an axis
through one end which makes an angle @ with the length
of the rod, as shown in Fig. 8-81.
=e Solution: Let the constant cross section of the rod be

FS: <a
a denoted by A and the mass per unit volume by y. The
differential mass dM of an element of length dr is equal
to yA dr. This element is at a distance r from the end
of the rod but at a distance p = 7 sin @ from the axis of
| inertia. Applying the fundamental definition of mass
Fic. 8-81. moment of inertia, we have
ve ip
[I = fp? dM} I ef. r? sin? O-yAdr= yA sin of r? dr
0 0
3 i

I = yA sin? 6 2 aX (yAL) - L? sin? 6

Since the mass of the entire rod is M = yAL, we obtain finally


[Chap. VIII
228
420) SEMoments OF INERTIA
=+ML*sin? 0 Ans.

When the axis is perpendicular to the rod, @ = 90° and sin@ = 1. This makes
the value of the moment of inertia
I=4ML Ans.

PROBLEMS .
864. By using the transfer formula and the result of Prob. 860, determine the
moment of inertia of a homogeneous right circular cylinder about an axis through
an element on its surface. The cylinder has a mass M and a radius r.
Ans. I = 3 Mr?
865. By using the transfer formula and the result of Prob. 862, determine the
moment of inertia of a homogeneous sphere of mass M and radius r with respect
to a tangent. Ans. =+¢Mr
866. By using the transfer formula and the result of Prob. 863, determine the
moment of inertia of a rod with respect to an axis through the center of gravity
perpendicular to the rod. Ans. [=75ME*
867. Derive the expression for the moment of inertia of a homogeneous right
circular cone of mass M, base radius r, and altitude h, with respect to its geometric
axis. Ans. I = 3 Mr?
868. By using the transfer formula and the result of Prob. 861, determine the
moment of inertia of the rectangular parallelepiped shown in Fig. 8-29 with respect
to a median line of the Z face. Take the median line parallel to the X axis.
Ans. I = 7 M(b? + 4c)
869. Determine the moment of inertia of the rectangular parallelepiped shown
in Fig. 8-29 with respect to an axis through one edge parallel to the Y axis.

8-17. Moments of Inertia of Composite Bodies


Many of the bodies used in engineering are combinations of the geometric
elements discussed in the preceding article. The moment of inertia of a
composite body is the sum of the moments of inertia of its elements, but
the moment of inertia of each element must be referred to the same axis
of inertia. For convenience, the mass moments of inertia of various bodies
are summarized in Table VIII-3.
Sometimes it is desirable to compute wezght moment of inertia, using
weight in place of mass and any convenient unit of length (either inches or
feet), the resulting unit being either lb-in.? or lb-ft?. However, this unit
must be converted into ft-lb-sec? before using the equations of dynamics
(developed later in Part I1) which require the use of mass moment of inertia.

ILLUSTRATIVE PROBLEMS
870. The right circular cylinder shown in Fig. 8-82 is made of steel weighing
490 lb per cu ft; it is 6 in. long and has a diameter of 24 in. Four holes, each 6 in.
Art. 8-17] Moments of Inertia of Composite Bodies 229
TABLE VIII-3. Mass Moments or INerRTIA FOR Bopres

Description of Body Axis Moment of Inertia

Solid right circular cylinder Longitudinal axis IMP


of radiusr

Hollow right circular cylinder Longitudinal axis 1M(R? + r)


of outer radius R and inner
radiusr

Sphere of radius r Any diameter 3Mr*

Spherical shell of mean Any diameter Mr


radius r

Uniform slender rod of At end, perpendicular to rod 4M RB


length L
Through center, perpendic- qeM L?
ular to rod

Elliptical cylinder of semiaxes Longitudinal axis LM (a +B)


aand b

Right circular cone of base Axis of revolution


radiusr
230 Moments OF INERTIA ’ [Chap. VIII

in diameter and equally spaced around


a circle 10 in. in diameter, are drilled
from the cylinder. Compute the mo-
ment of inertia about the geometric axis
of the body.
24 Solution:. The moment of inertia of
the body is equal to the moment of in-
ertia of the cylinder minus the mo-
ments of inertia of the masses drilled
from the cylinder.
The weights are computed as fol-
Fia. 8-32. lows:

; 490
[W = wAL] Cylinder: W = 1728 x (:x m7)<6 = WmIl>

Z , 490
Each weight drilled out: W = 1728 x* (7x 6)X 6 = 48.1 lb

The moment of inertia of the cylinder is found by means of the result in Prob. 860.
Remember that mass is defined as weight divided by the gravitational constant g.
The value of g is 32.2 ft per sec’.
| 1 i He 12\2
if = —5 Mr 2 ] it = —5 x ———
30.9 x G2)
— = 11.96 ft-lb-sec
-1D-! 2

The moment of inertia of each drilled-out mass with respect to its geometric
axis 1s
1 = ;eee 1 3
E = —5 ae 2 | If = — X 3595 x ia}= 0.0467 ft-lb-sec
Sl ve 2

Transferring this value to the geometric axis of the LL ILLALLLLL pe

body, we obtain for one drilled-out mass,


= 48.1 5\/
[I=I+M@ a ] I= 0.0467 + 355 — x (2)
l
I = 0.306 ft-lb-sec? l
The moment of inertia of the body is l
e
a
I = 11.96 — 4 X 0.306 8" 48"
I = 10.736 ft-lb-sec? Ans. F
871. Compute the moment of inertia of the cast-iron
flywheel with respect to the axis of rotation. The cross
section is shown in Fig. 8-33. The flywheel has six el-
liptical spokes 2 X 3 in. in cross section, which may be
considered as slender rods. Cast iron weighs 450 Ib per CLIELELLAA
cu ft. jee em, |
Solution: The moment of inertia of the flywheel is Fic. 8-33.
Art. 8-17] Moments of Inertia of Composite Bodies 231

equal to the sum of the moments of inertia of the rim, the hub, and the six
spokes. We begin by determining the weight of each element.
[W, = wr(k? —7*)L| Forrim: W, = tres X w X (247 — 22°) x 12 = 903 lb
(W, = wr(R?—7°)L] Forhub: W, = #85 X m X (42 — 2) X 10 = 98.1 Ib
[W. = w(rab)L] For onespoke: W, = jes X tr X 1X 1.5 X 18 = 22.1 lb

The moments of inertia of the elements with respect to the axis of rotation are next
computed.

1 all hws =a (=
a
E
L =56
= — VA(R 2 + 15)]
2 For reborn df csx3 x ( +t 19 = 103.2 Ses
ft-lb-sec 2
il
1 2 2;
E == M(R?+ “|For hub: J, ==5 x ale x (4
Ey + ia ]= 0.212 ft-Ib-sec?
2 32.2 12

ie= n(I + Md?) For n spokes:


ary
1 Vy ie)
(aah 2 he
1. eee
Jes
224, /is\? , aa
22.1. fay
n(fy ML + Me) 6 ey x (2) apa te ]
5.605 ft-lb-sec?

The moment of inertia of the flywheel is


{J =1,+1,4+ Is] I = 103.2 + 0.212 + 5.605 =109.017
I = 109 ft-lb-sec? Ans.

PROBLEMS

872. Determine the moment of inertia of a hollow steel cylinder with respect to
its geometric axis. The cylinder is 1 ft long and has an outside diameter of 3 ft and
an inside diameter of 2 ft. Steel weighs 490 lb per ft’.

873. By using the method discussed in Prob. 870, determine the moment of in-
ertia, with respect to the geometric axis, of a cylinder of radius R from which is
drilled a concentric hole of radius r. Denote the mass of the resulting hollow
cylinder by M, and the mass per unit volume by y. Ans. 1=3%M(R?+ 7°)

874. A slender rod 6 ft long rotates about an axis perpendicular to it at a point


2 ft from one end. The rod weighs 40 lb. Compute the moment of inertia about
the axis of rotation. Ans. I = 4.97 ft-lb-sec?

875. For a hollow cast-iron sphere of 20 in. outside diameter and 16 in. inside
diameter, compute the moment of inertia and the radius of gyration with respect to
a diameter. Cast iron weighs 450 lb per cu ft.
Ans. I = 6.32 ft-lb-sec?; k = 7.48 in.

876. The governor of an engine consists of two cast-iron balls 6 in. in diameter,
each of which is connected to the axis of rotation by a steel rod 1 in. in diameter and
16 in. long. Compute the moment of inertia and the radius of gyration of the
232 Moments oF INERTIA [Chap. VIII

governor assembly when the rods are inclined at an angle


4 of 30° to the axis of rotation.
Ans. I = 1.23 ft-lb-sec?; & = 9.28 in.
877. Aright circular cone made of steel has an alti-
tude of 20 in. and a base diameter of 24 in. A hole 6 in.
deep and 8 in. in diameter is drilled from the center of
the base of the cone and filled with lead. Lead weighs
710 lb per cu ft and steel weighs 490 lb per cu ft. Deter-
LLL. : eS
= at 10" 60” mine the moment of inertia of the resulting solid with re-
ate spect to its geometric axis.
878. Determine the moment of inertia of the cast-
iron flywheel shown in Fig. P-878 with respect to the
axis of rotation. The flywheel has six elliptical spokes,
3 X 4 in. in cross section, which may be considered as
slender rods. Cast iron weighs 450 lb per cu ft.
Ans. I = 280 ft-lb-sec?
WLU Y 879. Compute the moment of inertia of the flywheel
described in Prob. 878 by the following approximate
method: Neglect the hub and consider the flywheel to
Fia. P-878 and P-879.
have three double spokes each 56 in. long.

SUMMARY

The moment of inertia of an area is defined by the equation

l= fpdA
where p is the perpendicular distance from dA to the axis of inertia.
The radius of gyration k is defined by the relation

I = Ak (8-4)
where A is the total area of the figure.
The polar moment of inertia J with respect to an axis perpendicular to
the plane of a figure is the sum of the plane moments of inertia with respect
to two mutually perpendicular axes which intersect on the polar axis; i.e.,
J aaletal (8-3)
The transfer formula is useful to transfer the moment of inertia from a
centroidal axis to any parallel axis, or vice versa. The formula is
I=1+ Ad? (8-5)
The moment of inertia for a composite area is the sum of the moments
of inertia of its component parts. Note that the moments of inertia of each
Summary 233

part must be found with respect to a common axis before they can be added
(see Art. 8-6).
The product of inertia for an area is defined by the mathematical expres-
sion {xy dA and is denoted by the symbol P. With respect to an axis of
symmetry, the product of inertia is zero. The transfer formula below is
useful for transferring the product of inertia between a centroidal and a
parallel set of axes.
Poy 1 ARY (8-8)
When it is necessary to compute moments of inertia with respect to axes
inclined at an angle a with a set of reference axes for which J and P are
known, the following formulas may be used:

J, = BE 4 A cos 80 — Pay sin2a (8-9)

fp EE — cos 2a + Py sin2 (8-10)


Mohr’s circle (Art. 8-11) may be used as an alternative to these equa-
tions. This circle is especially convenient when determining maximum and
minimum moments of inertia and principal axes of inertia.
Moments of inertia for bodies, also known as mass moments of inertia,
are similar to moments of inertia for area. The essential difference lies in the

use of ev in place of dA, as may be seen from the definition In = fp” ie

The formulas for I, for several bodies are given in Table VIII-3 and are
applied to composite bodies in Art. 8-17.
ste A~,

pu-w) bie: an
“Rlyyy enlfiet wigh il
bey Chk rina get
; ne
mien toate nchitt wg
endion) wail eet talaga ade eee
* we sgl cevie “scan dabe i = Sei 7
re ®t u
ere dere& tty altacl ot unvig ad ; z “1 dal ~
"3h Stee
tiled adn
it eae > 1

wa

ig t&
: a ae

22 = A of Sah :
. :
ion a 7
7 7
a : -
; f vor ae paar
4 ta ji
@ a % ie) == au, ay
| ; é 7 : ~ - ; x : .

ote Us
1 Se
ber.Werd bo

DYNAMICS
a
aa?Re

7 = ne
7 ls
i=
aa : fay
.

ay
c
|
Chapter IX.
Principles of Dynamics

9-1. General

Dynamics is the branch of mechanics which deals with the study of


bodies in motion. Compared with statics, dynamics is relatively new; it is
generally considered to have been begun by Galileo (1564-1642). Its de-
velopment was greatly retarded by the lack of precise methods for measur-
ing time. The experiments which form the foundation of dynamics require
the use of three kinds of units: force, length, and time. Precise methods for
measuring force and length are relatively simple and account in part for
the early development of statics, in which only these units of measurement
are required. No accurate time-measuring devices, such as the pendulum
clock developed by Huygens in 1657 and the balance-wheel watch developed
by Robert Hooke around 1666, were devised until after Galileo’s death.
Dynamics was also retarded by the principles of natural philosophy which
were set up by Aristotle and in Galileo’s time were regarded as infallible.
Galileo’s experimental turn of mind led him to doubt these dogmas of
abstract thought. For example, he did not accept the notion that heavy
weights fall more rapidly than light ones. His experiments with dropping
weights from the Leaning Tower of Pisa exploded this theory but pre-
cipitated such bitter arguments that he was forced to leave Pisa.!
Galileo’s experiments with blocks sliding down inclined planes led to a
relation between force and acceleration which Sir Isaac Newton generalized
and incorporated into the laws governing the motion of a particle that are
named after him. Newton’s laws of motion are the basis for extending the
laws of motion from a particle to a body composed of a system of particles.
This extension is discussed in Arts. 9-7 and 9-8.
At this point definitions of the terms particle and body are pertinent.
The term particle usually denotes an object of point size. The term body
denotes a system of particles which form an object of appreciable size. In
other words, a particle is a body so small that any differences in the motions
of its parts can be neglected. The criterion of size is only relative, however;
the terms particle and body may apply equally to the same object. For
example, in astronomical calculations the earth may be assumed to be a
particle in comparison with the size of its path, whereas to an observer
1 The Star Gazer by Z. Harsanyi (Putnam) is an excellent account of Galileo’s life.
237
238 PRINCIPLES OF DYNAMICS [Chap. IX

on the earth, it is obviously a body of appreciable size. In this book the


term particle is used to mean an object whose dimensions are negligible
compared with the size of its path.

9-2. Kinematics and Kinetics


Dynamics is divided into two branches called kinematics and kinetics.
Kinematics is the geometry of motion. The term is used to define the motion
of a particle or body without consideration of the forces causing the motion.
Kinematics is essentially a treatment of the relations between displacement,
velocity, and acceleration.
Kinetics is the branch of mechanics that relates the force acting on a body
to its mass and acceleration. When the acceleration of a body caused by
the forces acting on it has been determined, the principles of kinematics
may be applied to determine the displacement or velocity at any instant.
In other cases, the kinematic motion of a body may be used to compute its
acceleration; from this, by means of the relations to be developed later,
the force or forces required to produce this motion can be determined.

9-3. Motion of a Particle


The kinematic discussion of the motion of a particle depends upon the
definitions of displacement, velocity, and acceleration. The displacement
of a particle is the vector distance from an origin to the position occupied
by the particle on its path of travel. The origin may be selected anywhere,
as at O in Fig. 9-la. There the displacements to positions A and B are
the vectors s4 and sg which change both in magnitude and inclination.
With straight line motion, however, it is best to select the origin O on the
path as in Fig. 9-1b so that only the magnitude of the displacement vector
can change but not its inclination.

O
Fic. 9-1. — Displacement in (a) curvilinear and (b) rectilinear motion.
Art. 9-3] Motion of a Particle 239

Observe carefully the distinction between a change in displacement and


the distance traveled along the path. In the curved path of Fig. 9-1la, the
change in displacement As is less than the are distance AB. In the straight
path of Fig. 9-1b, however, the change in displacement As and the distance
AB are numerically equal for a particle moving from A to B, but would be
unequal if the particle should ever reverse its direction. Thus if the par-
ticle in Fig. 9-1b travels from A to B and back to A, its final displacement
will be the vector su, its change in displacement will be zero, but the dis-
tance traveled will be the accumulated length from A to B and back to A.
This distinction between displacement (which is a vector measured from a
fixed origin) and distance is important because all the kinematic relations
we shall subsequently develop involve displacement and not distance.
Velocity is defined as the time rate of change of displacement, and ac-
celeration is defined as the time rate of change of velocity. The meanings
of these terms may be clarified by restricting the discussion here to straight-
line motion. A more general discussion is presented in Chapter XI.
Velocity. In Fig. 9-2, consider a a As
particle traveling in a straight line J
from A to B and let points 1 and
2 be two positions a small distance il 2 B
Fic. 9-2.
apart. If the displacement As is
traversed in the time At, it follows from the above definition that the
average velocity over that displacement will be

y) = As
ave — At

and the instantaneous velocity will be found as At approaches zero as a


limit; 1.e.,
As _ ds
v = limit Soar: (a)
At—0 At

Acceleration. Let us now assume that in the preceding discussion the


particle starts from A with a velocity v4 and arrives at B with a velocity vz.
The average acceleration, or the average rate of change of velocity, will be
the difference between vg and v4 divided by the elapsed time. The in-
stantaneous acceleration at any intermediate point cannot be determined
from these data. However, if the velocity at point 1 is v; and at point 2 is
v; + Av, the average acceleration during the time At will be

dave
_ Av
= At

and the instantaneous acceleration will be found as At approaches zero as


a limit; 1.e.,
240 PRINCIPLES OF DYNAMICS [Chap. IX
ELEN omen Ses eR SPN EIS ie, 5 ne ee

c= Umit S| oi ®
Since v = S the instantaneous acceleration may also be written

dv g (=) d’s
OPalaa\ite Mai Whar ©)
Elimination of dt in Eqs. (a) and (0) leads to a third equation
vdv = ads (d)

The three equations just developed are known as the differential kine-
matic equations of motion. These equations are really statements of funda-
mental definitions of instantaneous velocity and instantaneous acceleration.
By their proper application, the kinematic equations of motion for any
case can be determined. This will be done for several cases in the following
chapters. For the sake of compactness, the differential equations of kine-
matics are here restated.
ds
UT ap (9-1)

dv ad’s
a= dt = dé (9-2)

v du = ads (9-3)
The above definitions of velocity and acceleration include a fact which
may not be obvious, the fact that velocity and acceleration are vector
quantities involving direction as well as magnitude. As written here, they
consider only variations in the magnitudes of these quantities.2, But we
have shown earlier that displacement of a particle always takes place in a
definite direction. Therefore, if the definition of velocity, i.e., the time rate
See ‘ : 1 :
of change of displacement or v = aes rewritten in the form v = aia ds, it

is apparent that the vector displacement ds is multiplied by a scalar factor


1 ; : :
a which results in a new vector of different length than ds but of the same
direction. Obviously, if the displacement is constant in direction, the ve-
locity will likewise be constant in direction. In cases of rectilinear motion
where only the magnitude of velocity changes, the term speed is often used
to describe the velocity. Properly speaking, however, speed refers only to
the magnitude of velocity. The possibility of a change in the direction of
2 There isa vector calculus which considers variations in both magnitude and direction
but its discussion is beyond the scope of this book. ‘
Art. 9-5] Fundamental Equation of Kinetics for a Particle 241

velocity must not be overlooked (see Art. 11-5 on acceleration in curvilinear


motion). In this book the term velocity includes both the magnitude and
direction of the rate of change of displacement; speed is used only to mean
the magnitude of velocity.
Similarly, the definition of acceleration, i.e., the rate of change of veloc-
: dv . 1 :
ity, ora = a may be written a = ae dv. It is apparent that the velocity

: we 1 ; é
vector dv is multiplied by a scalar factor de resulting in a new vector of
G 5

different length but having the same direction as the velocity variation dv.
Units. The units that define displacement, velocity, and acceleration
depend upon the units chosen to represent length and time, such as foot,
centimeter, and mile, for length; and second, minute, and hour, for time.
Accordingly since displacement is synonymous with length, velocity with
change of length per unit time, and acceleration with change of velocity per
unit time, the common units for these terms are:
Displacement: foot, centimeter, mile.
Velocity: foot per second (ft per sec), centimeter per second (cm per sec),
mile per hour (mi per hr), etc.
Acceleration: foot per second per second (ft per sec”), mile per hour per
hour (mi per hr’), ete.

9-4. Newton’s Laws of Motion for a Particle


From his study of falling bodies, Galileo discovered the first two of what
are commonly called Newton’s laws of motion for a particle. Newton’s
name is associated with the laws of motion, however, because it was he who
generalized them and demonstrated their truth by his astronomical predic-
tions based on them.
Newton’s laws of motion for a particle have been stated in a variety of
ways. For our purposes we shall phrase them as follows:
1. A particle acted upon by a balanced force system has no acceleration.
2. A particle acted upon by an unbalanced force system has an accelera-
tion in line with and directly proportional to the resultant of the force
system.
3. Action and reaction forces between two particles are always equal and
oppositely directed.

9-5. Fundamental Equation of Kinetics for a Particle


Consider a particle of weight W acted upon by the forces shown in Fig.
9-3a. The resultant of these forces is obtained by a tip-to-tail addition as
in Fig. 9-3b; and because all the forces on a particle are assumed to be con-
current (the size of a particle is a point), its direction and position are
242 PRINCIPLES OF DYNAMICS [Chap. IX
eee
Wee
indicated by the dashed vector in Fig. 9-8a. By Newton’s second law of
motion, this resultant causes an acceleration a in line with & and directly
proportional to it, or
R= ka (a)
where k is some constant of proportionality.

R &

(c)
Fig. 9-3. — Resultant force on and acceleration of a particle.

If the same particle is now assumed to be in a vacuum, the resultant force


acting upon it is its weight W. By experiment, the acceleration produced
by W is found to be the value of the gravitational constant g which acts
in line with W, as shown in Fig. 9-3c. Again applying Newton’s second law
and assuming the same constant of proportionality, we have
W =kg (b)
Dividing Eq. (a) by Eq. (6) gives
W
If = O a (9-4)

Comparison with Eq. (a) discloses the value of the constant of proportion-
ality to be -- This ratio of weight divided by the gravitational constant
is often called the mass of the particle.
Since Eq. (9-4) is a vector equation in which each term except the scalar
Ww ce ‘ ;
term 7 represents direction as well as magnitude, it may be resolved into
components parallel to a set of rectangular coordinate axes yielding as a
variation of this equation

Tin
g
W
ta Giga (9-5)

Tea
Gee)
Art. 9-6] Effective Force on a Particle. Inertia Force 243

Here X, Y, and Z represent the components of R, and az, dy, and a, repre-
sent the 2, y, and z components of the acceleration.
A point of dispute between engineers and physicists is the system of units
to be used in these equations. To the physicist mass is the basic unit of
matter. The engineer considers force as the basic unit. These differences
can be reconciled by noting that the engineer is concerned primarily with
problems in which the variation in g is negligible; this makes the foot-
pound-second system of units convenient to use. The physicist, however,
requires an absolute system because he is concerned with problems in which
the value of g may vary widely; he accomplishes his purpose by using a
system of units based on mass, length, and time.
Perhaps the whole difficulty can be eliminated by observing that four
quantities are involved — force (Ff), mass (/), length (Z), and time (7) —
which are related by

To the engineer, the F-L-T system is most convenient, but the physicist
prefers the W-L-T system. Thus to the physicist, F is defined in terms of

es the resulting unit of force is called a dyne in the c.g.s. system or a


2
poundal in the English system. To the engineer, M is defined as ans the
resulting unit of mass is sometimes called a slug. Perhaps as good a con-
cept of mass as any is to consider that it represents the inertia of a body,
i.e., the resistance a body offers to a change in its motion.

9-6. Effective Force on a Particle. Inertia Force

The effective force on a particle is defined as the resultant force on the


particle. Since by Eq. (9-4) & and 2 a are numerically equivalent, either
Ww
may be said to be the effective force on a particle. The use of ae to
represent this force is especially convenient in cases where the acceleration
of the particle is known but the actual force system producing this accelera-
tion is not known. This concept is used in the following articles to extend
the laws of motion for a particle to the motion of a body composed of a
system of particles.
According to Newton’s third law, for every force there is an equal but
opposite reaction. In the case of a particle accelerated by a resultant force,
this reaction is called the inertia force of the particle. This force is the equal
but oppositely directed reaction to the resultant force acting on a particle.
244 PRINCIPLES OF DYNAMICS [Chap. 1X

It is convenient to think of the inertia force as a force numerically equal to


- a but directed oppositely to the acceleration.
If the inertia force is considered to act on a particle together with the
resultant force, the particle will be in a state of equilibrium. This will be
called dynamic equilibrium to distinguish it from static equilibrium in
which the particle is at rest or is moving with constant velocity.

9-7. D’Alembert’s Principle


D’Alembert’s principle expresses the relation between the external forces
applied to a system of particles and the effective force on each particle of
the system. It may be stated as follows: The resultant of the external forces
applied to a body (rigid or non-rigid) composed of a system of particles is
equivalent to the vector summation of the effective forces acting on all particles.
Both the resultant of the external force system and the summation of
effective forces are vectorial additions and may be summed up in the fol-
lowing vector equation:

R=—aub—ap---
WwW Ww

g g
The truth of this statement may be seen from Fig. 9-4a. The heavy out-
line denotes the boundary of a body which may consist of a system of

Fic. 9-4. — Resultant of external forces is equivalent to that of the effective forces.

particles either rigidly fastened together or composing a non-rigid body like


a gas or a liquid. The external forces acting on this body are its weight W
and several external forces of which two are shown, P; and Pe. Three
particles of the system composing the body are also shown. (The size of
these particles is exaggerated for convenience in representation.) One of
these particles is assumed to be located at the outer boundary of the body
where it is acted upon by the external force Pi, its own weight w;, and an
Art. 9-8] Motion of the Center of Gravity of Any Body 245

internal force A. The second particle is assumed to be adjacent to the first


particle but is drawn at a distance from it to show the forces more clearly.
This second particle is acted upon by an external force due to its own
weight w. and an internal force B due to the action of the first adjacent
particle, as well as by other internal forces caused by other adjacent par-
ticles. The third particle is assumed to be at the right boundary of the
body where it is acted upon by the external force P2, its own weight ws,
and the internal force D transmitted to it from the second particle by the
action of intermediate particles. For convenience, these intermediate par-
ticles are not drawn. In fact, the discussion is simplified if we temporarily
assume the body to be composed of just these three particles which act
mutually upon each other.
By Newton’s third law of motion, the internal forces A and B are equal
and oppositely directed. Hence any summation involving all the forces
acting on these two particles will automatically cancel such internal forces as
A and B. It is obvious therefore that any vectorial summation of forces
involving all the particles will cancel out all the internal forces between
particles since they occur in equal, opposite pairs. Figure 9-4b shows such
a vectorial summation of both the impressed forces such as P;, P2, and W
(note that W = w; + w. + w3 + - - -) as well as the effective force of each
particle. It is evident that the resultant R of the impressed forces is equiva-
lent to that of the effective forces. Hence we have

Beets Rite Ps pairs etic Gat? 2 * (9-6)

This expresses in equation form d’Alembert’s principle that the resultant of


the impressed forces acting on a body is equivalent to the vector sum of the
effective forces acting on all the particles composing the body.
A useful variation of this principle is obtained by assuming that each
particle is acted upon by a force just equal but opposite to the effective
force, thereby resulting in a set of reversed effective forces which will balance
the impressed forces. As discussed in Art. 9-6, these reversed effective
forces are the inertia forces of the particles. Thus d’Alembert’s principle
may also be stated as follows: The impressed forces acting on any body are
in dynamic equilibrium with the inertia forces of the particles of the body.

9-8. Motion of the Center of Gravity of Any Body


The vectorial relation expressed by Eq. (9-6) can be conveniently handled
by algebraic methods only if each term is resolved into its components.
Considering the X components of each term, for example, we have

2X = “a, + Fas, shares ay (a)


246 PRINCIPLES OF DYNAMICS [Chap. IX
e
OA oe a EE e eS
in which =X represents the algebraic summation of the X components of the
external forces W, Pi, Ps, etc., acting on the body, and 41,, a,, ete., are
the X components of the individual acceleration of each particle. The
; w
masses of the particles composing the body are denoted by me eiare etc.
Let the position of the particles at any instant be represented by (a1, y1, 21),
(a2, Yo, 22), etc. From statics, the X coordinate of the center of gravity of
the system (#) is given by the equation

g g g pa PO ©)
where W is the total weight of the system.
Differentiating Eq. (b) twice with respect to the time and remembering
dx Gia Clee
that ap Take and deo ap oe we have

W . Wi We W3
Og
‘ ee Fe OY ey
ee ey i” “—- home (c)

in which 4G, is the X component of the acceleration of the center of gravity.


Comparison of the right-hand terms in Eqs. (a) and (c) shows them to be
identical; hence we conclude that the left-hand terms must be equal, or

DX = ee (d)
Since a similar procedure can be followed with respect to the Y and
Z axes, the relations between the external forces acting on any body, the
mass of the body, and the acceleration of its center of gravity may be stated
by the following equations:

ox ¥ 4)
We
acid | (9-7)

=Z = diay)
9g
In terms of the resultant force R and the resultant acceleration @ of the
center of gravity, this becomes
W
R=—4
a 2
(9-8)
These equations express the acceleration of the center of gravity of any
body (rigid or non-rigid) in terms of the applied external forces. They
do
Art. 9-9] Applying the Principles of Dynamics 247

not locate the action line of the resultant force. This will be determined later
(see Arts. 10-7, 12-5, and 13-6).
In conclusion we may say that as far as the relation between the resultant
force and the acceleration of the center of gravity is concerned, any system
of particles may be considered equivalent to a single particle, provided that
the particle has the same mass as the mass of the system and the same
motion as the center of gravity of the system.
Whenever there is occasion to treat a body as though its mass were
squeezed to a point located at its center of gravity, we shall use the term
point diagram to define the diagram of the forces.

9-9. Applying the Principles of Dynamics


In this text, we shall apply the laws of dynamics only to bodies that are
considered to be particles or to bodies that are rigid. As we have seen, when
the dimensions of a body are small compared with its path, the body may
be assumed to be a particle. The flight of a projectile is an example. It is
also permissible to consider that any body is a particle having the body’s
weight and the motion of its center of gravity. This procedure is used in
discussing non-rigid bodies.
Although any body may be treated as a particle concentrated at its
center of gravity (Art. 9-8) and equations for its motion may be derived
from this concept, in many cases the angular motion of the body must also
be considered. This requires additional equations derived from the separate
paths followed by the particles composing the body. Such equations can
be obtained only for rigzd bodies in which the particles composing them are
fixed in position relative to each other, because only in rigid bodies can the
motions of particles be related to each other. The particles of a non-rigid
body, such as a jet of water, can and do follow separate, unrelated paths.
Although actual solid bodies are not really rigid, the change in shape
under disturbing forces is usually so small that no appreciable error is made
in assuming them to be rigid. In the following chapters we shall discuss the
various motions of an ideal rigid body. These motions are known as trans-
lation, rotation, and plane motion. The type of motion produced on a
rigid body by any given force system will be shown in later chapters to
depend upon the nature and position of the resultant of that force system.
For the present, we shall merely state the effect produced.
Consider a flat rigid body resting upon a smooth horizontal surface. If
the resultant of the applied force system is a single force passing through
the gravity center of the body as in Fig. 9-5a, the body will move in the
direction of the resultant R, but it will not rotate. If the direction of F is
constant, the motion of the body follows a straight-line path and is called
rectilinear translation. If the direction of R varies, although continuing
248 PRINCIPLES OF DYNAMICS [Chap. 1X

to pass through the gravity center, so will the motion of the body, resulting
in a curved path motion known as curvilinear translation. In either type
of translation, however, a straight line passing through any two particles
will always remain parallel to its initial position.
If the resultant of the applied force system is a couple M as in Fig.
9-5b, the body will spin about a vertical axis through its center of gravity,
but the center of gravity will remain stationary. All particles will describe
horizontal circular arcs about the vertical centroidal axis. This type of

WU OY
motion is called centroidal rotation.

(a) Translation (b) Rotation (c) Plane motion

Fic. 9-5. — Nature and position of resultant of applied forces determines type of rigid-
body motion.

Finally, the resultant of the applied force system may be a single force
which does not pass through the gravity center as in Fig. 9-5c. Recognizing
that the resultant can be transformed into a force acting through the
gravity center plus a couple (refer to Art. 2-8), it is easy to see that the
motion in this case will be a combination of translation and centroidal
rotation, commonly called plane motion.
The converse of the preceding discussion is also true, namely, the type
of motion specifies the nature and position of the resultant force. For ex-
ample, a body that is constrained so that its motion may only be a trans-
lation requires that the applied forces so distribute themselves that their
resultant passes through the center of gravity. Or if a body is constrained
so that it can only rotate about its centroidal axis, the resultant of any
applied force system must be a couple.

SUMMARY
The displacement of a particle is the vector distance from the origin to
the position occupied by the particle on its path of travel. If the particle
travels in a straight line, the displacement and the distance measured along
the path are numerically equal. If the path of travel is curved, the displace-
ment and distance along the path are not equal (see Art. 9-3). Velocity
is defined as the time rate of change of displacement; acceleration is defined
as the time rate of change of velocity.
Summary 249

From these definitions, the differential equations of kinematics (Art. 9-3)


are given by the equations
ds
= ae (9-1)

dv ad’s
a= dt = dé (9-2)

udu = ads (9-3)


The laws of motion for a particle (Art. 9-4) are extended, by means of
d’Alembert’s principle (Art. 9-7), to include a body (rigid or non-rigid)
composed of a system of particles. D’Alembert’s principle states that the
resultant of the impressed forces acting on a body is equivalent to the vector
sum of the effective forces acting on all the particles composing the body.
A useful variation of this principle states that the impressed forces acting
on any body are in dynamic equilibrium with the inertia forces acting on
the particles of the body.
The principle of the motion of the center of gravity of any body (Art. 9-8)
says that the resultant of the applied external forces is equivalent to the
product of the mass of the body and the acceleration of its center of gravity;
it is expressed by the equation
W
R pe (9-8)

This equation is valid either for non-rigid bodies or for the rigid-body
motions of translation, rotation, and plane motion. However, the location
of the resultant force is not specified by this equation.
Chapter X.
Rectilinear Translation

10-1. Definition and Characteristics of Translation


Translation is defined as the motion of a rigid body in which a straight
line passing through any two of its particles always remains parallel to its
initial position. Let Fig. 10-1 represent a translating body at any instant.
A straight line has been drawn through two particles A and B. The position
of the body at a later instant is shown
by the dashed outline. The line A’B’
passing through the same two particles
is parallel to its initial position. Also,
since the body is assumed to be rigid,
the distance separating the particles
remains constant and the particles
have no motion relative to each other.
The term translation cannot be ap-
Fic. 10-1.
— Translation of arigid body. plied to a non-rigid body such as a
liquid or gas because the position of
the separate particles cannot be controlled; the particles may and usually
do follow independent paths.
Translation may be either rectilinear or curvilinear, depending upon
whether the path described by any particle is straight or curved. The
motion of a translating body moving in a straight line is called rectilinear
translation. An example is a block sliding down a plane surface. If the
path of the translating body is curved, the motion becomes a curvilinear
translation. 'To illustrate further, if the path described by A in moving to
position A’ in Fig. 10-1 is straight, the body has rectilinear translation; but
if the path were curved, the body would have curvilinear translation. In
this chapter we shall consider only rectilinear translation; curvilinear
translation will be discussed in the next chapter.
The outstanding kinematic characteristic of the translation of a rigid
body is the fact that all the particles travel the same or parallel paths. It
follows that all the particles have the same values of displacement, velocity,
and acceleration, and the motion may be completely described by the
motion of any particle of the body. The particle usually selected is the one
250
Art. 10-2] Rectilinear Motion with Constant Acceleration 251

at the center of gravity of the body. In other words, a translating body


may be considered as a particle concentrated at its center of gravity.

10-2. Rectilinear Motion with Constant Acceleration


One of the most common cases of straight-line motion is that in which
the acceleration is constant. As will be seen later (Art. 10-6), this condition
arises when a body is acted upon by forces which remain constant in mag-
nitude and direction, such as a freely falling body or a train acted upon by
a constant draw-bar pull. Since it is so common, the student is urged to
memorize the kinematic equations of motion for this case.
The equations may be derived from the differential equations of kine-
matics by starting with the definition of acceleration in Eq. (9-2) written in
the form
dv = aadt (a)

and proceeding to integrate between definite limits, thus

{C =a idt (b)

Note that a is placed outside the integral sign because it is assumed con-
stant.
Let us examine the meaning of the limits. In relation to Fig. 10-2, Eq. (6)
indicates that at some initial po- v, 5
sition A from which time is to be “=> See
measured, there is an initial veloc 394B
ity vo, Whereas at some other po- B F |
sition B reached after a time s
; : : 1G. 10-2.
interval t, the velocity will be »v.
Integrating Eq. (b) and evaluating the limits gives

[vle, = alt;
v— Vo = at
or
v =v, + at (c)

Let us now consider the definition of velocity in Eq. (9-1) written in the
form
ds = uv dt (d)

and again proceed to integrate between definite limits:

fas = i = ike + at) dt (e)


252 RECTILINEAR TRANSLATION [Chap. X
E
aoe) OEE EE e
Note that the variable v must be replaced by its equivalent expressed in
terms of ¢t. In relation to Fig. 10-2, Eq. (e) indicates that at some initial
position A from which time is to be measured, there is a zero initial dis-
placement, whereas at some other position B reached after a time interval
t, the displacement will be s. Integrating Eq. (e) and evaluating the limits
gives .

[sls = [vot + 3 at?)


or
$= Oo 4 3 Ol (f)
Finally, let us consider the remaining differential equation of kinematics
Eq. (9-3) and again proceed to integrate between definite limits.

[oa = af ds (9)

The limits are written as before, since by referring to Fig. 10-2 it is obvious
that at zero displacement the corresponding velocity is v., while at a dis-
placement s it isv. Integrating and evaluating the limits, we obtain
y2 v

Le|,= ot
v2 — v2
= as
2
or
v= v2 + 2as (h)
The three kinematic equations of motion with constant acceleration!
may be summarized as follows:
v=v,-+at (10-1)
s = vot + pal (10-2)
v? = v, + 2as (10-3)
Signs. It is important to observe that these equations involve only the
magnitude of vector quantities. The direction of the vectors of displace-
ment, velocity, and acceleration is indicated by the following sign conven-
tion: The initial direction of motion represents the positive direction for
displacement, velocity, and acceleration. Hence a negative value of ve-
locity obtained in applying the equations would mean that the velocity is
directed oppositely to the initial direction of motion. A negative value for
displacement would indicate that the position of the moving particle is to
be measured back from the origin of displacement. Finally, if a particle
moving along a straight line returns to the starting position, the displace-
ment s in the above equations will be the vector drawn from the origin to
1 Motion with uniform velocity is a special case of constant acceleration in which
the acceleration is zero. Putting a =0 in the equations above reduces them to s = vt.
Art. 10-3] Freely Falling Bodies, Air Resistance Neglected 253

the final position; that is, s will be determined as zero, not the distance
actually traversed by the particle. Pause here to read again the discussion
on p. 239 concerning Fig. 9—1b.

10-3. Freely Falling Bodies, Air Resistance Neglected


It has been seen that the acceleration of a body is directly proportional
to the resultant force acting upon it (Art. 9-8). In the case ofva freely fall-
ing body, this resultant force is its own weight. The weight is a force that
results from the attraction between the mass of the body and the mass of the
earth; it varies inversely as the square of the distance separating the two
centers of mass and is directly proportional to the product of the masses.
Since the mass of any body on the earth is insignificant compared with the
mass of the earth, the gravitational force varies only with the position of
the body relative to the center of the earth’s mass. This variation in
distance assumes significance only when the body undergoes distinct
changes in its position on the earth’s surface, such as being shifted from the
equator to one of the poles or being carried high above the earth in an
airplane. For most cases over a given earth surface, however, the gravita-
tional force and the gravitational acceleration may be assumed to be con-
stant. For our latitude, this acceleration is approximately 32.2 ft per sec?
and is represented by the symbol g. This value of g will be used throughout
this book except as otherwise indicated.
In solving problems on falling bodies, a
specified direction (up or down) is assumed
to represent positive displacement. Velocity i \
and acceleration are positive when directed |
along positive displacement; they are nega- |
tive when pointing in the opposite direction 7
(Art. 10-2). It should be observed that a \
negative value of acceleration does not per |
se indicate a slowing down. We can only |
say that a body is accelerating if its velocity |
is increasing with time, and it is decelerating : aies
Sea
if its velocity is decreasing with time. This
is equivalent to stating that a body is speed-
ing up if the directions of velocity and acceler-
ation are the same, and slowing down if veloc-
ity and acceleration are oppositely directed. |
As an illustration, consider a stone thrown bp
vertically into the air from position A in Fic. 10-3. — Initial direction of
motion determines positive di-
Fig. 10-3. Let the displacement be meas- rections of displacement, ve-
ured as positive upward from A. After a locity, and acceleration.
254 RECTILINEAR TRANSLATION [Chap. X
n
a a S
certain time the stone will reach its topmost position B and then descend.
Throughout the motion, the stone is subjected to a gravitational acceler-
ation which is directed downward and therefore considered negative, 1.e.,
oppositely directed to +s. During the travel from A to B, the velocity
of the stone is positive, i.e., in the direction of positive displacement; but
thereafter the velocity is downward or negative.
During the travel from A to B, v and a have opposite signs and the stone
is slowing down, whereas after the stone leaves B, v and a have the same
sign and direction, and the stone is speeding up. Furthermore, during the
time of travel from A to C, the stone is above the initial position and will
have positive displacement; but after passing C it will have negative dis-
placement measured from A because it will be below the initial position.
In any motion involving freely falling bodies, the general equations of
motion for constant acceleration developed in Art. 10-2 may be applied by
replacing a by g. No restriction on the equations need be made other than
that velocity and acceleration are to be taken as positive in the direction
of positive displacement.

ILLUSTRATIVE PROBLEM
1001. Asshown in Fig. 10-4, a stone is thrown vertically into the air from a tower
100 ft high at the same instant that a
Sey Fea second stone is thrown upward from the
ground. The initial velocity of the first
stone is 50 ft per sec and that of the second
stone is 75 ft persec. When and where will
the stones be at the same height from the
Se ground?
Solution: The initial direction of motion
for each stone is upward. Using the con-
vention established in Art. 10-2, we there-
vo,= 75 ft/sec fore take the upward direction as positive
for s,v, and a. Applying Eq. (10-2) and
noting that the acceleration is g = 32.2
Fic. 10-4. ft per sec? directed downward and therefore
negative, we obtain
[s = vot + 4 at? For stone 1: s, = 50%— 16.12 (a)
For stone 2: s. = 75t — 16.1# (b)
From Fig. 10-4, sx — s, = 100. Hence subtracting Eq. (a) from Eq. (6) gives
Sy Gi S OO = Daz
t = 4 sec
Substituting ¢ in Eqs. (a) and (6), we have
s = 50 XK 4 — 16.1 X (4)? = 200 — 257.6 81 —57.6 ft
S. = 75 X 4 — 16.1 X (4)? = 300 — 257.6 So 42.4ft Ans.
Art. 10-3] Freely Falling Bodies, Air Resistance Neglected 255
c e ee
Hence the stones pass each other 57.6 ft below the top of the tower, or 42.4 ft
from the ground. Note that although we assumed that they would pags above the
tower, the negative sign of s; indicates otherwise. Since the terms involved in the
equations are vector quantities, an incorrect assumption of direction results merely
in a negative sign.

PROBLEMS
1002. On a certain stretch of track, trains run at 60 mph. How far back of a
stopped train should a warning torpedo be placed to signal an oncoming train?
Assume that the brakes are applied at once and retard the train at the uniform
rate of 2 ft per sec?.
1003. A stone is thrown vertically upward and returns to earth in 10 sec. What
was its initial velocity and how high did it go?
1004. A ball is dropped from the top of a tower 80 ft high at the same instant
that a second ball is thrown upward from the ground with an initial velocity of
40 ft per sec. When and where do they pass, and with what relative velocity?
Ans. After 2 sec at 64.4 ft from top of tower; relative velocity = 40 ft per sec
1005. A stone is dropped down a well and 5 sec later the sound of the splash is
heard. If the velocity of sound is 1120 ft per sec, what is the depth of the well?
Ans. 353 Tt
1006. Repeat Prob. 1005 if the sound of the splash is heard after 4 sec.
1007. A stone is dropped from a captive balloon at an elevation of 1000 ft. Two
seconds later another stone is projected vertically upward from the ground with a
velocity of 248 ft per sec. If g is 32 ft per sec?, when and where will the stones pass
each other? Ans. After 5 sec; 600 ft from the ground
1008. A stone is thrown vertically upward from the ground with a velocity of
48.3 ft per sec. One second later another stone is thrown vertically upward with a
velocity of 96.6 ft per sec. How far above the ground will the stones be at the
same level?
1009. A ball is shot vertically into the air at a velocity of 193.2 ft per sec. After
4 sec, another ball is shot vertically into the air. What initial velocity must the
second ball have in order to meet the first ball 386.4 ft from the ground?
Ans. Uo = 158.5 ft per sec
1010. A stone is thrown vertically up from the ground with a velocity of 300 ft
per sec. How long must one wait before dropping a second stone from the top of a
600-ft tower if the two stones are to pass each other 200 ft from the top of the tower?
Ans. t = 138.67 sec
1011. A ship being launched slides down the ways with a constant acceleration.
She takes 8 sec to slide the first foot. How long will she take to slide down the ways
if their length is 625 ft? Ans. t = 3 min, 20sec
1012. A train moving with constant acceleration travels 24 ft during the 10th
sec of its motion and 18 ft during the 12th sec of its motion. Find its initial velocity
and its constant acceleration. Ans. Uo = 52.5 ft per sec; a = —8 ft per sec?
256 RECTILINEAR TRANSLATION [Chap. X

1013. An automobile starting from rest speeds up to 40 ft per sec with a constant
acceleration of 4 ft per sec?, runs at this speed for a time, and finally comes to rest
with a deceleration of 5 ft per sec?. If the total distance traveled is 1000 ft, find the
total time required.
1014. A train travels between two stations 3 mile apart in a minimum time of
41 sec. If the train accelerates and decelerates at 8 ft per sec’, starting from rest at
the first station and coming to a stop at the second station, what is its maximum
speed in mph? How long does it travel at this top speed?
Ans. Max v = 60 mph
1015. Two cars A and B have a velocity of 60 mph in the same direction. A is
250 ft behind B when the brakes are applied to car B, causing it to decelerate at the
constant rate of 10 ft per sec?. In what time will A overtake B, and how far will
each car have traveled?
1016. An automobile moving at a constant velocity of 45 ft per sec passes a
gasoline station. Two seconds later, another automobile leaves the gasoline station
and accelerates at the constant rate of 6 ft per sec?. How soon will the second auto-
mobile overtake the first? Ans. t = 16.7 sec
1017. A balloon rises from the ground with a constant acceleration of 4 ft per
sec?, Five seconds later, a stone is thrown vertically up from the launching site.
What must be the minimum initial velocity of the stone for it to just touch the bal-
loon? Note that the balloon and stone have the same velocity at contact.

10-4. Rectilinear Motion with Variable Acceleration


When bodies are acted upon by variable forces, they move with variable
acceleration. To determine the kinematic equations of motion in such
cases, it is necessary to apply the given data to the differential equations of
kinematics (Art. 9-3). Since the acceleration may vary in many ways,
no general equations can be stated as was done in the case of constant
acceleration (Art. 10-2). At most we can only indicate the procedure to be
followed.
Basically we have three principal variables, s, v, and a, related by a com-
mon parameter ¢ as in the following box.

§
v t
a

Hach of these principal variables may be expressed in terms of the time,


or they may be expressed in terms of each other or even a combination of
the others. Let us consider here the simpler combinations.
Case I: The displacement is given in terms of the time; i.e., s = f(t), to
find v and a.
This is the simplest case and is easily solved by successive differentiation
Art. 10-4] Rectilinear Motion with Variable Acceleration 257

of the displacement with respect to the time according to the fundamental


ds
definitions of velocity ¢ = =) and acceleration («= 2 = Tay As an

example, let it be required to determine the velocity and acceleration of a


body after 3 sec, if the motion is defined by the relation s = 54+ 48, s
being in feet and ¢ in seconds.
Differentiating, we obtain the equations of velocity and acceleration

niceg 2 gee ay
at
and dy

Hence at t = 3 sec,
v= 5+ 12 X (3)? = 118 ft per a *
be ns.
a = 24 X 3 = 72 ft per sec?
Case II: The acceleration is expressed in terms of the time;i.e., a = f(t),
to find v and s.
The general procedure is to start with Eq. (9-2) written in the form
dv = adt and integrate to find the velocity in terms of the time.
We may now apply Eq. (9-1) written in the form ds = v dt and likewise
integrate to determine the displacement in terms of the time.” Eliminating
the time between these two relations will give an equation between velocity
and displacement.
As an example of Case II, let it be required to determine the velocity
and displacement of a body after 2 sec, if the motion is defined by the re-
lation a = 2 t, a being in feet per second? and ¢ in seconds, and if it is known
that s = 4ft and v = 2 ft per sec when t = 1 sec.
Applying dv = a dt and integrating between the given limits, we have
v t t

[oa
2
= [a=
1
J1 2eae
whence
OO
eae eat
or
p= 1 (a)
We now replace the variable v, just found in terms of the time, in the
differential equation ds = v dt and again proceed to integrate between the
given limits. This gives
8 t t

[as = ff(@+1)dt or [sk = E;+ |


4 1 1
whence
ee tart
Sai ah a eal en or ont
heey tal chal (b)

2The special case of motion with constant acceleration is a variation of Case II.
Exactly this procedure was followed in Art. 10-2.
258 RECTILINEAR TRANSLATION [Chap. X
o
SOS EIB yee
Hence, if ¢ = 2 sec, substitution in (a) and (6) gives

y= 5ftpersec and s=7.33ft Ans.

Case III: The velocity is given in terms of the time; ie., v = f(t), to
find a and s.
This case is a combination of Cases I and II. Differentiating the given
: : : d i oe
velocity-time relation determines the acceleration (ie. a= 4 while in-

tegrating it determines the displacement (i.e., f/ds = fv dl).


OTHER Cases: When the principal variables are not given as functions of
the time, direct differentiation or integration cannot be performed as above
without a preliminary treatment. There are two main variations which
follow:
Case IV: One of the principal variables is expressed in terms of an ad-
jacent variable; i.e., a = f(v) or v = f(s).
The procedure here is to use either a = = ory = e to relate the given

variables in terms of the time and thereby reduce the problem to one of the
three preceding cases. For example, if we are given a = f(v), apply
a= a = f(v). Separating the variables gives

t v dv

Je S56
which is integrated to give v in terms of the time t, which is Case III above.
Case V: The given variables are not adjacent; i.e., a = f(s).
In this case, we substitute the given
relation in v dv = ads, separate the
variables and integrate to obtain one
variable in terms of its adjacent var-
lable. Thus we obtain Case IV and
proceed as indicated therein.

ILLUSTRATIVE PROBLEM

1018. A rope of length L connects the


wheel A and the weight B by passing over
a pulley of negligible size at C as shown in
Fig. 10-5. At the instant when z = 9 ft,
the center of wheel A has a velocity
va = 10 ft per sec and an acceleration
Art. 10-4] Rectilinear Motion with Variable Acceleration 259

aa = 4ft per sec’, both rightward. What is then the velocity and acceleration of B?
Solution: If we denote the variable distance AC by z, the vertical length CB =
L — zand hence
h=L-e+y (a)
From the figure we also have
2= 2+ fh? (b)
By eliminating z from these relations, y may be expressed directly in terms of x so
that successive differentiation with respect to the time will relate the velocity and ac-
celeration of B to that of A. (See Prob. 1022 below.) However, a preferable method
is to retain z as a parameter and proceed as follows:
By differentiating Eq. (a) with respect to the time and noting that uy= Vp, we
obtain
dz dy _ dz
Dar ice iy. or Cay (c)

This result is not surprising if we note that the change in length of z determines the
rise (or fall) of B.
We next differentiate Eq. (b) with respect to the time which gives
dz dx
227 = 247 or A, = aan (d)

Another differentiation of Eq. (d) yields


dvz dz dvs de
A oka ae eae dt
or
zap +vp =ta,tovs (e)
By substituting the given data in Eqs. (d) and (e) and noting that z = 15 ft when
xz = 9 ft, we obtain

[From Eq. (d)] 15 vp = 9(10) vp = 6ftpersecup Ans.


[From Eq. (e)] 15 ap + (6)? = 9(4) + (10)? ap = 6.67 ft per sec? up Ans.

PROBLEMS
2 ie
1019. The motion of a particle is given by the equation s = 2 t — 6 +2
where s is in feet and ¢ in seconds. Compute the values of » and a when t = 2 sec.
1020. A particle moves in a straight line according to the law s = t® — 40 ¢ where
sisin feet andtinseconds. (a) Whent = 5sec, compute the velocity. (b) Find the
average velocity during the fourth second. (c) When the particle again comes to
rest, what is its acceleration?
Ans. (a) v = 35 ft per sec; (b) ave. v = —3 ft per sec; (c) a = 21.9 ft per sec’?
1021. A ladder of length L moves with its ends in contact with a vertical wall
and a horizontal floor. If the ladder starts from a vertical position and its lower
260 RECTILINEAR TRANSLATION [Chap. X

end A moves along the floor with a constant velocity v4, show that the velocity of its
upper end B is vg = —v, tan 6 where 6 is the angle between the ladder and the wall.
What does the minus sign mean? Is it physically possible for the upper end B to
remain in contact with the wall throughout the entire motion? Explain.
1022. Check the answers to Illus. Prob. 1018 by the following method: Write an
expression relating 2 and y, and by successive. differentiation show that
LVA CAA h We
Up = Veo ap = Wea s Vie
+ 2) Compute vz and az from these

relations.
1023. The rectilinear motion of a particle is given by s = v? — 9 where s is in
feet and v in feet per second. When t = 0, s = 0 and v = 8 ft per sec. Find the
s-t, v-t, and a-t relations.
1024. The velocity of a particle moving along the X axis is defined by v =
x’ — 4x?-+ 6x where v is in feet per second and z is in feet. Compute the value of
the acceleration when x = 2 ft. Ans. a= 8 ft per sec?
1025. The motion of a particle is defined by the relation a = 4 ¢, where a is in
feet per second? and ¢ in seconds. It is known that s = 1 ft and v = 2 ft per sec
when t = 1 sec. Determine the relations between v and t, s and t, v and s. is
Ans. 0 =20;38 =2641;0°? = (88 —1)V2
; See: : 8
1026. The motion of a particle is governed by the equation a = — 2 where a
is in feet per second? and s isin feet. Whent = 1sec,s = 4ftandv = 2 ft per sec.
Determine the relations between v and t, s and ¢, v and s.
1027. The motion of a particle is given by a = 6v'’””, where a is in feet per second?
and v in feet per second. When tis zero, s = 6ftandv = 0. Find the relations be-
tween v and ¢, s and ¢, v and s. Ans. S= 3 —- 639 = 92
1028. The motion of a particle is governed by the relation a = 4 #?, where ais
in feet per second? and t is in seconds. When fis zero, v = 2 ft persec and s = 4 ft.
Find the values of v and s when t = 2 sec.

10-5. Motion Curves

The motion of most machines is usually too complex to be analyzed


mathematically. Instead, a graphical analysis like that about to be de-
scribed is used. The method of graphical analysis is particularly useful
when applied to problems involving variation of displacement, velocity, or
acceleration with time.
Figure 10-6 represents the schematic outline of an oscillating arm quick-
return mechanism as used on shapers. As the crank A rotates clockwise
at a constant speed, block B slides along arm C thereby causing C to
oscillate between the two extreme positions C’ and C’’.. This action causes
the cutting tool carried by the ram F to move through the working stroke
Art. 10-5] Motion Curves 261

FL F F’

Sark
et me ori |

Bee we

ae a ae
oN 6
S83 Hew

.
vi

Fic. 10-6.

F’ to F” in a longer time than it takes to return the ram from F’”” back to
F’. Hence the name ‘“‘quick-return”’ mechanism.
If we now divide the crank circle into equal angular divisions, each
representing equal time intervals since the crank rotates at constant speed,
we can graphically determine the position of the ram F for each of these
equally timed intervals. These displacements of F from an origin at F’
have been plotted for one cycle in Fig. 10-7a to give the displacement-time
or s-t curve shown.

The slope of this s-t curve at any instant is evidently the ratio of at this
instant. But since as = v, determining the slopes of successive tangents to
dt
the s-t curve will permit us to calculate the corresponding velocities at these
positions. Plotting these velocities against the corresponding instants of
time gives the velocity-time or v-t curve shown in Fig. 10-7b. Having thus
graphically differentiated the s-t curve to obtain the v-t curve, the process
may be repeated with this derived v-t curve. Each of a succession of
tangents to the v-t curve has the slope 2 which, being equal to the accel-
eration a at the corresponding instant of time, will permit us to compute
the variation of acceleration with time. In this way, the acceleration-time
or a-t curve shown in Fig. 10-7c may be plotted. It will be noticed that at
time ft, the slope of the v-¢ curve is down to the right (i.e., has a negative
slope), and hence the acceleration a2 is plotted below the ¢ axis as a negative
262 RECTILINEAR TRANSLATION [Chap. X

Fig. 10-7. — Relations between s-t, v-t, and a-t curves.

value, whereas at time ¢t; the slope of the v-t curve is zero which denotes
zero acceleration. Similarly at time ¢,, the tangent to the s-t curve slopes
up to the right (i.e., has a positive slope), and hence the velocity v, at this
instant is plotted above the ¢ axis as a positive value.
From a practical viewpoint, this process of graphical differentiation is
quite tedious and requires drafting skill of a high order. It is not often
used. It has been described to emphasize the following relations between
these curves:

v= a = (Slope),_: (10-4)

=_ ©dv = (Slope), (10-5)


Art. 10-5] Motion Curves 263

They state that the slope of the displacement-time curve is equal to the
corresponding ordinate of the velocity-time curve and that the slope of the
velocity-time curve is equal to the corresponding ordinate of the acceler-
ation-time curve. Positive values of v and a result from positive slopes
(i.e., up to the right) of the s-t and v-t curves respectively, and conversely,
negative values of v and a are caused by negative slopes (i.e., down to the
right) of these curves.
Consider next the reverse of this procedure. By starting with the a-t
curve, it is possible to construct the v-t curve and then the s-t curve. This
procedure is called graphical integration and is achieved as follows:
If the definition of acceleration is written in the form dv = a dt and inte-
grated between the times ¢; and tf, we obtain
09 ty
av = f w = | a dt (a)

The geometric significance of the right-hand term is apparent from Fig.


10-7ce. During the infinitesimal time interval dt, the acceleration a may
be considered constant. Obviously a dt is the area of the unshaded rec-
ty
tangular strip. Since ifadt means the summation of such strips, we
ty
conclude that the shaded area under the a-¢ curve between the times ¢; and
ts represents the velocity change Av during this time interval, or
Vv. — v,; = Av = (Area)a_1 (10-6)
The value of Av is shown directed downward on Fig. 10-7b since the cor-
responding area on Fig. 10-7c for this time interval is negative.
Similarly we can show that the area under a v-t curve is equal to the
change in displacement during the time interval considered. Starting
with the definition of velocity written in the form ds = v dt, we integrate
between definite limits to obtain
8q tg
as= [as | v dt (b)

Here v dt is the area of the unshaded rectangular strip in Fig. 10-7b. The
ty
sum of such strips being i v dt, we find that the shaded area under the
ty
y-t curve between the times ¢, and f, represents the change in displacement
As during this time interval, or
So — S; = As = (Area),_; (10-7)

This value of As is shown directed upward on Fig. 10—7a since the corre-
sponding area on the v-t curve for this time interval is positive.
264 RECTILINEAR TRANSLATION [Chap. X

If desired, the change in displacement As may also be calculated directly


from the area under the a-t curve. From Eq. (a), the velocity at any instant
may be written as
‘ t

v=n+ [od (c)

If we substitute this general value of v in ds = v dt and integrate, we obtain


So te te t

As [ras= Poa= P(t f aa)a


8; ty 1 1
te "i
01 (te = ti) + { (fa at)
dt

which, after integrating the second right-hand term by parts reduces to

As = V1 (to ay t1) + ie (a dt) (to = t) (d)

To interpret the meaning of the second right-hand term, we note that a dt


is the area of an elemental strip under the a-t curve and that f — tis the
moment arm of this elemental strip about an ordinate through ft. The
integral is the summation of moments of all such strips and is equivalent to
the moment of area about the ordinate at f of that part of the a-t curve
included in the time interval tf — 4. Hence Eq. (d) may be rewritten as
As =. te tee (Area,ae G (10-8)

which states that the change in displacement As in any time interval tg — t


is equal to the velocity v; at the start of the interval multiplied by the time
interval plus the moment of area about & of that part of the a-t diagram
included in the interval t — t.
The application of these relations between motion curves is demonstrated
in the following illustrative problems. In this connection, Table X-1 will
be helpful in computing areas and moments of areas. Note the uniform
manner in which the coefficients of the terms increase.

ILLUSTRATIVE PROBLEMS
1029. A particle, starting with an initial velocity of 60 ft per sec, has a rectilinear
motion with a constant deceleration of 10 ft per sec?. Determine the velocity and
displacement at the end of 9 sec by sketching the a-t, v-t, and s-¢ curves and using the
relations between them.
Solution: This elementary problem is easily solved by using the equations of
constant acceleration, but its very simplicity will develop confidence in applying the
3 When this equation is applied to the special case of constant acceleration, we obtain
the familiar relation s = v,t + 4 al?.
Art. 10-5] Motion Curves 265

TaBLE X-1. PROPERTIES OF AREAS

Location of

(zero degree)

i (2nd degree)

(3rd degree)

y=hkx?

(nth degree)
266 RECTILINEAR TRANSLATION [Chap. X

relations between motion curves to more complex cases. Since the acceleration is
constant but negative, the slope of the v-t curve is likewise constant and negative
or directed down to the right as shown in Fig. 10-8. The velocity change in the
9 sec interval is
[Av = (Area) a—1] Av = —10 X 9 = —90 ft per sec
which decreases the velocity to —30 ft per sec.

—10 ft/sec?”

Slope =a=~—10 ft/sec?


i 3) .

— 30 ft/sec

ee
45’

Fic. 10-8.

Evidently the velocity is zero at t, = 6 sec so that the change in displacement can
be computed as the algebraic sum of the positive and negative areas A; and A» under
the v-t curve. Hence
ee ae As = 4(60)(6) — 4(30)(8) = 180 — 45 = 135 ft
The shape of the s-t curve is determined from Eq. (10-4); i.e., the velocity
ordinate equals the slope at the corresponding ordinate of the s-t curve. Thus at
t = 0, the tangent to the s-t curve is up to the right, becoming less steep and even-
tually horizontal at ¢ = 6 sec as the corresponding velocity ordinates gradually re-
duce to zero. Thereafter the tangents to the s-t curve are increasingly steeper down
to the right as the velocity ordinates become increasingly negative. Observe that
the s-t curve is a symmetric parabola with its vertex at ¢ = 6 sec because at equal
intervals to either side of this instant, the velocities are numerically equal but of
opposite sign, thereby producing equal slopes but of opposite directions.
The change in displacement may also be computed by means of Eq. (10-8). This
gives
Art. 10-5] Motion Curves 267

[As = v(t. — ty) + (Area)a_t- 3] As = 60(9—0) + (— 10 X 9) ($) = 185 ft


which is identical to applying s = vot + 4 at?.
Finally, we observe that the shape of the s-t curve indicates that the moving
particle reaches a maximum rightward displacement at ¢ = 6 see after which it
returns leftward. The total distance traveled is the sum of these travels and is
given by the sum of A; and A» or 180 + 45 = 225 ft.
1030. Draw the v-t and s-t curves for the two stones having the motions described
in Illus. Prob. 1001 on page 254. The first stone is thrown vertically up from a
tower 100 ft high with a velocity of 50 ft per sec at the same instant the second stone
is projected upward with a velocity of 75 ft per sec. When and where are the stones
at the same level?
Solution: Both stones move with a downward or negative acceleration of
g = —32.2 ft per sec?. Consequently from Eq. (10-5); i.e., a = (Slope),—z, both
v-t curves have the same slope. But since the stones start with different velocities,
their v-t curves are parallel as shown in Fig. 10-9. It is apparent that the stones
have a constant relative velocity of 75 — 50 = 25 ft per sec.
v (ft/sec)

et (sec)
Relative velocity
constant at
25 ft/sec

Choosing the initial position of stone 2 as a common origin for displacement, we


draw the s-t curves as shown. Since the initial relative displacement of 100 ft be-
tween the stones is being reduced at the constant relative velocity of 25 ft per sec,
the stones meet in 4 sec. This checks the value previously computed in Prob. 1001
and explains the physical significance of that computation. The common height s
at this instant is most easily computed from s = vot — 3 gt? = 75(4) — 2(32.2)(4)? =
42.4 ft.
268 RECTILINEAR TRANSLATION [Chap. X
00
a (ft/sec?)

13 t (sec)

-12

pee sec 3 sec + 6 sec

v (ft/sec) 38

Fig. 10-10.

1031. The a-t curve for a particle having rectilinear motion is shown in Fig.
10-10. Att = 0, the velocity is 8 ft per sec and the particle is 60 ft to the left of the
origin of displacement. Draw the v-t and s-t curves, specifying values of v and s at
t = 4sec, 7 sec and 18 sec.
Solution: The changes in velocity during the specified time intervals of 4 sec,
3 sec, and 6 sec are found by applying Eq. (10-6) to the area under the a-t curve as
follows:
Art. 10-5] Motion Curves 269
a
LO RY Ee
[Av = (Area)
,_ 1] Av: = 3 X 4 X 6 = 12 ft per sec
Av. = 3 X 6 = 18 ft per sec
oe6
Av; = -3x6--~" = —18
— 27 = —45 ft per sec

The velocity ordinates are found by adding Av, = 12 to the initial velocity of
8 ft per sec to obtain 20 ft per sec; to this result is added Av. = 18 to give 38 ft per
sec; from this value we subtract Avs; = 45 to obtain —7 ft per sec at ¢ = 13 sec.
The v-¢ curve is drawn through these points, its shape being determined by Eq.
(10-5) so that the slope of the v-t curve at any instant is equal to the corresponding
value of acceleration.
Now we may apply Eq. (10-7) to the area under this v-¢ curve to obtain the follow-
ing changes in displacement.
[As = (Area) »_4] As, = 8 X 4+ 4(12)(4) = 48 ft
As, = (20)(3) + 4(18)(3) = 87 ft
The last increment of displacement As; cannot be easily found from the area under
the v-t curve. Instead we find it by applying Eq. (10-8) to the last 6-sec interval.
The velocity at the start of this interval is 38 ft per sec. The moment of area under
the a-t curve is found as the sum of the moments of areas of its rectangular and
triangular parts. Hence we obtain
[As = v;(t2 — ti) + (Area)a—e- t3]

Ass = (88)(6) + (—3(6(51 x 6) + —9)(6)


—M® (2(2 6)
= 228— 54 — 108 = 66ft
- The required values of displacement are obtained by adding As; = 48 to the initial
displacement of —60 to give s = —12 ft att = 4 sec; then adding As, = 87 to this
result to obtain s = 75 ft at t = 7 sec; finally adding As; = 66 to this value to
obtain s = 141 ft att = 13 sec. The shape of the s-t curve which is drawn through
these points is determined by Eq. (10-4); i.e., the slope of the s-t curve at any
instant is equal to the corresponding value of velocity.

PROBLEMS
1032. Solve Prob. 1012 on page 255 by analyzing the v-t curve of its motion.
1033. From the v-t curve in Fig. P-1033, determine the distance traveled in 4
sec and also in 6 sec. Also sketch the a-t and s-t curves approximately to scale.
1034. The motion of a particle starting from rest is governed by the a-t curve
shown in Fig. P-1034. Sketch the v-t and s-t curves. Determine the displacement
at t = 9 sec. Ans. s = 228 ft
1035. The curved portions of the v-t curve shown in Fig. P-1035 are second degree
parabolas with horizontal slope at t = 0 and t = 12 sec. Sketch the a-t and s-t
curves if s, is zero. Check values of s using both Eqs. (10-7) and (10-8).
Ans. s = 282 ft att = 18sec
270 RECTILINEAR TRANSLATION [Chap. X

v(ft/eec )

a (ft/sec?)
12

4 6 ¢t(sec) 6 9

Fic. P-1033. Fic. P-1034.

1036. A particle having an initial velocity of 200 ft per sec decelerates according
to the a-t curve in Fig. P-1036. Compute the change in displacement in the time
interval of 30 sec.

v (ft/sec) 30 |a (ft/sec*)

30

6 12 18
Fra. P-1035. Fic. P-1036.

1037. A car moving at 60 ft per sec is brought to rest in 12 sec with a decelera-
tion which varies uniformly with time from 2 ft per sec? to a maximum deceleration.
Compute the distance traveled in stopping. Ans. s = 482 ft
1038. A car starts from rest and reaches a speed of 48 ft per sec in 15 sec. The
acceleration increases from zero uniformly with time for the first 6 see after which it
remains constant. Compute the distance traveled in 15 see. Ans. s = 294 ft

1039. A car accelerates for ¢ sec from rest to a velocity of 48 ft per sec, the accelera-
tion increasing uniformly from zero to 12 ft per sec. During the next 4 sec, the
car decelerates at a constant rate to a velocity of 4 ft per sec. Sketch the a-t, v-t,
and s-¢ curves.

1040. An object attains a velocity of 16 ft per sec by moving in a straight line


with an acceleration which varies uniformly from zero to 8 ft per sec? in 6 sec. Com-
pute its initial velocity and the change in displacement during the 6-sec interval.
Solve by using motion curves and check by calculus.
Ans. UV. = —8 ft per sec; As = 0
1041. The acceleration of an object decreases uniformly from 8 ft per sec? to
zero in 6 sec at which time its velocity is 10 ft per sec. Find the initial velocity and
Art. 10-6] Kinetics of Rectilinear Translation. Analysis as a Particle 271

the change in displacement during the 6-sec interval. Solve by using motion curves
and check by calculus. Ans. vo = —14 ft per sec; As = 12 ft

10-6. Kinetics of Rectilinear Translation. Analysis as a Particle


In rectilinear translation of a rigid body all particles of the body move
in parallel straight lines, and hence the displacement, velocity, and accel-
eration of any particle are parallel to the line of motion. The kinetic
equations of rectilinear translation are obtained from the general equation
governing the motion of the center of gravity (see Art. 9-8). There it was
shown that any body could be treated as though it were a particle which
had the same mass as the body and the same motion as the center of gravity
of the body, i.e., R = a When this equation is applied to the motion
of rectilinear translation in which all particles have the same acceleration,
the bar sign which refers to the center of gravity can be omitted. Thus we
obtain R = va, which is identical with Eq. (9-4) for the motion of a
particle.
It is convenient to take the line of motion as the X axis. Let this axis be
considered positive in the initial direction of motion. Using this convention,
we consider displacement, velocity, acceleration, and X components of
forces as positive when directed in the initial direction of motion. Since the
resultant force must act parallel to the direction of motion, its magnitude
is determined by 2X, which represents the summation of the X components
of the forces acting on the body.
From the preceding discussion, the general equations of kinetics, when
applied to rectilinear translation, become

EX - - " (10-9)
xY=xZ=0
Note carefully that the X axis coincides with the line of motion of the body,
and that it is considered positive in the initial direction of motion.
A general plan for the solution of problems consists of the following steps:
1. Draw a free-body diagram for each body involved in the problem.
Indicate thereon all forces, both known and unknown, representing the
latter by an appropriate symbol. If the direction of any unknown (except
friction) is incorrectly assumed, the solution will yield its correct magnitude
but with a negative sign. Friction forces, however, must always be directed
to oppose the motion.
2. Determine the direction of motion if not evident or specified and indi-
cate it by a dashed arrow near each free-body diagram.
272 RECTILINEAR TRANSLATION (Chap. X
Oe Oe ee eee
3. Determine the kinematic relations between the bodies involved in the
problem.
4. Select the X axis as positive in the direction of motion and apply
DY = 0 and 2X = “a to each body.
5. Solve for the unknowns, using such additional equations of kinematics
as may be required to determine relations betwéen s, v, and ¢.

ILLUSTRATIVE PROBLEMS
1042. The coefficient of kinetic friction under the 200-lb block in Fig. 10-11 is
0.20. The pulleys are assumed to be frictionless and weightless. Determine the
acceleration of each block and the tension in each cord.

(a) (b) () |x
Fig. 10-11.

Preliminary: We begin by determining the direction of motion of the system.


This is accomplished by assuming that one part of the system does not move and
computing the tension necessary to keep it at rest. With this assumption, if any
part of the stystem is found to be acted on by an unbalanced force, that part will
move in the direction of that force. When friction is involved, as in this problem,
its value to maintain equilibrium must be compared to the available friction. If
less than the available friction, motion will not occur. If the friction force required
for equilibrium is more than the available value, motion will ensue in a direction
opposite to the friction force which then acts at its maximum available value.
For example, the maximum tension necessary to keep the 300-lb block at rest is
ry

T. = 300 lb. Since the pulley is weightless, 2X = uda, when applied to it, reduces
to ZX = 0. Hence from the FBD of the pulley we obtain T, = 2 T; or T; = 150 |b.
This value of 7; can now be applied to the 200-lb block. Friction being tem-
porarily neglected, a force summation parallel to the incline is
2X = T; — 200 sin 30° = 150 — 100 = 50 lb
Thus apparently an unbalanced force of 50 lb is acting up the incline. Before con-
cluding that motion actually takes place we must consider the retarding effect of
Art. 10-6] Kinetics of Rectilinear Translation. Analysis as a Particle 273

the friction force. From Fig. 10-11b, ZY = 0 gives N = 173.2 lb, and the friction
relation determines the available friction to be
F =fN = 0.2 X 173.2 = 34.64 lb
However, this is not sufficient to balance the 50-lb force previously found, and we
therefore conclude that the 200-lb block moves up the incline. Obviously an up-
ward motion of this block must permit the 300-lb block to move downward, the
directions of motion being indicated by the dashed acceleration vectors a, and a»
adjacent to the FBD of each block. Observe that the actual tensions differ from
those used above with the assumption of no motion. 7, must be less than the weight
of the 300-lb block to permit it to move down while 7; must be less than 150 lb but
more than 100 + F = 134.64 lb to permit the 200-lb block to move up the incline.
It should also be noted that the 300-lb block moves only one-half the distance
of the 200-lb block; hence values of velocity and acceleration for the 300-lb block
are also one-half the corresponding values for the 200-lb block. Expressed mathe-
matically, this gives
8 =Fy ve = $y a2 =F (a)
the subscript 2 referring to the 300-lb block and the subscript 1 to the 200-Ib block.
Solution: Applying Eq. (10-9) to each block and noting that the X axis is positive
in the initial direction of motion, we obtain
: 200
[er - | 200-lb block: TT, — 34.64 — 200sin 30° = 39.9% (b)

300.5 1
300-lb block: 300 — 2 Es = 32.2 x 5 a (c)

Multiplying Eq. (6) by 2 and adding the product to Eq. (c) eliminates 7, and
yields
550
300 — 269.28 = 39.9 ay

from which
a, = 1.796 ft per sec? Ans.
Substituting this value of a; in Eq. (0) gives
200
T, —— 134.64 SS .796 = 11.18
39.9 OLE

whence
T, = 145.77 lb Ans.
Incidentally, it is highly improbable that the coefficient of friction will be pre-
cisely 0.20 or the value of g be exactly 32.2. Therefore the numerical results ob-
tained above contain too many significant places. More reasonable answers are
a, = 1.80 ft per sec?, and T; = 146 lb; a2 = 3a = 0.90 ft per sec’, and T, =
O29 2Moe
1043. In the system of connected blocks shown in Fig. 10-12, the coefficient of
friction under blocks A and C is 0.20. Compute the acceleration of each block and
the tension in the cord.
274 RECTILINEAR TRANSLATION [Chap. X
Oa

Th AT Oy ie
Xa poe ‘

Ve 320 Ib

240 Ib
1 -—Y aN.
| = N=3201b ,
dp 800Ib
1

|x
Fig. 10-12.

Solution: To simplify the computations, represent the weights of A and C by


components acting parallel and perpendicular to the inclines as shown on the free-
body diagrams. If we now temporarily assume B is at rest, 2 7 = 800 lb which
gives T = 400 lb acting on A and C. From force summations parallel to the in-
clines, the friction forces required to prevent motion of A and C are for A: 600 — 400
= 200 lb and for C: 400 — 240 = 160 lb. However, from ZY = 0 and F = fN,
we find the available friction forces on A and C to be F4 = 0.2 X 800 = 160 lb and
Fo = 0.2 X 320 = 64lb. These are insufficient to keep A and C at rest and are the
actual friction forces acting as A therefore slides down and C slides up.
We do not yet know the motion of B because although the downplane motion of
A tends to raise B, the upplane motion of C tends to lower B. The net motion of B
is the difference between the partial motion sz’ of B caused by C while A is held at
rest and the partial motion sg” of B due to A while C is held at rest. Noting that
these partial motions are only one-half the absolute motions of C and A, we obtain

with A at rest: sz’ = 3s8¢


with C at rest: sp’ = 4s,

Assuming that B moves down, its net motion is the difference of these partial
motions or
1 1
= 5 St Op SC) oy Gh
Art. 10-6] Kzinetics of Rectilinear Translation. Analysis as a Particle 275

whence, by differentiating this displacement relation twice with respect to the time,
we obtain the following relation between the accelerations:
ap =3ac —3aa (a)
We are now ready to apply Eq. (10-9) to each block. Note carefully that in each
case {X is applied as positive in the direction of motion indicated by the dashed
acceleration vectors adjacent to each FBD.

W 1000
r2X Fi«|
=—a Foror A: 600 60 —— TT = —
— 160 39.9 4 (b)

400
ForC:
or T — 6 64 — 240 estes
39,9 AC (c)

800
Foror B: 800 -— 2T = 39.9
—— 08 (d)

When az is replaced by 3 ac — aa from Eq. (a), these equations are easily solved
for their common term 7 by multiplying Eq. (6) by 7’5, Eq. (c) by —1, and adding
them, whence we obtain
ft = ByZ ile
and then, by substitution,
aa = 2.05 ft per sec?
ap l|= 1.89 ft per sec?
ac |= 5.83 ft per sec?
The positive values of these accelerations confirm our assumptions of the direction
of motion of each block.

PROBLEMS

1044. An elevator weighing 3220 lb starts from rest and acquires an upward
velocity of 600 ft per min in a distance of 20 ft. If the acceleration is constant,
what is the tension in the elevator cable? Ans. T = 3470 lb
1045. A man weighing 161 lb is in an elevator moving upward with an accelera-
tion of 8 ft per sec?. (a) What pressure does he exert on the floor of the elevator?
(b) What will the pressure be if the elevator is descending with the same accelera-
tion? Ans. (a) 201 lb; (b) 121 lb
1046. The block in Fig. P-1046 reaches a velocity of 40 ft per sec in 100 ft, start-
ing from rest. Compute the coefficient of kinetic friction between the block and the
ground.

Fic. P-1046. Fig. P-1047.


276 RECTILINEAR TRANSLATION [Chap. X

1047. Determine the force P that will give the body in Fig. P-1047 an accelera-
tion of 6 ft per sec?. The coefficient of kinetic friction is 0.20.
Ans. P = 135.4 Ib
1048. A magnetic particle weighing 3.6 grams is pulled through a solenoid with
an acceleration of 6 meters per sec?. Compute the force in pounds acting on the
particle. Note: 1lb = 454 grams and lin. = 2.54 cm. Ans. F = 0.00485 lb
1049. When a 644-Ilb boat is moving at 10 ft per sec, the motor conks out. How
much farther will the boat glide, assuming its resistance to motion is 2 v lb where v is
in feet per second? Ans. s = 100 ft
1050. A bullet weighing 1 lb is fired vertically upward with a muzzle velocity of
3000 ft per sec. If the velocity is 2950 ft per sec after 1 sec, what is the average
air resistance on the bullet? What maximum height will the bullet reach, assuming
that the air resistance remains constant? Ans. R = 0.55 1b; Max h = 90,000 ft
1051. Two blocks A and B are released from rest on a 30° incline when they are
50 ft apart. The coefficient of friction under the upper block A is 0.2 and that
under the lower block B is 0.4. Compute the elapsed time until the blocks touch.
Ans. t = 4.24 sec
1052. Determine the acceleration of the bodies in Fig. P-1052 if the fixed drum
eee Es
is smooth and A is heavier than B. Ans. a
~ WatWe %
1053. Referring to Fig. P-1052, assume A weighs 200 lb and B weighs 100 lb.
Determine the acceleration of the bodies if the coefficient of kinetic friction is 0.10
between the cable and the fixed drum. Ans. = 6.03 ft per sec?

Fie. P-1052 and P-1053. Fic. P-1054.

1054. Two bodies A and B in Fig. P-1054 are separated by a spring. Their
motion down the incline is resisted by a force P = 200 lb. The coefficient of
kinetic friction is 0.30 under A and 0.10 under B. Determine the force in the spring.
ATS She Los.4 ly
1055. If the pulleys in Fig. P-1055 are weightless and frictionless, find the
acceleration of body A.
Art. 10-6] Kinetics of Rectilinear Translation. Analysis as a Particle 277

1056. Determine the acceleration of body B in Fig. P-1056, assuming the pulleys
to be weightless and frictionless. Ans. ap = 5.85 ft per sec?

ic. P-1055. Fic. P-1056.


1057. The coefficient of kinetic friction under block A in Fig. P-1057 is 0.30 and
under block B it is 0.20. Find the acceleration of the system and the tension in each
cord.

Fia. P-1057. Fic. P-1058.

Fic. P-1059. Fic. P-1060.


278 RECTILINEAR TRANSLATION [Chap. X

1058. Determine the magnitude


of W in Fig. P-1058 so that the 200-lb
body will have an acceleration up the
plane of 4.025 ft per sec?.
Ans. W = 340 lb
1059. Compute the acceleration
of body B and the tension in the cord
supporting body A in Fig. P-1059.
~ 1060. In Fig. P-1060, compute
the acceleration of body B and the
Fic. P-1061 tension in the cord attached to it.
Ans. ap = 4.2 ft per sec?; T =
123.8 lb
1061. Compute the time required for the
100-lb body in Fig. P-1061 to move 10 ft starting
from rest.
1062. If the pulleys in Fig. P-1062 are weight-
less and frictionless, determine the acceleration
of each weight.
Ans. aa = 138.27 ft per sec?; ag = 9.49 ft per
sec?; dg = 1.89 ft per sec?
1063. Determine the acceleration of each
weight in Fig. P-1063, assuming the pulleys to
be weightless and frictionless.
1064. Determine the tension in the cord sup-
porting body C in Fig. P-1064. The pulleys are
frictionless and of negligible weight.
Ans. FT =171-61b
1065. Determine the maximum and mini-
Fic. P-1062.
mum weights of body C in Illus. Prob. 1043 on
page 273 that will keep C stationary. All other data remain unchanged.
‘Ans. Max. Wo = 924 lb; min. We = 535 Ib
1066. Referring to Illus. Prob. 1043 on page 273, let A weigh 600 lb but all other
data remain unchanged. Determine the time required for body C to move 20 ft
starting from rest. Ans. t = 2.78 sec
1067. In the system of connected blocks in Fig. P-1067, the coefficient of kinetic
friction is 0.20 under bodies B and C. Determine the acceleration of each body and
the tension in the cord.
1068. Repeat Prob. 1067, but change the weight of A to 600 lb, of B to 1000 lb
and of C to 500 lb.
Ans. T = 427.8lb;a4 = 9.24 ft per sec?; ag = 3.08 ft per sec?; ac = 3.08 ft per
sec?
Art. 10-6] Kinetics
ee of Rectilinear Translation. ee
Analysis as a Particle
s
279
a

Fic. P-1067 and P-1068.

1069. Two blocks A and B, each weighing


96.6 lb and connected by a rigid bar of neg-
ligible weight, move along the smooth surfaces
shown in Fig. P-1069. They start from rest
at the given position. Determine the accel-
eration of B at this instant. Hint: To relate
aa to ag, use the method developed in Illus.
Prob. 1018 on page 258.
Ans. ag = 11.6 ft per sec?

1070. Repeat Prob. 1069 if a leftward


horizontal force of 120 lb is applied to
block A.
Ans. ag = 7.62 ft per sec?
1071. The pulleys in the preceding prob-
lems have been assumed to be frictionless and Fic. P-1069 and P-1070.
280 RECTILINEAR TRANSLATION [Chap. X

weightless. What changes would there be in the solutions of these problems if the
pulleys (a) had friction? (b) had appreciable weight?

10-7. Dynamic Equilibrium in Translation. Analysis as a Rigid Body


We now consider a group of problems in which it is necessary to relate the
moment sum of the applied forces on a translating body to the moment
effect of their resultant. In the previous article, the magnitude of the
resultant force on a translating body was shown to be ; a but we did not
locate its position. To do this, we apply d’Alembert’s principle (Art. 9-7)
that the resultant of the applied forces acting on a body is equivalent to the
resultant of the effective forces acting on all particles composing the body.
Thus in Fig. 10-13, we select the origin of reference axes at the gravity

Z
Fie. 10-138. — Effective force on a particle of a translating body.

center of the translating body and take the summation of moments about
the reference axes of the effective force on each particle. The effective force
‘ ull) ‘ 3
on a typical particle such as D is eia where a is the common acceleration
of all particles. Since the effective force on this particle is parallel to the
X axis (or line of motion), it follows that the moment sum about the X
axis of all such particles is 2M, = 0. The moment sums about the Y and
Z axes are also zero as we see from

2M, == ghi2 = 2 3y +z Seis = (0


g g g
w a a
g y g y g Y

since 7 and Z must be zero because the reference axes pass through the
center of gravity of the body.
Art. 10-7] Dynamic Equilibrium in Translation 281
Therefore since the moment sum of the effective forces for all particles
is zero, the moment of the resultant effective force is also zero. Since this
resultant is a force, its moment is zero only because the force passes through
the center of moments. We conclude that the resultant effective force
W ; ;
raa (equivalent to the resultant of the applied forces) acts through the
center of gravity in the direction of the acceleration.
We are now ready to discuss dynamic equilibrium. This is a method
whereby a kinetics problem can be reduced to an equivalent statics problem.
We recall that in Art. 3-1, we originally defined a body in equilibrium as
one acted upon by a force system whose resultant is zero. We went on to
describe the physical meaning of static equilibrium as one in which a body
is either at rest or is moving in a straight line with constant velocity. In
other words, we were talking about bodies that had zero acceleration.
Vv
Applying the general equation R = = to them naturally gave a zero
resultant force.
It is obvious that accelerating bodies cannot be in static equilibrium. But
if we retain the definition of equilibrium to include any case in which the
resultant force is zero, a state of dynamic equilibrium can be created for
accelerated bodies. We merely add a dynamic reaction (i.e., inertia force)
which balances the resultant of the accelerating forces. This combination
of the applied forces together with the inertia force has a zero resultant;
hence all the equations and methods of static equilibrium may be applied to
this combination of applied and inertia forces.
We have shown above that the resultant of the accelerating forces in
senda , Ww ‘ ,
rectilinear translation is equivalent to Fi a acting through the gravity

center in the direction of the acceleration. Dynamic equilibrium can there-


fore be created by adding an equilibrant which is equal, opposite, and
collinear with this resultant. In other words, the equilibrant is an inertia
force having the magnitude a acting through the gravity center but
directed opposite to the acceleration. This procedure is known as the appli-
cation of d’Alembert’s principle (Art. 9-7).
The student should remember that a free-body diagram which includes
the inertia force aisa (sometimes called the reversed effective force) as well
g
as the real or applied forces represents a force system whose resultant is
zero. Since the condition of zero resultant governed the development of
static equilibrium, all the equations and methods of statics may be applied
to such a free-body diagram even though the body it represents actually is
282 RECTILINEAR TRANSLATION [Chap. X
Oe ne
accelerating. It is particularly advantageous to use dynamic equilibrium
to eliminate two unknown forces by taking a moment sum about their
intersection.

ILLUSTRATIVE PROBLEMS

1072. The car shown in Fig. 10-14 is given a rightward acceleration of 8.05 ft per
sec?. The sum of the friction forces 7, and /’, under the wheels is 20 lb. Find the
values of R,, Ro, and P.

a=8.05 ft/sec?
————x

W=1000 Ib
Fia. 10-14. — Dynamic equilibrium.

; Maret WV oe OU)
Solution: Adding the inertia force iia (ic,399 X 8.05 = 250 i)acting

through the center of gravity opposite to the acceleration reduces the system of
forces to a condition of dynamic equilibrium. Hence we may apply 2M 4 = 0 which,
by eliminating P and R2, permits us to find Ri. Thus
[2Ma4=0] 104,+ 20 X 2 — 1000 X 5 — 250 X1=0 R, = 521lb Ans.
Using this value of ; in a vertical foree summation, we obtain
ye—s0] R, + R. — 1000 = 0 R.=479lb Ans.
whereas from a horizontal force summation, we have
[2X = 0] P — 20 — 250 =0 P.=270lb Ans.
1073. The frame of a certain machine accelerates rightward at 4g ft per sec?. As
shown in Fig. 10-15, it carries a uniform bent bar ABC weighing 100 lb pinned to
it at C and braced by the uniform strut DE which weighs 50 lb. Determine the
components of the pin pressure at D.
Solution: Except for the application of the inertia forces, the analysis of this
problem is exactly the same as for three-force members (see p. 105). The free-body
Art. 10-7] Dynamic Equilibrium in Translation 283

eee eee me

a= g ft/sec?

Fig. 10-15.

diagram of each member is placed in dynamic equilibrium by including the inertia


force acting through the gravity center of each part directed opposite to the accelera-
tion. Note that instead of locating the center of gravity of ABC, it is more con-
venient to apply the inertia force acting on each segment of its length. The inertia
forces are
:
For AB: Y= (45) = 2m
g g \5
4
For BC: W = % (84) = 480
g g \9
4
For DE: Fy = (45) = 40m
g g \S
Consider now the FBD of ABC. A moment summation about C eliminates three
of the unknown forces and solves directly for Dj.
[SMe = 0] 4D, — 48(3) — 32(6) — 40(2) = 0 Dy, = 1041b Ans.
Turning to the FBD of DE in which D, is now known, we solve for D,. A moment
summation about E eliminates the unknown components acting at E.
[SMe =0) 4D, — (D, = 104)(4) — 402) — 502) =0 Dy, =149lb Ans.
284 RECTILINBAR TRANSLATION [Chap. X

If desired, the components of the pin pressures at C and # can now be computed
by applying a horizontal and vertical summation of forces first to the FBD of ABC
and then to that of DE.

PROBLEMS
1074. A juggler places the lower end of a vertical rod upon his finger. As it starts
to tip, explain how he keeps the rod in balance by moving his finger horizontally back
and forth.
1075. The cable of a cargo crane can support a maximum load of 2 tons. While
the crane is lowering a 1610-lb weight at uniform speed, the brake on the winch is
applied too rapidly, thereby causing a sudden deceleration of the weight equal to
100 ft per sec?. The cable snaps and the weight falls, badly injuring a workman.
For the purpose of establishing liability in this accident, is it likely that failure of
the cable was due to its being weaker than its test strength of 2 tons? Why?
1076. A uniform box is 2 ft square and 6 ft high. It stands on end upon a truck
with its sides parallel to the truck’s motion. If the box weighs 240 lb and the coeffi-
cient of friction between the box and the truck is 0.80, show whether the block will
slide or tip first as the acceleration of the truck is increased.
1077. The uniform block shown in Fig. P-1077 weighs 200 lb. It is pulled up the
incline by the force P = 800 1b. Determine the maximum and minimum values of d
so that the block does not tip over as it slides up
'E& P the incline. The coefficient of friction is 0.20.
Ans. Max. d = 2,32 ft; min. d = 1.253 ft

Fic. P-1077. Fig. P-1078 and P-1079.

. 1078. The 240-lb body in Fig. P-1078 is supported by wheels at B which roll
freely without friction and by a skid at A under which the coefficient of friction is
0.40. Compute the value of P to cause an acceleration of 4.
ANS a) = 136.3 Up
1079. If the value of P in Prob. 1078 is 100 lb, compute the acceleration. If this
value of P were applied at a higher position on the body, would the acceleration be
changed in any way? Explain your conclusion.
1080. An auto with a rear wheel drive has a wheelbase of 10 ft. The Og. 183 16
above the pavement and 4 ft ahead of the rear wheels. The coefficient of friction is
Art. 10-7] Dynamic Equilibrium in Translation 285

0.60 between the tires and the pavement. Determine the maximum acceleration
the auto could have when moving along a level road.
1081. An auto, equipped with only front wheel brakes, has a wheelbase of 120 in.
with its c.g. located 60 in. ahead of the rear wheels and 36 in. above the pavement.
If f = 0.80 at the tires, compute the minimum distance in which the auto can be
brought to rest from a speed of 60 mph if the driver’s reaction time before applying
the brakes is $ sec. Ans. $= 294 ft
1082. A car with a four-wheel drive weighs 3000 lb and has a wheelbase of 10 ft.
The c.g. is 3 ft above the pavement and 4 ft ahead of the rear wheels. Compute the
tractive force acting at the rear wheels when the car accelerates at 3g ft per sec’.
Assume the coefficient of friction is equal at all four wheels.
1083. The coefficient of kinetic K4 7 4
friction under the sliding supports
at A and B in Fig. P-1083 is 0.30.
What force P will give the 600-lb
door a leftward acceleration of
8.05 ft per sec?? What will be the
normal pressures at A and B?
Ans. P — 330 Ib; Na = 82.5
lb; Ng = 517.5 lb
1084. Repeat Prob. 1083 if f =
0.30 at A and f = 0.20 at B. W=600 lb
Ans. P = 281 lb; Na = 109.6 Fic. P-1083 and P-1084.
Ib; Ng = 490.4 lb
1085. In Fig. P-1085, find the angle 6 at which a uniform bar will be maintained
inside the smooth surface of a cylindrical drum accelerating leftward at 2g ft per
sec’. Ams. () = oil-

Fig. P-1085. Fic. P-1086.

1086. <A bar weighing 2 lb per ft is bent at right angles into segments 26 in. and
13 in. long. It takes the position shown in Fig. P-1086 when the frame F to which it
is pinned at A is accelerated horizontally. Determine this acceleration and the
components of the reaction at A.
Ans. a = 8.93 ft per sec?; A, = 6.5 lb; A, = 1.8 lb
286 RECTILINEAR TRANSLATION [Chap. X

1087. The uniform crate shown in Fig. P-1087 weighs 200 lb. It is pulled up the ©
incline by a counterweight W of 400 lb. Find the maximum and minimum values of
dso that the crate does not tip over as it slides up the incline.
Ans. Max. d = 3.35 ft; min. d = 1.61 ft
1088. Determine the value of W in
Prob. 1087 if the 200-lb crate is on the
verge of tipping forward as it slides up
the incline. Assume d = 3.32 ft.

1089. The frame of a machine is


accelerated leftwards at 2g ft per sec?.
As shown in Fig. P-1089, it carries a uni-
form angle ABC weighing 80 lb which
is braced by the uniform strut CD
weighing 60 lb. Determine the com-
ponents of the pin pressure at C upon
cD;
Ans. Ch = 54 lb right; C, = 18 lb
down
Fic. P-1087 and P-1088.
1090. Repeat Prob. 1089 if the frame
of the machine is accelerated rightwards at $9 ft per sec?.

Fic. P-1089 and P-1090.

1091. The uniform bar AB weighing 240 lb is mounted as shown in Fig. P-1091
upon a carriage weighing 480 lb. The center of gravity of the carriage is at C’ mid-
way between the wheels. If P = 180 lb and there is no frictional resistance at the
* wheels, find &, and F», and also the horizontal and vertical components of the pin
pressure at A. Ans. R, = 330 lb; R. = 390 lb; An = 180 lb; A, = 240 lb

1092. Repeat Prob. 1091 if the magnitude and sense of P is such that the reac-
tion of the carriage upon the bar at B is 60 lb leftwards.

1093. ‘Two bodies A and B, each weighing 96.6 lb, are connected by a rigid bar of
negligible weight attached to them at their gravity centers. The coefficient of
Summary 287

friction at the wall and floor is 0.268. If the bodies start from rest at the given
position, determine the acceleration of B at this instant. Simplify the solution by
creating dynamic equilibrium and taking a moment summation about the intersec-

Perfectly smooth

1 W= 480 lb [is

Fia. P-1091 and P-1092. Fic P-1093 and P-1094.

tion of the wall and floor reactions. Explain why these reactions pass through the
gravity center of B and A respectively. Hint: Relate the accelerations of A and B
by the method developed in Illus. Prob. 1018 on p. 258.
Ans. ag = 2.48 ft per sec?
1094. Repeat Prob. 1093 if a leftward horizontal force of 180 lb is applied to
body A. Ans. ag = 38.94 ft per sec?

SUMMARY
The rigid body motion of translation (Art. 10-1) is defined as motion in
which a straight line passing through any two particles of a body always
remains parallel to its initial position. All particles travel the same or
parallel paths and therefore the values of displacement, velocity, and accel-
eration are identical. If the path is straight, the motion is called rectilinear
translation; if curved, the motion is called curvilinear translation. The
latter is discussed in Chapter XI.
The kinematic equations of rectilinear motion with constant accelera-
tion are obtained by solving the differential equations of kinematics to
yield
v=v, + at (10-1)

s = vot + 4 af (10-2)
v? = v,’ + 2as (10-3)
In applying these equations, the following sign convention is adopted:
The initial direction of motion represents the positive directions of dis-
288 RECTILINEAR TRANSLATION [Chap. X

placement, velocity, and acceleration. Problems involving the kinematics


of freely falling bodies (Art. 10-3) are also solved by means of these equa-
tions; the acceleration a is replaced by the gravitational acceleration g taken
as 32.2 ft per sec’.
If the acceleration is variable (Art. 10-4), the kinematic equations may be
obtained by integrating the differential equations of kinematics. When
the motion can be expressed graphically (Art. 10-5), the area under a v-t
curve represents the change in displacement, and the area under an a-t
curve represents the change in velocity.
When the principle of the motion of the center of gravity is applied to
rectilinear translation, we obtain

xX = = a (10-9)

The X axis is chosen so that it coincides with the line of motion of the
body, and values of forces, displacement, velocity, and acceleration are all
considered positive in the initial direction of motion. The bar sign for
acceleration is omitted because the center of gravity has the same accel-
eration as all other particles in the body.
Dynamic equilibrium (Art. 10-7) may be created by adding an inertia
W : ; é
force —a at the center of gravity directed opposite to the acceleration.

When dynamic equilibrium is created, all the equations and methods of


static equilibrium can be used.
Chapter XI.
Curvilinear Translation

1i-1. Curvilinear Motion of a Particle. Fundamental Concepts


Translation of a rigid body has been defined as the motion in which a
straight line passing through any two points of the body always remains
parallel to its initial position. This definition requires that all particles
of the translating body have exactly the same motion. Hence in discussing
curvilinear translation, the body may be assumed to be concentrated at
its center of gravity and treated as a particle as in the case of rectilinear
translation.
In rectilinear motion, we chose the origin of motion on the path so that
only the magnitude of the displacement vector eould change, but not its
inclination. In curvilinear motion, however, the displacement vector will
change in both magnitude and inclination. Our concepts of velocity and
acceleration must be extended to include both these changes in displace-
ment. We shall find that although the velocity is always directed tangent
to the curved path of the motion, the acceleration is not tangent to the path-
Figure 11-1 shows the curved path traversed by a particle having
curvilinear motion. The displacement of any position is its vector distance

x B

O x

Fic. 11-1. — Displacement in curvilinear motion.

from the origin O. For example, the vector displacements of two positions
A and B are represented by s4 and sz. It is evident that the change in
displacement As is due to a combined change in the magnitude and inclina-
tion of these displacement vectors.
289
290 CURVILINEAR TRANSLATION [Chap. XI
athche cess rnc eae ie
For any position, the displacement s of any point may be expressed as
the vector sum of its X and Y coordinates as follows:
s=xby (11-1)

Recalling that v = anda = = we obtain by successive differentiation


of Eq. (11-1) :
v =U, Dd, (11-2)
and
a=a,4a4, (11-3)
In the following articles we shall discuss the geometric significance of
Eqs. (11-2) and (11-3).

11-2. Velocity in Curvilinear Motion


Let points A and B in Fig. 11-1 represent successive positions of a moving
particle after a small elapsed time At. The change in displacement As
during this interval is the chord distance between A and B. If the change in
displacement As is resolved into components parallel to the reference
axes, inspection of the figure shows that the geometric relation between
As, Az, and Ay is given by the following vector equation:
As = Ax 4 Ay (a)
If each term in Eq. (a) is divided by the elapsed time At during which
the displacement As was observed, we have
AS Te OL ey,
At OA Ee
Each term in the equation represents a vector displacement multiplied by
: 1 ‘ F
the scalar coefficient iN This creates a new vector having a different
magnitude but the same direction as the corresponding displacement. Each
of these new vectors, as we saw in Art. 9-3, represents the average velocity
in the respective directions of displacement. In the limit as At approaches
zero, we therefore obtain
ds
di
_dtde|.dyog
or

V = 0,4 dy - (11-2)
The geometric significance of Eq. (11-2) is as follows: As Aé approaches
zero, B approaches A and chord As coincides more completely with the
curve of travel so that, in the limit, As becomes ds which is directed along
Art. (11=3] Rectangular Components of Acceleration 291
SA R A atta,Alalate lahe cd
the path at A.
d. ;
Hence the term a represents the instantaneous velocity at
A directed tangent to the path at A. Figure 11-2 represents the path and
va

roe ‘ ny x
of
Fic. 11-2. — Rectangular components of velocity.

shows this velocity and its components v; and v,. The magnitude of the
velocity is given by the algebraic expression v = V/ (vz)? + (v,)?; its direction
with the X axis, by tan 6, = .
=z

Incidentally, if we replace v, and v, by their corresponding values

and d ses
7, we obtain
btai dy
v dt dy
tan 6, = 7
ae) — di oie

dt
It is evident that = is the slope of the curved path. This confirms our
previous conclusion that the velocity is tangent to the path.
11-3. Rectangular Components of Acceleration
Let Fig 11-1 be redrawn as in Fig. 11-3, showing the instantaneous veloci-
ties at points A and B which are sepa-
rated by the time interval At. From
Up Fig. 11-4, where these velocities are

hy Avy

O Ug
Fic. 11-4.:— Rectangular
Fic. 11-3. — Velocities at positions components of change
separated by a time interval At. in velocity.
292 CURVILINEAR TRANSLATION [Chap. XI

represented as free vectors, it is evident that a vector of magnitude Av and


the direction indicated must be added to v4 to obtain vg. In other words,
the change in velocity between v4 and vz is given by Av. Resolving Av into
components parallel to the reference axes gives the vector relation
Av = Avz 4 Ady
Dividing this relation by the elapsed time At during which Av was measured,
we obtain

IN IR
Each new vector in this equation represents the average acceleration in
the direction of each change of velocity. In the limit as At approaches zero,
this equation becomes
dv _ dvz re dvy
at Vedi dt
or
a=a,H4, (11-3)

in which a is the instantaneous accel-


eration at point A and a, and a, are its
components. This is shown in Fig.
11-5. Observe that the magnitude of
the acceleration at A may be ex-
pressed by the algebraic relation a =
V (az)? + (a,)?, and its angle with the
Fie. 11-5. — Rectangular components of :
acceleration. X axis by tan 6, = —
x=

an
Replacing a, and a, by their corresponding values of oy and e

2 gives

d’y

_ ae dy
et aadie ds

Therefore the direction of the acceleration vector is not tangent to the path.

11-4. Flight of Projectiles. Air Resistance Neglected


We shall assume the projectile to be moving without rotation in a vac-
uum. Such factors as wind velocity, air resistance, and rotation of the
earth, which have an effect on the actual flight of the projectile, will be
neglected. The principle of the solution is to resolve the curvilinear motion
of the path into rectilinear motions along the X and Y axes.
Let the path of the projectile be given by the curve OBCD shown in
Art. 11-4] Flight of Projectiles. Air Resistance Neglected 293
Fig. 11-6, with the origin of axes at the initial point of flight. The initial
velocity is taken as v, directed at an angle @ with the X axis. From the
initial direction of motion, displace-
ments will be taken as _ positive
both rightward and upward. Since
the only force acting on the pro- =
SS
jectile is its own weight, its total SS cee ix
acceleration at all positions is due SS
to gravity and is directed verti- X
\
cally downward with the value g. \
\

se
Hence the rectangular components
\
of this acceleration are constant at \

ik
MNT |
az = Oanda, = —g. D
Instead of considering the actual
path of the projectile, we combine Fie. 11-6. — Flight of projectile.
its simultaneous projections upon
the X and Y axes. The equations of these rectilinear components of the
path are found by substituting the X and Y components of s, v, and a in
the equations for rectilinear motion with constant acceleration as shown in
the accompanying table.

FLIGHT OF PROJECTILES ”

Rectilinear Motion with X Component of Flight Y Component of Flight


Constant Acceleration (az = 0, Vo, = Vo cos 8) (dy = —Q, Vo, = Yo sin @)

Uz = Vo, + Gat Vy = Vo, =) Oinfs


v=v.+ at or or
Vz = Uo cos 0 Vy = Vo sin 8 — gt

r= vot + % Al? y= Voyt + 4 a,t?


8 = vot + $ al? or or
x = v,cos6:t y = vosind-t
— 4 gt?

2 Tf air resistance, etc., were not neglected, a, and a, would become variable quan-
tities, and the differential equations of motion — Eqs. (9-1), (9-2), and (9-3) — would
have to be solved to obtain the proper equations for the flight of the projectile.

If, in Fig. 11-6, the time of flight is less than that required to reach C, the
projectile will be above its initial position and values of the Y displacement
will be positive. If the time of flight is more than that required to reach C,
the projectile will be on path CD and values of the Y displacement will then
be negative. At the topmost point of flight B, the value of vy will be zero.
294 CURVILINEAR TRANSLATION [Chap. XI
ag

ILLUSTRATIVE PROBLEM

1101. A projectile is fired from the top of a cliff 300 ft high with a velocity of
1414 ft per sec directed at 45° to the horizontal. Find the range on a horizontal
plane through the base of the cliff.

* U,= 1414 ft/sec

Imaginary / \
part of flight / | y=-300' .
t=-0.3 secpe
\
ue es
a
Fic. 11-7.

Solution: A diagram representing the conditions of the problem is shown in


Fig. 11-7. The Y displacement is positive upward and hence the downward accel-
eration g is negative; furthermore, the final position of the projectile is below the
origin and hence negative. Therefore we have

[y = vo sin 6-t — % gt] —300 = 1414 sin 45° x t —- 16.18 (a)


? — 62.1¢ — 18.63 = 0
= 62.4sec or ¢t = —0.3 sec
Using the positive value of ¢, we obtain

[x = vo cos 6 - t] x = 1414 cos 45° & 62.4 (6)


x = 62,400 ft Ans.

The negative value of ¢ obtained from Eq. (a), i.e., £ = —0.3 sec, may be inter-
preted as the time required for the projectile to leave the base of the cliff at A and
rise to the origin O. This is also the time required for the projectile to travel from
B to C. This observation may be checked by finding the time required to travel
distance OB from

[y = » sin 0-¢ — $ gt? 0 1414 sin 45° Xt — 16.18


t 62.1 sec

Adding 0.3 sec to this value gives the total time of flight, 62.4 sec, as before.

PROBLEMS
1102. A stone is thrown from a hill at an angle of 60° to the horizontal with an
initial velocity of 100 ft per sec. After hitting level ground at the base of the hill,
the stone has covered a horizontal distance of 500 ft. How high is the hill?
Art. 11-4] Flight of Projectiles. Air Resistance Neglected 295

1103. A shell leaves a mortar with a muzzle velocity of 500 ft per sec directed
upward at 60° with the horizontal. Determine the position of the shell and its
resultant velocity 20 sec after firing. How high will it rise?
Ans. x = 5000 ft; y = 2220 ft; v = 327 ft per sec; h = 2910/ft
1104. A projectile is fired with an initial velocity of 193.2 ft per sec upward at an
angle of 30° to the horizontal from a point 257.6 ft above a level plain. What hori-
zontal distance will it cover before it strikes the level plain? Ans. aw = 1340 ft
1105. Repeat Prob. 1104 if the projectile is fired downward at 30° to the hori-
zontal.
1106. A projectile is fired with an initial velocity of v, ft per sec upward at an
angle of 6° with the horizontal. Find the horizontal distance covered before the
projectile returns to its original level. Also determine the maximum height attained
2a 9 2 ain2
by the projectile. Ans. £= Ye AE h= Joa
g 29
1107. The car shown in Fig. P-1107 is just to clear the water-filled gap. Find
the take-off velocity vp. Ans. vo = 14.14 ft per sec

se
v5

Hel 22.2

Fic. P-1107.

1108. A ball is thrown so that it just clears a 10-ft fence 60 ft away. If it left
the hand 5 ft above the ground and at a angle of 60° to the horizontal, what was the
initial velocity of the ball? Ans. Vo = 48.3 ft per sec
1109. Determine the distance s at whicha
ball thrown with a velocity v, of 100 ft per sec
at an angle 6 = tan! # will strike the incline
shown in Fig. P-1109.
1110. In Fig. P-1109, a ball thrown down
the incline strikes it at a distance s = 254.5
ft. If the ball rises to a maximum height h =
64.4 ft above the point of release, compute its
initial velocity v, and inclination 8.
Ans. vo = 80.5 ft per sec; 6 = 53.1° Fra. P-1109 and P-1110.
296 CURVILINEAR TRANSLATION [Chap. XI

1111. Refer to Fig. P-1111 and find a@ to cause the projectile to hit point B in
exactly 4 sec. What is the distance x? Ans?) ao =\24.2% at=/3504t
U,= 96.6 ft/sec
u, =96 ft/sec Saat
re C~
es a
ss = ~ ae
Seg iu ——
Ex: N Constant ~\
\ u=20 = ft/sec SS

Pies P= Fig. P-1112.

1112. Boat A moves with a constant velocity of 20 ft per sec, starting from the
position shown in Fig. P-1112. Find @ in order for the projectile to hit the boat 5 sec
after starting, under the conditions given. How high is the hill above the water?
1113. Itis desired to pitch a golf ball across a trap to a green 100 ft away. What
is the best club to use if the initial velocity of the ball is 60 ft per sec? Assume that
the ball stops dead after striking the green, which is on the same level as the point
from which the ball is struck. Assume the clubs have slopes graduated at intervals
of 6° so that a No. 1 iron has a face inclined at 80° to the ground, the No. 2 iron at
74°, etc., down to a No. 9 iron inclined at 32°. Ans. Use a No. 9 iron
1114. A stone has an initial velocity of 100 ft per sec up to the right at 30° with
the horizontal. The components of acceleration are constant at az = —4 ft per sec?
and a, = —20 ft per sec?. Compute the horizontal distance covered until the stone
reaches a point 60 ft below its original elevation. Ans. «x = 448 ft
1115. A particle has such a curvilinear motion that its x coordinate is defined by
x = 5 # — 105 ¢t where z is in inches and ¢ in seconds. When ¢t = 2 sec, the total
acceleration is 75 in. per sec?. If the y component of acceleration is constant and the
particle starts from rest at the origin when ¢ = 0, determine its total velocity when
it = 4 sec. Ans. v = 225 in. per sec

11-5. Tangential and Nor-


mal Components of Accel-
eration
In many problems, the
most useful components
of acceleration are those
which are tangent and nor-
mal to the path. As we
shall see, these components
Fia. 11-8. — Velocities and normals to path. denote respectively the
Art. 11-5] =Tangential and Normal Components of Acceleration 297

rate of change of magnitude and of direction of velocity. Figure 11-3 has


been redrawn in Fig. 11-8 to show the normals to the path at points A
and B which are separated by a time interval At. These normals differ
by an angle of Aé@ radians and intersect at the center of curvature for the
segment AB. The radius of curvature at A is designated by r. The radius
of curvature at B will approach this value as A@ > 0 = dé.
In Fig. 11-9 the free vectors OC and OF representing the velocities at A
and B have been redrawn from a common origin. In addition, the length
OD has been laid off on vg equal to v4 and the change in velocity Av has
been resolved into the vectors CD and DE.
Cc
The distance DE represents the numer-
ical magnitude of the difference in velocity YA Mw
between vg and v4, for if vg and v4 were a NAD si
numerically equal, points Z and D would D Av,
coincide. The distance CD represents the Fic. 11-9. — Normal and tangential
difference in velocity between vp B and vqA C°MPonents of change in-velocity.
due only to the difference in direction of these velocities, for otherwise points
C and D would coincide. Thus we have resolved the total change in veloc-
ity Av into two components, one of which (DE) is due to a change in the
magnitude of the velocities and the other (CD) to a change in the direction
of the velocities.
Denoting CD by Av, and DE by Av, gives
Av = Av, H Av; (a)

Dividing this relation by the elapsed time At during which Av was measured
gives
Av Av, rs Av;
Ata Vie Ai At (b)
Each new vector in Eq. (b) represents the average acceleration in the direc-
tion of each change in velocity. In the limit as At approaches zero, this
equation becomes
dv im Wn re dv: (c)
dt dt dt
or
a=a,b4a, (11-4)
in which a is the instantaneous acceleration at point A and a, and a; are its
components. This is shown in Fig. 11-10. The magnitude of the resultant
acceleration is given by a = +/a,2 + a?, and its inclination with the normal
dv,
by tana = 7. Note carefully that a; = eh represents only the rate of
a n
298 CURVILINEAR TRANSLATION [Chap. XI

change in magnitude of velocity and


that it will be zero if the speed is
constant.
Let us now consider the geometri-
cal significance of permitting At to ap-
proach zero.as a limit. In Fig. 11-8,
poit B approaches to within a differ-
ential distance ds of point A; angle A@
' becomes d@; the radius of curvature at
~——__— | Bralsonbecomestia int emg oes
O
A@—0, angle OCD approaches 90°,
Fic. 11-10.— Normal and tangential and hence in the limit the direction
components of acceleration.
of CD is normal to the curve at A.
Likewise the direction of DE becomes tangent to the curve at A. This
accounts for the names normal and tangential components of acceleration.
Keeping these geometrical changes in mind, we have from Fig. 11-9
CD = dv, = v4 a0 (d)
and from Fig. 11-8
AB = ds = r dé (e)
Hence
_ dn 0adivs ds vz Cee
= oe
dineserUiek cha m) seer (f)
If we denote the velocity at any point by v, the normal acceleration is
v2

a, = — (11-5)
while the tangential acceleration is

a,=— (11-6)

Note that Eq. (11-5) expresses the


normal component of acceleration as
varying only with v andr. If the path
is straight, r will be infinite and a, will
be zero. For a curved path, a, will
be zero only if v is zero.
Figure 11-11 shows how the total
acceleration a may be resolved into
either rectangular components or
normal and tangential components.
Either set of components may be de-
rived from the other by projecting Fig. 11-11.
— Relations between rec-
tangular components and normal and
one set upon the other. tangential components of acceleration.
Art. 11-5] = Tangential and Normal Components of Acceleration 299

ILLUSTRATIVE PROBLEMS
1116. Point A in Fig. 11-12 moves in
a circular path of 20 ft radius so that its
are distance from an initial position Bis
given by the relation s = 6 t8 — 4 ¢, where
s is in feet and ¢ in seconds. Determine
the tangential and normal components of
acceleration of the point for the instant
when ¢ = 2 sec. Find also the resultant
acceleration in magnitude and inclination.
Solution: From Eq. (9-1) we have Fig. 11-12.

ds
E aN past
a | By BAd (6t Ce 4t)=18#@—-4
ae ph an (a)

From Eq. (11-6) we obtain

dv _d ; =
« = dt at = dt (18¢ 4) = 36 t (b)

whence substituting ¢ = 2 sec in Eqs. (a) and (6), we find

v 18 X (2)? — 4 = 68 ft per sec


and
a; = 36 Xx 2 = 72 ft per sec? Ans.

By substituting in Eq. (11-5) the numerical value of v just found, we find the
normal component of acceleration

E a 4
y2 v 2
| an = OR re 231 ft per sec? Ans.
r 20

The resultant acceleration is given by

Gp Va? a? | a= V (231)? + (72)? = 242 ft persec? Ans.

To find the inclination ¢ of the resultant acceleration, we have from the statement
of the problem

[s = 642 — s=6X (2)?-4xX2=


40ft
j=:J 6 = 49 = 2rad = 114.6°

Figure 11-13 shows a, az, and a, in their cor-


rect positions when t = 2 sec. From the figure
6=114.6 a 79 :
: tana
= — = — or a=17.3
O xX ih aM
Fig. 11-18. ¢ = 6—a=114.6° — 17.3° = O73. ANS:
300 CURVILINEAR TRANSLATION [Chap. XI

1117. A particle moves in such a manner that az = —6 ft per sec? and ay = —30
ft per sec?. If its initial velocity is 100 ft per sec directed at a slope of 4 to 3 as shown
in Fig. 11-14, compute the radius of curvature of the path 2 sec later.

Fig. 11-14.

Solution: Instead of determining the radius of curvature by applying the familiar


calculus formula to the equation of the path, a simpler procedure is to compute the
radius of curvature from Eq. (11-5) after first finding the velocity v and normal
acceleration dn.
The components of the velocity after 2 sec are
[v = v, + aft] vz = 100 X 3 — 6 X 2 = 48ft per sec
vy = 100 X $ — 30 X 2 = 20 ft per sec

Combining these components yields the resultant velocity v and the inclination 6 of
the tangent to the path.

[»= V2 + »,? | v I|= V (48)? + (20)? = 52 ft per sec


tan 6 = #8: @ = 22.6°
The normal component of acceleration is found by projecting the rectangular
components of acceleration upon the normal to the path. As shown in the figure,
these projections are oppositely directed so that a, is equal to their difference.
[an = ay cos 0 — a; sin 6] dn = 30 cos 22.6° — 6 sin 22.6° = 25 ft per sec?
Finally, Eq. (11-5) gives
v? (52)?
E = =| 25 = i r= 108 ft Ans.

PROBLEMS
1118. The normal acceleration of a particle on the rim of a pulley 10 ft in diame-
ter is constant at 8000 ft per sec?. Determine the speed of the pulley in rpm.
Art. 11-6] Kinetics of Curvilinear Translation. Dynamic Equilibrium 301

1119. At the bottom of a loop the speed of an airplane is 400 mph. This causes
a normal acceleration of 9 g ft per sec’. Determine the radius of the loop.
Ans. r = 1190 ft
1120. A particle moves on a circular path of 20 ft radius so that its are distance
from a fixed point on the path is given by s = 4 t? — 10 ¢ where s is in feet and ¢ in
seconds. Compute the total acceleration at the end of 2 sec.
Ans. a = 86.8 ft per sec?
1121. A particle is moving along a curved path. Ata certain instant when the
slope of the path is 0.75, a; = 6 ft per sec? and a, = 10 ft per sec?. Compute the
values of a; and a, at this instant and sketch how the path curves.
Ans. a; = 10.8 ft per sec?; a, = 4.4 ft per sec?
1122. A stone is thrown with an initial velocity of 100 ft per see upward at 60°
to the horizontal. Compute the radius of curvature of its path at the point where it
is 50 ft horizontally from its initial position. Ans. 7 = 251 ft
1123. A stone has an initial velocity of 200 ft per sec up to the right at a slope of
4 to 3. The components of acceleration are constant at az = —12 ft per sec? and
a, = —20 ft per sec?. Compute the radius of curvature at the start and at the top of
the path. Ans. At top, r = 28.8 ft
1124. A particle moves counterclockwise on a circular path of 400 ft radius. It
starts from a fixed point which is horizontally to the right of the center of the path
and moves so that s = 10 @ + 20¢ where s is the are distance in feet and ¢ is the
time in seconds. Compute the z and y components of acceleration at the end of 3
sec. Ans. Gz = —22.2 ft per sec’; ay = 12.7 ft per sec?
1125. A particle moves on a circle in accordance with the equation s = ¢' — 8 ¢,
where s is the displacement in feet measured along the circular path and ¢ is in
seconds. Two seconds after starting from rest the total acceleration of the particle
is 48+/2 ft per sec?. Compute the radius of the circle. Ans... .¢ = 12 ft

11-6. Kinetics of Curvilinear Translation. Dynamic Equilibrium


The kinetic equations of curvilinear translation are obtained from the
W
equation of motion for the center of gravity of any body, ie., k = 7

(Art. 9-8). It is convenient to resolve this equation into components which


are normal and tangent to the path. Calling the normal axis N and the
tangential axis T, we obtain
2

xN = Wie = we

g es (11-7)
a Wa, = — a;
g g

The bar signs which refer to the motion of the center of gravity can be
omitted from these equations because all particles in translation have
302 CURVILINEAR TRANSLATION [Chap. XI

identical values of acceleration as well as of displacement and velocity.


Instead of remembering that the positive senses of DN and DT in Kq.
(11-7) are in the directions of a, and a;, it is usually preferable to use
dynamic equilibrium. As was shown in Art. 10-7, the resultant of all
external forces on a translating body acts as the center of gravity of the
body. <A condition of dynamic equilibrium may therefore be obtained by
2
applying inertia forces of magnitudes = 2 and vo acting through the
center of gravity opposite in direction to the normal and tangential accel-
eration respectively. These inertia forces are known as the centrifugal
inertia force (or reversed normal effective force) and the tangential inertia
force (or reversed tangential effective force). It is usually more convenient
to solve problems in curvilinear translation by creating dynamic equilibrium.

ILLUSTRATIVE PROBLEM

1126. As shown in Fig. 11-15a, a bob of weight W is moving with a constant


velocity v in a horizontal plane at the end of a cord of length L. Because the string
generates a cone while in motion, the system is called a conical pendulum. It is
required to determine the tension in the supporting cord, its inclination with the
vertical, and the period or time required to complete one revolution.

Fig. 11-15. — Conical pendulum.

Solution: Dynamic equilibrium is created by applying the centrifugal inertia


WW OP ae ;
force a = acting radially outward from the center of the path. The forces acting on

the bob form the closed force polygon shown in Fig. 11-15b.
Art. 11-6] Kinetics of Curvilinear Translation. Dynamic Equilibrium 308
EE ae insiiaciaeiaat salicaria: hahha decalldladeded
The inclination @ of the cord with the vertical, as determined from Fig. 11-15b,
is
W v?
gr v?
CSAs race (a)

from which we note that the inclination @ is independent of the weight. Substitut-
ing r = L sin @ reduces Eq. (a) to
vy

os eh ae (b)

from which the value of cos @, and hence 0, may be obtained.


The tension T is also obtained from the force polygon. Solving Eq. (6) for 6, we
can use the value obtained to determine 7’ from the following relation:

W =Tecos@ or ce es
~ cos 6 (c)
The distance traveled by the bob in one revolution is 2 zr at a constant velocity
v, whence the period or time for a complete revolution is

Sie ai iP
$= -= — =92 ee
v v ms g tan0

From Fig. 11-15a, since tan 6 = s we have

h
t= an! (d)

As the angle @ decreases, the value of h approaches the limiting value L so that
the maximum time for a revolution becomes

L
tax = axe7 (e)
é€

PROBLEMS

1127. Solve Illus. Prob. 1126 using the following data: W = 100 lb; v = 8.08 ft
per sec; L = 18 in. Ans. @ = 57.6°; T = 187 lb; t= 0.998 sec
1128. A rod 4 ft long rotates in a horizontal plane about a vertical axis through
its center. At each end of the rod is fastened a cord 3 ft long. Each cord supports
a weight W. Compute the speed of rotation m in rpm to incline each cord at 30°
with the vertical. Ans. n = 22rpm
1129. A weight concentrated at the end of a cord forms a conical pendulum for
which the period is 1 sec. Determine the velocity v of the weight if the cord rotates
inclined at 30° with the vertical.
1130. In Fig. P-1130, the 20-lb ball is forced to rotate around the smooth inside
surface of a conical shell at the rate of one revolution in ¢ sec. Assuming that g =
304 CURVILINEAR TRANSLATION [Chap. XI

32.0 ft per sec, find the tension in the cord and the
force on the conical shell. At what speed in rpm will
the force on the shell become zero?
Aes, 10 = BiB los IN = 7/933 ioe Go 28) taoiaal
1131. A body of weight W rests on the smooth
inclined surface of the frame shown in Fig. P-1131.
A peg attached to the frame forces the body to rotate
with it about the vertical axis. Determine the speed
in rpm at which the tension in the cord is equal to the
weight of the body.
Fic. P-1130.
1132. The hammer of an impact testing machine
weighs 64.4 Ib. As shown in Fig. P-1182, it is attached to the end of a light rod
4 ft long which is pivoted to a horizontal axis at A. (a) What is the bearing re-
action on the pivot an instant after being released from the given position?
(b) What is the bearing reaction just before impact at B if the velocity of the ham-
mer is then 5.9 ft per sec? Ans. (b) R = 81.8 lb

alee

Fig. P-1131. Fig. P-1182.

1133. To check the radius of a railroad curve, the effect of a 20-lb weight is
observed to be 20.7 lb on a spring scale suspended from the roof of an experimental
ear rounding the curve at 40 mph. What is the radius of the curve?
Ans. r = 398 ft
1134. Figure P-1134 represents a schematic diagram of a Porter governor. Each
flyball weighs 16.1 lb and the central weight D is 40 lb. Determine the rotational
speed in rpm about the vertical axis AD at which the weight D begins to rise.
Ans. n= 109 rpm
1135. What counterweight W will maintain the Corliss engine governor in the
position shown in Fig. P-1135 at a rotational speed n = 120 rpm. Each flyball
weighs 16.1 lb. Neglect the weight of the other links.
1136. ‘The side rod of the engine in Fig. P-1136 is 8 ft long and weighs 100 Ib.
The cranks AD and BC are of length r = 18 in. and rotate at 300 rpm. Determine
Art. 11-7]
eBanking of Highway Curves
e 305

Fig. P-1134. Hire elisa

the maximum bending moment M in the rod if M = = where W is the total dis-
tributed load and L is the length of the rod.

Fic. P-1136.

1137. The segment of road passing over the crest of a hill is defined by the par-
4x a
abolic curve y = A car weighing 3220 lb travels along the road at a
10 ~=—-:100
constant speed of 30 ft per sec. What is the pressure on the wheels of the car when
it is at the crest of the hill where y = 4 ft? At what speed will the road pressure be
zero?
dy
1 dx?
Hint: Theradius of curvature is defined by — = a

© b+@)] dx

Ans. hk = 1420 lb; v = 40.1 ft per sec


306 CURVILINEAR TRANSLATION [Chap. XI

r 94" >< 36” ai a 1138. A homogeneous triangular plate


weighing 64.4 lb is attached to a rigid
A vertical support by two weightless links
as shown in Fig. P-1138. At the position
shown, the plate has an upward velocity
of 12 ft per sec. Compute the total force
acting on pin C.
Ans. C = 238 lb

11-7. Banking of Highway Curves


Ideal Angle of Banking. Consider
Fia. P-1138.
a car of weight W lb that makes a
horizontal turn on a curve of radius r ft while traveling at v ft per sec. The
curve is banked at an angle @ with the horizontal so that there is no tend-
ency to slide up or down the road. This angle is known as the ideal angle
gO)
of banking. Assuming the centrifugal inertia force =< applied through
the center of gravity to create dynamic equilibrium, the free-body diagram
is as shown in Fig. 11—16a; the resultant normal pressure against the wheels
is represented by NV.

Fic. 11-16. — Ideal angle of banking; friction unnecessary.

Because these forces are in equilibrium, a tip-to-tail addition gives the


triangle of forces shown in Fig. 11-16b, whence we have

Wv?
gr v?
tan@ = Wa = gr (11-8)

It should be observed that the dimensions of the car are negligibly small in
comparison to the size of the path, so that the car may be considered as a
particle.
Art. 11-7] Banking of Highway Curves 307

Equation (11-8) defines the ideal angle of banking in terms of the velocity
of the car and the radius of the turn and is independent of the weight of the
car. The velocity in this case is often termed the rated speed of the curve.
Friction Force on a Banked Curve. Let
us now determine the friction force ex- oN "8
erted by the road on the tires when the ye =
car is rounding a banked curve with a
velocity greater than the rated speed of
the curve. This frictional force F’ will
evidently act down the plane of bank-
ing and the point diagram of forces will
be as in Fig. 11-17. Taking 2X =0
parallel to the plane of banking will de- Fig. 11-17. — Friction force.
termine the friction force acting. Note
that the friction force adjusts itself to the amount required to prevent skid-
ding and will not have its maximum value unless skidding is impending. In
other words, the friction relation F = fN does not apply unless the car is
making the maximum speed.
When the car is traveling at its greatest speed and is about to skid up the
plane, the relation between F and N is given by F = fN. Remember that

(a) (b)
Fic. 11-18. — Maximum velocity, friction considered.

F and N are the components of the total road reaction # and that f is the
tangent of the angle of friction. Replacing / and N by their resultant
R (Fig. 11-182) therefore gives the tip-to-tail addition shown in Fig. 11-18b,
whence we have

tan (06+) = 27 == (11-9)


308 CuURVILINEAR TRANSLATION [Chap. XI

If the car is on the point of slipping down the plane of banking (because
of insufficient speed) we have
fan Oro ee = (11-10)
ILLUSTRATIVE PROBLEM
1139. The rated speed of a highway curve of 300-ft radius is 40 mph. If the
coefficient of friction between the tires and the road is 0.6, what is the maximum
speed at which a car can round the curve without skidding?
Solution: To find the angle of banking we apply Eq. (11-8).

v? (40 x $8)”
= —
tan d A tan 0 == ————__—_
359.9 x 300 = (),356 0 = 19'6"

From the definition of the angle of friction, we have


[tan @ = f] tan @ = 0.6 Oo= Bl”
Applying Eq. (11-9) gives

v?
|tan (6+ ¢) = =| tan (19.6° ° + 31°)
Ove v
= 39.9
<300

v? = 1.217 X 32.2 x 300 |= 108.5 ft per sec


v = 108.5 x $8 =
(— 74mph Ans.

PROBLEMS

1140. Why are railroad curves laid out in the form of a spiral with a gradually
decreasing radius of curvature instead of a circle of constant radius?
1141. A boy running a foot race rounds a flat curve of 50-ft radius. If he runs
at the rate of 15 mph, at what angle with the vertical will he incline his body?
ANS, (0) == NOS"
1142. A daredevil drives a motorcycle around a circular vertical wall 100 ft in
diameter. The coefficient of friction between tires and wall is 0.60. What is the
minimum speed that will prevent his shding down the wall? At what angle will the
motorcycle be inclined to the horizontal? What is the effect of traveling at a greater
speed? Ans. v = 35.3 mph; 6 = 31°
1143. The superelevation of a railroad track is the number of inches that the
outside rail is raised to prevent side thrust on the wheel flanges of cars rounding the
curve at rated speed. Determine the superelevation e for a track having a gauge of
4 ft 83 in. of 2000-ft radius and a rated speed of 60 mph. What is the flange pressure
P on the wheels of a 100,000-lb car that rounds the curve at 80 mph?
Ans. e = 6.73 in.; P = 9230 1b
1144. An airplane makes a turn in a horizontal plane without sideslip at 480
mph. At what angle must the plane be banked if the radius of the turn is 1 mile? If
the pilot weighs 150 lb, what pressure does he exert on his seat?
Summary 309

1145. A car weighing 3220 lb rounds a curve of 200-ft radius banked at an angle
of 30°. Find the friction force acting on the tires when the car is traveling at 60
mph. The coefficient of friction between the tires and the road is 0.90.
1146. Find the angle of banking for a highway curve of 300-ft radius designed to
accommodate cars traveling at 100 mph, if the coefficient of friction between the
tires and the road is 0.60. What is the rated speed of the curve?
Ans. 6 = 34.8°; v = 56 mph
1147. The rated speed of a highway curve of 200-ft radius is 30 mph. If the
coefficient of friction between the tires and the road is 0.60, what is the maximum
speed at which a car can round the curve with-
out skidding? W=3220 |b
1148. The coefficient of friction between the
road and the tires of the car shown in Fig. P-1148
is 0.60. The car weighs 3220 lb. It is rounding
the curve of 500-ft radius at maximum speed.
What is the value of the friction force acting un-
der each wheel? How high above the road must
the center of gravity be to limit this maximum
speed by the tendency to overturn?
Ans. At inner wheels, F = 435 lb; at outer Fiat P-1148:
wheels, F = 2980 lb; h = 4.02 ft
1149. Repeat Prob. 1148 if the road is banked at 20° instead of 30° as shown in
Fig. P-1148.

SUMMARY
A rigid body translation in which the paths followed by the particles com-
posing the body are parallel curved lines is known as curvilinear translation.
Since all particles of the body have identical motions, the motion of the
body is considered equivalent to that of a particle concentrated at the
center of gravity of the body.
Problems involving the kinematics of curvilinear translation can ‘be
solved by considering the rectilinear motions of the X and Y projections of
the center of gravity. The actual motion is related to its projections by the
equations
s=x py (11-1)

a=a,) 4a, (11-3)

When applied to the flight of projectiles with air resistance neglected


(Art. 11-4), these three equations can be used to determine the coordinates
of the projectile’s position. The components of the acceleration become
d,= 0 and a, = —g.
310 CURVILINEAR TRANSLATION [Chap. XI

The normal and tangential components of the acceleration in curvilinear


translation (Art. 11-5) are respectively normal and tangent to the path and
are related to the total acceleration by
a=a4,)4 (i-4)
The normal component of acceleration, expressed by
2
i haus (11-5)
r
is due only to the changing direction of the velocity. The tangential com-
ponent of the acceleration, given by

a, _weardv (11-6)
depends solely on the change in magnitude of the velocity.
The kinetic equations of curvilinear translation (Art. 11-6), obtained

mots
from the motion of the center of gravity, are
2

os (11-7)
xT = — as!
g
The WN and T reference axes are chosen through the center of gravity of the
body normal and tangent to its path. Dynamic equilibrium is obtained by
9

applying inertia forces of magnitudes oa and = a; acting through the


center of gravity opposite in direction to a, and a; respectively. These
forces are known as the centrifugal inertia force and the tangential inertia
force.
Problems involving forces acting on vehicles rounding a banked curve
are solved by adding the inertia forces, thereby creating dynamic equi-
librium (Art. 11-7). If the vehicle has no motion along the plane of bank-
ing, a force summation parallel to this plane will determine the necessary
resisting force. In the ideal case when no resistance is necessary, the angle
of banking is determined from the relation
v2
tan@ = — (11-8)
gr
When friction is considered, the maximum and minimum velocities are
found from the equation
v2
tan (9+ ¢) = (11-9, 11-10)
gr
Chapter XII.
Rotation

12-1. Rotation. Definition and Characteristics


Rotation is defined as that motion of a rigid body in which the particles
move in circular paths with their centers on a fixed straight line that is
called the axis of rotation. The planes of the circles in which the particles
move are perpendicular to the axis of rotation.
Figure 12-1 shows a body which is free to rotate about the fixed axis O.

Fic. 12-1. — All particles and lines have the same angular displacement.

If the radius to any point A is permitted to rotate through @ radians, point


A moves through the are distance s; = 7:6. Since the body is rigid, angle
AOB cannot change; hence the radius to any other point B will also rotate
through 6 radians and point B will move through the arc distance s. = 126.
From this we conclude that all particles of a rotating body have the same
angular displacement (i.e., the same 4) although their linear movements
(i.e., 8, and Ss) vary directly with their distances from the axis of rotation.
It should be observed that the angle between the line AB and its subse-
quent position A’B’ is also equal to 6. Consequently we define angular
displacement as the angular distance swept through by any line in a rigid
body. The units of angular displacement may be radians, degrees, or
revolutions, but radian measurement is preferred in order to correlate
angular displacement with linear displacement.
311
312 ROTATION [Chap. XII
eee
On
It was shown that in the motion of translation all the particles have
identical values of linear displacement, linear velocity, and linear accelera-
tion. The motion of rotation has a similar characteristic; all the particles
have the same values of angular displacement, angular velocity, and angular
acceleration. (These terms will be defined in the next article.) We shall
see that this conclusion depends upon the fact that. the angular displacement
is the same for all the particles.
It is important to note that the linear values of displacement, velocity,
and acceleration in a rotating body are not the same; they vary directly
with the distance of the particle from the axis of rotation.

12-2. Kinematic Differential Equations of Rotation


Consider a pulley free to rotate around an axle O under the action of a
weight W suspended from a cord wound around the pulley. Assume that
the weight descends s ft, as shown in Fig.
12-2. This will unwind from the pulley a
length of cord equal to s ft so that point B
on the rim will rotate to occupy the position
of point A. The angular distance @ through
which the pulley rotates 1s obviously sub-
tended by radii drawn to points A and B.
The relation between the linear displacement
of the weight and the angular displacement
(in radians) of the pulley is given by the
equation

s = 76 (a)
If we differentiate Eq. (a) with respect to
Fig. 12-2, — Relation between the time ¢, we have
linear and angular displace-
ment. ds de
apa ae (b)

Note that r is the constant radius of rotation. It will be remembered that


ds ; ; :
a representing the time rate of change of displacement, was defined as v, the
linear velocity of the weight. In this case, v must also be the linear velocity
of a point on the rim of the pulley. The term a represents the time rate
of change of angular displacement and hence, by analogy, will be called the
angular velocity and be represented by the symbol w. Thus the angular
velocity at any instant is defined by the equation
Art. 12-2] Kinematic Differential Equations of Rotation 313

o=a (12-1)
The common unit is radians per second (rad per sec) but other units such as
degrees per second (deg per sec) and revolutions per minute (rpm) are
also used.
Rewriting Eq. (b) as
v= 1ra (c)

and differentiating with respect to the time gives

ae (a)
pe k tie, 3
The expression a 2 Eq. (d) represents the time rate of change of the mag-
nitude of the velocity. It is preferable to denote this acceleration by az
because it not only represents the linear acceleration of the weight but is
also the tangential acceleration of a point on the rim of the pulley. The
eee : :
expression a represents the time rate of change of angular velocity and,
by analogy, will be defined as the angular acceleration a, according to the
following equation:
_ dw
Oo dt (12-2)

The common unit is radians per second per second (rad per sec”), but other
units are sometimes used, e.g., revolutions per minute per second (rpm per
sec), etc. Equation (d) may now be rewritten

a, = Ta (e)

Since v = rw, the normal acceleration of any point on the rim of the
pulley is given by

On = — = rw (f)

Although Eqs. (12-1) and (12-2) are the kinematic differential equations
of rotation, a third convenient relation may be found by eliminating df.
wo dw = a dé (12-3)

It will be helpful to summarize these differential equations of rotation


and compare them with similar expressions for rectilinear motion tabulated
on the next page.
314 ROTATION [Chap. XII

Rectilinear Motion Rotation


_ ds a We
di aE
dv _ d’s _ dw _ a6
Od de CS aE Ode
v dv = ads w dw-=
a dé

These relations differ only in the symbols used; they are therefore mathe-
matically identical. They can be transformed into each other by the rela-
tions deduced above, viz.:
76

ay (12-4)
aq; = Ta

a, = To?

Note that the angular displacement of a body is a vector quantity; e.g.,


the direction of rotation may be either clockwise or counterclockwise.
Since w and a have been defined in terms of @ by multiplying by the scalar
factor > it follows that w and @ are also vector quantities. Their direction
depends upon the direction of 6. We shall therefore use the following rule:
The sense of posite 0 determines the sense of positive w and positive a. This
agrees with the previously established convention for translation; 1.e., the
sense of positive s determines the sense of positive v and positive a.
The “right-hand rule” is used to represent graphically the vectors of 6,
w, anda. This rule applied to 6, for example, states that the vector is
directed along the axis of rotation as indicated by the extended thumb of
the right hand when the fingers are curled about the axis of rotation to
correspond with the direction of rotation.

12-3. Rotation with Constant Angular Acceleration


dv :
In Art. 10-2 we integrated the equation a = ae assuming constant ac-
celeration, to givev = v. + at. Since the form of the kinematic differential
equations of rotation are mathematically identical to the respective equa-
tions of rectilinear motion, integration of similar equations in rotation
will yield similar results. These results are tabulated below.

Rectilinear Motion Rotation


(related by)
v=v,+at 8S = 7) ®© = wow, + at
s=vt +4 al Vv = 7TH 6 = wot + 4 al?
v= vy. + 2as Oh, 3 ie w = w, + 2 a6
Art. 12-3] Rotation with Constant Angular Acceleration 315

When they are arranged in this form, the student should have no dif-
ficulty in remembering the equations of rotation with constant angular
acceleration. Analogous to rectilinear motion, w, is the initial angular
velocity; #, the final angular velocity after time ¢ and angular displace-
ment @.

ILLUSTRATIVE PROBLEM

1201. The initial angular velocity of the compound pulley B in Fig. 12-3 is 6
rad per sec counterclockwise and weight D is decelerating at the constant rate of 4
ft per sec?. What distance will weight A travel before coming to rest?

Fic. 12-3.

Solution: To correlate the given data, we start by finding the kinematic relations
between the bodies. Using Eq. (12-4) and denoting by EF any point on the cord
connecting B and C, we obtain
ito} SY SB thee 37210 — 2 UG; Sin = Lid Oe

Combining these relations into one continuous equation, we obtain


sa = 3060p = 0c = 3858p (a)

The velocity and acceleration relations between the bodies are obtained by merely
changing the symbols in Eq. (a) since they would be obtained from v = rw and
a, = ra which have the same mathematical form as s = 76. Therefore we may also
write
va = 3wp = Foc = 3vp (0)
aa = 3ap = $ac = 3ap (c)

Consider now the motion of A. From Eq. (6), 1, = 30, = 3 X 6 = 18 ft per


sec and from Eq. (c), a4 = 3ap = 3(—4) = —12 ft per sec’. We now obtain
[v2 = v,.” + 2 as] 0 = (18)? — (2)(12)s4 S4.=le.o1t Ans,
If desired, the displacement of the other bodies can now be found from Eq. (qa).
316 ROTATION [Chap. XII

PROBLEMS
1202. A flywheel 6 ft in diameter accelerates from rest at the constant rate of 4
rpm per sec. Compute the normal and tangential components of the acceleration of
a particle on the rim of the flywheel after 10 sec.
1203. The rim of a 50-in. wheel on a brakeshoe testing machine has a speed of
60 mph when the brake is dropped. It comes to rest after the rim has traveled a
linear distance of 600 ft. What are the constant angular acceleration and the num-
ber of revolutions the wheel makes in coming to rest?
Ans. a = —8.10 rad per sec?; 0 = 45.9 rev
1204. A gear is accelerated from rest to a speed of 900 rpm and then immediately
decelerated toastop. If the total elapsed time is 10 sec, determine the total number
of revolutions of the gear. Assume that both acceleration and deceleration are
constant but not necessarily of the same magnitude. Hint: Sketch an w-t curve.
1205. When the angular velocity of a 4-ft diameter pulley is 3 rad per sec, the
total acceleration of a point on its rim is 30 ft per sec?. Determine the angular
acceleration of the pulley at this instant. Ans. a = 12 rad per sec?
1206. Determine the horizontal and vertical com-
ponents of the acceleration of point B on the rim of
the flywheel shown in Fig. P-1206. At the given
position, w = 4 rad per sec and a = 12 rad per sec?,
both clockwise.
1207. Repeat Prob. 1206 except that a is changed
to 10 rad per sec? counterclockwise.
1208. A pulley has a constant angular accelera-
tion of 3 rad per sec?. When the angular velocity is
Fra. P-1206 and P-1207. 2 rad per sec, the total acceleration of a point on the
rim of the pulley is 10 ft per sec?. Compute the di-
ameter of the pulley. Ans. d = 4 ft

Fig. P-1209 and P-1210.


Art. 12-4] Rotation with Variable Angular Acceleration 317
1209. The step pulleys shown in Fig. P-1209 are connected by a crossed belt. If
the angular acceleration of C is 2 rad per sec?, what time is required for A to travel
180 ft from rest? Through what distance will D move while A is moving 240 ft?
AMS, th = Oi RIOR Gi, = WED Tt
1210. Repeat Prob. 1209 if the radii of pulley B are changed to 30 in. and 18 in.
1211. The rod BO in Fig. P-1211 rotates
in a vertical plane about a horizontal axis
at O. At the given position, end B has a
downward vertical component of velocity
of 6 ft per sec and also a downward verti-
cal component of acceleration of 9 ft per
sec?. Compute the angular acceleration of
rod BO and the total acceleration of point A.
Ans. a = 24 rad per sec?,.); a4 = 32.8
ft per sec? Fie. P-1211.

12-4. Rotation with Variable Angular Acceleration


Since the differential equations of rectilinear motion and of rotation are
mathematically identical, we need observe merely that the technique for
handling the case of rotation with variable angular acceleration is identical
to that developed in Arts. 10-4 and 10-5. One example will suffice.

ILLUSTRATIVE PROBLEM
1212. A body rotates according to the relation a = 3 # + 4, displacement being
measured in radians and time in seconds. If its initial angular velocity is 4 rad per
sec and the initial angular displacement is zero, compute the values of w and @ for
the instant when t = 3 sec. Solve analytically and graphically.
Solution: Rewriting Eq. (12-2) as dw = a dt and integrating between the given
limits, we have
w t

{| dw = fortaa
4 0

w—-4=+4+4¢
(69) I e+4t+4 (a)
Applying Eq. (12-1) in the form d# = w dt, substituting for w its value from Eq.
(a), and integrating gives
8 t
f= fotartaa
0 0

g= T+ 20441 (b)
Substituting ¢ = 3 sec in Eqs. (a) and (b), we have
= (3)?+4*3+4= 48 rad persec Ans.

= 42x BP +4 x 3 = 50.25 mad Ans.


318 ROTATION [Chap. XIT

Let us now check this solution by


means of the motion curves shown in
Fig. 12-4. Using an equation similar
to Eq. (10-6), the change in angular
velocity is
[Aw = (Area) e—z)]
Aw = 4 X 8 + 4(8)(27) = 124+ 27 =
39 rad per sec
Adding this value to w, = 4 gives w =
43 rad per sec at t = 3 sec as before.
The area under the w-t curve is sub-
divided into parts shaded to correspond
to the similarly shaded subdivisions of
the a-t curve. From an equation sim-
ilar to Eq. (10-7), we then obtain
[Ad = (Area) w+]
Ad=4xX38+4X3xXx124+24x3
Ye) SCOT = ADP Siac! (Claeele
oy
S
Ye)
If preferred, A? may be computed
by applying an equation similar to Eq.
ie (10-8) to the area under the a-t curve.
This gives
Fig. 12-4. — Motion curves.
[A@ —_ wr(te = t) ap (Area) ats to]
Ad=4X3+ (4X 3)@ X 3) + & X 8 X 27)G X 8) = 50.25 rad Check
which is identical term for term with the preceding computation although based on a
different concept.
Finally, a comparison of these computations obtained from motion curves will be
found to be identical term for term with those of the calculus solution.

PROBLEMS

1213. The rotation of a pulley is defined by the relation 6 = 2 t* — 30 + 6,


where @ is measured in radians and ¢ in seconds. Compute the values of angular
velocity and angular acceleration at the instant when t = 4 sec.
Ans. w = 272 rad per sec; a = 324 rad per sec?
1214. The rotation of a flywheel is governed by the equation w = 4V/f; w is in
radians per second and t is in seconds. @ = 2 rad when ¢t = 1 sec. Compute the
values of 6 and @ at the instant when t = 3 sec.
Ans. § = 18.21 rad; a = 1.154 rad per sec?
1215. A body rotates according to the relation a = 2 t, where a is in radians per
second per second and ¢ is in seconds. w = 4 rad per sec and @ is zero when f is zero.
Compute the values of w and 6 at the instant when t = 2 sec.
Art. 12-5] General Kinetics of Rotation 319

1216. Determine the number of revolutions through which a pulley will rotate
from rest if its angular acceleration is increased uniformly from zero to 12 rad per
sec? during 4 sec and then uniformly decreased to 4 rad per sec? during the next 3
sec. As, () = DQ aK
1217. The angular acceleration of a flywheel decreases uniformly from 8 rad
per sec? to 2 rad per sec? in 6 sec at which time its angular velocity is 42 rad per sec.
Compute the initial angular velocity and the number of revolutions made during
the 6-sec interval. Ans. >. = 12 rad per sec; 0 = 28.6 rev

12-5. General Kinetics of Rotation

The discussion will be limited to the case in which the rotating body is
symmetrical with respect to the plane in which the center of gravity moves.
The rotating body is represented by its projection upon this plane of motion,
and the point in which the axis of rotation intersects this plane is defined as
the center of rotation. Furthermore, we assume that all forces lie in the
plane of motion except for a few exceptional cases which we consider in
Art. 12-7.
Select reference axes through the center of rotation. The line joining
the center of gravity and the center of rotation will be called the N axis,
and the line through the center of rotation perpendicular to N will be called
the T axis. The axis of rotation will be called the Z axis. We shall then
have three mutually perpendicular axes, N, 7, and Z, with an origin at the
center of rotation. (See Fig. 12-5.) The Z axis is stationary, but N and T
rotate about Z as the body rotates. The rotating axes are called the normal
and tangential axes since they are respectively normal and tangent to the
path of the center of gravity.
Consider now the rigid body of weight W in Fig. 12-5 which is constrained
to rotate about a horizontal axis. The action of the external forces W,
P,, Ps, as well as the bearing reaction F, give the body the instantaneous
values of the angular velocity » and angular acceleration a shown. The
resultant of these applied forces (indicated by the dashed vector R) gives
the gravity center @ an acceleration @ whose components are @, = 7w* and
ad, = 7a. From the principle of the motion of the center of gravity (Art.
9-8), we know that F acts in the direction of @ and is related to it by the
-

equation R = —d. Taking normal and tangential components of this


g
vector equation, we obtain

or
320 ROTATION [Chap. XI]

where the summations 2N and DT apply to all external forces, including


the bearing reaction F at O, and are positive in the directions of a, and @.
The moment effect of the resultant force is found by equating the moment

Fic. 12-5.

sum of the impressed forces to the moment sum of the effective forces for all
particles. The moment of the effective force on any typical particle A with
; Vigne dW
respect to the axis of rotation is given by r - ra. This moment is due
only to the tangential component of the effective force. The normal com-
ponent ey: passes through the axis of rotation and hence has no
|
moment about it. Equating the moments of impressed forces about the
axis of rotation (thereby eliminating the generally unknown bearing re-
action) to the moment sum of all effective forces gives
4 7
zu, = |neot yale
g g
or
IM = la

winene JL, = iow and has been defined as the mass moment of inertia

with respect to the Z axis (see Art. 8-13). Note that a is placed outside the
integral sign because it is independent of the position of the particle.
Art. 12-5] General Kinetics of Rotation 321
In summary, the general equations of rotation of a symmetrical body
with respect to a set of N, 7, and Z reference axes are

EN = * Fo? (12-5)
ET = = Fa (12-6)
=M. = La (12-7)
A convenient rule for signs is to take YN and YT as positive in the direc-
tions of d, and a@;. The direction of d@, will always be toward the center of
rotation; that of Gd, is determined from the sense of a. The sense of >M,
is positive in the initial direction of rotation, that is, in the sense of a.
Unlike translation, however, the resultant force R does not pass through
the gravity center G. To determine the position P where it intersects the
N axis, we apply the principle that the moment of a resultant force is equal
to the moment sum of its components. Of the two components of R, only
: : J ;
the tangential component (i.e., = Fa) has a moment effect about the axis
‘of rotation. Denoting its moment arm by gq, we therefore obtain

fa-g = 2M, = 1a

Since J, = - k,*, in which k, is the radius of gyration with respect to the


axis of rotation, we may write

W fa-q= W k,’a
g g
whence
k 2

q= = (12-8)

Therefore the resultant of effective forces (or of all the impressed forces)
; é kee :
passes through a point P on the WN axis at a distance of es from the axis of

rotation.
Point P at which the resultant of the applied forces intersects the normal
axis is called the center of percussion. The center of percussion may be
thought of as equivalent to the “center of gravity” of the effective forces
acting on all particles of the body, just as the actual center of gravity is the
point through which passes the resultant weight of all particles of the body.
322 RoraTION [Chap. XII

12-6. Centroidal Rotation


One of the commonest cases of rotation is centroidal rotation, in which
the axis of rotation passes through the center of gravity. Consequently
the distance between the center of rotation and the center of gravity
becomes zero (i.e., 7 = 0), and the equations of rotation reduce to
=IN=0
xT =0 (12-9)
=M = Ia
The bar sign indicates that both 2M and J are taken with respect to a cen-
troidal axis of rotation.
Since both ZN and YT equal zero for centroidal rotation, the resultant
of the impressed forces always is a couple of magnitude 2M. The converse
of this observation is that if the force system applied to a body reduces to
a couple, the body will undergo a centroidal rotation. The resultant
couple will create centroidal rotation even if there is no fixed axis through
the center of gravity. We shall use this observation later in Art. 13-6.

ILLUSTRATIVE PROBLEMS
1218. The pulley assembly shown in Fig. 12-6a weighs 161 lb and has a cen-
troidal radius of gyration of 2 ft. The blocks are attached to the assembly by cords
wrapped around the pulleys. Determine the acceleration of each body and the
tension in each cord.

W=1611lb
k=2!

Th q,
7, T:

cra ele alee |arte


(a) 1 00 Ib (b) 200 Ib
Fie. 12-6.

Solution: The moments of the weights about the center of rotation give an un-
balanced moment in a clockwise sense. The 200-lb block will therefore move down
while the 100-Ib block rises. The FBD of each part of the system showing the
direction of motion of each body can now be drawn as in Fig. 12-6b. In writing
Art. 12-6] Centroidal Rotation 320

the equations of motion, remember that )X and DM are taken plus in the direction
of motion.
The moment of inertia is given by
- We. = 161 os
[72s 7 k2 I = — X (2)? I = 20 ft-lb-sec?

For the 200-Ib block, which moves down, take the X axis as positive downward.
In accordance with Art. 12-2, the linear acceleration of the 200-Ib block is expressed
in terms of the angular acceleration of the pulley by the relation a; = ra or, in this
case, a, = 2a. Applying the equation of translation, we have

|=x = ral 200 — T; uu zu X 2a (a)


= 39.9 “999
Since the 100-lb block moves upward, take the X axis as positive upward. From
the relation a; = ra, we have a2 = 3a. Applying the equation of translation gives

ap W
eee ae —_ 100
et pba 100
no) one

The equation of rotation applied to the pulley assembly is taken positive in the
sense of rotation; hence

[2M = 1a) 27, —3T. = 20a (c)


Multiplying Eq. (a) by 2 and Eq. (b) by 3 and then adding Eas. (a), (6), and (c)
to eliminate the tensions, we obtain

200 x 2 — 100 XK 3 = 24.84a 4+ 27.95a + 20a


a = 1.38 rad per sec? Ans.

Substituting the value of a in Eqs. (a) and (6) gives


a = 2a = 2.76 ft per al
do = 3a = 4.14 ft per sec?
ne \| 182.9 Ib ane
T, = 112.8 lb

The reaction on the support may be found by applying

[ZY = 0] R, — 161 —7,-—


TT, =0
R, = 456.7 lb Ans.

1219. The rotating drum in Fig. 12-7a has a centroidal mass moment of inertia
of 10 ft-lb-sec?. The coefficient of friction at the brake is 0.25. At the instant the
brake is applied, block B has a downward velocity of 20 ft per sec. What is the
constant brake force P required to stop block B in a distance of 10 ft?
Solution: The FBD of each part of the system is shown in Fig. 12-7b. We first
find the linear acceleration of block B from the given data.
[v2 = v,? + 2 as] 2 Xa X 10
0 = (20)?+ a = —20 ft per sec?
324 ROTATION [Chap. XII

(b)
Fie. 12-7.

From the kinematic relation between the block and the pulley, we find the angular
acceleration of the pulley.
[at = ra| —20 = 2a a = —10rad per sec?
For block B, the X axis being taken as positive in the downward direction of
motion

|2x = 7 100 — T = a5 xX (—20) (a)

For the pulley, 2M is positive in the direction of rotation; this gives


(2M = fol 2T —3F = 10 X (—10) (b)
Solving Eqs. (a) and (6), we have
0 = G21 Mo
F = 141.41b
From the friction relationship,
[F = fN| 141.4 = 0.25N N = 565.6 lb
Finally, from the FBD of CD, we obtain
[2Mc = 0] 565.6 X 1+ 141.4 x % —4P =0 P=159.1lb Ans.

PROBLEMS
1220. A weight of 96.6 lb is fastened to a cord which is wrapped around a solid
cylinder of 3 ft radius weighing 322 lb. The cylinder rotates about its horizontal
centroidal axis. Compute the angular acceleration and the tension in the cord;
also the total bearing reaction.
Art. 12-6] Centroidal Rotation 325

1221. What torque applied to the cylinder of Prob. 1220 will raise the weight
with an acceleration of 12 ft per sec?? What will be the total bearing reaction?
Ans. M = 578 ft-lb; R = 454.6 lb
1222. During the operation of a punch press, its flywheel decelerates uniformly
from 400 rpm to 200 rpm in 1 see. The rim of the flywheel weighs 1288 lb, its inside
and outside diameters are 56 in. and 60 in., and it is attached to its hub by 6 spokes.
What average shearing force is developed between the rim and each spoke during
the 1 sec interval? Ans. F = 350 lb
1223. The compound pulley in Fig. P-1223 has a centroidal mass moment of
inertia of 20 ft-lb-sec?. Find the tension in the cord supporting the 161-lb weight.
T= 20 ft-lb-sec?
ie=322 Ib
k =2ft

Fig. P-1223. Fia. P-1224 and P-1225.

1224. Determine the time required for the compound pulley shown in Fig.
P-1224 to reach a speed of 600 rpm starting from rest. Ans. t = 37.3 sec
1225. To what value should the weight of A in Fig. P-1224 be changed to give
it a downward acceleration of 9 ft per sec??
1226. If the weight shown in Fig. P-1226 is descending freely, determine the
tension in the cord both before and after a brake force P = 100 lb is applied.
Neglect thickness of brake. Ans. Before, T = 65.2 lb; after, T = 146 lb

P T=12ft-lb-sec”
T = 40 ft-lb-sec?

Fia. P-1226 and P-1227. Fiq. P-1228,.


326 ROTATION [Chap. XII

1227. If the drum in Fig. P-1226 is rotating clockwise at 120 rpm, solve for the
brake force P required to bring the system to rest in 5 sec. Assume the brake block
to be 6 in. thick. The coefficient of kinetic friction at the brake is 0.20.
1228. Find the tension in the cord attached to block A in Fig. P-1228. Neglect
the weight of the floating pulley supporting weight B. Ans. Ta = 192 lb
.
Wz = 3001b

{ae lb
k =1.732 ft

Fig. P-1229.

1229. Compute the tension in the cord attached to block A in Fig. P-1229. The
coefficient of kinetic friction under both blocks is 0.20. Ans. Ta = 60.5lb
1230. Determine the maximum weight of A that will permit the 400-lb block
B to slide without tipping over. Ans. Wa = 199 lb

{We 644 Ib
3 = Pte,
Fie. P-1230.

1231. In the system in Fig. P-1231, block A has a downward velocity of 48 ft


per sec at the instant the brake is applied. What is the tension in the cord between
A and B after the brake is applied? How far will block A have moved 2 sec after
the brake is applied? Neglect thickness of brake.
Ans. T = 488.8 lb; s = 62.6 ft
1232. Assume the maximum strength of the cord supporting block A in Fig.
P-1231 is 700 lb and of that joining drums B and C is 1200 Ib. If the brake is ap-
plied too suddenly, one of these cords will fail. Which one will it be and at what
brake force P? Ans. P= 304 Ib
Art. 12-7]
cee Non-CentroiNi
dal Rotation. Dynamic Equilibrium
aaa aitll ld 327

W =483 i
k=1.414'
P=200 Ib
ee lb
k=2'

W=322 lb
Fig. P-1231 and P-1232.

12-7. Non-Centroidal Rotation. Dynamic Equilibrium

In applying the equations 2N = ae Tf = ee and 2M, = Ia to


non-centroidal rotation, we must be very careful to choose the positive
senses of the N, 7, and 7 summations to agree with the positive senses of
Gn, G:, and a respectively. These equations are used with a free-body
diagram which shows only the applied forces. (See Illus. Prob. 1233 below.)
An alternate solution using dynamic equilibrium permits axes to be
oriented at will and moment summations to be taken about any center
without any restrictions as to sign. When the equations of dynamic
equilibrium are used, they refer to a free-body diagram which includes
both the applied forces and the inertia forces.
A condition of dynamic equilibrium may be created, as in the ease of
translation, by imagining that an equilibrant is applied which is equal,
opposite, and collinear to FR, the resultant of the applied forces. Unlike
translation, this equilibrant does not act through the gravity center;
instead, it can balance R only by acting through the center of percus-
sion P.
This is shown in part (a) of Fig. 12-8 where the equilibrant is represented
by its components , Fu? and 7a which are directed respectively opposite
to Gd, and d;. This representation of dynamic equilibrium is not especially
2
Zz
convenient since we must remember that P is located a distance q = =
from the axis of rotation. The best state of dynamic equilibrium is shown
in part (c) which consists of the components of the inertia force acting
through the gravity center G plus an inertia couple. It is derived from
: ; [ewe
part (a) by adding a pair of equal opposite forces of magnitude 7 Fa at G
328 ROTATION [Chap. XII

(b)
Fig. 12-8. — Dynamic equilibrium created by inertia forces acting at center of percus-
sion in (a) or by inertia forces at gravity center plus inertia couple in (c).

as shown in part (b). The addition of these balanced forces permits us to


AT

replace 7 Fa acting at P by an equivalent force at G plus a couple. The


couple has the moment

—Wo Fa (=
fk
—#)
a
= (2k?
Wea
—+
Week
P)a =
;
To
vi g g
: et
since by the transfer formula, the expression Ae ey i =), — Vd

I which is the centroidal moment of inertia. The inertia couple Ja is


represented by the directed are and may be applied anywhere on the body
since the moment of a couple is independent of the moment axis.
Summarizing, we create dynamic equilibrium in rotation by the following
procedure:
1. Apply the reversed normal effective force ' Fw” (also called the cen-
trifugal inertia force) acting radially outward through the center of gravity
opposite to the direction of d,.
2. Apply the reversed tangential effective force 7 (also called the
tangential inertia force) acting through the center of gravity opposite to the
direction of a.
3. Apply the inertia couple Ja anywhere on the body acting opposite to
the sense of a.
The only exception to these rules concerns the location of the centrifugal
inertia force in bodies which are unsymmetrical with respect to the plane
of motion described by the center of gravity. In these cases, the centrifugal
Art. 12-7] — Non-Centroidal Rotation. Dynamic Equilibrium 329
a a EY
inertia force does not act through the center of gravity although its magni-
=
: Wer
tude is always = 7w*. One example of such unsymmetrical rotation is the
case of a slender rod rotating an angle with the axis of rotation as in Fig.
12-9. In part (a) the centrifugal inertia force acts radially outward from

(a) (b)
Fic. 12-9. — Centrifugal inertia forces acting on a rod inclined to the axis of rotation.

the axis of rotation through the eentroid of the triangularly distributed


a ae:
forces SS rw and equals vt Fw? = i (
5)w*. In part (b) the rod is dis-
g g g
placed a distance a from the axis of rotation so that the centrifugal inertia
force consists of two parts acting as shown and determined by
LPs
Phat = (a+ a Balle
5)ot= autld RON a
+ 7 (5)

ILLUSTRATIVE PROBLEMS
1233. A uniform slender rod, 8 ft long and weighing 96.6 lb, rotates in a vertical
plane about a horizontal axis 1 ft from its end. When it is in the horizontal position
shown in Fig. 12-10, its angular velocity is 4 rad per sec clockwise. What is then
its angular acceleration and the bearing reaction at A?
Solution I: In this first solution, we shall not use dynamic equilibrium. Accord-
ingly, the FBD in Fig. 12-10 shows only the externally applied forces. In applying
the equations of rotation to this non-centroidal rotation, be careful to take ZN,
>T, and =M as positive in the respective directions of Gn, d1, and a.
We start by computing the moment of inertia Z4 about the axis of rotation at A.
Using the transfer formula, we obtain
2 Ma] 96.6\,.., 96.6
(i =I14+ joes (298)9) + 35908)? = 16 + 27 = 43 ft-lb-sec’
330 ROTATION [Chap. XII

w = 4 rad/sec a

Fig. 12-10.
The value of a is found by applying Eq. (12-7). This moment summation about
the axis of rotation eliminates the unknown bearing reaction. Thus we find
(2M, = 1, a] 96.6 X 3 = 48a a = 6.74 rad per sec? . Ans.
The reaction at A has been resolved into the normal and tangential components
R, and R;. Taking positive summations in the directions of a, and G1, we have
Ww 96.6
Es
> = —j Fu?
Fa j lip SS 39, (3) (4) 2
SS Rn == 144 lb

Since d@; is directed downward, DT is also positive downward. Hence we have

Se = Ue Fa 96.6 — Ri = 96.636, 74) Ri = 35.9 lb


g B22
Combining its components, the total bearing reaction is

E =VR,+ R | R = ./(144)? + (85.9)? = 148.5 lb Ans.


Solution II: In this solution we create dynamic equilibrium by applying the
centrifugal and tangential inertia forces at the gravity center G and add the inertia
, Fra = 9a lb
| eee
[~~ Ja =16e ft-lb
ive A G

W = 96.6 lb
Fig. 12-11. — Dynamic equilibrium.

couple Ja as shown by the dashed vectors in Fig. 12-11. Each of these inertia
components act respectively opposite to @,, @:, and a. Their values are:
W ge _ 96.6
Lo Sirat= 144 lb
; Fo) moo
39,9) (4)?

WwW. 96.6
— fa = 99.9°8) ¢ = 9a lb

= 1 96.6
lo=
a 7 5 Mis)a = yee
( 2qQ = 16a ft-lb
39,9 (8)?a
Art. 12-7] Non-Centroidal Rotation. Dynamic Equilibriwm 331

Applying the equations of dynamic equilibrium, we find a from a moment sum-


mation about A, thereby eliminating the unknown bearing reaction.

[2M = 0] 96.6 X 3 — 39a) —1l6a=0 a = 6.74 rad per sec? Ans.


This moment summation is really identical to 2M4 = I4q used in the preceding
solution because the sum of the a terms is 27a + 16a = 43 aas before. Actually
the sum of the inertia couple and the moment of the tangential inertia force is
A W = b :
Ia + —fa-F = UI + MP)a which, by the transfer formula, is 74a. Hence either
g
the moment equation of rotation or a moment summation of dynamic equilibrium
may be used to find a, depending on one’s preference. The real advantage of
dynamic equilibrium becomes evident when we want to take force summations
along other than the N and T directions, or a moment summation about some other
axis than the axis of rotation. These advantages will be more apparent in the next
sample problem.
Here we can show one advantage of dynamic equilibrium by finding R; from a
moment summation about the gravity center G. This gives
16(6.74)
(2M, = 0] 3R:-—lIa=0 or R= 3 =o). ONL

We can also use force summations to obtain

ie — 0] R:+ 9(6.74) — 96.6 = 0 1h = B50) ihe


(2H = 0] R, — 144=0 In, = WETS

whence, as before, the total bearing reaction is R = 148.5 lb.

1234. A turntable rotating in a horizontal plane about a vertical axis O carries a


bent bar weighing 16.1 lb per ft attached to it at A and forced to rotate with it by
a smooth peg at C. At the instant shown in Fig. 12-12, w = 4 rad per sec and
a = 6 rad per sec? both clockwise. Determine the forces acting at A and C.
Solution: The equations of rotation are unwieldy in this problem so we resort to
dynamic equilibrium which permits a free choice of axes and moment centers. In-
stead of locating the center of gravity of the bent bar, it is more convenient to apply
the inertia components acting as shown in Fig. 12-13 at the gravity center of each
segment.
The values of the inertia forces are

For AB: For BC:


Ww. 16.1ee X 6 eye? a= 96 1b
T put = (EXP m Fat = (=
Lee) )03)(a)?4)?= 961 lb
= 96

Ww x 4
16.1 aes W x 4)
(ex di
LEER Sod (te = 36 lb —fa = (3)(6) = 361b
Gg ( 32.2 /) 4 32.9 _))6)
The resultant inertia couple /a is the sum of the inertia couples acting on each
segment. Its value is
332 RorTaTIon [Chap. XII

ky itok 1 (008) 63 1 (S48)ay


Ta = 70 ft-lb
We now apply the equations of dynamic equilibrium. The value of P is de-
termined from a moment summation about A.
w= 4 rad/sec i
PE « = 6 rad/sec” To = 70 ft-lb W-=, —
aan g°o5 36 lb

—— >
Wr? =96 lb

As 1 Fe =361b
Ay
Wrw? =96Ib
Fic. 12-12, Fig. 12-13.
[5M,=0] 4P + 36(6) — 96(2) — 96(3) + 70 =0 P =48.5lb Ans.
Using force summations directed along the perpendicular components of the
reaction at A, we obtain
[2X = 0] A, + 48.5 — 96 — 36 = 0 dle 87.5 lb
[ZY = 0] A, — 96 + 36 =0 Any, 60 Ib
from which the total reaction at A is found to be

A = V (87.5)? + (60)? = 106.2Ib Ans.

PROBLEMS
1235. A 3220-lb flywheel is fastened to the midpoint of a shaft 6 ft long. The
center of gravity of the flywheel is 0.01 in. from the axis of rotation. The flywheel
rotates at a constant speed of 1800 rpm. Determine the maximum and minimum
values of the bearing reactions at each end of the shaft.
Ans. Max. i = 3090 lb> Mans y= 130ilb
1236. A uniform slender rod 6 ft long that weighs 64.4 lb is suspended vertically
atoneend. A horizontal force of 32 lb is applied at the midpoint of the rod. Deter-
mine the horizontal reaction of the axis on the rod. Where should the force be
applied to make the horizontal reaction
As ae toe zero? (This point is called the center of
ie g B percussion.)
a 8 1237. The uniform slender rod in Fig.
Fig. P=1237.
P-1237 weighs 96.6 lb and is supported on
knife edges at A and B. Determine the reaction at A the instant after the support
at B is suddenly removed. Ans. Ra = 46.5 lb
Art. 12-7] Non-Centroidal Rotation. Dynamic Equilibrium 333

1238. A man weighing 161 lb is seated


on a horizontal turntable 2 ft away from
the vertical axis of rotation, as shown in
Fig. P-1238. The coefficient of friction 161-lb man
between him and the turntable is 0.40.
If the turntable starts from rest and ac- Rea8
celerates at the rate of } rad per sec?, how
many seconds will elapse before he starts
to slide? Determine the angle 6 of the di-
rection in which he will slide.
Ans. t = 5.08 sec; 0 = 4.45°
Fia. P-1238.
1239. A uniform slender rod is hinged
to a frame rotating about a vertical axis as in Fig. P-1239. Show that the angle

between the rod and the axis is defined by cos @ = 3g


2
Le?
1240. A uniform slender rod weighing 96.6 lb is fastened to the rotating frame
in Fig. P-1240 by a smooth hinge at A and a horizontal cord at B. The frame
rotates about its vertical axis at a constant speed of 4 rad per sec. Find the tension
in the cord and the horizontal and vertical components of the hinge reaction.

Fic. P-1239. Fic. P-1240 and P-1241.

1241. Determine the speed of rotation in rpm at which the cord in Prob. 1240
will have a tensile force of 200 lb. Ans. n = 51 rpm
1242. A uniform slender rod 6 ft long and weighing y lb per ft is fastened at its
midpoint to a horizontal shaft as shown in Fig. P-1242. The rod is attached to the
shaft midway between two bearings A and B a distance L ft apart. Compute the
dynamic reactions at A and B when the shaft is rotating at w rad per sec.
3g)?
Ans. Rs = Re = ——
334 RoTATION [Chap. XII

Fia. P-1242. Fia. P-1243 and P-1244.

1243. Two blocks having the weights and positions shown in Fig. P-1243 rest
upon a frame which rotates about its vertical axis at a constant speed. The co-
efficient of friction between the blocks and the frame is 0.20. The weight and friction
of the pulley being neglected, at what speed in rpm will the blocks start to slide?
What is the tension in the cord at this instant?
Ans. n = 31.4rpm; T = 22.5 lb
1244. Repeat Prob. 1243 if the weights of the blocks
are interchanged.
1245. Three bars, each 2 ft long and weighing 9.66 lb,
are pinned together to form the equilateral frame shown in
Fig. P-1245. They rotate in a horizontal plane about a
vertical axis at A. What torque is required to cause an an-
gular acceleration of 12 rad per sec?? What is the reaction
Fig. P-1245. at A when the frame reaches a speed of 38.2 rpm?
Ans. M = 21.6 ft-lb; A = 20.8 lb
1246. The bent bar shown in Fig. P-1246 weighs 16.1 lb per ft. It rests on a
smooth horizontal surface and rotates about a vertical axis through A. Compute
the torque required to cause a counterclockwise acceleration of 6 rad per sec?. What
are the X and Y components of the reaction at A when the speed is 3 rad per sec?
Ans. M = 568 ft-lb; Az = 15 lb; Ay = 198 lb

Hee 64.4 Ib
L=6h

W=832.2 lb
Diam.= 2 ft

Fie. P-1246 and P-1247. Fig. P-1248.

1247. The bent bar shown in Fig. P-1246 weighs 16.1 lb per ft and is free to
rotate in a vertical plane about a horizontal axis at A. Compute the X and Y
Art. 12-7] Non-Centroidal Rotation. Dynamic Equilibrium 335
a ehae eet Lape ae dt
components of the bearing reaction at A an instant after it is released from rest at
the given position. Ans. A, = 49.0 lb: A, = 133.4 lb
1248. The system shown in Fig. P-1248 consists of a circular disk welded to the
end of a uniform bar. The assembly rotates in a vertical plane about a horizontal
axis at A. At the given position, the angular velocity is 4 rad per sec. Compute
the magnitude of the bearing reaction.

k,=1.5'
w=5 rad/sec

W =193.2 lb

W=322 lb

Fig. P-1249.

1249. At the instant shown in Fig. P-1249, the body B has a clockwise angular
velocity of 5rad persec. The horizontal cord joining A and B passes over a weight-
less and frictionless pulley. Determine the horizontal and vertical components of
the axle reaction at Z. AMS, Iii, == BMWS Tir, = BRE Ile

1250. The rotating assembly shown in Fig. P-1250 consists of an unbalanced


pulley to which is bolted a uniform rod carrying a sphere at its end. The pulley
rotates about a horizontal axis at Z and
has a 1 ft radius of gyration about its W=644 oa
gravity center G. Show that the mass k=1ft
moment of inertia about Z is I, = 45.4
ft-lb-sec? and then compute the angular
acceleration of the pulley and the tension
in the cord.
Ans. a = 4.64 rad per sec?; T =
137.5 lb
1251. At the instant shown in Fig. P- W=96.6lb 193.2 Ib
1250, the system has a clockwise angular W=4021h
velocity of 4 rad per sec. Using the re-
sults of Prob. 1250, compute the horizontal
and vertical components of the bearing re-
action at Z.
Ans. Rp = 156.5 1b; R, = 378.5 lb Fic. P-1250 and P-1251.
336 ROTATION [Chap. XII

1252. Two eccentric weights, W: = 100 lb and W, = 200 lb, are fastened to
the rotating horizontal shaft AB shown in Fig. P-1252. Compute the values of
balance weights concentrated 1 ft from the shaft and rotating in vertical planes

18” W,=2001b

Fig. P-1252.

through A and B that will balance the dynamic effects of W; and W»2. What are
the angular positions of the balance weights measured from the plane containing
W, and axis AB? Ans, Win = IS ilo, Oa = ZOOS Wig = 228i Ilo, Oe = 717

SUMMARY
Rotation (Art. 12-1) is the motion of a rigid body in which the particles
move in circular paths with their centers on a fixed straight line called the
axis of rotation.
Angular displacement is measured in radians by the angular distance
swept through by any radius of or line in the rotating body. The kinematic
characteristics of rotation are as follows: All particles have identical
values of angular displacement, angular velocity, and angular acceleration.
The linear values of displacement, velocity, and acceleration vary directly
with the distance of the particle from the axis of rotation.
The kinematic differential equations of rotation are tabulated below with
similar equations for rectilinear motion.

Rectilinear Motion Rotation


aa _ dé

peouy.d’s _dw _ d’6


Sale ct ot aP ce
vdv = ads wo dw = a dé (12-3)
Summary SSH

These equations are related by


= iit)
= TP
Coe ter (12-4)

a, = Tw”

The initial direction of rotation is taken as the positive sense of 0; this


also determines the positive sense of w and a.
From the above relations it is easy to determine the equations of rotation
with constant angular acceleration (Art. 12-3). These equations are tabu-
lated below.

Rectilinear Motion Rotation


(related by)
v=v,+at s = 7r6 ® = Ww. +at
s = vo + 3 al? v= To 6 = wot + 4 al?
v2 =v, + 2as a=rTra w = wo” + 206

Problems involving variable acceleration (Art. 12-4) are solved by inte-


grating the kinematic differential equations of rotation. The technique is
similar to that developed in Arts. 10-4 and 10-5.
Problems involving the rigid body motion of rotation are referred to a
set of reference axes rotating with the body. The WN axis is chosen to pass
through the center of gravity and the center of rotation; the T axis passes
through the center of rotation perpendicular to the N axis; the Z axis
coincides with the axis of rotation.
The general kinetic equations of rotation (Art. 12-5) are given by

=N = bg. nplll To” (12-5)


g g

== W as = ey (12-6)
g g
iM, = la (12-7)
In the case of centroidal rotation, 7 = 0 and the above equations reduce to
=M = Ia (12-9)
Dynamic equilibrium in rotation (Art. 12-7) is created by applying the
inertia forces i Fw and ufFa acting through the center of gravity directed
‘ J . .

g
respectively opposite to d, and d;, plus an inertia couple Ia opposite in
sense to a applied anywhere on the body. Note that J is the centroidal
moment of inertia.
Chapter XII.
Plane Motion

13-1. Definition and Analysis of Plane Motion


Plane motion is that motion of a rigid body in which all particles in the
body remain at a constant distance from a fixed reference plane. Con-
sequently all particles move in parallel planes and all particles lying on the
same straight line perpendicular to the reference plane have identical
values of displacement, velocity, and acceleration. The plane in which
the center of gravity moves is the plane of motion. In the articles which
follow, the bodies are assumed to be symmetrical with respect to their
planes of motion.
Examples of bodies having plane mo-
tion are rolling wheels, the connecting
/ ; rod of a reciprocating engine, or the links
; joining rotating elements of machines.
In all these cases, the identifying char-
. \ acteristic of plane motion is that a re-
ee ee ney ference line connecting any two points
SB een ihe plane of motion simultaneously
Fie. 13-1. — Plane motion. undergoes an angular displacement and
a linear displacement. Actually, a plane
motion is a simultaneous combination of the motions of translation and
rotation. For example, consider the rigid body represented in Fig. 13-1
by its projection upon a fixed reference
plane coinciding with the plane of mo- eae
tion. Assume that the body changes its aes A }
position from the full to the broken out- ! i
line during the time At. Assume the mo- r (
tion of two points A and B in the body \ ss
to be as shown and to have the respec- ‘ \
tive displacements s4 and sz. ‘aa B Pee Tors B’
To determine a relation between the yyq 13-2.— Plane motion re-
movements of points A and B, assume solved into a combined rotation and
that the body is rotated through an an- imaua tion,
gle @ about an axis passing through A until line AB is parallel to line
A’B’ (Fig. 13-2). The body may now be given the curvilinear trans-
338
Art. 13-1] Definition and Analysis of Plane Motion 339

lation defined by the motion of A, thereby causing the body to coincide


with its final position. The motion of B is the geometric sum of sg/4 and
sa and may be expressed by the vector equation
SB Il 84 Db SB/a
or (13-1)
SB Sa b 76

The symbol sz;4 is to be read as the displacement of B assumed rotating


about A; it is equal to 76, where r is the distance between A and B.
In a finite time interval, the actual movement of point B will only ap-
proximately equal s4 +4 sz/4 since this equation assumes that the trans-
lation and rotation of the body take place independently, whereas they
actually occur simultaneously. For purposes of analysis, however, there is
the valuable concept that plane motion is equivalent to a rigid body rota-
tion about any point plus the rigid body translation of that point. No error
will occur when the time interval between any two successive positions ap-
proaches zero as a limit. Then the actual motion is equivalent to successive
infinitesimal displacements from one instantaneous position to the next.
Each such infinitesimal displacement consists of the vector sum of the trans-
lation of an arbitrarily selected reference point plus a rotation about that
reference point.
Differentiating Eq. (13-1) with respect to the time (i.e., assuming
At — 0), we have
VB = VA UB/A
or (13-2)
Vp = Va dT

The symbol vg, is read as the velocity of B rotating about A. It is equal


to rw where the angular velocity w is defined as a ie., the time rate of
change of the body rotation 6, and r is the distance between A and B.
In like manner, differentiating Eq. (13-2) with respect to the time yields

ap = G4 + aBya
or (13-3)
Gp = 44H 7To* bra

where the symbol az,a is to be read as the acceleration of B assumed rotating


about A. This acceleration may in turn be resolved into a normal com-
ponent rw* directed from B toward A and a tangential component ra
directed perpendicular tov. (See Arts. 11-5 and 12-2.) The angular accel-
eration a is defined by a i.e., the time rate of change of the body angular

velocity w.
340 PLANE MOoTION [Chap. XIII

To summarize this discussion, the equations will be rewritten in the


following form:
Sp = Sab (Saya = 10) (13-1)
Vg = Va DH (Vea = Tw) (13-2)
ap = a4p (apy = to? b 1a) (13-3)

The use of parentheses to enclose equivalent terms may be confusing at


first. However, if the student remembers that the terms in the parentheses
are equivalent, the advantage of a continual reminder that a relative
motion, such as ag/4, may be expressed in terms of its equivalent rotational
components will more than offset any initial confusion. Of course, if he
prefers, he may use the more conventional form of these equations derived
earlier. He should note also that Eqs. (13-1), (18-2), and (13-3) are really
the equations of relative motion of a point B with respect to another point
A. They may be used to determine the relative motion between any two
points.
Finally, it is very important that the student understand that the rota-
tional components of plane motion are independent of the choice of a
reference point. Our discussion of pure rotation on page 311 shows that
6, w, and a represent the angular properties of any line in the rotating body.
Similarly in plane motion, 6, w, and a are defined in terms of the angular
motion of any line in the body. It is easy to demonstrate that such a
reference line has the same angular motion regardless of the choice of any
arbitrarily selected reference point. For example, if the body in Fig.
Rotation about B as
ne reference point
~<a

Rotation about A as ~-——


reference point
Fig. 13-3. — Angular motion is independent of reference point.

13-3 were rotating about B instead of A as previously assumed in Fig. 13-2,


its motion would be equivalent to a rotation about B, plus the translation of
B. The reference line AB would then rotate about B to the position BA”
Art. 13-2] Interpretation of the Kinematic Equations
I R ACE Oe aa, Sitch lee 341
parallel to the final position B’A’, after which the body would be trans-
lated along the path BB’’B’ to its final position. Obviously the angle @
defined by ABA” is counterclockwise about B and equal to the angle
BAB" which defines the same counterclockwise rotation @ about A as a
reference point. Hence in plane motion, the values of 0, w, and a@ refer to
the body and have the same values and directions regardless of what refer-
ence point is selected.

13-2. Interpretation of the Kinematic Equations


Before the kinematic equations of plane motion can be applied, it is
essential that we understand their physical significance. Consider first the
angular component of the plane motion of a rigid body shown in successive
positions in Fig. 13-4. It is evident that the angular position of line AB
is changing in a counterclockwise
sense; hence the angular velocity
w is also counterclockwise as shown.
The rate of change w, which is the
angular acceleration a, is assumed
to be counterclockwise also (al-
though a may be oppositely directed
to w if the body’s angular velocity
is decreasing).
Velocity Equation. Let it be as-
sumed that a rigid body has the / ¢ / ‘Path ofB
angular velocity w and that a refer- ‘ ._-~
ence point A has the linear velocity Fic. 13-4. — Change in angular position
v4ity shown in Fig. 13-5a. The veloo- ° Hing,4Bscgompenied
of any other point B is deter-
byangular ve-
mined by Eq. (13-2) [i.e., vs = va4 (ve/4 = Tw)| which, expressed in
words, says that the resultant velocity of B is equal to the vector sum of
the translational velocity of the reference point A and the rotational ve-
locity of B about A. Figure 13-5b illustrates the translational effect of the
reference point A while Fig. 13—5c shows the rotational velocities about A.
The vector summation giving the velocity of B is shown graphically in
Fig. 13—5d.
If desired, the velocity of B may be determined analytically from the
vector polygon of Fig. 13-5d. Since vz is the resultant of the vector sum of
va and vg/a, we may project these components of vg upon arbitrarily selected
reference axes and use the principle that the projection of the resultant is
the sum of the corresponding projections of its parts. Therefore, selecting
X and Y axes as shown, we write vg, = Zv, and vg, = Zvy whence the
magnitude of vg is given by V (vz,)? + (vz,)? and its inclination with the X
342 PLANE Motion [Chap. XIII

UpjA=TW

UB/A= TW
a of B
(b) (c) (d)
(a)
Velocity of — Velocity of Velocity of
Plane Motion ~ ‘Translation Rotation
Fig. 13-5. — Interpretation of velocity equation.

axis by tan 0, = a When the directions of all the velocity vectors are
i

known, as well as the magnitude of one of them, the simplest solution is to


apply the sine law directly to the velocity polygon to determine the magni-
tudes of the other two velocities.
Acceleration Equation. Let it be assumed that w, a, and the acceleration
aa of the reference point A in Fig. 13-6 are known and have the directions
shown in (a). In parts (b) and (ce), the plane motion of the body is resolved
into a translation (in which every point has the same acceleration as the
reference point A), plus a rotation about the reference point A. The ac-
celeration of any other point B is the vector combination of these motions
as defined by Eq. (13-8) [i.e., ag = aa BH (@z/4 = Tw?4 ra)] which says
that the resultant acceleration of B is due to the vector sum of the accelera-
tion of the reference point A, the normal acceleration of B rotating about A,
and the tangential acceleration of B rotating about A. This summation is
shown graphically in (d).
For analytical calculations, we may draw the acceleration components
of ag at acommon point as shown in (e). Now apply the principle that the x
and y components of the resultant are equal to the corresponding summa-
tions of its parts; i.e., as, = Za, and ag, = La,, whence the magnitude of ag
is given by ag = V(az,)? + (az,)? and its inclination with the X axis by
tan 6, = op
as,
An alternate analytical method is to transpose the resultant acceleration
az to the right side of Eq. (13-8) giving 0 = a4 4 ag/;4 — ag. Since sub-
Art. 13-3] Translation and Rotation for Freely Rolling Disksand Spheres 343
eee eee Segue tenie PS en gyee rg YE eRe

(a) (b) (c) (e)


Acceleration — Acceleration Acceleration
of Plane Motion of Translation of Rotation
Fic. 13-6. — Interpretation of acceleration equation.

tracting the vector ag is equivalent to adding its reversed value, Eq. (13-8)
may then be rewritten as 0 = a4 4 agya HD Gz,,, Which is an addition of
vectors whose sum is zero. This summation is equivalent to equilibrium
of the acceleration vectors. The application of this concept is covered in
detail in Illus. Prob. 1803 on page 349.

13-3. Relation Between Translation and Rotation for Freely Rolling Disks
and Spheres
In the majority of cases involving plane motion, the linear displace-
ment of any point in the body (A—A’ in Fig. 138-1) cannot be readily corre-
lated with the angular displacement of the body. Therefore relations
between s, v, and a of the point in the body and 6, w, and a of the body are
not generally known, but in certain cases they can easily be found.
For example, consider a disk or sphere of radius r rolling freely (.e.,
without slipping at the point of contact) along a plane as in Fig. 13-7.
Resolve the plane motion of the disk into a rotation about A plus the
translation of A. Then any reference line such as AB may be assumed to
rotate about A through the angle 6 after which it may be translated from
AB, through the distance s, to its final position A... If the plane motion
is resolved into a series of infinitesimal rotations and translations, the actual
path of B will be the cycloid shown dashed on the figure. The geometric
center of the disk obviously has a rectilinear translation s4, whereas the
344 PLANE Motion [Chap. XITI

angular displacement of the disk is 6. If the disk rolls freely along the plane,
distance BC = s4 = arc distance CDB, = 16.
The relation between linear and angular displacement having been es-

Daatencna S45 parts Sec


Fig. 13-7. — Free-rolling disk or sphere.

tablished, successive differentiation yields the following results which are


true at any particular instant (i.e., At — 0):

S80 (a)

dia. a dé fe
df Boon pete (b)

Gilg ROpeOns WAN ee (c)


dt GE Mana ne i reba
Note carefully that these relations between the linear motion of the geo-
metric axis of a disk or sphere and the angular motion of the body are true
only when the disk or sphere rolls freely without slipping. If it slips, these
relations are not true. Note also that positive w and a have the same direc-
tion as positive 6, i.e., clockwise for a rightward motion of the disk and
counterclockwise for a leftward motion. These directions are illustrated in
Fig. 13-7.

ILLUSTRATIVE PROBLEMS
1301. The wheel of 3-ft radius shown in Fig. 13-8 rolls freely to the right. At
the given position, w = 3 rad per sec anda = 5rad per sec?, both clockwise. Com-
pute the velocity and acceleration of the point B which is 2 ft from the center A of
the wheel.
Solution: By using the relations developed in Art. 13-3, the velocity and accel-
eration of the center A of the free-rolling wheel are respectively v4 = rw = 3 X 3 =
9 ft per sec and a4 = ra = 3 X 5 = 15 ft per sec®, both rightward.
Since the motion of A is known, resolve the plane motion of the wheel into a rota-
Art. 13-3] Translation and Rotation for Freely Rolling Disksand Spheres 345

tion about A plus the translation of A. Then the velocity of B is determined as


shown in Fig. 13-9a. In this figure, the sketches below the velocity equation give
the magnitude and sense of each known term. Note
that A is not actually stationary but is shown so
only to indicate the relative rotation of B about A.
If the terms in this vector equation are now plot-
ted to scale as in Fig. 13-9b, the values of vg and
6, are scaled off as 13.1 ft per sec and 23.4° re- eZ:
spectively. Analytically, the vector polygon can be
used to compute these results either by the cosine
law or by projection upon conveniently selected
reference axes. Using the latter method, we obtain rahe
vp, = Zvz = 9 + 6 cos 60° = 12 ft per sec
bg, = Lv, = —6sin 60° = —5.2 ft per sec Riceisis.
whence
ve = V(vz,)? + (vz,)? = 18.1 ft per sec down to the right at 0, = 23.4° Ans.

vU3= U4 +> (Upya = rw) es


va=9 x

6 ft/sec
(a) (b)
Fic. 13-9.

To determine the acceleration of B, use the acceleration equation shown in Fig.


13-10a. The sketches in part (a) show that the acceleration of B rotating about A
a,=15

Fig. 138-10.
346 PLANE Motion [Chap. XIII

consists of the normal component rw? = 2(3)? = 18 ft per sec and the tangential
component ra = 2(5) = 10 ft per sec? directed as shown. Plot the vector equation
in tip-to-tail fashion as shown in part (b) from which we scale the values ag = 18.3
ft per sec? and 6, = 75.4°.
For an analytical solution, it is convenient to draw the components of ag acting
at a common origin as shown in part (c). By taking summations with respect to
the reference axes shown, we obtain :
ap, = Laz = 15 + 10 cos 60° — 18 cos 30° = 4.6 ft per sec?
ap, = Za, = —10 sin 60° — 18 sin 30° = —17.7 ft per sec?
whence
ap = V(az,)? + (az,)? = 18.3 ft per sec? down to the right at 6, = 75.48° Ans.
1302. The dimensions of the reciprocating engine shown in skeleton outline in
Fig. 13-12 are as follows: crank AO = 1 ft, connecting rod AB = 3 ft. The crank
rotates with a constant clockwise speed of 10 rad per sec. Determine the accel-
eration of the piston, and w and @ of the connecting rod when the crank is in the
given position.
Preliminary: Since the piston has a motion of translation, every point in it has
the same motion. Therefore its acceleration will be that of point B which is com-
mon to both the piston and the connecting rod. The other end A of the connecting
rod has a motion of rotation about the crank center O. Its velocity v4 is perpen-
dicular to the crank AO and has the magnitude rwa9 = 1 X 10 = 10 ft per sec.
Also since A rotates about O with constant angular velocity, its acceleration a4
consists only of a normal component directed from A toward O and having the
magnitude rw45 = 1 X (10)? = 100 ft per sec?.
Graphic Solution: Since the motion of A is known, resolve the plane motion of the
connecting rod into a rotation about A plus the translation of A. To determine the
velocity of B, use the vector equation shown in Fig. 138-11 in which the sketches
specify the velocity vector of each term. The relative velocity vga of B assumed
rotating about A is perpendicular to the connecting rod AB.

Ug — Us cee (Upja = TWy4z)

v, =10 ft/sec

Up/a = 34yB
Fie. 13-11. — Interpretation of velocity equation.

The true directions of these velocity vectors are determined by the scale layout
of the engine in Fig. 13-12. Begin the velocity polygon by laying off v4 to any
convenient scale as shown in part (b). Through the tip of v4 draw VB/A perpen-
Art. 13-3] Translation and Rotation
for Freely Rolling Disks and Spheres 347
oa
FS a tema Ree ilies alee eats

Wa4o=

10 rad/sec
|
|

pw3p7 258
Fic. 13-12. — Velocity and acceleration polygons.

dicular to AB and through the tail of v4 draw vg. The intersection of vg and vg/a
determines their magnitudes which are scaled off to be
vp = 6.5ft persec and vga = 8.8 ft per sec
Placing the arrows on the velocity polygon so that vg is the resultant of the tip-
to-tail sum of v4 and vg,4, shows that vg;4 is directed down to the right. Since B
was assumed to rotate about A (see Fig. 13-11), the angular velocity of the con-
necting rod is counterclockwise and has the magnitude

) ) 8.8 ;
E = 4 OAB = a = sa = 2.93 rad per sec counterclockwise. Ans.

To determine the acceleration of B, we use the acceleration equation shown in


Fig. 13-13 in which the sketches indicate the acceleration vector of each term.
2
az — a, +> = (Apa = TWig +> TK)

‘AO TrOAB
my aea4 = TW 7?
ap =100 7 Wie
AB
ooS
°

.)

Fic. 13-13. — Interpretation of acceleration equation.

Obviously ag is constrained to be horizontal. As discussed previously, aa is directed


from A toward O with the magnitude 100 ft per sec’. The relative acceleration
apa means that we assume B to be rotating about A. It has the normal and tan-
gential components shown. The normal component is rw, and is directed from
B toward A with the magnitude 3 X (2.93)? = 25.8 ft per sec?. The tangential
348 PLANE Morion [Chap. XIII

component raap is perpendicular to AB. We must complete the acceleration


polygon before we can tell whether the direction of raz is up to the left as shown
(which we shall soon see is correct) or in the opposite direction.
To start the acceleration polygon, lay off a4 to scale as shown in Fig. 13-12c.
Through the tip of aa lay off the known value of rw,z. Now draw ra4z through the
tip of rw, perpendicular to AB and draw ag through the tail of a4. The inter-
section of ag and ra4g determines their magnitudes which are scaled off to be

ap = 104 ft persec? and (ar)p/a = roap = 47 ft per sec?

The arrows are placed on the acceleration polygon so that ag is the resultant of
the tip-to-tail sum of a4, rw4,, and raag. Now it is confirmed that the direction
of raap in Fig. 13-13 was correct, and hence a4 must be clockwise. Its magnitude
is given by

E = 2| AB = ets = 2 = 15.7rad per sec? clockwise Ans.

It will be observed that w4z and a4z are in opposite directions. The explanation
is that in the position considered, the connecting rod is slowing down as it approaches
its top position; shortly it will reverse its angular motion as the piston continues to
move to the right.
Analytical Solution: To obtain an analytical solution, we first compute the func-
tions of angle @ between AB and the horizontal. Referring to Fig. 13-12, we have
AD = AO sin 30° = 1 X 0.5 = 0.5; therefore sin ¢ = ie = = mG and cos @ =

ae =). 9S872

TH ap=30 4B Y
is Twig = 25.8
Up/a = 3Wap | d x

= 100
(a) (b) rf
Fig. 13-14. — Velocities and accelerations determined analytically.

Referring to the velocity polygon redrawn in Fig. 13-14a, assume X and Y ref-
erence axes (not shown) to be selected so that the X axis coincides with vg. Since
the components of the resultant velocity equals the summations of the components
of its parts, we have

[ve, = Zoy] 0 = 10 sin 60° — vg/4 cos


0 = 10 X 0.866 — 3 wag X 0.987
wap ll= 2.93 rad per sec counterclockwise Ans.
Art. 13-3]
re Translatio n-and
etree ree Rotation
ee for Freely Rolling Disksand Spheres 349
ee a erro NE ELE SOE
The positive value of waz confirms the direction of vg yA in Fig. 13-11 and hence
wap must be counterclockwise if B is considered to be rotating about A. Con-
tinuing with an X summation of the velocity vectors, we obtain

lv, = Zvz] ve = 10 cos 60° + (3 waz) sing


= 100.543 X 2.93 x $
vp = 6.47 ft persec Ans.
To determine the accelerations analytically, it is convenient to represent the
vector components of ag acting at a common origin as shown in Fig. 13-14b. The
reference axes are chosen so that the X axis coincides with the direction of ag whence
we obtain
[az, = Za,] 0 = 3a4z cos + 25.8 sin @ — 100 sin 30°
0 = 3a4B X 0.987 + 25.8 xX ¢ — 100 X 0.5
a4p = 15.5 rad per sec? clockwise Ans.
Since B is assumed rotating about A, the positive value of ra4g can be obtained
only by a clockwise sense of a4g about A as indicated in Fig. 13-13.

[ap, = Laz ap = 100 cos 30° + 25.8 cos @ — (8 a4z) sing


ap = 100 X 0.866 + 25.8 X 0.987 — 3 x 15.5 X =
apg = 104.4 ft per sec? rightwards Ans.
The positive value of ag shows that it acts in the positive sense of the X summa-
tion, 1.e., to the right.
1303. A straight rod 10 ft long is in a horizontal position with its ends in contact
with the inclined planes as in Fig. 13-15. At the instant shown, the angular ve-
locity is 2 rad per sec clockwise, and the an-
gular acceleration is 1 rad per sec? counterclock- w=( rad/sec
wise. For the position given, determine analyti- oc=1) rad /sec?
cally the linear velocity and linear acceleration of
the end points A and B.
Solution: The linear velocity and linear accel-
eration of the end points of the rod are evidently
constrained to be directed along the inclined Fig. 13-15.
planes. Assume A as the reference point, and re-
solve the plane motion of the rod into the translation of A plus the rotation of
the rod about A.
The direction of each vector in the velocity equation is indicated in Fig. 13-16a.
Starting with the known value of vz/4, we sketch the velocity polygon as shown in
part (b). Note that the arrows are placed so that vg is the resultant of vg;a and va.
Since all the angles and one side of the velocity triangle are known, we can apply
the sine law to obtain
UB VA we 20

sin 45° sin 60° sin 75°


whence
vp = 14.6ftpersec and v4 = 17.9ftpersec Ans.
350 PLANE Morion [Chap. XIII

Uz a Us
(=)
N
ll
UA <

30° a =
Ta as
Up
Uga= 10x 12
(a) = 20 ft/sec (b)
Fig. 13-16.

The acceleration equation is ag = aa (ap/a = Tw?+ ra) in which the magni-


tudes of ag and a4 are unknown. These accelerations must act along the respective
inclines, but may be directed either up or down them, depending on whether points
A and B are speeding up or slowing down.
In this example it may be advantageous to transpose the resultant acceleration
ap to the right side of the acceleration equation and then, instead of subtracting
the vector ag, to add its reversed value. Thus the acceleration equation becomes
0 = aa apap az, Since this summation of vectors is zero, the summations
of their components with respect to any set of reference axes must also be zero.
In effect, we establish equilibrium of the acceleration vectors by adding the reversed
value of ag to its components.
In Fig. 13-17b, we draw the vectors of the acceleration equation acting at a com-
mon origin. In part (a), the relative acceleration ag/4 of B assumed rotating about
ra =10x1 ra=10 x
=10 ft/sec? a
a

= 10(2)*
40 ft/sec?

(a)
Fig. 13-17.

A has the normal and tangential components shown. In part (b), we add to these
components the unknown vectors a4 and ag (reversed) directed as shown to create
equilibrium of this vector system. Then selecting the X axis through az,,, to
eliminate it from a Y summation, we obtain
Pah: = O 10 sin 60° + 40 sin 30° — a4 sin 75° = 0
aa = 29.7 ft per sec? Ans.
after which an X summation yields
[Zaz = 0] ap + 29.7 cos 75° + 10 cos 60° — 40 cos 30° = 0
ap = 21.95 ft per sec? Ans.
Art. 13-3] Translation and Rotation for Freely Rolling Disks and Spheres 351

Note that since ag was shown reversed, its true direction is down the 30° incline.
If the direction of either a4 or ag had been incorrectly assumed, negative values
would have been obtained. The positive results show that the accelerations of A
and B are both directed down the inclines. These results, in conjunction with those
for velocity, show point B to be speeding up, whereas point A is slowing down.

PROBLEMS
1304. The wheel shown in Fig. P-1304 rolls without slipping on the horizontal
plane. The angular velocity is 4 rad per sec clockwise and the angular acceleration
is 6 rad per sec? clockwise. Solve for the absolute accelerations of points A and C.
Ans. aa = 23.75 ft per sec?; ag = 32 ft per sec?
1305. The wheel described in Prob. 1304 rolls and slips on the horizontal plane.
The linear acceleration of the center O is a, = 4 ft per sec? to the right and w = 3
rad per sec and a = 5 rad per sec? both clockwise. Determine the absolute accel-
erations of points A and C.

Fic. P-1304 and P-1305. Fig. P-1306.

1306. Block B causes the compound disk shown in Fig. P-1306 to roll without
slipping up the 30° incline. If the angular acceleration of the disk is 4 rad per sec’,
find the vertical acceleration of B. What is the general relation between az, and a?
Upon what does ag, depend? In what
way are these results changed if the
disk rolls freely down the incline? As-
sume that the cord supporting B re-
mains vertical.
Ans. dp, = 8 ft per sec?
1307. Block B causes the com-
pound drum D in Fig. P-1307 to roll
without slipping up the incline. If the
linear acceleration of B is 2 ft per sec?
down, compute the vertical accelera-
tion of block A. Assume that the cord
supporting A remains vertical.
Ans. aa, = 6 ft per sec’? 4 Fic. P-1307.
352 PLANE Morion [Chap. XIII

1308. The compound wheel shown in


Fig. P-1308 rolls without slipping be-
tween the two parallel plates. At the given
position, plate A has a rightward velocity
and acceleration of 8 ft per sec and 12 ft
per sec? respectively. At this same instant
the wheel has an angular velocity of 4 rad
per sec clockwise and an angular accelera-
tion of 10 rad per sec? clockwise. Compute
the linear velocity and acceleration of plate
B and of point D on the wheel. Hint: First
find vg and ag.
Fig. P-1308. Ans. ap = 8 ft per sec? left;ap = 7.07
ft per sec?
1309. The 10-in. bar AB shown in Fig. P-1309 moves with its ends in contact
with the vertical wall and the horizontal floor. At the instant when 6 = 30°, the bar
has a clockwise angular velocity of 4 rad per sec
and a clockwise angular acceleration of 6 rad per
sec?. Determine the velocity and acceleration
of points A and B.
Ans. vp = 34.6 in. per sec; ag = 132 in.
per sec?
1310. Referring to Fig. P-1309, suppose that
when angle 6 = tan 4, the velocity of A is 24
in. per sec leftward and the acceleration of A is
38 in. per sec? rightward. Compute the accelera- Fig. P-1309 and P-1310.
tion of point B.
1311. The triangular plate ABC in Fig. P-1311 moves so that its vertices A and
B slide on the horizontal and vertical surfaces shown. If w = 1/2 rad per sec and
a = 4rad per sec’, both clockwise at the given position, compute the acceleration of
point C. Ans. ac = 23.35 ft per sec?

Fig. P-1311. Fie. P-1312.

1312. A ladder of length L is sliding down a vertical wall and along a horizontal
floor, as shown in Fig. P-1312. Its lower end A is moving with a constant velocity
Art. 18-3] Translation and Rotation
for Freely Rolling Disks and Spheres 353

va. Determine the angular velocity and angular acceleration of the ladder in terms
of va and the angular displacement 6. What are these values expressed in terms of
va and time ??
2 3

Ans. Een ee Fc = ~* sec? 6 tan 6; » = ——"4_sq = spielen


L i (L? — vg #)'? (L? — vat?)
1313. The rod in Fig. P-1318 slides with its ends in contact with the floor and the
inclined plane. At the instant shown, the velocity of A is 20 ft per sec leftward,
and the acceleration of A is 6 ft per sec? rightward. Compute the linear velocity and
linear acceleration of point B for the given instant.
Ans. vg = 20 ft per sec; ag = 40.2 ft per sec?

Hes P1313: Fig. P-1314.

1314. A bar 4 ft long slides with its ends A and B in contact with a horizontal
floor and an inclined plane as shown in Fig. P-1314. The angular velocity of the rod
is 3 rad per sec and its angular acceleration is 5 rad per sec”, both counterclockwise.
Find the acceleration of the end B and of the midpoint M of AB.
Ans. dp = 47.0 ft per sec?; ay = 27.3 ft per sec?

1315. Ina rigid body having plane motion, let A and B be any two points in the
plane of motion and M be a point midway between A and B. Prove that the accel-
eration of M is one-half the vector sum of the accelerations of points A and B; i.e.,
am = 4(aa+ ag). Use this result to check the midpoint acceleration of bar AB in
Prob. 1314 if it is known that a4 = 11.4 ft per sec? right and ag = 47.0 ft per sec?
down the incline.
1316. The 6-ft diameter disk shown in Fig. P-1316 rolls back and forth over a

Fig. P-1316. Fig. P-1317.


354 PLANE MOTION | [Chap. XIII

short distance without slipping. A 5-ft bar BC is pinned to the rim of the disk at B
and its other end C drags on the ground. If the center of the disk has a constant
rightward velocity v4 = 12 ft per sec at the given position, compute ve and ac.
Ans. vc = 21 ft per sec; ac = 104.3 ft per sec?
1317. In the four-link mechanism shown in Fig. P-1317, the driving crank AB
has a clockwise angular velocity of 4 rad per sec and a counterclockwise angular
acceleration of 10 rad per sec?. Determine the angular acceleration of the driven
crank CD for the given position. Ans. Qcp = 17 rad per sec?
1318. The reciprocating engine shown in skeleton outline in Fig. P-1318 has a
crank AQ 1 ft long and a connecting rod AB 4 ft long. Crank AO rotates clockwise
with a constant angular velocity of 10 rad per sec. Find the linear acceleration of B
and of the midpoint M of AB when in the position given.
Ans. ap = 71.1 ft per sec?; ay =
79.3 ft per sec?; a4gp = 17.4
rad per sec?
1319. Refer to the reciprocating
engine described in Prob. 1318. Figd
Fic. P-1318, P-1319, and P-1320. the acceleration of the pistonB at the
instant the crank has rotated 120°
clockwise from its extreme left position.
1320. At the instant when the crank AO of the engine shown in Fig. P-1318 has
rotated 90° clockwise from its extreme left position, the connecting rod AB has the
clockwise values wag = 3 rad per sec and aag = 5 rad per sec. For this position,
compute the velocity and acceleration of both the piston B and the crank AO.

Fig. P-1321.

1321. An inversion of the slider-crank mechanism has the slider B fixed as shown
in Fig. P-13821. The rod OD slides back and forth through the fixed block B as the
link AB oscillates about B and causes AO to describe plane motion. At the instant
shown, the rod OD has a constant leftward velocity of 7.24 ft per sec. Determine
graphically the angular acceleration of link AO for this position.
Ans. aso = 160 rad per sec? counterclockwise

13-4. Instantaneous Center and Instantaneous Axis of Rotation


In the preceding articles, we considered the plane motion of a rigid body
as equivalent to successive infinitesimal displacements from one instanta-
neous position to the next. Each such infinitesimal displacement consisted
of the vector sum of the translation of an arbitrarily selected reference
Art. 138-4] Instantaneous Center and Instantaneous Axis of Rotation 355

point plus a rotation about that reference point. If we could find a reference
point that is momentarily at rest, however, each such successive displace-
ment would consist only of a rotation about that reference point.
Consider now the body shown in Fig. 13-18 in which, at a particular
instant, the velocities of points A and B have the given directions. If
there is a center momentarily at rest
about which the body rotates, it will
be located at the intersection C of two
lines drawn through A and B perpendic-
ular tov, and vg respectively. The point
C is on the instantaneous axis of rota-
tion. This is the line, perpendicular to
the plane of motion, which joins those
points in the body which are instantan- Ty |
eously at rest and have zero velocity.
The intersection of this instantaneous NIC
axis with the plane of motion is called Bre iBoie. 2 Graptier determination
the instantaneous center of rotation or, of instant center.
more briefly, the instant center.
The value of using the instant center to determine velocities is evident
from a consideration of the kinematic equation for velocity. Thus, denot-
ing the instant center by C and assuming the plane motion to be resolved
into a rotation about C plus the translation of C, we can find the velocity
of any point B from the equation vg = ve H (vajc = Tw). The velocity of
C, however, must be zero by the definition of an instant center. Hence
the kinematic equation for determining velocities reduces to vg = ra,
which is equivalent to the kinematic equation of rotation. During the in-
stantaneous rotation, all points in the body have the same angular velocity
about the instant center, and the direction of the instantaneous velocity of
any point is perpendicular to the line joining the point with the instant
center.
Returning to Fig. 13-18, if the velocity of one of the points, say va, is
known, we can use the following equation to determine the angular velocity
of the body and the linear velocity of any other point B in the body:
VA VB
QoQ=-— => —
TA TB

If the two points A and B have parallel but unequal velocities, as shown in
Fig. 13-19 (a) or (b), the instant center C lies on the common perpendicular
to these velocities and is located by direct proportion.
The instant center can also be located if the linear velocity of one point
and the angular velocity of the body are both known at any instant. Thus
356 PLANE MOTION [Chap. XIII
ON a EE es

Fig. 13-19. — Determination of instant center when velocities are parallel, but unequal.

if point A in Fig. 13-20 has a known rightward velocity v4 and a known


clockwise value of w, the instant center C is located along the line passing
through A perpendicular to v4 and lying
very 2 distance r below A as given by the
relation
VA
r= —
a)

In the case of a wheel or sphere of


| radius r rolling freely along a plane,
AL the velocity of the geometric center
| is expressed by va = rw, as shown in
Art. 13-3. Hence we conclude that
bc the instant center of a wheel is located
Fic. 13-20.— Analytical determina-
tion of instant center.
at a distance r from the geometric
> ‘
center and, from the above discussion,
that it is at the point of contact with the plane. There is visual evidence to
confirm this, as for example the clear imprint left by a tire after it has run
over a patch of wet pavement.

13-5. The Instant Center Does not Have Zero Acceleration


If the instant center had no acceleration, it would remain fixed in space
and plane motion would become rotation about a fixed axis. Actually, the
space position of the instant center is not fixed, but always moves with some
acceleration. Consequently accelerations in plane motion cannot be com-
puted as though the body were rotating about the instant center of zero
velocity. This is a common error of the beginner who assumes that the
instant center of zero velocity is also the instant center of zero acceleration.
Although an instant center of zero acceleration does exist, there is no simple
method of locating it. Accelerations must be determined by using the
methods described in Art. 13-2.
However, in one important special case, that of a free-rolling disk, the
Art: 13-5] Use of the Instant Center 357

tangential acceleration of certain points can be computed as though the


disk were actually rotating about the instant center of zero velocity. These
points lie on the line joining the geometric center of the disk with its instant

(b)
Fic. 13-21. — Acceleration of any point B on the line joining the geometric center O
with the instant center C.

center of zero velocity. Thus consider the free-rolling disk in Fig. 13-2la
whose geometric center has the acceleration ag = ra as shown in Art. 13-3
on page 344. If the plane motion is resolved into a rotation about O plus
the translation of O, the acceleration of a point B is given by the equation
ag= (ao => ra) pb (azo => bo? b ba)

The graphic interpretation of this equation is shown in Fig. 13-21b. Obvi-


ously, the tangential acceleration of B (directed perpendicular to the line
BOC) is given by
dp, = Tra+
ba = (r+ b)a = na

where 7; is the distance from B to the instant center C.


If we disregard the normal component of acceleration, we can conclude
from the preceding discussion that the motion of any point along the line
joining the geometric center of a free-rolling disk with its instant center is
the same as though the disk were actually rotating about an axis through
the instant center. This concept gives us a convenient way to relate the
plane motion of a free-rolling disk with the motion of a body connected to
it. Several examples are given in the following ra
illustrative problems.
Finally, let us show that the acceleration of |
rw
the instant center C of the disk is not zero. °¢1
Figure 13-22 shows the graphic interpretation
of the equation dg=ra
Fic. 13-22. — Acceleration of
ac = (do = ra) # (acjo = Ta” b ra) the instant center C.

from which it is evident that the acceleration of C is equal to rw* and is


directed from C toward O.
358 PLANE MoTION [Chap. XIII
Se

ILLUSTRATIVE PROBLEMS

1322. The compound wheel shown in Fig. 13-28a rolls without slipping between
two parallel plates. The velocity of the upper plate A is v4 = 12 ft per sec to the
right and that of the lower plate B is vg = 6 ft per sec to the left. Find the angular
velocity of the wheel and the linear velocity of point D on it.

Ug = 12 ft/sec

Ug = 6 ft/sec

(b)
Fig. 13-23.

Solution: The points on the vertical centerline of the wheel which are in contact
with the plates have the velocities shown in Fig. 13-23b. The instant center C lies
on the line AB which is the common perpendicular to v4 and vg. Its location is
determined from

V4 _OB B_ Ss
= or TA =27rB
TA TER TA TB

which, substituted into r4 + rg = 24 in. obtained from the dimensions of the wheel,
gives r4 = 16 in. and rg = 8in. Therefore the angular velocity is
6 :
eee > = 9 rad per sec clockwise Ans.
TB 12

Returning to Fig. 13—23a, we find the velocity of D is perpendicular to the length


CD drawn from the instant center and its magnitude is

[v = ral vp = (2)? + (6)? (9) = 57 in. per sec = 4.75 ft per sec Ans.

It is useful to observe that the horizontal and vertical components of vp are


proportional to the vertical and horizontal components of the length CD; i.e.,

vp, = 2 X 9 = 18 in. per sec left and vp, = 6 X 9 = 54 in. per sec up

1323. The disks in Fig.13-24 are free to roll on the inclined planes. Disk A is
fastened to pulley P by cord Q, which is wrapped around the drum of the disk and
around the smaller pulley. Disk B is similarly attached to the outer wheel of the
pulley. The linear acceleration of the center of disk A is constant at 10 ft per sec?
Art. 13-5] Use of the Instant Center 359

down the incline. Determine the linear displacement, velocity, and acceleration of
the center of disk B after 4 sec, starting from rest.

Fic. 13-24.

Solution: The motion of pulley P is related to the motions of disks A and B by


cords Q and T. All the bodies can be considered as in a state of rotation — P about
its axle, A about its instant center C4, and B about its instant center Cz. Apply-
ing the relation between linear and angular displacement in rotation, we may ex-
press the linear displacements of points on cords Q and T in terms of the angular
displacements of the bodies by
sor hy = BOA = WX Oe (a)
sp = 20p = 463 (b)

From Eqs. (a) and (6) we obtain


304 > Op = 2 6p (c)
Velocity and acceleration relations between the disks and the pulley have similar
forms because »v = rw and a; = ra have the same mathematical form as s = r@.
Therefore we may also write
304 = Op = 2 OR (d)
DOA — Op — 2 Op (e)

Not all of the above discussion is really necessary to solve this problem; however,
it will be found useful in applying the work-energy method in Art. 14-11.
From the given data the angular acceleration of disk A can be expressed in terms
of its linear acceleration by
|d, = Fa] 1K) arom aa = 2 rad per sec?

Applying the relation (e), we now compute the angular acceleration of disk B.
[3a4 = 2 az] BK 22 ear ap = 3 rad per sec?
whence the acceleration of the geometric center of disk B is found from
[a, = 7a} ap =3xX3 dp = 9 ft per sec?
360 PLANE Morion [Chap. XIII

Since the motion of the center of disk B is rectilinear with constant acceleration
dp = 9 ft per sec, we may apply Eqs. (10-1) and (10-2) of Art. 10-2 to obtain

[v = v, + af] 0p =0+9X4 dp = 36 ft per sec Ans.


[s = vot + $ at] ssn = 0+4xX9 xX (4) 88
= 72 ft Ans.
In this problem as disk A rolls down the incline, its center travels the distance
34 = 564 while the cord Q travels sg = 364. Hence for each radian of angular dis-
placement of A, 2 ft of cord are unwound from the drum of A. Actually the un-
winding of the 2 ft of cord from A is the result of the rotation of the drum about the
geometric center of A. Similarly we may show that for each radian of angular
displacement of B, 1 ft of cord is unwound from B as it rolls up the incline. If the
motion of the system were reversed with A rolling up and B rolling down their
respective inclines, cords Q and T would wind up these amounts on A and B respec-
tively. In either case, the geometric center of A would still move faster than cord Q
while cord 7 would still move faster than the geometric center of B.
1324. Ata certain instant a four-
link mechanism occupies the position
shown in Fig. 13-25. The dimensions
are as shown. The angular velocity
of AB is wag = 11 rad per sec. Lo-
cate the instant center of BC and
WAaB= use it to determine the linear velocity
11 rad/sec es Ne of C and the angular velocity of CD.
= Solution: Points B and Care con-
i XQ | strained to rotate about the fixed
i NI bearings A and D respectively. Since
Ozc_
the velocity of B is therefore perpen-
Fig. 13-25. dicular to AB and the velocity of C
is perpendicular to CD, the inter-
section of AB and CD prolonged determines the instant center Ogc (Art. 13-4).
From the diagram, we have

DO = AD tan 30° = 3.464 X 0.577 = 2 ft


ae AD _ 3.464
4 ft
, cos 30° —-0.866
These distances could have been obtained graphically by drawing the mechanism
to scale. The graphic method is generally used in this type of problem.
Applying the principle that the velocity of any point is equivalent to rotation
about the instant center (Art. 13-4), we have
[vp = TaBwAB = TBOWBC] Mo OL = o.00ne wac = 3 rad per sec
lve = rcowsc] ve = (8+ 2) X3 = 15 ft persec Ans.
Also
luge = repwep| 15 =3 X wep wcp = 5rad persec Ans.
Art. 13-5] Use of the Instant Center 361

The velocity of any other point M is found by scaling the distance MO from a
scale diagram of the mechanism and applying vy = ryowsc. The direction of vy
is perpendicular to MO as shown.

PROBLEMS
1325. Using the instant center method, resolve Prob. 1312 on page 352 for w and
then differentiate w with respect to the time to obtain a in terms of w.
1326. Block B in Fig. P-1326 drops 48 ft with constant acceleration in 4 sec,
starting from rest. Compute the displacement, velocity, and acceleration of the
center of the free-rolling disk D after this time. What length of cord is wound on or
off the disk in this time?

Fic. P-1326.

1327. In Fig. P-1327, body B descends with a constant acceleration of 8 ft per


sec?. Its initial velocity is 12 ft per sec. After 5 sec have elapsed, what will be the
displacement and velocity of the center of the free-rolling disk? What length of cord
is wrapped on or off the disk in this time?
Ans. s = 200 ft; v = 65 ft per sec; L = 120 ft

Fie. P-1327.

1328. In the system described in Prob. 1307 on page 351, assume block B has
an upward velocity of 8 ft per sec and a constant downward acceleration of 3 ft
362 PLANE Morion [Chap. XIII

per sec?. Determine the vertical movement of block A after 2 sec have elapsed.
What is then the velocity of A? Ans. va = 7.21 ft per sec
1329. Make the following changes in Illus. Prob. 1323: The radii of disk A are
changed to 3 ft and 5 ft; of disk B to 2 ft and 4 ft; of pulley P to 9 in. and 18 in. If
the angular acceleration of disk A is 3 rad per sec? clockwise, determine the time for
the center of disk B to travel 60 ft from rest. Ans. t = 3.87 sec
1330. The initial downward velocity of block D in Fig. P-1330 is 12 ft per sec
and its acceleration is constant at 6 ft per sec?. Determine the linear displacement
of the center of the free-rolling disk A after 2 sec. Ans. sa = 24 ft

Fie. P-1330.

1331. The rectangular box is moving with its ends in contact with the floor and
wall shown in Fig. P-1331. The velocity of A is 4 ft per sec to the left. Using the
instant center method, determine graphically the velocity of points B, C, D, and of
M, the midpoint of CD.
Ans. vg = 6.93 ft per sec; vc = 6.02 ft per sec; vp = 2.07 ft per sec; vy = 2.07
ft per sec

Fig. P-1331. Fic. P-1332.


Art. 13-6] Equations of Plane Motion 363
1332. A 4-ft bar AB moves with its ends in contact with a horizontal floor and
an inclined plane as shown in Fig. P-1332. If the velocity of A is 9.6 ft per sec, use
the instant center method to find the velocity of the midpoint of the bar.
1333. The compound wheel described in Illus. Prob. 1322 on page 358 has a
clockwise angular velocity of 3 rad per sec at the instant that plate A has a right-
ward velocity of 9 ft per sec. With these conditions, compute the linear velocity
of plate B and of point D.
1334. Crank AO in Fig. P-1334 is A
rotating at 300 rpm. Locate the in-
stant center of the connecting rod 60°
AB. Use this to determine the veloc- : : of \
ity of B and of the midpoint of AB.
Check these values by the graphic | Fic. P-1334.
method given in Art. 13-2.
Ans. vg = 23.7 ft per sec; vw = 26.7 ft per sec
1335. Rods AB and CD are pinned together at B as shown in Fig. P-1335 and
move in a vertical plane with the absolute angular velocities w4g = 4 rad per sec
and wep = 5rad per sec. Locate the instant center of CD and use it to compute
the velocities of C and D. Ans. v¢ = 28.3 in. per sec; vp = 25 in. per sec
1336. Figure P-1336 shows the pitch circles
of two gears A and B connected by the arm OC.
Gear A turns counterclockwise at 12 rad per sec

erie Fie. P-1335. Fig. P-1336 and P-1337.

while arm OC turns clockwise at 8 rad per sec, thereby causing gear B to roll
around gear A. Locate the instant center of gear B and compute its angular
velocity. Hint: Study Fig. 13-19b.
Ans. 8 in. to left of C; wp = 18 rad per sec clockwise
1337. Repeat Prob. 1336 if gear A turns counterclockwise at 9 rad per sec while
arm OC also turns counterclockwise at 1 rad per sec.

13-6. Equations of Plane Motion


Before discussing the kinetics of plane motion, let us review the effect of
various types of force systems upon rigid bodies free to move in space,
364 PLANE MorIon [Chap. XIII

starting from rest. We have learned (see Art. 10-7) that if the resultant
of a force system is a single force which passes through the center of
gravity, a motion of translation ensues: rectilinear translation if the direc-
tion of the resultant force is constant, or curvilinear translation if the direc-
tion of the resultant force varies. A force system whose resultant is a
couple will cause the body on which it acts to have a motion of centroidal
rotation (see Art. 12-6). The resultant couple will create centroidal rota-
tion even if there is no fixed axis in the body about which it must rotate.
Consider now the non-concurrent force system acting on the body shown
in Fig. 13-26a. Let it be assumed that the resultant of this system reduces
ome P, oz aem~A

a a
a e a
ae 5

BP,
/2
. (a) (b) (c)
Fic. 13-26.

to the single force R shown in (b), which does not pass through the center of
gravity. This resultant may be replaced (see Art. 2-8) by the equivalent
force and couple shown in (c). There it is evident that R is the vector sum
of the applied forces shown in (a) while 2M is their moment sum about a
centroidal axis perpendicular to the plane of motion. Applying the
discussion of the preceding paragraph, we conclude that the body in (c)
will therefore simultaneously rotate about its center of gravity with an
angular acceleration a caused by 2M and translate with an acceleration
a of its center of gravity caused by R. In other words, the resulting motion
is a plane motion equivalent to a combination of a pure rotation about
a centroidal axis and a translation of its center of gravity.
We conclude that the rotational component of plane motion is defined
by the equation of centroidal rotation 2M = Ja in which J is the centroidal
mass moment of inertia of the body and a is its angular acceleration. The
translational component of plane motion is expressed by R = ue G@ as de-

rived in Art. 9-8 or by the components of this equation; viz., 2X = We Gs


e WwW. . ees :
ane ae eod,. ‘To summarize this discussion, the kinetic equations of
plane motion are
Art: 13=7| Rolling Bodies 365
eee

ux =a,
g
EY = a, (13-4)
=M = Ia
in which the positive directions of forces, moments, and accelerations are
chosen to agree with the initial direction of motion.

13-7. Rolling Bodies


Let us consider now the application of Eq. (138-4) to rolling bodies, post-
poning the discussion of other cases of plane motion to the next article. The
case of the rolling wheel is simplified by the fact that its geometric center
has a rectilinear translation parallel to the surface on which it rolls. Con-
sequently, for a homogeneous rolling wheel having its gravity center coincid-
ing with its geometric center as in Fig. 13-27, the direction of @ is known.

Then the resultant effective forces consist of a single force ul @ acting as


g
shown through the gravity center G, and
the couple Ja. The best reference axes to
choose are those which pass through the
gravity center of the body with the X
axis directed parallel to the surface on
which the body rolls and positive in the
initial direction of motion. The compo-
nents of d are then given by d, = a@ and
ad, = 0. Applying d’Alembert’s principle
that the resultant of the applied external
forces is equal to that of the effective for-
Fic. 138-27. — Resultant effective
ces, the motion of the rolling body is de- forces.
termined by

=X - 53 (13-5)
rY = (13-6)
SM ea (13-7)
Before discussing the application of these equations, there are two
important special cases to consider. The first is that in which the wheel
rolls freely without slipping. It can do so only if sufficient frictional resist-
ance acts at the instant center C to hold that point instantaneously at rest.
The value of this static frictional resistance is generally unknown since it
may have any value ranging from zero up to the limiting static friction
366 PLANE Motion [Chap. XIII

force f;N. It is therefore convenient to eliminate the unknown static


frictional force by taking moments about the instant center C. Hence
by equating the moment sum about C of the applied external forces (.e.,
~M¢) to the moment of the effective forces, we obtain

~Me = Ia + (Ha) (a)

If free rolling exists, then @ = ra so that Eq. (a) becomes

2M = Ta+ (Hra)e = (1+ Fhe (b)

in which we recognize that the term in parentheses means that the moment
of inertia is transferred from G through the distance r to the instant center
C. Consequently for free-rolling wheels, we obtain
EMe = Ica (13-8)
in which >M¢ is the moment sum of the ap-
plied forces about C and I¢ is the mass mo-
ment of inertia about C. If slipping occurs, the
position of the instant center is unknown and
the relation =Mc = Ica cannot be used, al-
though the other relations developed previous-
ly apply whether or not free rolling exists.
The second special case is that of the un-
balanced wheel or cylinder in Fig. 138-28 whose
Fic. 13-28.— Unbalanced gravity center G does not coincide with the
wheel. geometric center O. Here the direction and
magnitude of the acceleration of the center of
gravity is not known and consequently Eqs. (18-5), (13-6), and (13-8)
do not apply. ‘This case is one of the types discussed in the next article.
Nevertheless, it is an interesting exercise to
show that the moment equation about the 1001b
instant center [Eq. (13-8)] may still be ap-
plied to the two positions in which G crosses
the line containing the geometric center O
and the instant center C.

ILLUSTRATIVE PROBLEMS
1338. The solid cylinder in Fig. 13-29 is 4 ft
in diameter and weighs 644 lb. It is acted upon
by an upward force of 100 lb applied by a cord
wrapped around it. Find the coefficient of stat-
ic friction required to prevent slipping. Fre. 13-29.
Art. 138-7] Rolling Bodies 367
Solution: Since free rolling of the cylinder is implied in the statement of the
problem, the instant center is at the point of contact with the ground, and Kq.
(13-8) may be used to obtain a.

[2Me = Ica} 100><423— (3


2 x 399
SHx2") a = 1.67 rad per sec?

Selecting X and Y axes as shown, the components of @ are given by d; = & and
dy = 0. Then, using the relation @ = ra which applies to free-rolling disks, we
find that Eqs. (13-5) and (13-6) yield

W W 644
E> = — ; aa= jera | F = 39.9 SOX
— 167) F =
= 66.7 lb

my 0 100 + N — 644 =0 N = 544Ib

E = =| fs = sal = 0.1231 Ans.

If the coefficient of static friction were 0.20, what would be the friction force?
The available friction force would be found from the relation F = f,N = 0.2(544) =
108.8 lb. But, as has been seen, not all of this is required to prevent slipping; only
66.7 lb would be acting as before. This example shows that the friction force
that may be acting cannot be found from the relation defining the available friction
force.
If f;= 0.12 and f;, = 0.10, what would happen? What are the values of @ and a?
Under these conditions, the available static friction of 0.12 « 544 = 65.3 lb would
be less than that required to prevent slipping. Therefore slipping would occur and
a kinetic friction force = f,N = 54.4 lb would act. 2Me¢ = Ica cannot be used
because it applies only to free-rolling disks. Therefore we must use Eqs. (13-7) and
(13-5) to obtain

why 1 44
[2M = la] 100 X 2— 544K 2= i:x wa x 2)a a = 2.28 rad per sec?

|2x = iis«| 54.4 = ies xX Ga G = 2.72 ft per sec?


g 32.2

Note that the relation between @ and a for free rolling does not apply when
slipping occurs, 1.e.,
[a ¥ ral 2.72
#2 X 2.28
1339. In the system shown in Fig. 13-30, the floating pulley D is supported by
a cord wound around the drum and a second cord wound around its outer radius.
The second cord, after passing over a frictionless pulley of negligible weight, is
wound around the drum of disk A. Determine the angular accelerations and the
tensions 7’ and P if both bodies roll without slipping.
368 PLANE Motion [Chap. XII]

W=193.2 lb
hk =2.45 ft

Fic. 13-80.

Solution: To determine whether motion occurs and in which direction, tempo-


rarily assume disk A to be at rest. Setting a moment summation about A’s instant
center C equal to zero, we find 27 = 322 (8 sin 30°) from which 7 = 241.5 lb.
Consider now disk D whose instant center is at B. If 2Mz, = 0, there will be equi-
librium; but if 2M, + 0, the disk D will move in the direction of the unbalanced
moment. Thus 2M, = 241.5 X 5 — 193.2 X 2 indicates an unbalanced counter-
clockwise moment acting on disk D. It therefore moves up, climbing along cord P
which is wound up on its drum. Simultaneously, cord 7, moving up, is unwound
from disk D. The absolute motion of 7' is also down the incline, its point of attach-
ment with disk A moving slower than the center of gravity of A, so that JT also
unwinds from A as disk A rolls down the incline. These directions of motion
specify the positive directions of moments and forces.
The kinematic relations between disks A and D are found as described in Prob.
1323 on p. 359. The bodies may be assumed to be instantaneously rotating about
their instant centers B and C so that the motion of the cord which connects them is
given by
6 = fm) sr = 50p = 204

whence successive differentiation yields the following relation between their angular
accelerations:
5ap = 2a4 (a)
The centroidal mass moments of inertia are
= = = 22
[I = Mk] oreARmy ime oe (2)? = 40 ft-lb-sec?
B22
sey KRY
For D: I = —— (2.45)? = 36 ft-lb-sec?
32.2
Art: 13-7] Rolling Bodies 369

whence, using the transfer formula, the values with respect to the instant centers
are
22
ZT =I4+ Me] For A: I¢ = 40+ (3)? = 180 ft-lb-sec?
oe2.2
aa
For D: ip = 36+, 9 2) = 60 ft-lb-sec?

We are now ready to consider the kinetics of each disk. The friction force F
acting on disk A is a static friction holding point C momentarily at rest. Since a
static friction may vary from zero to its maximum available value, F will be un-
known even though f; may be specified. Only when slipping impends will F be
determined by f,V. However, this unknown friction force can be easily eliminated
by taking advantage of the condition of free rolling and applying [Me = Ica
which permits a moment summation about the instant center C through which F
acts. Similarly the unknown tension P is eliminated by a moment summation
about the instant center B of disk D. Taking moments positive in the initial di-
rections of motion, we obtain
[2Mc = Ica] 322 X 3 sin 30° — 2 T = 130 a4 = 130(8 ap) (b)
[2Mez = Iza] 5 T — 198.2 X 2 = 60ap (c)
The tension T is easily eliminated from these equations by multiplying Eq. (c)
by2 and adding them, whence
ap = 0.94 rad per sec? Ans.
QA Zap = 2.35 rad persec? Ans.
after which either value of a may be substituted in (6) or (c) to determine
T = 88.7 lb Ans.
To compute P, we apply Eq. (13-7) to the FBD of disk D.
2M = Ia] 88.7 X 3 — 2 P = 36 X 0.94 P=116lb Ans.
As a check, we may also determine P by applying Eq. (13-5) to the free-body
diagram of disk D. Taking the X axis as positive upward in the initial direction of
motion, we obtain

|2x = ral P.-+-.88.7-—-193:2 = ae5= 2 x 0.94) P =116lb. Check


g

PROBLEMS
1340. A solid cylinder of weight W and radius r rolls without slipping down a
plane inclined at 6° with the horizontal. Determine the minimum coefficient of
static friction to prevent slipping. What is the acceleration of the center of the
cylinder? Ans. fs = +4 tan0;a = $gsin 0
1341. Repeat Prob. 1340 using a homogeneous solid sphere in place of the
cylinder.
370 PLANE MorTion (Chap. XIII

1342. A solid cylinder 2 ft in diameter and weighing 64.4 lb is at rest on a 30°


incline. The coefficient of static friction is 0.25 and of kinetic friction is 0.20. De-
termine the acceleration of the center of gravity, the friction force acting, and the
time for the cylinder to move 20 ft, when released from rest.
1343. Solve Prob. 1342 if the cylinder is on a 45° incline.
Ans. G = 18.2 ft per sec?; F = 9.11 lb; ¢ = 1.4838 sec
P=1001b

W=128.8 1b

P=501b

f=0.30 f-0.20
Fie. P-1344. Fie. P-1345.
1344. The body shown in Fig. P-
1344 consists of two solid cylinders
between which is a weightless drum.
Find the acceleration of the center of
gravity, the friction force acting, and
' the direction of motion. State whether
ae Ib slipping or perfect rolling occurs.
a Sy ¥ Ans. G& = 6.06 ft per sec?; F = 25.76
eet ey Ib;;a2 == 0.19
0.19 rad
rs per sec?;2 slips
sl
1345. The compound disk and drum
shown in Fig. P-1345 is acted upon by a
force P = 100 lb which always remains
horizontal. Assuming free rolling, de-
termine @ and the required friction
force.
Ans. a = 7.08 ft per sec?; F = 84 1b
1346. Determine the friction force
acting on the disk shown in Fig. P-
1346 if it rolls without slipping. The
small fixed pulley is frictionless and
has negligible weight.
ATiSae Ha 2028 lp
1347. If the disk shown in Fig. P-
1347 rolls without slipping, determine Fic. P-1347.
Art. 13-7] Rolling Bodies =)

the acceleration a of its center of gravity. Assume the fixed pulleys to be fric-
tionless and of negligible weight. Ans. @ = 4.51 ft per sec?
1348. A solid cylinder A (Fig. P-1348) that rolls without slipping is connected
to a block B by a cord passing over a frictionless pulley of negligible weight. The
cord is fastened to an axis passing through the center of the cylinder which has a
radius of 1 ft and a weight of 161 lb. Block B weighs 200 lb. Determine the ac-
celeration of B, the tension in the cord, and
the friction force acting on A. P
Ans. dg = 2.38 ft per sec?; 7’ = 98.3 lb;
F = 5.95 lb 26°

30° = ae a.
Fig. P-1348. Fig. P-1349 and P-1350.

1349. The 80-lb plank shown in Fig. P-1349 rests on two cylindrical rollers of
radius r each weighing 30 lb. Determine the force P that will accelerate the plank
up the incline at 6 ft per sec?. Assume no slipping occurs either at the plank or at
the incline. Alas, IP = ixe.7 Ilo
1350. In the preceding problem, if the force P is removed, compute the ac-
celeration of the plank. Ans. a = 11.8 ft per sec?
1351. Determine the acceleration of
the 48.3-lb block shown in Fig. P-1351.
este lb Ans. a = 2.8 ft per sec?
k=12in
W=322 4
k=2ft

Fic. P-1351. Fira. P-1352.

1352. The 64.4-lb weight causes the compound disk shown in Fig. P-1352 to
roll and slip on the horizontal floor. If the coefficient of friction between the disk
and the floor is 0.20, determine the acceleration a of the center of the disk.
Ans. G@ = 3.96 ft per sec?
372 PLANE MorTIoNn [Chap. XIII

{We 644 Ib 1353. The gear and concentric drum


k=2ft shown in Fig. P-1353 rests on a rack B.
The rack, which weighs 128.8 lb, is free
to slide on smooth horizontal guides. A
force P = 200 lb applied to the rack causes
gear A to roll along the rack. Determine
the tension in the cable and how much
cable is wound on or off the drum in 2 sec
starting from rest.
Hie. P-1353.
1354. The disk D shown in Fig. P-1354
rolls freely upon the inclined plane. Compute the tension in the cord connecting
disk D and pulley P.

W=128.8 Ib
k= 2ft

Fig. P-1354.

1355. Referring to Fig. P-13855, determine the minimum coefficient of static


friction between the disk and the incline required to prevent the disk from slipping.
Ans. fs = 0.333
W=200 Ib; k=1ft W=96 Ib
k=2ft

Fic. P-1355. Fic. P-1356.


Art. 13-8] Dynamic Equilibrium in Plane Motion 373

1356. Determine the weight of block B that


will make the compound wheel shown in Fig. P-1356
roll freely up the 30° incline at 4 rad per sec?. Assume
g = 32.0 ft per sec?. Neglect the backward ineli-
nation of B caused by inertia. Ans. W = 128 lb
1357. As shown in Fig. P-1357, a cord passes from
the rim of a solid cylinder of weight W and radius r
over a weightless and frictionless pulley and back
to the center of the cylinder. Determine the tension
h cord t
of the ee :

1358. Assuming that the disk in Fig. P-1358


rolls without slipping, determine the tensions in the
cords and the acceleration of block B.
Ans. dp = 2.84 ft per sec?; Tz = 95.5 1b; Tp =
133.8 lb
1359. In the compound gear and rack assembly
shown schematically in Fig. P-1359, a clockwise
torque of 1180 ft-lb is applied to the gear at the in-

W=322 lb; k=1.414 ft

W=161]b; k=2ft

\30° Fic. P-1358.

stant shown. If the racks slide in friction-


less guides, compute the acceleration a of
the center of the compound gear.
Ans. G& = 4 ft per sec?

13-8. Dynamic Equilibrium in Plane


Motion
From Fig. 13-26 on page 364 and
the accompanying discussion in Art. I
128.8e
Ve
eee) =
13-6, we know that plane motion 1s
caused by a resultant R equal to Fig. P-1359.
374 PLANE MOorTIon [Chap. XIIT

- a acting through the center of gravity and a couple =M equal to Ia.

These effects of the applied forces can therefore be balanced and a state of
: t , WwW. :
dynamic equilibrium created by applying an inertia force me acting
through the center of gravity in an opposite direction to @ and an inertia
couple Ja acting opposite to the sense of a.
The value of ad, and hence of = d, is not usually known but must be >
determined by combining the known acceleration a4 of any reference
point A with the rotational components of the acceleration of the gravity
center about that reference point. Thus temporarily denoting @ by ac, we
obtain
ag = a4 + AGA
or
G = aah fw HD Ta

in which 7 represents the distance between the center of gravity and the
reference point A. Multiplying this equation by S gives

Wi = — a, PW — Fwbee
— &@
W Ww
H — fa
g g g
in which the terms on the right-hand side represent the components of the
resultant effective force = a. Dynamic equilibrium is created by applying
these components through the center of gravity. As shown in Fig. 13-31a,
these components act respectively opposite to the corresponding accelera-

ra

(b)

Fig. 13-31. — Dynamic equilibrium created


by applying the inertia components in (a)
respectively opposite to the corresponding components of acceleration in (b).
Art. 13-8] Dynamic Equilibrium in Plane Motion 375

tion components of the gravity center shown in (b). In addition, the inertia
couple Ja is applied acting opposite to the sense of a.
From another viewpoint, what we have done is to combine dynamic
equilibrium of a non-centroidal rotation about a reference point A with the
inertia force caused by translation of the reference point A. This may be
recognized by referring to Fig. 12-8 on page 328.

ILLUSTRATIVE PROBLEMS

1360. A certain engine has a connecting rod AB which is 3 ft long, is of constant


cross section and weighs 322 lb. The crank AO is 1 ft long and rotates at a constant
speed of 10 rad per sec. The pressure on the cross head at the instant shown in
Fig. 13-32 is 1000 lb. Neglecting friction and the weight of the cross head, de-

W = 322 lb

Fic. 13-382. — Accelerations and applied forces.

termine the guide pressure B, and the horizontal and vertical components of the
crank-pin pressure A, and A, on the connecting rod.
Preliminary: The kinematic properties of the connecting rod must first be de-
termined. This was done in Illus. Prob. 13802 on page 346. The values found
analytically were: w = 2.93 rad per sec counterclockwise; a = 15.5 rad per sec?
clockwise; and ag = 104.4 ft per sec? rightward.
Since AB sin @ = AO sin 30°, we also have

sin @ = sin ALY = ; and cos ¢ = ee = 0.987

Solution: The FBD of the connecting rod shown in Fig. 13-83 is in dynamic
equilibrium created by applying the inertia force — @ at the center of gravity act-
g
ing opposite to @ and applying the inertia couple Ta acting opposite to a. The
value of @ is found in terms of the known acceleration of B using the relation

G = ap Hb Tw? H fra

in which 7w? and 7a are respectively the normal and tangential acceleration com-
ponents of G assumed rotating about B. These accelerations act in the directions
376 PLANE Motion [Chap. XIII

Ay
Wrec = 232 lb Ag

Ic = 116 ft-lb

1.48” >
By W = 322 lb
Fig. 13-33. — Dynamic equilibrium.

shown in Fig. 13-82 and are redrawn for reference at the upper left of Fig. 13-33."
The corresponding components of the inertia force are:
W ao
Ses
‘ aB 39.9 ~*~1044A = 0)1044 Ib

W 322
—ae
fu = 39.9 * esDG 2293)
2.93)?= 129 |b

W322 2
mp ou ea

which are applied through G in the opposite directions to the corresponding


accelerations.
The centroidal mass moment of inertia of the connecting rod is

oe tee Tee ee I = 7.5 ft-lb-sec?


~ 19 or 250 se SEU es
whence the inertia couple is

[ej 55x -15,0 = L1G itp


and is represented by the directed are opposite in sense to a.
Applying the conditions of dynamic equilibrium, we first take moments about A,
thereby eliminating the forces A, and A, while obtaining the value of B,. This gives
[>Ma = 9]
2.96 By, — 1000 X 0.5 — 322 X 1.48 + 1044 x 0.25 + 232 x 1.5 — 116 = 0
from which
Jey = Wee Mo Alo.

1 An identical state of dynamic equilibrium, having the same inertia components, may
also be created by using the concept that the plane motion of the connecting rod is a
combination of the translation caused by the motion of a reference point B and a non-
centroidal rotation about B.
Art. 13-8] Dynamic Equilibrium in Plane Motion 377

Using this value of B, ina Y summation of forces, we have


[ZY =0] A, = 232 cos¢ + 129sin d — 322+164=0 A, = —92lb Ans.
while from an X summation of forces, we obtain
[ZX = 0] 1000 — 1044 — 232sing@+129cos¢ —Az=0 Az=45lb Ans.
Alternate Solution: For the special case in which the center of gravity coincides
with the midpoint of the connecting rod, it is especially convenient to compute the
acceleration of the gravity center by using the result
given in Prob. 1315 on p. 353. There it was indi- 4
cated that the midpoint acceleration of a rod is
equal to one-half the vector sum of its end point ac- 1 A
celerations. Here it is known that the end point pee ORAS
accelerations area4 = 100 ft per sec2?andthatag = 30° ame
104.4 ft per sec? so that, applying the aforementioned
principle in Fig. 13-34, we find the X and Y com-
ponents of @ to be = a, = 50 ft/sec”
Gz = 52.2 + 50 cos 30° = 95.5 ft per sec? right Fia. 13-34.
G, = 50 sin 30° = 25 ft per sec? down
Dynamic equilibrium is now created by applying the inertia force components

Usa, and z a, at G acting respectively opposite to G, and @, and the inertia couple

I a acting opposite toa. Their values are shown in Fig. 13-35. The student should

Ay
Ax

E 0.25!
Ia =116 ft-lb

1.48"
By W = 322 Ib
Fig. 13-35.

verify that application of the conditions of equilibrium to this system of forces


gives as before
B, = 1641b; A, =45lb; A,=—92lb Ans.
1361. A slender uniform rod 6 ft long is sliding with its ends in contact with a
frictionless wall and floor. The rod weighs 161 lb. At the instant shown in Fig.
13-36, the angular velocity of the rod is 2 rad per sec counterclockwise. Determine
the values of a, Nu, and Nz.
378 PLANE MOTION [Chap. XIII

Fic. 13-386. — Acceleration of center of Fig. 138-37. — Dynamic equilibrium.


gravity.

Solution: If dynamic equilibrium is created, the unknown forces N4 and Ng can


be eliminated by a moment summation about their intersection C. However, to
create dynamic equilibrium, we must know the acceleration @ of the center of
gravity G. The value of @ may be found by means of the relative acceleration equa-
tions and methods developed in Art. 13-2. However, a shorter method is available
in this problem based on the fact that G describes a circle about O. Thus as long
as the ends of the uniform rod are in contact with the wall and floor, OG = GB =
3 ft. Hence the center of gravity G moves along a circular path of 3 ft radius having
O as a center. The radius OG always makes the same angle with the vertical as
does the rod, but the angular motion of this radius is measured clockwise from the
wall whereas the angular position of the rod is measured counterclockwise from the
wall. Consequently w and a for the radius OG are numerically equal, but oppositely
directed to w and @ for the rod.
Having established that the center of gravity G moves in a circular path about O,
we see that its acceleration,has the normal and tangential components a, = 7w? =
3w* and a; = fa = 3a directed respectively from G toward O and perpendicular
to GO as shown in Fig. 13-36. The direction of Gis assumed to be down to the right.
This agrees with a clockwise sense of aog for OG and with an opposite counter-
clockwise sense of az for the rod.
Choosing the center of gravity as the reference point, we create dynamic equi-
librium by applying the inertia forces as shown in Fig. 13-37. These inertia forces
: WW acer :
consist of the components raGn, — 4, plus the couple Ja directed opposite to the
senses Of G@,, d:, and a4p respectively. Their values are
Art. 13-8] Dynamic Equilibrium in Plane Motion 379

W _ Wie 161 Fe
9g. an = 9 To? = 399 x38 X (22 = 60 1b

WwW W 161
Ae Ta XM
C= IH eallo
guk iaog me 32.2
= iy 1 161
la = (4 1)0 ap x 30.9 SOE See = Wh e@rlloaity

In applying the conditions of dynamic equilibrium, we first take moments about


C, thereby eliminating the unknown quantities N4 and Ng. This gives
=

[=Mec = 0] 161 X 8 cos 60° — a, x 8 — Ta = 0

161 X 3 cos 60° — 15a X 3 — 1l5a=0


a@ = 4.025 rad per sec? counterclockwise Ans.
The positive value of a indicates that the sense of a4g as shown in Fig. 13-37 was
correctly assumed to be counterclockwise.
Equating a vertical summation of forces to zero eliminates N.4 and gives
ye) Nz — 161 + 60 sin 60° + (15 X 4.025) sin 30° = 0
Ng =788lb Ans.
From a horizontal summation of forces we obtain
xe —10)] Na + 60 cos 60° — (15 X 4.025) cos 380° = 0 Na 22.3 lb Ans.

PROBLEMS
1362. A uniform slender rod is sliding with its ends in contact with a vertical
wall and a horizontal floor. Prove that, at any instant, the angular acceleration a
may be computed from the equation 2Mc¢ = Ica where 2M¢ is the moment sum
of the applied forces about the instant center C and /¢ is the mass moment of inertia
of the rod about C. Warning: This relation is valid only when the center of
gravity is at the midpoint of the rod.
1363. The slender uniform rod AB weighs 96.6 lb and slides in frictionless guides
along the wall and floor, as shown in Fig. P-1368. At the instant shown, the body

W=64.4 lb

Na
Fia. P-1363. Fia. P-1364.
380 PLANE Motion [Chap. XIII

angular velocity is 4 rad per sec counterclockwise. Determine the values of N4


and Nz. Ans. Na = —99.6 lb; Np = —75.4 lb
1364. The ladder AB in Fig. P-1364 is considered as a slender uniform rod
weighing 64.4 lb. The wall and floor are assumed to be frictionless. At the instant
given, the body angular velocity is 2 rad per sec counterclockwise, and the body
angular acceleration is 3 rad per sec? counterclockwise. These values were created
by the force P pushing the ladder up the wall and along the floor. Determine the
values of P, Na, and Nz. Ans. P = 32.44]b; Na = 52.6 lb; Ng = 4.84 lb
1365. A uniform slender bar, 6 ft long and weighing 96.6 lb, is free to move with
its ends A and B in contact with a horizontal floor and an inclined plane as shown in
Fig. P-1365. Determine its angular acceleration an instant after it is released from
rest in the given position.

Fig, P-1365. Fig. P-1366.

1366. The non-homogeneous rod AB shown in Fig. P-1366 weighs 193.2 Ib. Its
center of gravity is at G and its centroidal mass moment of inertia about an axis
perpendicular to the plane of the figure is 15 ft-lb-sec?. At the given position,
w = 2 rad per sec and a = 5 rad per sec? both clockwise. Determine the normal
pressure acting at B on the smooth incline and the frictional and normal com-
ponents of the reaction at A on the rough floor.
2. 1367. The connecting rod AB of an engine
is a slender uniform rod 36 in. long weighing
Ws 100 1b. The resultant F of all forces acting on
oo AB is 400 lb directed and located as shown in
S82: 30/ Si o Fig. P-13867. Determine the linear accelera-
‘ : 7\ tion of the center of gravity of the connecting
Fic. P-1367. rod and its angular acceleration. (Hint: Re-
fer to Fig. 138-26 on page 364.)
Ans. @& = 128.8 ft per sec?; a = 57.2 rad per sec?
1368. The connecting rod of the engine in Fig. P-1368 is assumed to be a slender
uniform rod 4 ft long weighing 322 lb. The crank AO is 1 ft long and rotates with a
constant angular velocity of 10 rad per sec. The pressure on the cross head at the
instant given is 2000 Ib. Neglecting friction, determine the guide pressure B on
the cross head and the horizontal and vertical components of the crank pin pressure
Art. 13-8] Dynamic Equilibrium in Plane Motion 381
at A. (From Prob. 1318 on p. 354, w4g = 1.8 rad per sec, a4g = 17.4 rad per sec?
clockwise, and ag = 71.1 ft per sec?.)
Ans. B = 340 lb; Az = —1290 lb; A, = —371 1b
1369. Repeat Prob. 1368 if the 4-ft connecting rod varies in cross section so
that its center of gravity G is located 1.5 ft from A and its mass radius of gyration
is 1.0 ft about an axis through G perpendicular to the plane of motion. Assume all
other data to remain unchanged.

Fic. P-1368, P-1369, and P-1370. Fic. P-1371.

1370. When the piston of the engine described in Prob. 1368 is at its extreme
left position, the cross head pressure is zero. What is then the guide pressure B and
the horizontal and vertical components of the crank pin pressure at A?
1371. The circular disk shown in Fig. P-1371 weighs 161 lb after the semicircular
hole is cut out of it. Its radius of gyration of mass is 1.2 ft with respect to a hori-
zontal axis through the gravity center at G.. Determine the angular acceleration
at the given position if the disk is released from rest and does not slip.
Ans. a = 2.83 rad per sec?
1372. The unbalanced wheel shown in Fig.
P-1372 weighs 322 lb and has a mass radius of
gyration of 0.866 ft with respect to a horizontal
axis through its gravity center G. At the given
instant, w = 3 rad per sec and a = 6 rad per
sec?, both clockwise. If the wheel does not
slip, determine the value of P which is directed
parallel to the incline. Ans. P = 251 \b

1373. If the unbalanced wheel described in


Prob. 1372 starts from rest when at the posi-
tion shown in Fig. P-1372 and does not slip,
determine the initial angular acceleration caused
by a pull P = 240 lb.
1374. The wheel shown in Fig. P-1374 rolls
without slipping under the action of the pull
P. At the given position, w = 3 rad per sec and
a = 4rad per sec*, both clockwise. The wheel Fig. P-1372 and P-1373.
382 PLANE MorTIon [Chap. XIII

carries a uniform slender rod weighing 96.6 Ib


which is pinned to the wheel at B and rests
between two smooth pins at A. At the given
position, compute the horizontal and vertical
components of the pin pressure at B and the
force exerted by the active pin at A.
Ans. A = 14.02 lb; B, = 49.0 lb; B, =
95.4 Ib
1375. Set up dynamic equilibrium for the
combined system of wheel and rod described
in Prob. 1374 and, without solving for the
pressures at A and B, determine the value of
Ftq. P-1374 and P-1375. the pull P at the given position. The wheel
weighs 128.8 lb. Ans. “P= 58:9lb

SUMMARY

Plane motion (Art. 13-1) is the motion of a rigid body in which all par-
ticles move in parallel planes. The fundamental concept of plane motion
is that it may be considered equivalent to a combination of rotation about
any reference point plus vectorially the translation of that reference point.
From this concept, we obtain the following kinematic equations which are
true at any particular instant:

Sp = SavH (SBA = 70) (13-1)


Ve = Vad (VBA = Tw) (13-2)
Qp = a4 (Gaza = To? H TQ) (13-3)

In these equations A is the reference point and B is any other point in the
body. Note that values of 6, w, and a for plane motion are independent
of the choice of the reference point. The application of these equations is
indicated in Art. 13-2.
For free-rolling disks or spheres (Art. 13-3), relations between the trans-
lation of the geometric center of the body and rotation of the body are
given by
s4 = ré

vA = TW

aa = 7?ra

in which A represents the geometric center of the body and r is the distance
from A to the surface on which the body rolls.
The instantaneous axis of rotation in plane motion is the line joining
the points having zero velocity. The instant center (denoted by C) is
located at the intersection of this line with the plane of motion. The
Summary 383

velocity of any point may be found as though the body were actually
rotating about the instant center.
The instant center for free-rolling disks or spheres lies at the point of
contact with the surface on which the body rolls. The location of the
instant center in other cases was discussed in Art. 13-4.
When the center of gravity is selected as the reference point (Art. 13-6),
the kinetic equations of plane motion become

she dee
g

sey
s, g
as
13-4

=M
= Ia
These will be recognized as a combination of the equations for translation
and centroidal rotation. The positive directions of forces, moments, and
accelerations are chosen to agree with the initial direction of motion.
In the case of free-rolling disks or spheres (Art. 13-7), the reference axes
are also chosen through the center of gravity. The X axis is directed
parallel to the surface on which the body rolls and is considered positive
in the initial direction of motion. This results in the following equations:

x= a (13-5)
ry = (13-6)
=M = Ia (13-7)
mMce = Ica (13-8)
Equation (13-8) applies only when free rolling exists; the others can be
used whether or not free rolling is present.
Other problems in plane motion can be handled conveniently by creat-
ing dynamic equilibrium so that the equations and methods of static
equilibrium may be used. Dynamic equilibrium (Art. 13-8) is created by
applying inertia forces acting through the center of gravity, plus an inertia
couple Ja. The inertia couple is directed opposite to the sense of a. The
: | eal Dies: :
inertia forces consist of the components of a acting respectively oppo-
site to the acceleration components which determine @ in terms of the ac-
celeration a4 of any reference point A and the rotational components of the
acceleration of the center of gravity about A.
Chapter XIV.
Work and Energy

14-1. Introduction

One type of problem that frequently occurs in dynamics involves a rela-


tion between force, displacement, and velocity. A technique known as the
work-energy method is particularly adapted for solving such problems. As
we shall see, this method eliminates consideration of acceleration and leads
directly to the desired solution. In most applications it also eliminates
consideration of forces which are internal to a system of bodies. Generally
speaking, the work-energy method will usually be faster and easier than
the force-inertia method developed in the preceding chapters. However,
the force-inertia method is indispensable for determining the instantaneous
acceleration caused by variable forces or moments.
The terms work and kinetic energy are used to define certain mathe-
matical expressions. A precedent for this was found when we defined the
moment of inertia of an area as equivalent to the mathematical expression
JS pe dA. In the following articles we shall do two things: (1) derive the
mathematical expressions which define work and kinetic energy as applied
to translation, rotation, and plane motion; (2) discuss the technique, use,
and advantages of the work-energy method as applied to these several
motions.

14-2. Fundamental Work-Energy Equation for Rectilinear Translation


The mathematical expressions defining work and kinetic energy as ap-
plied to translation are easily obtained by considering the following equa-
tions which have been already derived:
jn & |
2X =—a
Gao (a)
ads = vd!)

Note that the first member of Eq. (a) equates the resultant force acting at
any instant to the corresponding acceleration. Obviously the value of a
depends on the corresponding value of {X; it may be constant or variable
depending on whether =X is constant or variable. The second member
of Eq. (a) expresses the instantaneous value of the acceleration in terms of
the instantaneous velocity.
384
Art. 14-2] Work-Energy Equation for Rectilinear Translation 385

Eliminating a in the two equations results in


a

2X -ds = 7 v dv (b)

which is defined as the differential form of the work-energy relationship.


Assuming that the initial velocity is v. at zero displacement and that the
final velocity is v at a final displacement s, we may integrate Eq. (b) to
obtain

fox ds = ye v dv (c)
0 g %,

which gives us

? ee ne Wee
Jzx dsy= oe; v ag v,2 = a9 (v? — v,?) (14-1)

This is the fundamental work-energy equation.


s

The expression { >X - ds is defined as resultant work, whereas the ex-


0

ae i e ere
pression 5 rs v? is defined as kinetic energy. In other words, Eq. (14-1)
may now be interpreted as meaning that the resultant work on a translating
body ts equal to the corresponding change in kinetic energy.
Work-Energy Equation for Constant Forces. If the forces acting on a
body are constant, the resultant force 2X will be constant, whence Eq.
(14-1) becomes

xrX-s==-—v' v2 = =~ (v? — v,*) (14-2)

The expression LX - s is called the resultant work done on a body as it


moves through a linear displacement s.
Units. Substituting the dimensional units of the terms in LX - s, we
find the unit of work to be force multiplied by distance. Expressing force
in pounds and distance in feet, the unit of work is ft-lb. Other dimensional
units can be used, and work can also be expressed as lb-in., dyne-cm, ton-ft,
etc. Usually it is more convenient to express force in pounds, distance in
saith ey es ;
feet, and time in seconds. Substituting these units in Wl vy’ gives the unit
of kinetic energy.
1G = 49

Sie K BEC XK ea = ft-lb


g t sec

Thus the work-energy equation is seen to be dimensionally correct; this is a


check of the validity of any equation.
386 Work AND ENERGY [Chap. XIV
nL

14-3. Meaning of Work


Consider a body subjected to the constant forces shown in Fig. 14-1
which move the body up the incline. Selecting the X axis as positive in the
direction of motion, the resultant of
x the unbalanced force system is
=X = Pcosé—Wsind
—F (a)
whence, multiplying both sides of Eq.
(a) by s, we find the resultant work
after moving a displacement s is ex-
pressed by
DX -s = (P cos 6)s — (W sin #)s —
F-s (b)
Fig. 14-1.
The first expression in the right-hand
terms of Eq. (0) is called accelerating work or positive work. The other
expressions are known as retarding work or negative work. The alge-
braic sum of positive and negative work is called resultant work (ab-
breviated as RW).
A clearer concept of the meaning of work may be obtained from visualiz-
ing the physical meaning of the first expression, (P cos #)s. According to
this, as shown in Fig. 14-2a, work may be defined as the product of the

Work=(P cos @)s Work = P(s cos 6)


(a) (b)
Fig. 14-2. — Two interpretations of work done by a force.

component of force in the direction of displacement multiplied by the dis-


placement. Note that in accordance with this definition, forces perpen-
dicular to the displacement (such as N in Fig. 14-1) do no work because they
have no component in the direction of displacement. If the first expression
is rewritten as P(s cos @), we see from Fig. 14-2b that work may also be
interpreted as the product of the force multiplied by the component of
displacement in the direction of the force. Similarly, the work done by W
Art. 14-4] Application of the Work-Energy Method. Constant Forces 387
may be rewritten as —W(s sin 6) = —Wh since s sin 0 = h, the vertical
rise of the block. Thus the work done by gravity forces is the product of
the weight multiplied by the change in vertical displacement, being negative
(as in this case) when the direction of W is opposite to the vertical displace-
ment, and positive when they are both downward.

14-4. Application of the Work-Energy Method. Constant Forces


If a body is subjected to different sets of forces during different phases of
its motion, the resultant work summed up for all these phases may be
equated directly to the total change in kinetic energy. It is not necessary to
compute the velocity at the end of any phase so that it may be used as the
initial velocity for the next phase. For example, assume that the car
of weight W lb in Fig. 14-8, starting with a velocity v., reaches the bottom

Fic. 14-3. i

of an incline s; ft long with a velocity 1, and then rolls a distance s: ft


along the level to a position at which its velocity is v2. Writing the work-
energy relation for the displacements s; and s2, we have

Eyed Feats: Vie Pile aI


29 29 (a)
eae ways
eae g V2 29 1

Adding these two equations gives


1W i,
DX481 “bE DX 282 = fax hy = Nae D) g OF (b)

The left side of this equation represents the sum of the resultant work
during the displacements s; and s.. If the displacements are considered to
ESpees small, such as ds, and dss, this side becomes DX, - ds: +
- ds, + - ++, which will be recognized as equivalent to {2X - ds pre-
ees dened as resultant work. It is therefore not surprising to find that
the right side of Eq. (b) expresses the change in kinetic energy in terms of
388 Work AND ENERGY [Chap. XIV

the initial and final velocities v, and vv. In spite of the fact that two
different accelerations are involved during the displacements s, and s2, we
may therefore sum up the work done during the several phases of a motion
and equate this resultant work to the total change in kinetic energy.
An outstanding advantage of the work-energy method is that it may be
applied directly to a system of bodies without the need for considering any
of the internal forces, such as tensions in connecting cords. This is par-
ticularly convenient in cases where the internal forces vary (as in Illus.
Prob. 1403 below). To show why the internal forces are automatically
eliminated, consider a system of two bodies connected by a link of negligible
weight. From Newton’s third law, the reactions of the link on the two
bodies are equal and oppositely directed. If the length of the link does not
change, each end has the same component of displacement along the link,
and hence the positive work done by the connecting link on one of the bodies
is equal to the negative work done on the other body. Consequently the
total change in kinetic energy of a system of bodies is caused only by the
resultant work of the external forces, provided the length of the connecting
members does not change and they are of negligible weight.
A general plan for applying the work-energy method consists of the fol-
lowing steps:
1. Determine the direction of motion. Confirmation is obtained by
noting that the resultant work must be positive to speed up a system, and
vice versa.
2. Determine the kinematic relations between the bodies composing the
system.
3. Apply the work-energy equation to the entire system.
4. If the internal force in a connecting member is desired, apply the
work-energy equation to a free-body diagram of that part of the system
on which this force then acts as an external force. If the internal force is
not constant, however, this step will determine only its average value. The
instantaneous value of a variable force must be found by the force-inertia
method.

ILLUSTRATIVE PROBLEMS
1401. The 300-lb block in Fig. 14-4a rests upon a level plane for which the
coefficient of kinetic friction is 0.20. Find the velocity of the block after it moves
80 ft, starting from rest. If the 100-lb force is then removed, how much farther
will it travel?

Solution: The FBD of the block in its first phase of motion is shown in Fig.
14-4b. Computing the normal and frictional forces in the usual manner, we apply
the work-energy equation to phase 1.
Art. 14-4] Application of the Work-Energy Method. Constant Forces 389

eee

F,= 601b

N,= 250 1b N,=8001b


(a) (b) Phase 1 (c) Phase 2
7 Fig. 14-4.

“ Ww . 300
|2x: -3= 29 (v? — 12) | (100 cos 80° — 50)(80) = 644 v;

from which
v; = 25.1 ft persec Ans.
The FBD of the block during the second phase of the motion is shown in Fig.
14-4c. To determine how much farther the block will travel after the 100-lb force
is removed, we equate the resultant work done during both phases of the motion to
the total change in kinetic energy. This change in kinetic energy will be zero since
the final and initial velocities are zero. We obtain

|2x + TX. = us(v.? — 02) | (100 cos 30° — 50)(80) — 60 ss = 0

from which
S. = 48.7 ft Ans.
1402. As shown in Fig. 14-5, the 300-lb counterweight B pulls the 200-lb block
A up the 30° incline; the coefficient of kinetic friction is 0.20. If the pulleys are
considered frictionless and weightless, determine the velocity of block A after it has
moved 20 ft, starting from rest.

Wa=200Ib x
oe

N =1731b

Wz = 300 lb
Fia. 14-5.
390 WorkK AND ENERGY [Chap. XIV

Solution: Only the external forces acting on the system are shown. As noted on
page 388, the internal forces (tensions in the cords) do no work on the system. The
kinematics of the problem indicate that the displacement and velocity of B are one-
half the corresponding values of A.
If we apply the work-energy equation to the system and note that the resultant
work RW = (2X -s)p + (2X°s)4 where the X summations are positive in the
indicated directions of motion, we obtain

|Riv= 2) = (v2 — 0 | 300 X 10 + (—34.6 — 200 sin 30°)(20) =


200 300 1
644°4 + 644 (n, = Jo’)
2

3000 — 2692 = Gi (200 + 75)

from which
; 2 _ 308 X 64.4
= 72.1 and v4 = 8.5ftpersec Ans.
a 275
If the tension in the cord pulling A should be desired, it can now be easily found
by applying a work-energy equation to the FBD of A. This gives

[2x-2=
DX -s =—Foo
Ww
(wv —- 02)
0, | (a
TA —1346)1340)20)
(20) ==200BY 72:
Ta = 145.8lb Ans.
One final point is worthy of note. Since the forces and accelerations are constant,
we may apply Eq. (10-3) to compute a4. Doing this gives
[v2 = 0,’ + 2 as] 721 = 2 a4(20) aa = 1.80 ft per sec?
Now compare these results with those obtained by the force-inertia solution of
this same problem on page 272.
1403. Body A is pulled along the frictionless level surface in Fig. 14-6 by a
flexible, inextensible cord passing over smooth fixed pegs at C and D to the weight B.
If A starts from rest at the given position, determine its velocity when it is in the
dashed position.

Fic. 14-6.
Art. 14-4] Application of the Work-Energy Method. Constant Forces 391

Solution: The only active force doing work on the system is Wz. The vertical
movement of B is equal to the difference in length of AC as it moves between the
two specified positions. Designating this variable length by z, and noting that the
velocity of B is equal to the rate of change of this length (ie,vB = aA we have

2=a?+ 42 whence 2 = 1/8 + 4 = 8.95 ft


and 2f=+/2?+ # = 5.00 ft

To relate v4 to vg, we differentiate the preceding expression with respect to the


time. This gives

whence for the dashed position


Dvpi— 3 VA

Applying the work-energy equation, we obtain

WwW 2 2 = 200 2 100 ( = 9 2 )


|einmoog” =») | 100(8.95 — 5) 64.44 Te Vp 95 A

from which
v,’ = 108 and v4 = 10.4ftpersec Ans.

PROBLEMS
1404. A constant force P = 150 lb acts on the body shown in Fig. P-1404 during
only the first 20 ft of its motion starting from rest. If f, = 0.20, find the velocity
of the body after it has moved a total distance of 30 ft.
P=1201b

Fic. P-1404. Fi¢. P-1405.

1405. After the block in Fig. P-1405 has moved 10 ft from rest, the constant
force P is removed. Find the velocity of the block when it returns to its initial
position. Ans. v = 21.2 it per sec
1406. What force P will give the system of bodies shown in Fig. P-1406 a
velocity of 30 ft per sec after moving 20 ft from rest? Ans. P = 445 |b
392 Work AND ENERGY [Chap. XIV

Fic. P-1406.

1407. Find the velocity of body A in Fig. P-1407 after it has moved 10 ft from
rest. Assume the pulleys to be weightless and frictionless.
Ans. v4 = 7.65 ft per sec

Fig. P-1407. Fig. P-1408.

1408. Through what distance will body A in Fig. P-1408 move in changing its
velocity from 6 ft per see to 12 ft per sec?
1409. Determine the velocity at-
tained by block A in Fig. P-1409 after
moving a distance of 12 ft starting
from rest.
1410. In what distance will block
A of Fig. P-1409 attain a velocity of
12 ft per sec, starting from rest?
Ans. sa = 34.4 ft
1411. Body A starts from rest in
the position shown in Fig. P-1411. De-
termine its velocity after it has moved
Fic. P-1409 and P-1410. 15 ft along the frictionless surface.
Art. 14-5] Resultant Work. Variable Forces 393
r e ee ee BEY
1412. Find the velocity of body A in Fig. P-1411 after it has moved, starting
from rest at the given position, for 9 ft along the frictionless surface shown.
Ans. v4 = 15.57 ft per sec

W, = 3001b

Fig. P-1411 and P-1412. Fie. P-1413 and P-1414.

1413. Two sliders, connected by a light, rigid link 10 ft long, move in the
frictionless guides shown in Fig. P-1413. Determine the velocity of B when x = 6
ft if B starts from rest when vertically below A. Assume W4 = Weg = 100 lb and
We = 50 |b. Ans. vp = 12.5 ft per sec
1414. Repeat Prob. 1413 when x = 8 ft.

14-5. Resultant Work. Variable Forces


The general form of the work-energy equation has already been shown
to be

itEX-ds = x7(v — v,) (14-1)


and it has been applied to cases in which 2X remained constant. In many
cases, however, =X varies, and it is with them that we are concerned here.
An examination of the first term in Eq. (14-1) shows that if 2X is variable
it must be expressed in terms of s before 5x
the resultant work can be found mathe-
matically. In many cases a mathematical
relation between =X ands cannot be found, U;
although experimental methods may be
used to express the relation graphically.
Figure 14-7 shows such a force-displace-
ment diagram. It is evident that for a dif- Uj
ferential displacement ds, the resultant LA
Sil
” wh

force xX may be considered constant. 4 44-7. — Force- rapidaerient


Since the area of the small shaded rec- diagram.
394 WorkK AND ENERGY [Chap. XIV

tangle represents the term 2X - ds, it follows that the resultant work

{"EX - ds is represented by the area under the force-displacement curve.


0

This principle gives us a simple method of computing the work done by


forces which are directly proportional to displacement, as in springs. The

O s $1 52 Displacement
Fic. 14-8. — Force-displacement diagram for a spring.

force-displacement diagram for a spring (Fig. 14-8) is a straight line


determined from Hooke’s law and expressed by the equation
Paths (a)
in which k is known as the spring modulus. This modulus or spring con-
stant represents the force required to deform a given spring through a unit
distance.
Noting that the resultant work, {"Ike ds, is represented by the area
0

under the force-displacement diagram, we see from Fig. 14-8 that the
work done in deforming a spring from its free or unloaded length to an
extension (or compression) of s units is the area of the triangle OAB, or

[zx -ds = 4 (ks)(s).= 2 ks? (b)

The work done may also be found by evaluating the integral f 2X - ds, in
7)

which =X is equal to ks. This gives

fax-as = [esas = ike?


0 0

as before.
The force-displacement diagram is especially useful in determining the
work required to stretch a spring from an initial deformation s, to a larger
deformation s». In this case, the resultant work is the area of the trapezoid
Art. 14-5] Resultant Work. Variable Forces 395
nee ee ee ee
CDEF, which is equivalent to the average force multiplied by the change
in deformation, or
, ks + ks
ee hore (Se — 81) (c)

Since the force exerted by a spring depends only on the spring constant
and the magnitude of spring deformation, the work done is due only to the
change in length of the spring and is independent of any rotation of the
spring. For example, consider the spring shown in Fig. 14-9a which has a

Free length=2 in

4 k=10lb/in Force (lb)

re, EN 30 Ib
nc UU
ihMi

an
mn 101b
fale 5. . s(ft)_
$
(ft)

Bees eves oe | -ar=2 p+


(a) (b)
Fic. 14-9. — Method of computing work done on a spring.

free length of 2 in. and a modulus of 10 1b per in. As one end is moved from
the dashed position A to the solid position B, the initial force on the spring
is P = ks = 10(3 — 2) = 10]band the final force is P = 10(5 — 2) = 30)b.
Meanwhile, the spring undergoes a change in length AL = 5 — 3 = 2 in.
Since the work-energy equation requires that work be in ft-lb units, we
avoid confusion in units by expressing the change of length shown in Fig.
14-9b as AL = 2 ft. Now the area of the shaded trapezoid gives the work
done on the spring as
ee se ase
5 a = 3.33 ft-lb | (d)

The illustrative problems which follow show the application of a force-


displacement diagram to work-energy problems involving variable forces.

ILLUSTRATIVE PROBLEMS
1415. An elevator weighing 4000 lb is being lowered at 10 ft per sec when the
hoisting drum is suddenly stopped. If the elastic properties of the supporting cable
are such that it is equivalent to a spring with a modulus of 2000 Ib per in., determine
the maximum tensile force produced in the cable.
Solution: The maximum tension will be the sum of the initial tension of 4000 Ib
plus a spring force ks in which s is the stretch of the cable. This stretch is found
396 Work AND ENERGY [Chap. XIV
SeES eS eee

by applying the work-energy equation to the elevator while its velocity changes
from 10 ft per sec to zero. Expressing the stretch in inches, the average spring force
is (2000 s) lb and we obtain

_W F ah (3) - 20007 uaa


[zw = wn | (4000 —3X 2000 s)(55) = gggO — 10)
s ll 10.87 in.

The maximum cable tension is


[T= W ksi T = 4000 + 2000(10.87) = 25,740 Ib
Thus the sudden stop of the hoisting drum momentarily increases the cable tension
by 6.43 times. Now read Prob. 1075 on page 284.
1416. At the instant shown in Fig. 14-10a, an external force has pushed the
500-lb block against the spring, thereby compressing the spring 6 in. If the spring
constant is 100 lb per in., how far will the block be projected along the level plane
(for which f; is 0.20) when the external force is released? What will be its maximum
velocity?

fo iB
er 6=

600 lb

a -100 Ib
Fic. 14-10.

Solution: The initial kinetic energy of the block is zero and it will again be zero
when the block has come to rest after traveling its maximum distance along the
plane. The resultant work done may be obtained from a force-displacement dia-
gram in which the effects of the variable spring force and of the constant frictional
resistance have been plotted separately in Fig. 14-10b. At the start the spring
force is 600 Ib; this diminishes to zero in 6 in. We now obtain
[RW = AKE] 2(600) (8) — 100s = 0 s=1.5ft Ans.
The block will have a maximum velocity at the instant the net force on it is zero.
As the net force-displacement diagram of Fig. 14-10c shows, maximum positive
work is done on the block up to this position; thereafter negative work slows it
Art. 14-5] Resultant Work. Variable Forces 397

down. Since the spring force decreases at 100 lb per in., the critical position occurs
after a movement of 5in. Applying the work-energy equation, we find

PUR Ww
a =51 (500) ( 2) 500 a
[ 29 (vyey?v, | aoe)
12 Career
Ae v 5= 3.66 ft per sec Ans.

PROBLEMS

1417. A weight is dropped from a position just above, but not touching, a spring.
Show that the maximum deformation produced will be twice that if the same weight
is gradually lowered upon the spring.
1418. A block weighing 96.6 lb is dropped from a height of 4 ft upon a spring
whose modulus is 100 lb per in. What velocity will the block have at the instant
the spring is deformed 4 in.? Ans. v = 15.3 ft per sec
1419. A 600-lb block slides down an incline having a slope of 3 vertical to 4
horizontal. Itstarts from rest and, after moving 4 ft, strikes a spring whose modulus
is 100 lb per ft. If the coefficient of kinetic friction is 0.20, find the maximum
velocity of the block. Ans. v = 12.3 ft per sec
1420. Through what distance must the 600-lb block of Prob.
1419 slide from rest before touching the spring if its velocity is 10
ft per sec at the instant the spring is deformed 3 ft? Assume the
spring constant is changed to 30 lb per in.
1421. A weight of W lb is suspended from a vertical spring
(Fig. P-1421) whose modulus is k lb per ft. The weight is pulled
down s ft from its equilibrium position and then released. Deter-
mine its velocity when it returns to the equilibrium position.

Ans. v=svy/= Fria. P-1421.

1422. The rigid horizontal bar shown in Fig. P-1422 is supported by two springs.
A 300-lb weight is then placed upon the bar at B. A sudden blow projects the weight
toward A with an initial velocity of 6 ft per sec. What is its velocity when it
reaches A? Neglect friction and the weight of the bar.
Ans. va = 4.46 ft per sec

Fig. P-1422. Fia. P-1423 and P-1424.


398 Work AND ENERGY [Chap. XIV

1423. Neglect friction of the 40-lb collar against its vertical guide and compute
the velocity of the collar after it has fallen 7 ft, starting from rest in the position
shown in Fig. P-1423. The unstretched length of the spring is 3 ft.
Ans. v = 18.2 ft per sec
1424. Repeat Prob. 1423 if the unstretched length of the spring is 2 ft.
Ans. v = 16.1 ft per sec
1425. The car in Fig. P-1425 is moving toward the bumper spring and has a
kinetic energy of 100,000 in.-lb. The main bumper shield (aa) is connected to the
main spring, which has a modulus of 1000 lb per in. The two auxiliary bumper

Fic. P-1425.

shields (b) are 12 in. behind aa and are attached to secondary springs, each of which
has a modulus of 500 lb per in. When the car is brought to rest, what will have
been the greatest movement of aa? What percentage of the energy has been ab-
sorbed by the main spring? Ans. s = 14in.

14-6. Power. Efficiency


Power is the time rate at which work is done on a body. For example, if
a train is being pulled by a locomotive, the work done on the train may
be measured by the work done by the drawbar pull. Expressed mathe-
matically,
Work
Power =
Time

This gives the average power. If F is the net force doing the work, then
work during any instant is given by
Dx <ds = fF -ds
and the power exerted at any instant is

Work Fds
Power = Tinie edie Fv (14-3)

ie., the power exerted at any instant is the product of the net force mul-
tiplied by the instantaneous velocity.
The unit of power depends on the units of work and time. Common
units are ft-lb per sec and kg-m per sec in the gravitational system, dyne-
cm per sec (erg) or the joule per sec in the absolute system. These units
are usually too small for use in engineering. The units commonly used
Art. 14-6] Power. Efficiency 399

here are the horsepower (hp) and the watt and kilowatt (kw). The horse-
power is a traditional unit equivalent to 550 ft-lb of work per sec, or
33,000 ft-lb of work per min. The watt equals 107 ergs per sec, and the
kilowatt equals 1000 watts.
The watt and kilowatt are used in electrical engineering. Relations
between horsepower and kilowatts are as follows:

1 hp = 0.746 kw
1 kw = 1.34 hp
For large quantities of work, the units are the horsepower-hour (hp-hr)
and the kilowatt-hour (kw-hr). These units indicate the amount of work
done in one hour at the constant rate of 1 hp or 1 kw.
Because of losses resulting from friction and other causes, the power
delivered from a machine or other device is never equal to the power put
into it. Efficiency is the ratio of power output to power input. This ratio
is usually multiplied by 100 so that efficiency may be given as a percent-
age. Since power output and power input are measured during the same
time, efficiency may also be defined as the ratio of energy output to energy
input or of work output to work input. The overall efficiency of a ‘number
of machines placed in series is equal to the product of their individual
efficiencies.

ILLUSTRATIVE PROBLEM
1426. A 1000-ton train is accelerated at a constant rate up a 2% grade. The
train resistance is constant at 10 lb per ton. The velocity increases from 20 mph
to 40 mph in a distance of 2000 ft. Determine the maximum horsepower developed
by the locomotive.
Preliminary: The inclination of a grade is expressed as the rise in feet measured
in a horizontal distance of 100 ft. Thus a 2% grade means a rise of 2 ft in a hori-
zontal distance of 100 ft. For grades up to 10%, the length of the incline is prac-
tically equal to its horizontal projection so that the cosine of the angle of the grade
is practically equal to unity. Therefore the
normal pressure may be considered equal to
the weight of the train; hence the train
resistance remains constant. Furthermore, for
small angles the sine may be considered equal
to the tangent, so that in this case of a 2%
grade, the sine will be equal to too.
Solution: The constant drawbar pull ex-
erted by the locomotive is found by applying
the work-energy equation. From the point
2,000,000
lb
diagram of the forces acting on the train
(Fig. 14-11) we have Fig. 14-11.
400 WorkK AND HINERGY [Chap. XIV
Tenn ee EE

|2x 5 = 1. (v2 — v2) |


29
2 2,000,000
(p— 10,000 — 2,000,000 x | LON ae 64.4
O8:67.— 29.37
100 \
P = 90,000 Ib
The maximum power is developed when the train is moving at maximum speed.
Therefore :

e Fv 90,000 X 58.6
ULES hp = 9600 Ans.
E 550 me 550 y rn
PROBLEMS
1427. A train weighs 1600 tons. The train resistance is constant at 12 lb per ton.
If 6000 hp are available to pull this train up a 2% grade, what will be its speed in
miles per hour? Ans. v = 27mph
1428. A train weighing 1200 tons is pulled up a 2% grade at 10 mph. If the
train resistance is constant at 12 lb per ton and the power exerted by the locomotive
remains constant, what will be the speed of the train on a level track? What horse-
power does the locomotive develop?
1429. A train weighing 100 tons is being pulled up a 2% grade. The train resist-
ance is constant at 10 lb per ton. The speed of the train is increased from 20 ft per
sec to 40 ft per sec in a distance of 1000 ft. Find the maximum horsepower de-
veloped by the locomotive. Ans. 635 hp
1430. Water flows through a nozzle 1 in. in diameter under a head of 400 ft to
drive a turbine. The turbine is 90% efficient and is connected to a generator which
is 94% efficient. Whatis the power output in kilowatts? Ans. 25.1 kw
1431. Water enters a hydraulic reaction turbine with a velocity of 12 ft per sec
and leaves it 3 ft lower with a velocity of 4 ft per sec. If 100,000 lb of water flow
through the turbine each second, compute the horsepower output. Assume the
turbine is 80% efficient. Ans. 724hp

14-7. Work-Energy Applied to Curvilinear Translation


Many bodies are relatively so small compared with their paths of
travel that they may be treated as particles. This assumption applies to
the curvilinear motion of all bodies discussed in this article. The derivation
of the work-energy equation for rectilinear translation is also valid for
curvilinear translation provided we assume that the X axis is tangent to the
path during each distance ds measured along the path. The resultant work
is then also f‘=X - ds and the kinetic energy is x (v? — v2) where v and
oO

v. are the final and initial velocities directed tangent to the path. The
work done by gravity forces is best computed as the weight multiplied by
Art. 14-7] =Work-Energy Applied to Curvilinear Translation 401
the change in elevation (see Art. 14-8) whereas the work done by a spring
may be found as in Art. 14-5.

ILLUSTRATIVE PROBLEMS
1432. A 100-lb weight is swung in a vertical circle at the end of a 4-ft cord. The
topmost velocity of the weight is 12 ft per sec. Find the tension in the cord at 120°
past top position. What minimum velocity at the bottom will keep the weight in
the circular path at the top?
Solution: The tension is found by U,=12 ft/sec
creating dynamic equilibrium at B as
shown in Fig. 14-12. The velocity at
B is found by applying the work-energy
equation. Note that the tension T is
everywhere normal to the path and
hence does no work. We obtain

[1a = Fes! = v4) |


: a 100
100(4 + 4 sin 30°) = oa WB — 12?)

5 = 530 W= 100 lb c
It may be observed that if W is cancelled Fia. 14-12.
out of this equation, the terms may be
rearranged to give vg? = va? + 2 gh which is equivalent to the equation for free-
falling bodies (see Art. 10-8). This result will be obtained for any frictionless
curvilinear translation in which only the weight does work. However, the time
elapsed will, in general, differ from that required for free fall, and moreover, the
velocities are always tangent to the path.
Taking summation of forces along the normal to the path at B gives
W ove
[ZN = 0] 1 5 — 100 cos 60° = 0

100 _ 530
7; = 30.9 x aes
4 + 100 x 0. 0.5 dy = 461
61 1lb Ans :

To find the minimum velocity at the bottom C to just keep the weight in the
circular path at the top A, we note that the tension in the cord will be zero at A.
7 2
The centrifugal force Lid"4" will then just balance the weight W so that va? = gr.
g
If we apply the concept of the motion being equivalent to a free fall, we obtain
[vc? = va? + 2 gh] ve? = gr +2g9(2r) = 5gr
= 5(32.2)(4) = 644 vo = 25.4 ft persec Ans

PROBLEMS
1433. A 10-lb weight is swung in a vertical circle at the end of a 6-ft cord. The
maximum strength of the cord is 40 Ib. Determine the minimum velocity at which
402 WorkK AND ENERGY [Chap. XIV

the cord will break. Can the weight be swung through a complete circle? What
must be the minimum strength of the cord for the weight to be swung in a complete
circle? Ans. v = 24.1 ft per sec; no; 7’ = 60 lb
1434. A weight of 20 lb is swung in a vertical circle at the end of a 4-ft cord. At
the lowest position of the weight, the tension in the cord is 80 lb. (a) How high
above the lowest position will the weight rise on the circular path? (b) What would
this result be if the cord is replaced by a stiff rod of hegligible weight?
Ans. (a) h = 5.33 ft; (b) h = 6 ft
1435. A 100-Ib weight is attached to a stiff rod of negligible weight that is
hinged at one end. The rod is released from rest in a horizontal position and allowed
to swing freely in a vertical are. Through what angle must it swing to cause a
tension in it of 200 lb? Ans. § = 41.8°
1436. A weight W swings in a vertical circle at the end of a stiff rod of negligible
weight and length L. The weight is displaced an angle 6 from its lowest position
and released from rest. Find @ so that the tension in the rod at the lowest position
is four times that at the start.
1437. A car starting from rest at A is to attempt the frictionless loop-the-loop
shown in Fig. P-1437. The car weighs 150 lb and carries a 150-lb man. Determine
the height / in order for the car to just clear the gap. What will be the normal
pressure of the track against the car at B? What is the maximum force exerted by
the man against his seat during the entire trip?
Ans. h = 55 ft; Ne = 150 lb; max. N = 975 lb

Fic. P-1437.

1438. A particle of weight W, moving at a velocity of Vt ft per sec at the top,

slides vertically along the surface of a smooth cylinder of radius r. Find the vertical
height the particle falls before it leaves the cylinder. Ans. at
4
1439. A particle of weight W, moving with a velocity of v, ft per sec at A (Fig.
P-1439), slides vertically along the surface of a smooth cylinder of 8 ft radius. It
leaves this surface at B. Determine the initial velocity v,. Measured from the
center of the cylinder, how far will the particle travel horizontally before it strikes
the ground? Ans. Uo = 12.44 ft per sec; x = 13.83 ft
Art. 14-8] Fundamental Work-Energy Equation for Rotation 403

Fic. P-1439 and P-1440.

1440. If v, = 5 ft per sec at A in Prob.


1439, determine the angle AOB at which the
particle leaves the surface of the cylinder. Fig. P-1441.
1441. A 100-lb weight rotates in a vertical plane at the end of a light rigid rod
as shown in Fig. P-1441. The spring, whose modulus is 5 lb per in., does not act
on the rod until it exceeds its free length of 3 ft. Determine the velocity of the
weight after it has swung from rest at the dashed position to the given position.
Ans. v = 18.3 ft per sec

14-8. Fundamental Work-Energy Equation for Rotation


The fundamental equation of work-energy applied to rotation is ob-
tained by the same procedure as in translation. (See Art. 14-2.) We begin
by eliminating a in the equations
2M = Ia (a)
and
ad@=w-dw (b)
and we obtain
DM - dé = Iw dw (c)
This is integrated between the limits or conditions that the angular dis-
placement varies from zero to a final value of 6 while the angular velocity
changes from an initial value of w, to a final value of w. We thus obtain
r w
[mas = 1)ode
7) w

or
t]

This is the fundamental work-energy equation for rotation.


404 WorK AND ENERGY [Chap. XIV

Each of the terms on the right side of the equation represents the kinetic
energy of the rotating body, expressed in foot-pounds, at the beginning and
end of an angular displacement. The left-hand term represents resultant
work of rotation during an angular displacement 6, expressed in foot-pounds.
The entire equation indicates that the resultant work is equal to the change
in kinetic energy.
If >M is variable, the resultant work can be found only if 2M can be
expressed in terms of 6 or from the area under a 2M —6@ diagram. If 2M
is constant, the resultant work may be expressed as 2M - 6, in which case
the work-energy equation becomes
=M - 6 =f Iw? — 0?) (14-5)
As in translation, no acceleration is involved; hence the resultant work
on the body (however computed) during any motion is always equivalent
to the resultant change in kinetic energy. Also, as in translation, work is
considered positive in the direction of motion; i.e., moments in the direc-
tion of motion do positive work on the body, and vice versa.

Fig. 14-13. — Velocity of a typical particle.

The expression for the kinetic energy of a rotating body can also be
developed from the fact that the total kinetic energy is the summation of
the kinetic energies of all particles of the body. Thus in Fig. 14-13, if A is
a typical particle, its kinetic energy is
1dw 1 dW
KE = Deg y= ae (d)
For the entire body
Total KE = ‘(pees me, (eran
20g g
or, as before KE =i It )
Art. 14-9] Work-Energy Method with Constant Moments 405

14-9. Work-Energy Method with Constant Moments


In the following illustrative problems the forces causing rotation are
constant. Since the moment arms of these forces are also constant, rota-
tion is due to constant moment.
As in the case of translation, the work-energy equation may be applied
directly to a system of connected bodies without considering any of the
internal forces in the connecting elements. Then after the relation between
force, displacement, and velocity for the original system has been deter-
mined, we may isolate any part of the system and reapply the work-energy
equation to that part.

ILLUSTRATIVE PROBLEMS
1442. Determine the distance that body D in Fig. 14-14 must move in order to
reach a velocity of 24 ft per sec starting from rest. What tension is acting in the
cord joining the step pulleys B and C during this movement?

Wp = 100 lb
Fia. 14-14.

Solution: A preliminary calculation of the tensions, assuming the system to be


at rest, discloses an unbalanced upward force on D which therefore rises while the
rest of the system moves as shown by the displacement arrows. Using s = 79, the
kinematic relations among the bodies are
8A = 2 Op; Op = 2 0c; 3 4c = 8D

Combining these results gives


gs, = 203 = 400 = $ 8D (a)

and since v = rw is of the same mathematical form as s = 76, the velocities are
similarly related as follows:
va =2up=4uc= FD | (b)
406
Ee
Work AND HINERGY [Chap. XIV

The following equates the resultant work of external forces to the total change in
kinetic energy. We apply DX - s to the external forces on each translating body, and
=M - 6 to the moments of external forces on the rotating bodies (of which there are
none in this case). Remember that 2X and =M are positive in the direction of
: W
motion. The kinetic energy change in the system is 2 2a (v? — v,) for the translat-
ing bodies and D4 [(w? — w,’) for the rotating bodies. We then have, using Eq. (6)
to compute the velocities,

4 so)— 100 sp oO
(180 — 18)(4 = 644 beso
(32)? oct + 4
5 (8)(16)? wqet
+ 5 (12)(8)? 2 +4 100
64.4 (24) fo4yo,

which reduces to
176 sp — 100 sp = 3180 + 1024 + 384 4+ 895 (c)
76 sp = 5483 Sp = 72.1 ft Ans.

To determine the tension in the cord be-


tween B and C, we cut this cord and isolate
the system of bodies C and D as shown in Fig.
14-15. As this system moves through sp =
72.1 ft, its change in KE is the sum of the last
two terms on the right side of Eq. (c). The
tension 7’ is an external force on this system,
and its work is computed from 2M - 6c where
6¢ = 38p. Applying the work-energy equa-
tion to this system, we obtain
[RW = AKE]
2 T(% X 72.1) — 100(72.1) = 384 + 895
T=41.6lb Ans.
If desired, the acceleration of D may be
computed from Eq. (10-8).
Wp = 100 1b
[v2 = v,.” + 2 as]
Fig. 14-15.
(24)? = 0 + 2 ap(72.1)
ap = 4.0 ft per sec
whence relations similar to Eqs. (a) or (b) will give the accelerations of the other
bodies.

1443. The initial velocity of the block in Fig. 14-16a is 20 ft per see downward
at the instant the brake is applied. If the kinetic coefficient of friction at the brake
is 0.25, how far does the block descend before stopping? Neglect the dimensions
of the brake block.
Solution: From the FBD of the brake lever in Fig. 14-16b, we have
2Ma = 0] 250 X4-1XN=0 N = 1000lb
[F = f,N] _ F=0.25 x 1000 F = 250 lb
Art. 14-9] Work-Energy Method with Constant Moments 407
i ee
P =250 lb T= 20 ft-lb-sec2 250 lb

1D |Up= 20 ft/sec
(b)
W=200 Ib
Fic. 14-16.

If we apply the work-energy equation to the moving bodies and neglect the
effect of the tension which is an internal force, we obtain

[2x-s+2M-6 = 0 Uo”) + 51(u? — ae) |

200
200 x s — (250 X 3) X @= > x (0 — 209) + 5X 200 — 4 (a)

: 8 20
Sinces = 2 0andv = 2a,or@ = 5 and w, = — = 10 rad per sec, substitution in
2
Kq. (a) gives
200 X s — 375 X s = —1242 — 1000
from which, ;
s=12.8ft Ans.

PROBLEMS
1444. Body A in Fig. P-1444 drops 12 ft from rest before striking the ground.
Pulley B is mounted on an axle 6 in. in diameter, and the axle friction is constant at
40 lb. How many turns will B make after A stops?
Also compute the tension in the cord before A hits
the ground. Ans. 0p = 7rev; T = 41.6 lb

1445. The centroidal mass moment of inertia of


the pulley assembly in Fig. P-1445 is 20 ft-lb-sec’.
Determine the velocity of body A after body B has
moved 8 ft from rest. What is the tension in the wy=j9g8 lb,
cord supporting A? k=1.414 ft

1446. Determine the distance moved by body A


in Fig. P-1446 in changing its velocity from 12 to 24 W=96.6 lb;
ft per sec. Fig. P-1444.
408 Work AND HNERGY [Chap. XIV

T=20 ft-lb-sec?

Fig. P-1446.

W=100 lb W=400 lb 1447. Assume the system described


Fia, P1445. in Fig. P-1228 on page 325 moves
through any convenient distance from
rest, say sa = 12 ft and, after finding the corresponding velocity, use the
work-energy method to check the tension in the cord attached to block A.
1448. Apply the work-energy method outlined in the preceding problem to
check the tension in the cord attached to block A in Prob. 1229 on page 326.
1449. The compound pulleys in Fig. P-1449 are connected by a crossed belt.
Determine the velocity attained by body A after it has moved 12 ft from rest,
and the tension in the cord supporting it.

T= 24 ft-lb-sec”

T= 36 ft-lb-sec”

Fic. P-1449 and P-1450.

1450. Let body D be omitted from the system shown in Fig. P-1449. Determine
the distance in which body A will reach a velocity of 24 ft per sec, starting from rest.
1451. Determine the weight of body D in Fig. P-1451 that will permit body A
to reach a downward velocity of 15 ft per sec after moving 18 ft from rest.
Ans. W = 301 lb
1452. In Fig. P-1451, if body D weighs 644 lb, what velocity will it attain after
moving 9 ft from rest?
Art. 14-10] Work-Energy Method with Variable Moments 409

T=24 ft-lb-sec”

T= 36 ft-lb-sec”

Fig. P-1451 and P-1452.

1453. The 300-lb body in Fig. P-1453 has a downward velocity of 15 ft per
sec at the instant the brake is applied. Determine the value of the brake force to
stop it in 30 ft. Neglect the thickness of the brake block.
1454. The elevator A in Fig. P-1454 is moving down at a velocity of 30 ft per
sec at the instant the brake is applied. What brake force will stop the elevator in
a distance of 12 ft? Also determine the tension in the cable between the drums.
Neglect the thickness of the brake block. Ans. P = 1900 lb

W=193.2 Ib;
R= ib
W= 966 Ib;
k=2tt
U,=15 ft/sec
Fic. P-1453. Fig. P-1454.

14-10. Work-Energy Method with Variable Moments


Although the forces causing rotation may be constant in the following
illustrative problems, their moment arms are not; this causes rotation
under the action of a variable moment. Angular velocities in such cases
410 Work AND ENERGY [Chap. XIV

are best obtained by the work-energy method which


eliminates consideration of variable angular accel-
eration.
The work done by gravity forces will be the
product of the weight W by its change in eleva-
tion h as discussed in Art. 14-3. It is worth while
to check this statement by considering Fig. 14-17
in which the body rotates through @ radians from
Fig. 14-17.
the horizontal. Although not so convenient, the
t)
work done by W can be found by applying { 2M - dé which gives
0
6 t)

iy QO = ffW7 cos
6 dé = W7{sin 6]§ = W7 sin 0
0 0

and as? sin @ = h, we finally obtain Wh as stated before.

ILLUSTRATIVE PROBLEMS
1455. A uniform rod 6 ft long weighing 96.6 lb carries a 32.2-lb weight at one
end. As shown in Fig. 14—18, it starts from rest in a horizontal position and rotates
through a vertical are of 60°. At this instant determine the bearing reaction at A.

eA ae ae.
T
|
/ 30.3 lb
y
hi
yA
=105 lb

ae
(6) Wee =20.21
Sige
32.2 1b 32.2lb g Fw°=70\b
Fic. 14-18. Fic. 14-19.— Dynamic equilibrium.

Solution: The moment of inertia of the assembly about the axis A is +ML? for the
W
rod plus — k? for the weight, giving
g
1 (96.6
ft-lb-sec?
lk 3 (a6) a = 5(6) = 72
Art. 14-10] = Work-Energy Method with Variable Moments 411
CE ef pa a INSa alee laa ll
To determine the instantaneous acceleration, refer to Fig. 14-18 and apply
[2M = Ia] 96.6(3 cos 60°) + 32.2(6 cos 60°) = 72a@ a = 3.36 rad per sec?
The value of w? is determined from the work-energy equation. The only external
forces doing work in this problem are the gravity forces which are constant in di-
rection. Multiplying each weight by the vertical component of displacement of
its center of gravity, we obtain
[RW = 21a(@ — w,")] 96.6(3 sin 60°) + 32.2(6 sin 60°) = 3(72)w? w? = 11.63
We now use these values of a and w? to obtain the inertia forces which create
dynamic equilibrium as shown in Fig. 14-19. Note that the inertia forces are ap-
plied at the center of gravity of each weight. The inertia couple Ja need not be
computed because it has no components to affect the following normal and tan-
gential summations:
[ZN = 0] R, — 105 — 70 — (96.6 + 32.2) cos 30° = 0 Rn = 292.7 lb
[2T = 0] Re + 30.3 + 20.2 — (96.6 + 32.2) sin 30° = 0 R; = 18.9 lb
By combining these components, the total bearing reaction is 292.8 lb which we
round off to 293 lb.
1456. Body A weighs 200 lb and has a moment of inertia of 120 ft-lb-sec? about
the axis of rotation at Z. Body A starts from rest in its topmost vertical position
and rotates through 60° under the action of block B as shown in Fig. 14-20. De-
termine the values of w, a, and the tension in the cord at the given instant. Then
determine the bearing reaction on the axis of rotation Z.

W=2001b

I,=120 ft-lb-sec? W= 400]b

Fia. 14-20.

Solution: The angular velocity is determined by equating the resultant work of


external forces to the total change in kinetic energy. Body A rotates through

60 X ads rad; hence the vertical distance that block B falls is given by s = r0 =
180
y («0s< -) = 0.524 ft. The vertical component of displacement of the center
180
of gravity of body A is (4 — 4 cos 60°). From the work-energy equation, we obtain
.
12
412 WorkK AND ENERGY [Chap. XIV

[RW = 29
FEot = 9)
Pe + G,5 Ht — a 2 |
400
400 X 0.524 + 200(4 — 4 cos 60°)= 644 SK a +3 < 120) XK ae (a)

Since v = rw and therefore v = $ w or v? = fo, (ae i in Eq. (a) yields


210 + 400 = 1.55 w? + 60 w?
from which
w 2 = 9.9 w = 3.15 rad persec Ans.
The tension in the cord and the angular acceleration of the body at the given
instant are found by applying the kinetic equations of translation and rotation to
the block and to body A respectively. This gives
W 400
|2x - =


AN) as — =
mie (0)
b

(2M, = yo] aos ap + 200 x 4 sin 60° = Ade (c)

Since a = 4a, solving Eqs. (6) and (c) yields


a= 7.25 rad persec? and 7 = 355]b Ans.
Observe that the angular acceleration cannot be found from w? = w. + 2 a6 since
a is not constant in this problem. Furthermore only an average value of 7 would
be obtained by applying the W-E equation to the FBD of B or A.

a=180lb

Ry
Fig. 14-21.

To determine the bearing reaction on body A, we create dynamic equilibrium as


shown in Fig. 14-21. The values of the inertia forces are
Ww 2D eee ae
plied
5 Fw ss29 ag, 9) = 2 46 lb

Fiaue= a05 ay(7 .25)= 180 lb


Art. 14-10] Work-Energy Method with Variable Moments 413

Note that the value of the inertia couple Ja need not be computed because it has
no effect on the following vertical and horizontal summations of forces:
pee (0) 180 cos 30° + 246 sin 30° — 200 — R, = 0 R, = 79 |b
[>x = 0] 355 + 246 cos 30° — 180 sin 30° — R;, = 0 Ry, = 478 lb
Combining these components, the total bearing reaction is
[e =VRi+ R?| R =\/(478)?
+ (79)? = 4851b Ans.
PROBLEMS
1457. A slender rod 10 ft long is hinged at its upper end and hangs vertically.
The rod is struck a sudden blow which causes it to rotate through an angle of 90°.
Compute the initia! velocity of the lower end of the rod. Since the gravity center
travels a frictionless path, why can’t you find its velocity by using the concept of
free fall developed in Prob. 1482 on page 401? Ans. v = 31.1 ft per sec
1458. A solid cylinder of radius r and weight W is
free to rotate in a vertical plane about a fixed horizon-
tal axis passing through a point on its rim, as shown in
Fig. P-1458. If the cylinder starts from rest in a
vertical position when 6 = 0, determine the value of w
in terms of 6. Then by differentiating w, obtain the
value of a in terms of 8. Check this value of a@ by ap-
plying 2M = Ia.

Ans. 0 =v ADE pee 6 = eae Fie. P-1458.


3r 3r
1459. A slender uniform bar of length L and weight W is free to rotate in a verti-
cal plane about a horizontal axis passing through one end. If the rod rotates through
an angle of 6 rad from rest, starting in a horizontal position, determine the value of

k=2.5 lb/in

W= 644 |b;
k,=1.414 ft Pe 96.6 Ib
att

Fiaigre Fic. P-1461.


Fic. P-1460.
414 Work AND ENERGY [Chap. XIV
ee
w for any value of 6. Then find @ by differentiating w and check by means of
2M = la. _ _ fsgsing _ 3gcos0
Ans. w= TM ee

1460. A heavy belt 80 ft long and weighing 10 lb per ft passes over the cylinder
shown in Fig. P-1460. At the start, d = 10 ft. Neglecting the thickness of the belt,
determine its velocity at the instant when d = 40 ft. Ans. v = 14.7 ft per sec
1461. At the instant shown in Fig. P-1461, the spring is horizontal. Determine
the clockwise angular velocity of the uniform 96.6-lb rod at the given position so
that it will just reach a horizontal position. Ans. w = 2.56 rad per sec

T=36ft-lb-sec”

W=150 lb
ae

I,=81 ft-lb-sec?

Fig. P-1463.

1462. Resolve Prob. 1441 on page 403 if, in addition to the given data, the rod
weighs 64.4 lb. Ans. v = 20.4 ft per sec
1463. Find the linear velocity of block
C in Fig. P-1463 after body A has rotated
60° from its topmost vertical position, start-
ing from rest.
Ans. v¢ = 4.83 ft per sec
1464. The system described in Prob.
1250 on page 335 is repeated here as Fig.
P-1464. It is known that J, = 45.4 ft-lb-
4 ho sec” for the rotating assembly. Compute
193.2lb the velocity of the 193.2-lb body after a
90° rotation from the given position if the
initial angular velocity is 7 rad per sec (a)
clockwise and (b) counterclockwise.
1465. From Fig. P-1465, determine
Fie, P-1464, the clockwise angular velocity w, at the
lowest position of the body that will
make the body swing through a complete revolution.
Ans. Wo = 5.4 rad per sec
Art. 14-10] Work-Energy Method with Variable Moments 415
a A A AA 6 te i lm a ll la ll

W =193.21b

W=822 lb

Fie. P-1465.

1466. A pole 6 ft long and weighing 96.6 lb hangs vertically from a horizontal
axis through its upper end. A blow gives the pole an initial angular velocity of 6
rad per sec. Determine the horizontal and vertical components of the bearing reac-
tion at the instant the pole has rotated through 60°.
Ans. Rp, = 248 lb; R, = 168 lb
1467. A uniform rod with rounded ends is placed upright on a rough horizontal
surface. It is permitted to fall from rest to the ground. Compute the coefficient of
friction at the ground if the lower end slips when the rod has a slope of 4 vertical to 3
horizontal. ASO O7,
1468. A uniform rod 8 ft long weighing 96.6 lb has a 32.2-lb weight fastened to
oneend. It rotates about the other end through a vertical arc of 60°, starting from
rest in its topmost vertical position. At this instant, compute the total bearing
reaction. Ans. R = 43.5 lb
1469. Toa 161-lb cylinder is bolted a 128.8-lb uniform rod 6 ft long which carries
a 32.2-lb weight at itsend. A cable wrapped around the cylinder passes horizontally
over a small pulley to support a 322-lb body. The system starts from rest when the
rod is in its lowest vertical position and rotates through 60° to the position shown in

Fia. P-1469 and P-1470.


416 Work AND ENERGY [Chap. XIV

Fig. P-1469. At this instant compute the centrifugal inertia force acting on the
rotating assembly and the velocity of the 322-lb body.
Ans. C.I.F. = 108.5 lb; v = 4.8 ft per sec
1470. Find the bearing reaction on the system of Prob. 1469 after the rod has
rotated 180°, starting from rest in its lowest position.

14-11. Work-Energy Applied to Plane Motion ©

We have shown in Art. 13-6 that a plane motion is equivalent to a


combination of pure rotation about its centroidal axis and a translation of
its center of gravity. We shall now show that the work-energy equation
for plane motion is the sum of the work-energy expressions previously
developed for the motions of translation and of centroidal rotation.
For the translational component of plane motion, taking the X axis
tangent to the path of the gravity center during each displacement ds
along the path, we have

RW = fox -d= W 2 — 0) (a)


0 2 g

and for the rotational component,


6

RW = ['sit- ao = 11(w? — w,”) (b)


0

Equating the sum of the left-hand terms of these equations to the sum of the
right-hand terms, we obtain the following work-energy equation for plane
motion:

RW = SU
W
(= — eg)
Dy) + 5 Ilo? — w')
1;
(14-6)
This arithmetical addition is possible because both work and kinetic energy
are scalar quantities expressed in the same foot-pound units. The resultant
work (RW) of constant or variable forces and moments is computed as
explained in previous articles. The kinetic energy is expressed in terms
of the velocity 0 of the gravity center and I which is the centroidal mass
moment of inertia.
If the instant center of plane motion can be located, it is usually more
convenient to express the kinetic energy in terms of an instantaneous
rotation about the instant center. Thus expressing the kinetic energy at
any instant as

hep (c)
Art. 14-11] Work-Energy Applied to Plane Motion 417
aE RE i 1 al odeh eee ltt ee
we may replace 0 by 7w where 7 is the distance from the instant center to
the center of gravity. Doing this gives

KB = 57
1W
Pat +5 lot
=
= 5e(1“ + Ye)
which reduces to

KE = 4 Ic (c)

W
since, by the transfer formula, J + ai 7 = Ic which is the mass moment
of inertia about the instant center C.
Except for free-rolling wheels, the position of the instant center relative
to the body usually changes, so that the change in kinetic energy must be
expressed as
AKE = $ Icw@? — 3 Ic,@,? (14-7)

in which Jc and Jc, represent respectively the moments of inertia about the
final and initial positions of the instant center C.
This last expression suggests a special but convenient form of the work-
energy equation for homogeneous free-rolling wheels in which [¢ is constant.
Considering the plane motion as equivalent to rotation about the instant
center, we may write
EMc + 0 = 1 Ic(w? — 0,2) (14-8)
in which 2M¢ is the moment about C of the constant external forces acting
during an angular displacement 6.1. This moment summation about C
automatically eliminates the static frictional force holding the instant
center momentarily at rest and indicates that such friction forces (as the
term ‘‘static’’ indicates) do no work on free-rolling wheels. If the wheel
slips, Eq. (14-8) will not be valid; we shall then have to use Eq. (14-6) and
also consider the work done by kinetic friction forces.

ILLUSTRATIVE PROBLEMS

1471. The system in Fig. 14-22 has the values shown. Assuming disk D to roll
without slipping, determine the linear velocity of its center after body B has moved
2 ft, starting from rest. Also determine the friction force acting on the disk D to
prevent slipping.
1 Equation (14-8) can also be obtained by eliminating a from between 2Mc = Ica
and a d@ = w dw and then integrating according to the procedure outlined in Art. 14-8.
Naturally, the ensuing result is valid only for homogeneous free-rolling wheels to which
~Mc = Ic a may be applied.
418 WorK AND ENERGY [Chap. XIV
na a ea a a

W=161 lb
kR=1.414f Te

W = 193.2 lb

Fig. 14-22.

Solution: We first determine the mass moments of inertia.

T= Wig Fores i= meu (1.414)? = 10ft-lb-sec?


9 32.2
Ho petPBs oe ;
Imtoye JOS IP = 39,9 (2)? = 40 ft-lb-sec

2 22
Uc =1+ Ma} ForD: Ic = 40 + z (3)? = 180 ft-lb-sec?
32.2
The directions of motion may be found by the method of temporarily assuming
the system to remain at rest, as in Prob. 1339 on page 368. These directions are
shown by the dashed arrows adjacent to each body which specify the positive direc-
tions of forces and moments. On this basis, the resultant work calculated below is
positive and speeds up the system, thereby confirming these directions of motion.
The kinematic relations between the bodies are
[s = 70] sr, = 4060p =1XOp also sr, = sp = 2 Op
which are combined into the following relation
4060p = Op =H 5B (a)
whence, by differentiation, the velocities are similarly related by
4 wp = we = Foz (b)
We now apply the work-energy equation to the system, considering only the work
of external forces. Using Eq. (a) and the given value of sg = 12 ft, we have 6p =
Art. 14-11] Work-Energy Applied to Plane Motion 419

% sg = 1.5rad. The resultant work is the sum of the left-hand terms in Eqs. (14-2),
(14-5), and (14-8) used respectively for the translational, rotational, and plane
_ motion parts of the system. The total change in kinetic energy is the sum of the
right-hand terms of these equations. Thus we obtain
[2X - sla + [2M - |p + [2M - Op =

E (ve — ve) |ob EI? — 2) | + EIo(@? — w,?


2g B 2 P 2 D

Substituting vg = 8 wp and wp = 4 wp from Eq. (6) in Eq. (c), gives


2320 — 724 = 3(64 wp?) + 5(16 wp?) + 65 wp? = 337 wp?
whence
wp? = 4.73 and wp = 2.18 rad per sec
The velocity of the center of disk D therefore is
[i = Tw] dp = 3 X 2.18 = 6.54 ft per sec Ans.
The friction force acting on D is found by applying the following work-energy
relations to the FBD of the disk.
[2>Mc- 6 = 3 Ic — w,”)] (4 T, — 322 X 3 sin 30°)(1.5) = 3 (130)(4.73) =(d)
[2M -0=3T @ — w,*)] (1 xX T; — 3 F)(1.5) = 3 (40)(4.73) (e)
Multiplying Eq. (e) by (—4) and adding the result to Eq. (d) eliminates 7; and
gives
18 F — 724 = (130 — 160)(4 X 4.73) = —71
F = 36.3\lb Ans.
1472. The circular disk shown in Fig. 14-23 weighs 161 lb after the semicircular
hole is cut out of it. Its radius of gyration about the gravity center G is 1.2 ft.
Determine the angular velocity of the disk after it has rolled without slipping for
450° or 14 turns clockwise, starting with an initial angular velocity of 4 rad per sec.
W, = 4 rad/sec
Ww
Pe ae SS
Za
Sian “S
a
161lb ~\

Fig. 14-23.
420 Work AND ENERGY [Chap. XIV

Solution: During the movement from the initial to the dashed final position, the
work done is caused only by the center of gravity dropping a net vertical distance
of 6 in. = 0.5 ft. Since free-rolling is assumed, the static friction F does no work.
Before equating the resultant work to the change in kinetic energy given by Eq.
(14-7), let us compute J¢, Fe Ic. sisthe transfer formula, we obtain

(=I+4+ Ma] ihe 50


s 2)? +3
—s5(0. 5++ 2°)= 27.45 {t-lb-sec”

ie = a5 (-£2) 2+ +55
——— (1.5)
(1.5)? = 17.45
17.45 ft-lb-sec?
ft-lb-sec

The work-energy equation now gives


[RW = 3 Icw? — 3 Ic," 161 X 0.5 = $(17.45)w? — $(27.45) (4)?
w? = 34.4 and w= 5.87 rad per sec Ans.
1473. Sliders A and B, connected bya
uniform bar 6 ft long, move in the friction-
less guides shown in Fig. 14-24. If the
system starts with negligible velocity when
A is vertically below B, determine the ve-
locity of A in the given position.
Solution: Using the instant center C,
the kinematic relations between the bodies
are
P=R)]| OrpSaee@ ancl on =]cise
W,=128.81b The resultant work done on the system
is caused only by Wg and W4pz. The
kinetic energy change in the system is
eer
i (v? — »,?) for the translating sliders
Fig. 14-24, A and B and 4Jcw? — 4 Ic¢,w,? for the
plane motion of the bar AB. As it hap-
pens, not only is the initial velocity negligibly small, but in this example J¢, also
equals I¢ so that only J¢ need be computed. Using the transfer formula, its value is
= 64.4 64.4
[IT=I+Md
he ] Ig = wee‘)(6)? 2+ + =
30.9 (3) (3)? = 244 ft-lb-sec
ft-lb-sec?

Applying the work-energy equation, we have


[RW = AKE]
96.6 128.8
96.6(6( — 4.8 =F 64.4(3( — 2.4 — 64.4 36%w)? 2 + 64.4 (4.8w)?ond
+ 4(24)w?

whence
= 1.99 and w = 1.41 rad per sec
Hence the velocity of A is
lua = 4.8] va = 4.8 X 1.41 = 6.77 ft per sec Ans.
Art. 14-11] Work-Energy Applied to Plane Motion 421

PROBLEMS

1474. As shown in Fig. P-1474, a cord from a rigid support is wrapped around a
solid homogeneous cylinder of radius r and weight W. Find the velocity of the
center of gravity after it has dropped h ft, starting from rest. h
Ans. 1 = 2. [9%
3

k=250 lb/in

Fic. P-1474. Fig. P=1475.

1475. The car represented schematically in Fig. P-1475 weighs 1610 lb exclusive
of its four wheels, each of which is 30 in. in diameter, weighs 64.4 lb, and has a
centroidal radius of gyration of 12 in. Determine the maximum deformation in
the bumper spring if the car hits it with a speed of 10 ft per sec.
Ans eese—al h4 108

Fia. P-1476.

1476. Assuming free rolling of the disk in Fig. P-1476, find the velocity of body
A after it has moved 8 ft, starting from rest. Assume the fixed pulleys to be
frictionless and of negligible weight.
422 Work AND ENERGY [Chap. XIV

1477. The disk shown in Fig. P-1477 rolls freely on a horizontal track. Com-
pute the angular velocity of the disk after it has rolled 12 ft from rest. Neglect
the backward inclination of the block caused by its inertia.
Ans. w = 3.9 rad per sec

W=161 Ib;
hz2ft

Fig. P-1477. Fic. P-1478.

1478. Disk A is composed of a 3-ft radius cylinder sandwiched between two


cylinders each of 2-ft radius. It rolls without slipping between two parallel rails as
shown in Fig. P-1478. Determine the angular velocity of disk A after its center has
rolled freely for 12 ft, starting from rest. Assume body B weighs 128 lb and g =
32.0 ft per sec. Neglect the effect of the inertia force on the inclination of B.
Ans. w = 6.07 rad per sec

1479. Assume the body A in Fig. P-1479 moves 12 ft, starting from rest. After
determining the velocity corresponding to this displacement, compute the tension in
the cord joining disk D and pulley P by applying the work-energy method to a
FBD of disk D.

Fic. P-1479,
Art. 14-11] Work-Energy Applied to Plane Motion 423
1480. The solid homogeneous sphere A and the compound drum B shown in
Fig. P-1480 roll without slipping on the inclined planes. Determine the velocity of
the center of sphere A after it has moved 12 ft from rest.
Ans. ta = 12 ft per sec

Sphere, W=64.4 lb W=128.8 lb


k =1.414 a

{P= 96.6 Ib
k=1ft

Fig. P-1480. Fig. P-1481.

1481. The compound drum A and the floating pulley B shown in Fig. P-1481 roll
without slipping. Compute the velocity of the center of A after it moves 12 ft
from rest.
1482. The floating disk D in Fig. P-1482 rolls without slipping on the cord
passing from the fixed support and around it to pulley P. Compute the linear
velocity of body A after A has moved 12 ft from rest. What is the tension in the
cord connecting pulley P with disk D?
Ans. va = 15.6 ft per sec; T = 144 lb

T= 24 ft-Ib-sec?

W=161 lb
k=1.414 ft

Fic. P-1482. Fic. P-1483.

1483. A homogeneous slender rod L ft long and weighing W lb slides down a


smooth vertical wall and along a smooth horizontal floor as shown in Fig. P-1483.
Assume that the ends of the rod remain in contact with the wall and the floor.
If the rod is at rest when 6 = 0°, determine the value of the instantaneous angular
424 Work AND ENERGY [Chap. XIV

velocity w in terms of the angular displacement 6. Then by differentiating w, obtain


the value of win terms of 6. Will the value of a be any different if the lower end of
the rod is not at rest when 0 = 0°? Why?

Ans, |@)— 7 — cos ba = 28sind

1484. An elliptical plate has semi-axes a (major) and


b (minor). It is held with the major axis vertical, as in
Fig. P-1484. The plate is released from rest and rolls
freely on the horizontal surface shown. Determine the
angular velocity at the instant the major axis is hori-
zontal.

Ans. w= 7 eee rad per sec


Fic. P-1484. Ca
1485. In Fig. P-1485, the uniform rod AB is 6 ft long and weighs 193.2 lb. The
solid cylinder fastened to it at B has a radius of 1 ft and weighs 96.6 lb. The system
starts from rest when AB is vertical. Compute the angular velocity of the cylinder
when at the position shown. Ans. w = 4.65 rad per sec

Fig. P-1485. Fia. P-1486.

1486. The system shown in Fig. P-1486 consists of a crank OA joined to a solid
cylinder B by a connecting rod AB. The crank and connecting rod may be con-
sidered to be uniform slender rods, and the cylinder rolls without slipping. When
crank OA is vertical, its angular velocity is 2 rad per sec clockwise. Determine the
angular velocity of the crank when it is horizontal. Ans. woa = 7.69 rad per sec
1487. Replace body A in Fig. P-1411 on page 393 by a solid cylinder 3 ft in diam-
eter weighing 322 lb. The cord is attached to the cylinder at its center. Determine
the linear velocity of the center of the cylinder after it has rolled without slipping for
9 ft, starting from rest at the given position. Ans. da = 12.75 ft per sec
1488. The unbalanced wheel described in Prob. 1372 on page 381 is acted upon
by a constant force P = 200 lb directed parallel to and up the incline. If the wheel
has a counterclockwise angular velocity of 2 rad per sec at the given position and
Summary A425
does not slip, determine its angular velocity after its geometric center has traveled
Po malt.

SUMMARY
By eliminating the acceleration from the equations >X = Lid and
ads = v dv, we obtain the relation

s ia Vent WV Palisa
J zx-as moar By = yu — U,°) (14-1)

This is the fundamental work-energy equation for translation.

The expression {)=X - ds is defined as resultant work (or as =X -s if


0

the forces are constant) and is considered positive in the direction of motion.
7
: rt Se 0%, :
The expression 35v? is defined as kinetic energy. The usual unit for work

and energy is the foot-pound.


In Art. 14-3 it is shown that the geometric significance of work is the
product of the component of force in the direction of displacement multi-
plied by the displacement, or the product of the component of displace-
ment in the direction of the force multiplied by the force. Forces perpen-
dicular to the displacement do no work.
In applying the work-energy equation, it is best to equate the resultant
work done by external forces on a system to the total change in kinetic
energy. This makes it unnecessary to consider the work done by the in-
ternal forces in a system. The resultant work due to variable forces (and
also constant forces) can be easily obtained from the area under a force-
displacement curve.
Power is defined mathematically by the expression F -v where F is the
resultant force doing the work at any instant and v is the corresponding
value of the instantaneous velocity. The unit of power (ft-lb per sec) is
too small for use in engineering; the more common unit is the horsepower
(hp), defined as equivalent to 550 ft-lb per sec or 33,000 ft-lb per min.
The work-energy equation for rotation and the methods employed are
analogous to those applied to translation. The fundamental equation is
obtained by eliminating the angular acceleration from 2M = Ja and
adé = w dw; this gives
6
f EM-d6 = 4 Io? — 2 Io,? = 3 Mo? — 0,2) (14-4)
0

6
The expression fsa - dé (or 2M - 6 if the moments are constant) is
0
defined as resultant work in rotation. Work is considered positive in the
426 WorkK AND ENERGY [Chap. XIV

direction of rotation. The expression } Jw? is defined as kinetic energy in


rotation. The unit for both expressions is usually the foot-pound.
When the work-energy method is applied to systems of bodies some of
which translate and others rotate, the resultant work done by external
forces on the system is equated to the total changes in kinetic energy of
each body of the system. This principle may be written in equation form as

> (2X +s) + (2M - 6) = EAC — v2) + Du (w? — w22)


The larger summation sign shows that the expressions for work and kinetic
energy for each body in the system are to be added as indicated.
The work-energy equation applied to plane motion is the sum of the
work-energy expressions for translation of the gravity center plus that for
centroidal rotation, viz.,

RW = oe (D2 + 0,2) + 5Ho? — @") (14-6)


If the instant center of rotation can be located, it is usually more convenient
to express the change in kinetic energy as
AKE — 4 Tow? == 4 [o,e00 (14-7)

in which I¢ and Ic, are the moments of inertia about the final and initial
positions of the instant center C.
For homogeneous free-rolling wheels, the most convenient form of the
work-energy equation is
=Mc : os 3 I¢(@? aa Wo) (14-8)

in which 2M¢ is the moment about the instant center C of the constant
forces acting during a displacement 0.
Chapter XV.
Impulse and Momentum

15-1. Introduction
We have seen that the work-energy method eliminates consideration of
acceleration in problems relating force, displacement, and velocity. There
is another class of problems which relate force, velocity, and time in which
the acceleration may also be eliminated. This is particularly convenient
when forces act for very small time intervals during which the forces may
vary, as in an impact or sudden blow.
Such problems are conveniently solved by means of the impulse-momen-
tum method. As in work energy, the terms 7mpulse and momentum are
used to describe certain mathematical expressions which we shall derive
in the following articles for the motions of translation, rotation, and plane
motion. We shall also discuss the technique, use, and advantages of the
impulse-momentum method applied to the several motions.

15-2. Fundamental Impulse-Momentum Equation for Translation


Proceeding as in work energy, let us eliminate the acceleration from the
equation for the motion of the center of gravity (Art. 9-8) and from that
defining the acceleration of the center of gravity in terms of its velocity.
These equations are:
fa ily
4 (a)
DOG
Eliminating @ leaves W
R dt = dF di (b)

which is the differential form of the impulse-momentum equation. Assum-


ing that the velocity has the value 5, when 1 is zero and the value 0 at any
other value of ¢, integration of Eq. (b) between these limits gives the general
form of the impulse-momentum equation, viz.,
t

[rat=¥v Wo, (15-1)


0 g g
The symbol — indicates vector subtraction of the two terms on the right
side of the equation. We shall show in the following articles that each
427
428 IMPULSE AND MOMENTUM [Chap. XV

term of this equation represents a vector quantity; hence the necessity


for vector instead of algebraic subtraction.
Note that Eq. (15-1) refers to the motion of the center of gravity of any
system just as did Eq. (a) from which this result was derived.
t

Resultant Linear Impulse. The expression -fR dt is known as the re-


“0

sultant linear impulse. Obviously this expression can be evaluated only if


R is constant or can be expressed in terms of ¢. Examining the expression
dimensionally shows the unit of linear impulse to be lb-sec.
It is also important to note that the expression FR dt consists of a vector
quantity (R) multiplied by a scalar quantity (dt). This results in a new
vector having the same sense and inclination as R but differing from it in
magnitude. In other words, linear impulse is a vector quantity and, as
such, may be resolved into X, Y, and Z components when desired.

Linear Momentum. The quantity = dis the linear momentum of a body

at any instant and may be symbolized by U. The quantity 7,dt may be


represented by dU. Substituting the dimensional equivalents in the ex-
pression for linear momentum gives
Wee _ ]lb X see 2 5 ft _ eee
g ft sec
This shows the dimensional unit of momentum to be identical with that of
impulse.
As in the case of impulse, it is important to note that the expression Mgd
g
consists of a vector, 0, multiplied by a scalar coefficient, = this results
in a new vector having the same sense and inclination as d but a different
magnitude. In other words, linear momentum is also a vector quantity
which may be resolved into X, Y, and Z components when desired.

15-3. Special Cases of the Fundamental Equation


When considering the dynamic action of jets or eccentric impact, etc.,
it is convenient to deal with the components of impulse and momentum.
Equation (15-1) may be written in the following form:

(Seas
0
t

g g
t

[sy
0
at a g
ye lt
g
(15-2)
t

[osz-at digi aa Wed.


0 g g
Art. 15-4] Linear Impulse-Momentum Applied to Translation 429
0 i a le eect ee te al RM ba tk| id
Tt will be seen that these equations could also have been derived by con-
sidering the components of the vector Eq. (a) (Art. 15-2), namely

eae di
ZX = —: aLae aLe = sR

< W . " do.


AS ar and Gi Ta

W dd
2Z =
ee:Lz

selz =
cit
—_

and eliminating d:, @,, and @., as was done there.


Equation (15-2) may be summarized in the following principle: The com-
ponent of the resultant linear impulse along any axis is equal to the change in
the components of the linear momentum along that axis.

15-4. Linear Impulse-Momentum Applied to Translation

Since the condition of translation means that all the particles have the
same displacement, velocity, and acceleration, there will be no need to
use the bar sign to distinguish the motion of the center of gravity of the
body. Furthermore, if we continue to specify that the positive sense of
the X axis is taken in the initial direction of motion, the resultant force
causing motion will be given by 2X; however, ZY and YZ will be zero.
Hence the general equation of linear impulse-momentum as applied to trans-
lation reduces to
t

Hismedie on. Yo, = Wp (15-3)


0 g g g
When the applied forces are constant, this equation becomes

OXT fm = (vy — v,) (15-4)


Be careful to substitute correct signs for v and v,; remember that these
quantities are considered positive in the initial direction of motion.
The following problems show the application of these equations to trans-
lation when =X is constant and also when LX is a function of the time ¢.

ILLUSTRATIVE PROBLEMS

1501. A 200-lb block in contact with a level plane is acted upon by a horizontal
force P equal to 100 lb as shown in Fig. 15-1a. The coefficient of kinetic friction is
0.20. In what time will the velocity of the block be increased from 4 ft per sec to
10 ft per sec?
430 ImpuLsE AND MOMENTUM [Chap. XV
(e be aiS BES A a i
Soyiritubinsiair R
W = 200 lb; ly
i |
N= 200 lb

F=40 lb P=100 lb

(a) (b)
Fie. 15-1.
Solution: The point diagram of the free body is shown in Fig. 15-1b. Applying
the impulse-momentum equation (2X constant) gives
200
|2x she - (= v9 | (100 — 40)¢ = 355 (10 — 4)
t = 0.621 sec Ans. -
It will be observed that a similar result would be obtained by applying 2X = a a
to find the acceleration and substituting this in the kinematic equation v = v, + at
to find the time. In the present application, the impulse-momentum equation is
used merely to eliminate the acceleration in the two equations just mentioned.
The impulse-momentum method is of value because it eliminates the acceleration,
especially when this is variable. The advan-
2X = Resultant force tage of the method will be more apparent in
the next problem.
1502. A 400-lb block is in contact with a
level plane whose coefficient of static friction
is 0.40 and whose coefficient of kinetic friction
is 0.20. The block is acted upon by a force P
which varies with the time according to the
relation P = 20t. What velocity will the
Fic. 15-2. — Force-time diagram. block attain as the time varies from zero to
10 sec?
Preliminary: The resultant impulse when the applied forces are variable is given
by f2X - dt. If a force-time diagram is plotted, asin Fig. 15-2, it will be apparent
that the expression 2X - dt represents the area of the shaded elemental strip. Hence
the resultant impulse ("2X - dt is equivalent to the area under this diagram between
the proper time limits. This graphic interpretation of the resultant impulse will
help in solving the problem.
Solution: Figure 15-3b shows the point diagram of forces acting on the block.
Applying 2Y = 0 gives
N —400=0 or WN = 400lb
The maximum static friction force is obtained from
F=f,N = 0.4 X 400 = 160 lb
Art. 15-4] Linear Impulse-Momentum Applied to Translation
Ee ene ee— eS431
W= 400 lb; X-forces (Ib)

Time (sec)
F }
N |400 lb

(a) (b) (c)


Fig. 15-3.

But this static value will act only when P = 160 lb or, since P = 201, after 8 sec
have elapsed. At any time between zero and 8 sec, the static friction will adjust
itself to the net force tending to cause motion. As the time increases beyond 8 sec,
the block will start to slide, and the kinetic friction force will act at its maximum
value, as found from
F=f,.N= 0.2 X 400 = 80 lb
We shall assume that the transition from static friction to kinetic friction takes
place instantaneously, although this is not strictly true. With this assumption, a
graphic representation of the X components of the forces acting, plotted against
time, will be as shown in Fig. 15-8c.
Applying the impulse-momentum equation and noting that {2X - dt is equiv-
alent to the areairae the force-time diagram, we have
ee 160 400
[Jez. dt =—=o = | ee ae el Sn x
v = 16.1 ft per sec Ans,
Referring to Fig. 15-3c, observe that during the initial 8 sec the resultant force
(and hence the resultant impulse) is zero.

PROBLEMS
1503. A 300-lb block is in contact with a level plane whose coefficient of kinetic
friction is 0.10. If the block is acted upon by a horizontal force of 50 lb, what time
will elapse before the block reaches a velocity of 48.3 ft per sec, starting from rest?
If the 50-lb force is then removed, how much longer will the block continue to move?
Ans. t = 22.5 sec; t = 15 sec
1504. A horizontal force of 300 lb pushes a 200-lb block up an incline whose
slope is 3 vertical to 4 horizontal. If f, = 0.20, determine the time required to
increase the velocity of the block from 10 to 50 ft per sec.
1505. Determine the value of P that will give the system of blocks in Fig. P-1505
a leftward velocity of 20 ft per sec, 10 sec after starting from rest.
1506. Thesystem shown in Fig. P-1505 has a rightward velocity of 10 ft per sec,
What value of P will give it a leftward velocity of 20 ft per sec in a time interval of
20 sec? Ans. P = 378 lb
432 IMPULSE AND MOMENTUM [Chap- XV

Fic. P-1505 and P-1506.

1507. Determine the velocity of body B in Fig. P-1507 after moving for 5 sec.
(a) starting from rest and (b) starting with a downward velocity of 6 ft per sec.
Ans. (b) vp = 8.64 ft per sec up
1508. A 64.4-lb block is in contact with a smooth
level plane. It is acted upon by a horizontal force
P which varies according to the relation P =
12¢ — 3, where P is in pounds and ¢ in seconds.
Determine the maximum positive velocity of the
block, and the time when the block will again be at
rest.
Ans. v = 16 ft per sec; t = 6 sec

15-5. Displacement from Force-Time Curve


We have seen that the area under the force-
time curve may be used to determine the ve-
Fic. P-1507. locity. We shall now demonstrate how this
area may be used to determine the displace-
ment. This method is especially convenient when dealing with variable
forces.
The differential form of the impulse-momentum equation

DX -dt = Ly dv
may be rewritten as 9
dv = 75-BX dt (a)
This expresses the velocity change caused by DX in the time dé.
Let us suppose that 2X is applied to a body for a time dt and then re-
moved, and that no other force acts on the body after this time. Therefore,
by Newton’s first law of motion, it will continue to move with the velocity
increment given by Eq. (a). Since the equation of motion is s = vt for uni-
form velocity, it follows that the displacement during the time interval
(t; — t) from the instant ¢ when this velocity increment occurs to some later
time ¢; may be written as
g
Art. 15-5] Displacement from Force-Time Curve 433
E a ee a
The effect of continuously varying the value of =X over the time interval
from zero to t; will be a summation of terms similar to Eq. (b). This gives
ty
= ants
lf (t, Las
— t)DXet- dt (c)

A graphic representation of this


integration is given in Fig. 15-4.
The value 2X - dt is the area of the
elemental strip, and (t — ?) is its
moment arm with respect to the
ordinate at 4. The integral is the
summation of moments of all such
strips and is equal to

s=SAY
75:4 t (d)

where A is the area under the DX-t Fic. 154, — Displacement computed
: i from a force-time diagram.
curve between the proper time lim-
its and ¢ is the distance from the centroid of this area to the ordinate at
the later instant.
To find the total displacement in any time interval t, we add the dis-
placement due to the initial velocity of the body. Thus we obtain

$= ot +o AE (15-5)

This result is almost identical with Eq. (10-8) on page 264 which determines
the displacement from an acceleration-time curve. This similarity is to be
expected since a force-time curve can be reduced to an acceleration-time
: W
curve by merely dividing the force ordinate 2X by the mass # of the body

involved.

ILLUSTRATIVE PROBLEM

1509. The force-time curve for a body weighing 161 lb is given in Fig. 15-5.
Assuming that », = 6 ft per sec, find the velocity and displacement after 10 sec,
Solution: The velocity is given by

W
|s2xa= 4 =F o-m|
1 ; 12
a 6 X 2 Pig.nate 161
pa 39.9
est * ¥ i )
ada

vy = 5.2ftpersec Ans.
434 IMPULSE AND MOMENTUM [Chap. XV

The displacement is found from Eq. (15-5). Be careful to take moments about
the later time ordinate, i.e., at £ = 10 sec in this instance. This gives

[e-+ fat]

Sh 4 1 @ )
8 = Se Nae Ss =
a [3x4x10x(F+6) = 6 X 2(1+ 4) 22553 X 6X 4x x \5
(= x $e4 ]

= Yiltn, AWDR.

PROBLEMS
1510. Referring to the force-time curve in Illus. Prob. 1509, determine the 161-lb
body’s velocity and displacement after 6sec. Ans. v = 7.6 ft persec;s = 46.9 ft
1511. A 40-lb body is free to slide on a horizontal frictionless surface. It starts
from rest and is struck three successive blows of 100 lb, 50 lb, and —100 lb at
intervals of 2 sec. Assuming that each blow is constant for 0.1 sec, what is the
velocity and displacement of the body after 8 sec?
Ans. v = 4.025 ft per sec; s = 57.1 ft
1512. A body initially at rest is acted upon by a constant force of 18 lb for 5 see,
after which an opposite force of 12 lb is applied. In what additional time will the
body come to rest? In what additional time will it return to its initial position?
Ans. t= 7.5 sec; t = 17.18 sec
1513. A 322-lb body moves under the action of a force given by the relation
P = 18 — 3¢, where P is in lb and ¢ in sec. If the body starts from rest, in how
many seconds will it return to its initial position? What is then its velocity?
1514. A 322-lb body moves under the action of a variable force P according to
the law P = 12¢ — 4, where P isin lb and tinsec. If the body starts from rest,
determine its velocity and displacement after 8 sec.
Ans. v = —29.9 ft per sec; s = —34.1 ft
1515. The body in Fig. P-1515 is moving leftward at 4 ft per sec when P = 6 @
is applied for 4 sec and then removed. P isin pounds and tinseconds. Find v and s
at 6 sec after P is first applied.
Art. 15-6] Dynamic Action of Jets 435

1516. In Fig. P-1515, P = 10 ¢ — @ where P is in as P


pounds and ¢ in seconds. How far has the 322-lb body 322
lb
traveled from rest before it starts to reverse its direc- | Kah
tion of motion? Ans. s = 140.8 fi Roath
Fic. P-1515 and P-1516.
1517. A 150-lb body is at rest on a horizontal floor
for which the coefficient of static and kinetic friction is 0.20. A horizontal force of
P = 15 tis applied for 6 sec and then removed. P is in pounds and ¢ in seconds.
How far will the body be from its initial position when it comes to rest?

15-6. Dynamic Action of Jets


The force produced by a jet of water or other fluid impinging upon a
stationary object is easily obtained by applying the concept that the com-
ponent of resultant impulse is equal to the change in the corresponding
components of momentum discussed in Art. 15-3.
Although general formulas can be developed, it is preferable to consider
such problems as special cases of the impulse-momentum method. The
procedure is illustrated by the following problems:

ILLUSTRATIVE PROBLEMS
1518. A jet of water 2 in. in diameter issues from a nozzle with a velocity of
100 ft per sec, and impinges tangentially upon a perfectly smooth stationary vane
which deflects it through an angle of 30° without loss of velocity, as shown in Fig.
15-6. What is the total force exerted by the jet upon the vane?
Solution: Since it is more convenient to treat the jet of water as the free body,
we represent the effect of the stationary vane upon the jet by means of its compo-

‘¥¢

v=100 ft/sec

oe sae
30°
v=100 ft/sec
—EE

a i Sata p:
2l"dia -
a
Fic. 15-6. — Force exerted by a stationary vane upon a jet.

nents P, and P,. These forces are equal but opposite to the effect of the jet upon
the vane. W represents the total pounds of water passing over the vane in ¢ seconds.
W = wAv,t, where w is the density of water (taken as 62.5 lb per cu ft for fresh
436 IMPULSE AND MOMENTUM [Chap. XV
ha cde n Seek a
Se
water), A is the cross-sectional area of the jet in sq ft and v, is the velocity of the
jet in feet per second. Hence

WW _ wAvetwAro W
ae eee = 4.23 tsl
E 9 ] 7 oo ey) a per
Taking X and Y axes as shown (considered positive in the direction of motion),
and applying the condition that the X component of resultant impulse equals the
change in X components of momentum, we have
;
|2x ‘t= * i; = re) | —P,-t = 4.23 ¢(100 cos 30° — 100) P, = 56.7 |b

Similarly, considering the Y components of the impulse-momentum equation,


we have

Ba 2h = = My - re) | P,-t = 4.23 t(100 sin 30° — 0) Py = 212 Ib

The resultant force is found from

ppowaes: P| P = 56.7 F 212 P=219lb Ans.


A similar result can be obtained graphically by plotting the vectors in the impulse-
momentum equation, 1.e,
Ww ie Ww
|Re= V3 — U9 = »#(-7.,)] R-t = 423 ¢ 4 (— 423 #)
g g g g
R = 423 4 (— 423)
- 423 lb This vector equation is plotted in Fig. 15-7,
whence by scaling
R=219lb Ans.
NY
R se 1519. Consider that the vane in Illus. Prob.
1518 is moving to the right at a velocity u = 40
30° ft per sec; all the other data remain un-
Pret 157, changed. Determine the total force exerted
by the jet upon the moving vane.
Preliminary: Usually the deflecting vane is not stationary but is moving in the
same direction as the jet with a smaller velocity of say wu ft per sec. In this case,
W’, the total pounds of water passing over the moving vane, is expressed by
wA(v, — u)t, where (v. — u) is the relative velocity with which the water approaches
the vane. In terms of W, the total pounds of water issuing from the nozzle, we also
U
have Wee wi 22: )so that the mass of water whose momentum is being

changed is found from either


WENT WAOh 2) eeG25 (nw X 2?
Ja00 — 40)t = 2.54 ¢ slugs
gon g 32.2 \4 x 144
Art. 15-6] Dynamic Action of Jets 437

or, using the result of the preceding problem,


W Ww ae - *)
; = ; Se ze = 4,23 3t ¢ |\——___—__]
100 = 2 .54¢t slugs Check

The final absolute velocity of the water v is the vector sum of the velocity relative
to the vane (i.e., v. — u) plus the absolute velocity wu of the vane itself. The compo-
nents of v can be easily obtained from the vector addition in Fig. 15-8b.

Velocity
relative to vane

(a) (b)
Fic. 15-8. — Jet impinging upon a moving vane.

Solution: Taking axes as shown in Fig. 15-6, we obtain


WwW’
|2x “t= 5 (vz — re) | —P,-t = 2.54 t{[(100 — 40) cos 30° + 40] — 100}

—P,-t = 2.54t(52 + 40 — 100)


i — eZisal

Similarly, the Y component of the impulse-momentum equation gives

|2 ai - (vy — ra) | P,-t = 2.54 t[(100 — 40) sin 30° — 0]


P, = 76.0 )b

As before, the resultant force is found from

[p-vPe+ P| P= V2.3? +763" P=79lb Ans.


Power is developed by the X component of P which acts in the direction of
motion of the vane. The power developed is found from

F-v 20.3 X 40
oA, pietille = ee hp = 1.48 Ans.
E | PETE J iS
This result is the power developed by one vane. If a series of moving vanes are
used in which the next vane intercepts the jet just after the first vane has moved
out of the way, all the water from the jet is utilized, and we should use wAv,t instead
of wA(v, — u)t in the preceding computations. Doing this changes the power
developed to

seam Af aa
MAO Eg 60
aR = 2.47 Ane.
438 IMPULSE AND MOMENTUM [Chap. XV
nnn.—

PROBLEMS

1520. What force would a 2-in. jet of water flowing at the rate of 2 cu ft per sec
exert on a stationary plate set at right angles to the jet? Ans. P = 356 lb

1521. A 2-in. diameter jet of water (w = 62.5 lb per cu ft) moving with a velocity
of 100 ft per sec impinges tangentially upon a smooth stationary vane which deflects
it through 180°. Find the total pressure of the water on the vane, and the theoretical
horsepower available in the jet. Ans. P = 843 lb; hp = 38.45

1522. <A jet of water flowing at 161 Ib per sec and moving with a velocity of
100 ft per sec impinges tangentially upon a smooth stationary vane which deflects it
through 135°. Find the total force exerted on the vane.
1523. A machine gun fires a steady stream of 0.5-oz bullets at the rate of 500 per
min against a stationary target at right angles to the direction of fire. The velocity
of the bullets is 2000 ft per sec. Compute the average force on the target.
IMS Io = IES Ilo

1524. A tank weighing 1000 lb rests on platform scales. It is being filled with
water from a vertical jet having a velocity of 150 ft per sec and a cross-sectional
area of 0.2 ft?.. What will be the total scale reading after 5sec? Ans. 19,100 1b
1525. A chain 80 ft long that weighs 10 lb per ft is suspended vertically with its
lower end just touching a platform. If the chain is dropped, determine the impulsive
force exerted on the platform at the instant the upper end has dropped 60 ft.
AWTS, IP = AV lio

1526. A jet of water flowing at 64.4 lb per sec and moving rightward at 100 ft
per sec is deflected through 120° by a curved vane moving leftward at 30 ft per sec.
What force is exerted on the vane?

1527. A stream of water flowing at 96.6 lb per


sec and moving rightward at 90 ft per sec strikes
the vane shown in Fig. P-1527. If half of the water
flows across each part of the vane, determine the
force required to hold it in place.
Ans. P = 457 lb
1528. If the vane of Prob. 1527 is moving right-
ward at 40 ft per see and one-third of the water
Z flows over the upper portion while two-thirds flows
Fia, P-1527 and P-1598, 2°T88 the lower portion, determine the power de-
veloped. Ans. 19.1 hp
1529. A 12-in. pipe carries water with a velocity of 10 ft per sec around a 60°
elbow. The pressure intensity at both ends of the elbow is taken as 40 Ib per sq in.
Compute the resultant force exerted by the water on the elbow. Neglect the weight
of the water in the elbow. Ans. P = 4680 lb
Art. 15-7] Conservation of Momentum 439

15-7. Conservation of Momentum


Let us write the differential equation from which the fundamental im-
pulse-momentum equation was derived in the following form:

fr-a= fa = fav
in which dU represents differential momentum. Observe that if R is zero
during any time interval, this equation reduces to fdU = 0 and therefore
the total momentum U must be zero or a constant.
The condition that U be zero merely implies a static condition involving
uniform motion or rest; it arises when a condition of static equilibrium
exists in a system of particles. The condition that U (the momentum of a
system of particles) be constant arises when the resultant is zero because of
mutual action and reaction between the particles composing the system.
Although the total resultant force on the system may be zero, this does not
mean that the force on any single particle is zero. For example, when a
shell is discharged from a gun, the force acting on the shell is always equal
and opposite to the force acting on the gun. The resultant force acting on
the system composed of shell and gun will be zero although a propulsive
force is acting on both shell and gun if either is considered as a free body.
The condition that the momentum be constant applies only to the system as a
whole, never to its component parts. This is known as the principle of conserva-
tion of momentum.
Therefore, if a system is composed of particles of weight Wi, Ws, etc.,
having velocities v1, v2, etc., and after a mutual reaction between the
particles they possess new velocities 1’, v2’, etc., the condition that the
momentum of the system be constant is expressed by means of the follow-
ing vector equation:

se er mmm oe War ne 2 (15-6)


g g g g
Since the direction of each particle in the system is unspecified and since
momentum is a vector quantity, the addition of the momenta must be
vectorial. However, if all the particles move in the same direction, Eq.
(15-6) becomes an algebraic summation. For example, the tip-to-tail
addition of the X components of a force system is a vector summation re-
duced to an algebraic summation.

ILLUSTRATIVE PROBLEMS

1530. A 160-lb man, moving horizontally with a velocity of 10 ft per sec, jumps
off the end of a pier into a boat which is at rest. If the boat weighs 640 lb, what is
440 ImMpuLSE AND MoMENTUM [Chap. XV

the final horizontal velocity of the boat and the man? Neglect the resistance of
the water.
Solution: Since the boat and the man exert mutual forces on each other, the
principle of conservation of momentum may be applied to the system of particles
constituting the man and the boat. The final velocity of the boat and man will
be the same; hence, applying Eq. (15-6), we have _
W zs ei ae W 2) 160 640 160 + 640
| 32.0 < 10 + 395 X9= “355
v=2ftpersec Ans.
The positive value of the final velocity indicates that the direction of the man
and the boat agree with the direction of his initial velocity.
Observe that the gravitational constant g is aeconstant factor, and hence may be
canceled out in numerical computations.
1531. Solve the preceding problem if the boat has an initial velocity of 3 ft per
sec toward the pier.
Solution: In this case the initial velocity of the boat is opposite to that of the man
and hence is considered negative when we apply the principle of conservation of
momentum.

Eeoon Ms 2 W2) “|S 640 160 + 640


x 10 + 359 (- ay eae = Xv
32.2
5
v = —(.4 ft per sec
The minus sign indicates that the final velocity is directed toward the pier.
1532. A ballistic pendulum consists of a sand box weighing 59 lb that is sus-
pended from a cord 10 ft long. A 1-lb shell is fired horizontally into the box and
remains embedded in it. Because of impact, the sand box swings through a maxi-
mum angle of 30°, as shown in Fig. 15-9. Determine the velocity with which the
shell strikes the box.
Solution: The initial velocity of the sand box with the shell embedded is found

aN
oan

irre =10-10 cos 30°


Art. 15-7]
eConservatie
on of Momentum 44]
by the work-energy method. The work done is negative because the gravity force
acts opposite to the upward rise.
Ww 2 W
f oar
= = > ee
ve) | =
W(10 — 10 cos 30°) = —7 (0 — »')
°

0 — 164.4 le 4. Vo = 9.3 ft per sec


This value of velocity represents the common velocity of the shell and box directly
after impact. The velocity of the shell before impact can be found by applying the
principle of conservation of momentum.
fae Wo. WitW: | EiAg Dong penne x 9.3
g g i] g g
vs = 558 ft persec Ans.

PROBLEMS
1533. A 2000-lb shell is fired from a gun weighing 300,000 lb. If the muzzle
velocity of the shell is 1500 ft per sec and the recoil of the gun is checked by a nest
of springs having a modulus of 2000 lb per in., what is the maximum recoil distance?
INS, 8 = (OR Ty
1534. A 1000-lb shell is fired from a 200,000-lb cannon with a velocity of 2000 ft
per sec. Find the modulus of a nest of springs that will limit the recoil of the cannon
to 3 ft.
1535. Direct central impact occurs between a 20-lb body moving with a velocity
of 10 ft per sec and a 30-lb body moving in the opposite direction with a velocity
of 6 ft per sec. The 20-lb body rebounds in the opposite direction with a velocity
of 5 ft persec. Compute the amount and direction of the velocity of the 30-Ib body.
Ans. v = 4 ft per sec

Perfectly smooth surface


Fig. P-1536.

1536. The spring shown in Fig. P-1536 has a normal length of 12 in. It is com-
pressed to half its length and the blocks are suddenly released from rest. Determine
the velocity of each block when the spring is again 12 in. long.
Ans. vs = +6.6 ft per sec; v3 = 11 ft per sec

1537. A cannon is mounted on one end of a railroad car and a rigid target is
mounted on the other end. The car is free to roll without resistance on a level track.
A shell is fired from the gun when the car is at rest. Discuss the motion of the car
and what happens when the shell hits the target. Assume that the shell is a solid
ball and that the car is frictionless.
442 IMPULSE AND MoMENTUM [Chap. XV

1538. A wooden pile that weighs 500 lb is driven into the ground by successive
blows of a hammer weighing 1000 lb that falls freely through a distance of 6 ft upon
the head of the pile. The average resistance to penetration is 4000 lb. How far
does a single blow of the hammer drive the pile into the ground? Assume that the
hammer and pile cling together after impact. (Hint: Resultant work done equals
kinetic energy lost in impact.) Ans. s = 1.6 it
1539. A 1200-lb hammer falling freely through 3 ft drives a 600-lb pile 6 in.
vertically into the ground. Assuming the hammer and pile to cling together after
impact, determine the average resistance to penetration of the pile.

15-8. Elastic Impact


When two elastic bodies collide, they are deformed at first; then they
spring apart because of the action of restoring elastic forces. Throughout
this elastic impact, there exist mutual action and reaction forces. Hence
the equation of conservation of momentum may be applied here to express
one relation between the unknown final velocities of the bodies. Another
relation between them is obtained from the definition of the coefficient of
restitution. The coefficient of restitution (symbolized by e) is defined as
the ratio of the relative velocities of colliding bodies after impact to their
relative velocities before impact. The relative velocities are measured
along the line which is the common normal to the bodies.
As an example, assume direct central impact of the bodies shown in
Fig. 15-10a. Body W, is overtaking body We»; hence the relative velocity
v; Ug U, U2
eee ee a —_ ——"

————— cane ieee ee


V) U3 vi Us
(a) (b)
Fic. 15-10. — Two extreme cases of direct central impact.

before impact is v; > v2, where — denotes vectorial subtraction. After


impact, assume that the bodies continue moving in their original direction.
In order for them to separate, body We. must have a larger final velocity
than W,; therefore the relative velocity after impact is v2’ — vy’.
Figure 15-10b shows another extreme case of direct central impact in
which the bodies approach each other before impact and have opposite
velocities after impact. In this case, the relative velocity before impact will
remain the vector difference v; — v2, which corresponds to the arithmetical
sum of the magnitudes of the velocities. Consequently, since the sign of v2 is
minus (i.e., opposite to v; assumed as plus)
Art. 15-8] Elastic Impact 443

Vj Ve = V1 — (—by) = v1 + w%

Similarly the relative velocity after impact is also given by

Ve’ > vy = v9 — (—04') = v9! + vy!

A general equation defining e can therefore be given by


Relative velocity after impact — v,' —> v,’
~ Relative
velocity before impact v,> % ee)
In direct central impact in which the velocities of both bodies are directed
along the same straight line, Eq. (15-7) is the algebraic difference of the
respective velocities. For oblique impact, the velocities in Eq. (15-7) must
be the components of velocities along the common normal of the colliding
bodies. Note that the components of velocities perpendicular to the com-
mon normal are unchanged by the impact if the bodies are smooth; that is,
no forces act perpendicular to the common normal.
The coefficient of restitution is really a measure of the elastic properties
of a body. For example, perfectly elastic bodies will have exactly the same
relative velocity after impact as before; that is, the value of e will be
unity. Perfectly inelastic bodies (those which cling together after im-
pact) will have the same final velocity and the value of e is obviously zero.
The coefficient of restitution will therefore always lie somewhere between
the limits of zero and unity. Values of e for different materials may be
found in any standard engineering handbook.
Imperfect elastic action is due to the internal frictional forces developed
in the impacting bodies. The effect of these forces is manifested by a small
rise in temperature caused by the loss in kinetic energy. The rise in tem-
perature of a hammer repeatedly struck against an anvil demonstrates this.

ILLUSTRATIVE PROBLEMS
1540. A ball dropped from a height of 10 ft upon a stone pavement rebounds to
a height of 8 ft. What is the coefficient of restitution between the ball and the
pavement?
Solution: The initial velocity with which the ball strikes the pavement is found
from the kinematic equation
v=0, +2gh or 1 =V2gh
Similarly, the velocity of rebound is given by
oy! = +/2gh’
Taking the direction of 1; as positive and applying the definition of the coefficient
of restitution, we have
444 IMPULSE AND MoMENTUM [Chap. XV

Vol — v;! CS (=v 20h) h’


[e- 2a" | = v2gh —0 ae h

e= 4) = 0.894 Ans
1541. The 10-lb and 20-lb bodies
sede USsiak are approaching each other with the
velocities shown in Fig. 15-11. Ife =
0.60, what will be the velocity of each
body directly after impact?
v,=40 ft/sec v,=10 ft/sec
Solution: Applying the equation of
Fie. 15-11. conservation of momentum with the
factor g canceled out gives
[Wiwib W v2 = Wi’ bd Wv!] 10 x 40 — 20e< (—10) = 10) vy! + 20 vo!

or
vi’ + 20,/ = 20 (a)
From the definition of coefficient of restitution,
[vo
> v1! = e(v1 > %)] vl — vi’ = 0.6[40 — (—10)]
or
vo) — v:/ = 30 (b)
Solving Eqs. (a) and (6) yields
v)’ = —13.33 ft per sec
s ‘ |= +16.67 ft per a BUS.

The minus sign before v;’ indicates that the 10-lb body rebounds to the left after
impact.

PROBLEMS
1542. Direct central impact occurs between a 60-lb body moving rightwards at
10 ft per see and a 30-lb body moving leftwards at 20 ft per sec. If the coefficient
of restitution is e = 0.6, what is the average impact force for a time of contact
lasting 0.02 sec? Ans. F = 1490 lb
1543. Direct central impact occurs between a 100-lb body moving to the right
at 5 ft per sec and a body of weight W moving to the left at 3 ft per sec. The co-
efficient of restitution e = 0.5. After impact the 100-lb body rebounds to the left
at 2 ft per sec. Determine the weight W of the other body. Ans. W = 140 1b
1544. A golf ball is dropped from a height of 20 ft upon a hardened steel plate.
The coefficient of restitution is 0.894. Find the height to which the ball rebounds on
the first, second, and third bounces.
1545. The balls A and B in Fig. P-1545 are attached to stiff rods of negligible
weight. Ball A is released from rest and allowed to strike B. If the coefficient of
restitution is 0.6, determine the angle @ through which ball B will swing. If the
Art. 15-9] Fundamental Impulse-Momentum Equation for Rotation 445

impact lasts for 0.01 sec, also find the


average impact force. , lee
Ans. -@ = 65°:.P: = 1070 Ib 60°
1546. The system shown in Fig. P- ee
1545 is used to determine the coeffi- W,=301b I
cient of restitution. If ball A is released i wed
from rest and ball B swings through o 1
6 = 53.1° after being struck, deter- Ns Ws =20 1b
ery Fic, P-1545 and P-1546.
1547. A ball is thrown at an angle @
with the normal to a smooth wall, as shown in
Fig. P-1547. It rebounds at an angle 6’ with
the normal. Show that the co-efficient of res-
tan 6
titution is expressed by e =
tan6’
1548. A ball is thrown with a velocity of 50
ft per sec directed at 60° with the horizontal
against a smooth vertical wall. The ball is re-
leased from a position 40 ft from the wall and 6
Fie. P-1547. ft above the level ground and travels in a ver-
tical plane. The coefficient of restitution be-
tween ball and wall is 0.6. How far from the wall does the ball strike the ground?
ANS —S.anG
1549. As shown in Fig. P-1549, a 40-lb ball moving horizontally to the right
with a velocity of 8 ft per sec, collides
obliquely with a 30-lb ball moving up W. =40 lb
to the left at 30° to the horizontal at : W2= 30 Ib
10 ft per sec. If the coefficient of res-
titution is 0.6, determine the amount
v,=8 ft/sec 30°
and direction of the velocity of each
ball directly after impact.
Ans. vs» = 3.43 ft per sec to left; v2=10 ft/sec
v3 = 8.22 ft per sec up to Fia, P-1549.
right at 37.3° with horizontal
1550. Two bodies of masses M, and M, moving in opposite directions with
velocities v; and v» collide. If ¢is the coefficient of restitution, show that the energy
loss in direct central impact is given by
1 MM,
Loss in KE = (T= 20, — 0)"
2M,+ M,
15-9. Fundamental Impulse-Momentum Equation for Rotation
Proceeding as in the case of linear impulse-momentum (Art. 15-2), we
shall eliminate the angular acceleration a from the following equations:
446 ImMpuLsE AND MoMENTUM [Chap. XV

2M = Ta
_ de (a)
am
This gives
2M -dt = I dw (b)
which is the differential form of the impulse-momentum equation for rota-
tion. If we assume that the angular velocity has the value w, when ¢ is
zero and the value w at any other value of t, integration of Eq. (6) between
these limits gives the general form of the impulse-momentum equation for
rotation, viz.,
t
[sm -dt = Iw — Ia, (15-8)
0
When the resultant moment
2M is constant, this equation becomes
=M -t = Iw — Io, = I(o — w) (15-9)
t

Resultant Angular Impulse. The expression f YM - dt is called the re-


0

sultant angular impulse. Examining the expression dimensionally shows


that the unit of angular impulse is ft-lb-sec.
If 2M is variable, the resultant angular impulse can be found if 2M can
be expressed mathematically in terms of time ¢, or if a graph can be obtained
showing its variation with time. In the latter case, the resultant angular
impulse is equivalent to the area included between the proper limits of a
~M-t diagram. In many cases, however, 2M is constant and the resultant
angular impulse is expressed simply as 2M - t.
It is also important to note that the expression consists of a vector
quantity (i.e., 27) multiplied by a scalar quantity (.e., dt). This results
in a new vector having the same sense and inclination as 2M but differing
in magnitude. In other words, angular impulse is a vector quantity (de-
fined by the right-hand rule) and may be resolved into X, Y, and Z compo-
nents when desired.
Angular Momentum. The expression Jw is defined as angular momentum
and is also expressed in units of ft-lb-sec. The right-hand term of Eq. (15-8)
represents the difference between the final and initial angular momenta and
is called the change in angular momentum. The change in angular mo-
mentum depends only upon the mass moment of inertia and the final and
initial angular velocities (J is assumed constant; for J varying, see Art.
15-10). Therefore, whether the motion has variable or constant accelera-
tion, the change in angular momentum is independent of it.
Likewise, as in angular impulse, it is important to note that the expres-
sion Jw consists of a vector w multiplied by a scalar coefficient J. This
Art. 15-11] Angular Impulse-Momentum Applied to Rotation 447
ES SE ei ct Sel lel
results in a new vector having the same sense and inclination as w but
differing in magnitude. The vector of angular momentum (governed by
the right-hand rule) may also be resolved into X, Y, and Z components
when desired.
The vector properties of angular impulse and angular momentum are
usually of little importance in cases that involve rotation about a fixed
axis. However, where there is rotation about variable axes of rotation, as
in a gyroscope, these properties are of considerable value. (See Art. 15-14.)

15-10. Rate of Change of Angular Momentum


A more general interpretation can be given to the fundamental impulse-
momentum equation by defining the angular momentum Jw at any instant
by H. Equation (6) in Art. 15-9 may be written
2M - dt = dH = d(Iw)
or

=M = d(Io) (15-10)
This states that the resultant moment is equal to the rate of change of
angular momentum.
This equation is especially useful when J varies; if J is constant, the
equation at once reduces to
du
2M =17 = Ia

15-11. Angular Impulse-Momentum Applied to Rotation


The derivation of the impulse-momentum relations for rotation is exactly
the same as for translation; hence all of the equations previously developed
for translation may be written for rotation by merely replacing linear terms
by their corresponding angular terms. In particular, the rotational counter-
part of Eq. (15-5) is
@ = wt + 5(Area) yf (15-11)
which enables us to compute the angular displacement in terms of the area
under a moment-time curve.
It is essential to notice an important difference between work-energy and
impulse-momentum methods. It was possible to apply the work-energy
equation directly to a system of translating and rotating bodies, thereby
eliminating internal forces, because the terms involved were scalar quan-
tities expressed in the same units which could be added arithmetically.
This procedure is not possible with the impulse-momentum equations. The
linear impulse-momentum terms and the angular impulse-momentum terms
448 IMPULSE AND MoMENTUM en [Chap. XV
faked een reaseninnein innit eerie
are not scalars; they are vector quantities having different directions and
units. To combine them, it is necessary to transform the linear vectors into
angular vectors. Usually this transformation is accomplished automatically
if the impulse-momentum equations are applied to each separate body of the
system as is discussed in the following illustrative problems.

ILLUSTRATIVE PROBLEMS

1551. A pulley of 2-ft radius is free to rotate about its centroidal axis. Its cen-
troidal mass moment of inertia is 10 ft-lb-sec?. A cord wrapped around the pulley
is fastened to a 200-lb block. The initial velocity of the block is 20 ft per see up-
ward. Determine the linear velocity of the block after
4 sec.
Solution: A FBD of the system is shown in Fig.
15-12. The directions of impulse and momentum are
chosen positive in the initial directions of motion. As-
sume that the final velocity (and hence the final mo-
mentum) corresponds in direction with the initial
velocity. A positive result will confirm this assump-
tion; a negative result will indicate that the vector
q direction has been incorrectly assumed, and that the
final velocity is opposite to the initial velocity.
From the kinematics of rotation (Art. 12-2) we
have
u,=20 ft/sec [v = ra] 20 326, w@> = 10 rad per sec
W= 200 lb salvo ‘ele
Fic. 15-12. 2
Substituting in Eqs. (15-4) and (15-9), we obtain

Ww 200
fZX -t=—(v—v,
7 (v —v | (T ~— 200) x 4 = ——
39,9 (py
(v —— 20) (a)

[2M -t = I(w — w,)] (-2T) x 4=10 (:= 10) (b)


Multiplying Eq. (a) by 2 and adding the equations eliminates the tension and
gives
v= —71.8ftpersec Ans.
The minus sign indicates that the final velocity is directed downward.
Shorter Method of Solution. If we consider the pulley and disk as a system of
particles, we can apply the principle of angular impulse-momentum directly. The
tension 7’, being an internal force, occurs as an equal opposite pair of forces in the
system, and hence cancels out. The resulting conclusion may be stated as follows:
The resultant angular impulse of all external forces about the fixed axis of rotation is
equal to the resultant change in angular momentum of the system.
Art. 15-11] Angular Impulse-Momentum Applied to Rotation 449

In the present problem, the angular momentum of the block is the moment of
its linear momentum about O, and the angular impulse of the external forces on
the block is the moment of linear impulse also taken about O. If the moment arm
of linear impulse and momentum about the fixed axis of rotation is denoted by r,
this principle can be expressed in equation form as

YEX- r+ SEM 4 = >To —») 7+ Silo — 0) (16-12)


Applying this equation to the external forces in Fig. 15-12 yields
200
(v — 20) x2+10(% 10]
whence, as before
v = —71.8 ft per sec
1552. The initial velocity of the block
in Fig. 15-13 is 20 ft per sec downward.
The coefficient of kinetic friction at the
brake is 0.25. The centroidal mass mo-
ment of inertia of the drum is 10 ft-lb-
sec?. Find the time required for the
system to come to rest after the brake
is applied.
Solution: For the forces acting on the
brake lever, 2M4 = 0, or N = 400 lb.
Hence
F =f,N = + X 400 = 100 lb
We may now apply the principle that
the moment of impulse of external forces |v,=20 ft/sec
equals the total change in angular mo-
mentum. Fic. 15-18.

[ex-p-r+2m-1= 2 —2)-r+ To) |


( 4) X 22 —
(100 ¢100 x« 4)
4 X pois
= 309 (0 — 20) X 2+ 10 x (0 — 10)
#= 1.121 sec Ans.

1553. A variable force P is applied tangentially to the outside of a disk of 2 ft


radius constrained to rotate about its centroidal axis. The centroidal mass moment
of inertia of the disk is 10 ft-lb-sec?. The force P varies according to the relation
P = 32, where P is in pounds and ¢ in seconds. Compute the angular velocity of
the disk in rpm after 2 sec, starting from rest. What is then the angular displace-
ment in rev?
Solution: Applying Eq. (15-8), we have (2M variable)
450 IMPULSE AND MoMENTUM [Chap. XV
POU EEE a

[feed = eo | [sex2u- 10(w — 0)


w = 1.6 rad per sec

@ = 1.6 X y= = 153 rpm Ans.

We may also use the moment-time curve shown in Fig. 15-14. Noting that
t

ifDM - dt is equivalent to the area under this curve, we obtain


0

M t
[Jew a=r60- 29]

4x
24 X 2 = 10— 0)
w = 1.6 rad persec Check

|< 9 sec—>| ; By using Eq. (15-11) and measuring the moment -


Fic. 15-14. —Moment-time 2'™ / from the later time ordinate, the angular
: curve. displacement is

1 1 1
&= wt + 7
= (Area)ar-ae i| Ga) + al Se DYE SK 2)
e x 2)|

(i) = BO mal = oe =(0.5lrev Ans.


27

PROBLEMS

1554. A solid cylinder 4 ft in diameter and weighing 644 lb rotates on its cen-
troidal axis at a speed of 100 rpm. Assuming that the moment of the friction on
the bearings is 24 in.-lb, find the force which, if applied tangentially to the rim, will
increase the speed to 300 rpm in 30 sec.
If the accelerating force is removed
when the speed reaches 300 rpm, what
f,=0.20 time will elapse before the cylinder
comes to rest?
Ans. P = 14.95 lb; t = 628 sec
T=10 ft-lb-sec?
1555. Find the value of P that will
cause the 200-lb block in Fig. P-1555 to
pY reach a velocity of 20 ft per sec after
Fig. P-1555. moving from rest for 5 sec.
INS IPS TAS NG
1556. A system of weights and pulley has the data given in Fig. P-1556. Find
the time required for the pulley to reach a clockwise speed of 500 rpm, starting
from a counterclockwise speed of 100 rpm.
Art. 15-12]
a es Conservat
Na ion of Angular Momentum 451

T=100 ft-lb-sec? I =10 ft-lb-sec?

Fig. P-1556. Fig. P-1557.

1557. The drum in Fig. P-1557 is rotating at 10 rad per sec at the instant the
brake is applied. What is the friction force developed at the brake in order to stop
the system in 5 sec? Aupis, 10 = (Bil Mle
1558. A pulley of 2.5-ft radius is rotated about its centroidal axis by a tan-
gentially applied force P = 24t — 4# where P is in pounds and ¢ is in seconds. If
the pulley has a centroidal mass moment of inertia of 12 ft-lb-sec?, find the number
of radians through which the pulley rotates from rest before starting to reverse its
direction. Also find its angular velocity when it returns to its starting position.
‘' Ans. 6§ = 151.7 rad; w = —120 rad per sec
1559. Repeat Prob. 1558 changing the radius of the pulley to 3 ft and the value
of the tangentially applied force to P = 6 # — 0.5 # where P is in pounds and t is
in seconds.
1560. A wheel 6 ft in diameter is rotated about its centroidal axis by a tan-
gentially applied force P = 40 + 6 # where P is in pounds and ¢ is in seconds. The
rotation is resisted by a brake (f = 0.40) which exerts a constant normal pressure
of 100 lb on the rim. If P acts for 6 sec and is then removed, through how many
revolutions will the wheel turn before stopping? Assume the centroidal mass mo-
ment of inertia is 6 ft-lb-sec?. Ans. @ = 238 rev

15-12. Conservation of Angular Momentum


Let us rewrite the differential equation from which the fundamental
angular impulse-momentum equation was derived in the following form:
t a)

[ema= fra. (a)


From this equation we see that if, during a rotation, 2M is balanced (ie.,
2M = 0), we will obtain
i Idw =0 (b)

Therefore the angular momentum must be zero or constant.


452 IMPULSE AND MoMENTUM [Chap. XV

The condition that the angular momentum be zero implies a static con-
dition involving either no rotation or rotation at a uniform velocity; it
arises when a condition of static equilibrium exists. The condition that the
angular momentum be constant arises when the resultant moment is zero
because of mutual action and reaction between the particles or bodies com-
posing the rotating system. This condition applies only to the system as a
whole, never to its component parts. This is known as the principle of con-
servation of angular momentum.
Therefore, if a system is composed of bodies of inertia J;, /2, etc., having
velocities w1, w:, etc., and if after a mutual reaction between the bodies they
possess new velocities w1’, ws’, etc., the condition that the angular momen-
tum of the system be constant is expressed by the following equation:
ho, b he, Pb +++ = ho’ b he.’ p++: (15-13)
Since the axis of rotation of each of the bodies is unspecified and since
angular momentum is a vector quantity (directed along the axis of rotation),
the addition of the angular momentum must be vectorial. However, if the
bodies are rotating about the same or parallel axes, Eq. (15-13) becomes an
algebraic summation.
The principle of conservation of angular momentum can also be applied
to a translating body striking a rotating body. In this case the linear
momentum of the translating body is multiplied by its moment arm about
the axis of rotation when we substitute in the equation.

ILLUSTRATIVE PROBLEM
1561. The shafts on which disks A and B rotate can be connected by a clutch as
shown in Fig. 15-15. Disk A rotates at 2 rad per sec counterclockwise, and disk B
rotates at 6 rad per sec clockwise. The mass moment of inertia of disk A is 4 ft-lb-
sec? and for disk B it is 8 ft-lb-sec?. If the clutch suddenly fastens both shafts to-
gether, compute the final angular velocity of the system directly after being joined.
Clutch

w=2 rad/sec w=6 rad/sec

Counterclockwise Clockwise

Fig. 15-15.

Solution: Since there is mutual action and reaction between the disks, we may
apply the equation of conservation of angular momentum. If we assume the angular
velocity of A to be positive, We obtain
[Lio H Tow. = Ui + I2)o] 4X2-8xX6= (4+ 8)w
= —3.33 rad persec Ans.
The minus sign indicates that the final rotation corresponds with that of disk B,
ie., clockwise.
Art. 15-12] Conservation of Angular Momentum 453

PROBLEMS
1562. When the speed becomes excessive, the balls in a certain type of governor
are released by a catch, and they move to the places shown by the dotted outlines
in Fig. P-1562. J for the rod is 4 ft-lb-sec?, and
each ball weighs 20 lb. If the speed of the as-
sembly is 20 rad per sec when the governor
operates, what will be the final speed of the
system?
1563. A timber 4 ft long of uniform cross
section weighs 100 lb. It is suspended verti-
cally downward from a horizontal axis. A bul-
let weighing 0.2 lb is fired directly at the center Fig. P-1562.
of percussion of the timber, causing it to
swing through 30° from the vertical. Determine the velocity of the bullet before
impact. Ans. v = 1800 ft per sec
1564. A uniform bar, 3 ft long and weighing 20 lb, is suspended vertically from a
horizontal axis at its upper end. It is struck at 2 ft below this axis by a 2-lb ball
of putty moving horizontally at 60 ft per sec. If all the putty clings to the bar,
compute the loss in kinetic energy. Ans. Loss = 98.6 ft-lb
1565. Determine the maximum angle through which the bar described in Prob.
1564 will swing.
1566. A weight is attached to one end of a cord as shown
in Fig. P-1566. At the position indicated, the weight is ro-
tating about the vertical axis of the hollow support in a
circular path of 4-ft radius at 10 rad per sec. If the cord is
slowly drawn into the support so that the radius of rotation
is 2 ft, what will be the new angular velocity of the weight?
Further, determine and explain the increase in kinetic
energy.
Ans. w = 40 rad per sec
Fie. P-1566. 1567. Two disks A and B are mounted loosely on a hor-
izontal shaft. The mass moment of inertia for disk A is 6
ft-lb-sec?, and for B it is 2 ft-lb-sec?. A spring wound
loosely around the shaft connects the disks. One disk
T =12 ft-lb-sec?
is used to wind up 300 ft-lb of energy in the spring,
after which both disks are released from rest. Neglect-
ing the mass of the spring, compute the angular velocity
of each disk at the instant when the spring has released
all its stored energy to the disks.
Ans. wa = +5 rad per sec;
wp = 15 rad per sec
1568. The 64.4-lb block in Fig. P-1568 is lifted ver-
tically 4 ft and then dropped from rest. Determine the
angular velocity of the disk immediately after the cord
supporting the block becomes taut. What velocity will Fic, P-1568.
454 IMPULSE AND MOMENTUM [Chap. XV
Ne

the block attain if it then drops an additional 3 ft? Neglect any vibration that
may occur.

15-13. Impulse-Momentum Equations for Plane Motion


As was brought out in Art. 9-8 on the motion of the center of gravity, any
system of particles may be considered to be equivalent to a single particle
having the motion of the center of gravity of the system and its mass. This
relation, however, is true only for the relations between the resultant force
and the acceleration of the center of gravity; it does not apply to any rota-
tional motion caused by applied forces.
When applied to plane motion, therefore, the equations of linear impulse-
momentum are exactly the same as those derived in Arts. 15-2 and 15-3 for
translation, viz.,

ss) = T D, (15-1)
or

M ht = | (Cle (15-2)
= >

M Ny Q ~~ I =i=]n
| i=}ae nS
=—~ .—)

The angular impulse-momentum equation for plane motion applies to the


rotation effects caused by the impressed forces. In this respect it is the same
as the equation derived for rotation in Art. 15-9 and may be stated as fol-
lows: The resultant angular impulse is equal to the change in angular mo-
mentum.
The resultant angular impulse in plane motion is the resultant of the
moments of linear impulse about the reference axis.
The expression for angular momentum H4 about any reference axis A,
from which more convenient expressions may be obtained, is derived as
follows:
Choose any base point A, whose velocity v4 is known, as an origin for
reference axes X and Y. The velocity of any particle B in the body is
obtained from the kinematic equation vg = v4 4 (vpy4 = Tw) as in Art.
13-2. The velocities composing vg are shown in component form in Fig.
15-16a. The linear momentum of particle B, oFVg, 1S represented in this
form in Fig. 15-16b.
The angular momentum of the body is obtained from the principle that
Art. 15-13] Impulse-Momentum Equations for Plane Motion 455
e e es
dw

Fig. 15-16. — Components of velocity and of linear momentum of a typical particle B.

the total angular momentum is equal to the moments of linear momentum


of all particles composing the system, viz.,

Ha = | p-—vp (a)

where p is the moment arm (not shown) of the linear momentum Ms Up.
From Fig. 15-16b, taking moments positive in the sense of w and re-
membering that the moment of a resultant is equal to the sum of the
moments of its parts, we have

dw dw dw dw
-—— 0p = —Tw-r
+ —04,°Y — —V4,°2 b
eo ae Eth ©)
Integrating gives

dw dWw W _ dW We
—r=I,, y =—J, and ———Pe
g g g g
we have

Hy, = Inn + es fee dsp x (15-14)


g g
The positive sense of H4 is chosen to agree with w; if w is counterclockwise,
the signs of the last two terms in Eq. (15-4) will be reversed.
Angular Momentum with Respect to Axis through Center of Gravity. If
the reference axis is chosen through the center of gravity, the values of
& and g in Eq. (15-4) become zero, and J4 becomes I. This results in
H =Io (15-15)
456 IMPULSE AND MoMENTUM [Chap. XV

Angular Momentum with Respect to Axis through Instant Center. If the


reference axis is chosen to pass through the instant center C, the values
of va, and v4, in Eq. (15-4) become zero by definition of the instant center,
and J, becomes Jc, giving
Hotee (15-16)

ILLUSTRATIVE PROBLEM

1569. The system shown in Fig. 15-17 is released from rest. Disk A rolls without
slipping. Using the impulse-momentum method, determine the velocity of block B
after 4 sec.
Preliminary: The various mass moments of inertia are first determined as follows:

e e. Wes = ie
i = m k2 Home 39.9 x (1)? = 6 ft-lb-sec?

- Es 96.6
Kor Az : 39,9 x (1.414) me 6 ft-lb-sec
-lb-sec?

(i =1+ Md For A: Ig =6+ 55 X CB)? = 33 ft-lb-sec?

The kinematic relations between the velocities are

[v = rw] Up = 2wp wp = =

UB
or, = 1 X we = 404 Aare)

W=96.6 lb
h=1.414 ft

qT,

| B_ |W=32.21b
UB

32.2 1b

Fig. 15-17.
Art. 15-13] Impulse-Momentum Equations for Plane Motion 457
Solution: Using the methoddiscussed in Illus. Prob. 1339 on page 368, we
find the directions of motion to be as shown. The impulse-momentum equation is
applied to each of the bodies in the system. For block B this gives
W 2.2
|2x cis a (vo — ro | ($2.25 I) ean — as X vB (a)

For the rotating pulley


2M -t = Iw — w,)) @TM%-1XT) xX4=6xF (b)
For the rolling disk, whose motion is equivalent to rotation about the instant
center C, we obtain
ee TS etl (4 T, — 96.6 X 3sin 30°) X 4 = 38 X a (c)
The tensions are eliminated by multiplying (a) by 2 and (c) by + and adding.
This gives
vg = 18.8ftpersec Ans.

PROBLEMS
1570. A cylinder 2 ft in diameter starts up a 30° incline with an angular velocity
of 100 rpm. Assuming free rolling, how soon will it come to rest?
Ans. t = 0.975 sec
1571. The disk shown in Fig. P-1571 rolls freely. The pulley is assumed to be
weightless and frictionless. If the disk is rolling initially to the left with an angular
velocity of 4 rad per sec, in what time will the block reach a downward velocity of
12 ft per sec? Ans. t = 1.705 sec
W=193.2]b

W=193.2Ib
a =1.414 ft

W=96.6 lb
k =1.414 ft

Fig. P-1571. Fiq. P-1572.

1572. In the system shown in Fig. P-1572, assume that the disk rolls without
slipping. Compute the time required for the block to reach a velocity of 20 ft per
sec, starting from rest. Ans. t = 20.7 sec

1573. Solve Illus. Prob. 1569 if the cord in Fig. 15-17 is attached to the disk at
the under side of the drum instead of at the upper side as shown. Assume that the
cord is still parallel to the 30° incline.
458 IMPULSE AND MOMENTUM [Chap. XV
ee —— ———— —— — ————— —

15-14. Gyroscope
The gyroscopic effect is present whenever a body that rotates about one
of its axes also rotates about another axis not parallel to the first one. This
effect appears, for example, in the wheels of a locomotive which is rounding
a curve or in the propeller of an airplane that is making a turn.
The usual concept of a gyroscope is a solid of revolution that rotates
rapidly about its axis of symmetry. Let us consider the case in which the
gyroscope is a disk supported at point O on an axle fastened to it; this is
shown in Fig. 15-18. The disk is spinning with a large angular velocity w
about its axis of symmetry. This axis will henceforth be called the spin
axis.
Y (Precession axis)

: xX
(Spin axis)

(Torque axis)

Fig. 15-18. — Gyroscope.

Although the axle of the disk is free to rotate in any direction about O,
the momentum generated by the spinning disk will not permit it to fall
but will make it rotate about the vertical axis Y. This rotation is known as
precession and the Y axis is called the precession axis.
The external forces, consisting of the weight W of the disk and the verti-
cal reaction R at O, create a torque about the Z axis; this axis is called the
torque axis. Under these conditions, the spin axis, the precession axis, and
the torque axis are mutually perpendicular. Our discussion of the gyro-
scope will be limited to this case. :
The resultant angular momentum of the disk is the vector sum of the
momentum Jw of the disk rotating about its spin axis, plus the momentum
caused by the external torque M@. In the differential time dt, the angular
impulse caused by the external torque is M dt; this produces an equivalent
angular momentum having the same vector direction (see Art. 15-9). Using
the right-hand rule to determine the vector directions of these quantities,
we find that the resultant angular momentum of the disk is inclined at an
angle d¢, as shown in Fig. 15-19. The direction of the resultant angular
momentum vector is the axis about which the disk tries to spin. In order to
spin about this new axis, the disk turns through the angle dé. This causes
Art. 15-14] Gyroscope . 459
ee eee
precession. If M and Jw are constant, the resultant momentum vector
remains inclined at a constant angle with Jw and the disk precesses with a
constant angular velocity Q.

Resultant angular momentum


Mat or new axis of spin (lies in XZ plane)

Fig. 15-19. — Precession.

The angular velocity of precession is easily obtained from the vector


diagram shown in Fig. 15-19. Since d¢ is a small angle whose tangent is
equal to the angle in radians, we have

M dt = Iwdd
or

M _= 7, a¢
loa,

Since 2 = a is the rate of change of angular displacement of the spin


axis, we obtain
M = IQ (15-17)
Generally the rate of precession 2 is small compared with w.
If we attempt to hurry the rate of precession by applying a force in the
XZ plane, we shall merely make the rotating disk and axle rise. This rise
is caused by the angular momentum of the hurrying torque and is repre-
sented by a vector directed along the Y axis. The new resultant momentum

ip New resultant momentum

Momentum
caused by
hurrying torque

Da Previous resultant momentum

Fia. 15-20. — Effect of hurrying torque.

of the system is shown in Fig. 15-20. Similarly a force in the XZ plane


exerted to retard the precession will make the axle fall.
We have seen that a torque about an axis perpendicular to the spin axis
460 IMPULSE AND MoMENTUM [Chap. XV

causes a precession about a third axis perpendicular to the other two.


Conversely, a forced precession about one axis will induce a torque about
a third axis perpendicular to the spin and precession axes. For example,
the wheels of an automobile going around a horizontal curve are forced to
precess about a vertical axis through the center of curvature. This forced
precession causes a torque about a third axis perpendicular to the spin and
precession axes, i.e., about an axis which is tangent to the curve. Because
of this torque, the reaction on the outer wheel is increased and that on the
inner wheel is decreased; these effects are additional to the effects of cen-
trifugal inertia.

ILLUSTRATIVE PROBLEM
1574. A car is rounding a horizontal curve of 200-ft radius at a velocity of 30 ft
per sec. Each wheel weighs 60 lb, is 30 in. in diameter, and has a radius of gyration
of 12in. If the tread (1.e., the distance separating the wheels on the axles) is 5 ft,
compute the change in pressure on each pair of wheels caused by the gyroscopic
effect.
Solution: The moment of inertia of a pair of wheels is found to be
W 2°60 i
|
if = 5 k f2
— I — 39,9 x 12 = 3.73
ie ft-lb-see
-|p- 2

The values of w and Q are

[= | w eeu = 24 rad per sec


r 13

v 30
f ;| 500 0.15 rad per sec

In accordance with the right-hand rule, the direction of the vector representing
the angular momentum of a pair of wheels as the car approaches the observer is
shown by OA in Fig. 15-21. As the car rounds the curve, this vector rotates about
the vertical Y axis to position OB. The change in angular momentum AB must be
produced by an induced torque acting on the car wheels. If we again apply the

Fig. 15-21.
Art. 15-14] Gyroscope 461
eee

right-hand rule and extend our thumb in the direction AB, our fingers will curl
about the vector AB in a counterclockwise sense, thereby indicating that the
gyroscopic reactions on the car wheels are directed as shown.
From Eq. (15-17) we obtain
[M = Io] 5 R = 3.73 X 24 X 0.15 R = 2.69lb Ans.

PROBLEMS
1575. A solid disk 12 in. in diameter weighing 322 lb rotates at 1800 rpm. It is
keyed to a shaft 10 in. long which is supported by a frictionless pivot attached to
the outer race of a ball bearing, as shown in
Fig. P-1575. If the disk rotates in a clock-
wise sense when viewed from the right, de-
termine the angular velocity and sense of
the precession.
Ans. Q = 1.14 rad per sec clockwise when
viewed from above
1576. A pair of locomotive driving wheels
and their axle weigh 4830 lb. Their diameter
is 6 ft and their radius of gyration is 2.5 ft.
Determine the changes in wheel reactions
caused by the gyroscopic effect when the lo- W=322 Ib
comotive is rounding a curve of 3000 ft radi- Fra. P-1575.
us at 60 mph. Assume the rails to be 5 ft
apart. Ans. R = 161 1b
1577. The wheel in Fig. P-1577 weighs 322 lb and has a radius of gyration of
1.732 ft. It rotates about its horizontal axis AB with a constant speed of 200 rad
per sec. Axis AB is mounted in a yoke which
when viewed from above rotates at 1 rad per
sec clockwise about the Y axis passing through
the center of gravity of the wheel. Compute
the forces on the bearings at A and B.
Ans. Ra, = 1161 lb up; Rg = 839 lb down

1578. Compute the magnitude of the force


P acting 2 ft to the right of O in Fig. P-1577
that is required to increase the rate of preces-
sion in Prob. 1577 to 2 rad per sec. In what
direction must it be applied?
Ans. P = 3000 lb acting vertically down

1579. A steam turbine in a ship is mounted


Fic. P-1577 and P-1578. with its axis parallel to the propeller shaft. The
turbine weighs 3220 lb, rotates at 1800 rpm, and
is mounted in bearings 4 ft apart. If it has a radius of gyration of 3 ft, compute
the changes in bearing reactions caused by the gyroscopic effect of the ship mak-
462 ImpuLsE AND MoMENTUM [Chap. XV

ing a turn of 2000 ft radius at 20 knots. (Note: 1 knot = 1.69 ft per sec.) _
Ans. R.= 717Zdb

SUMMARY
er er : : W _ - Oe
By eliminating acceleration from the equations Rk = hy and @ = a

we obtain the relation


t

ler ate aera, (15-1)


0 g g
which is defined as the fundamental impulse-momentum equation.
t

The expression f R-dt (or R- tif R is constant) is known as the result-


0
ant linear impulse. It is a vector quantity having the sense of FR and is
Ser e Wie
expressed in lb-sec. Similarly the quantity mi v is defined as linear momen-
tum. This is also a vector quantity having the sense of 3 and expressed in
Ib-sec.
The component of resultant impulse along any axis is equal to the change
in the components of linear momentum along the axis. This important
principle is expressed as an equation by
t

ie di = Ua - Le , etc. (15-2)
0 g Fiat
When this equation is applied to translation problems in which all particles
have identical values of s, v, and a, the bar signs are unnecessary.
If the acting force is plotted against time to obtain a force-time curve,
the resultant impulse is equal to the area under the curve. The moment of
the area under this curve about the later time instant multiplied by a gives
the displacement of the body in the corresponding time interval (Art. 15-5).
The dynamic action of jets is described from the principle that the com-
ponents of resultant impulse are equal to the change in the corresponding
components of momentum (Art. 15-6).
The principle of conservation of momentum may be applied to any system
of particles acted upon only by mutual reactive forces. This principle is
expressed by the equation

pega = Typ Boa. (15-6)

Problems involving direct elastic impact are solved by means of the prin-
ciple of conservation of momentum in combination with the definition of
the coefficient of restitution (e). The coefficient of restitution is defined
by the equation
Summary 463
Vo! ak v,!
e = ————
See (15-7)
a
In oblique impact, the velocities used in the above equations are the com-
ponents of velocities along the common normal to the colliding bodies.
The impulse-momentum equation for rotation is obtained by eliminating

the angular acceleration in the equations =M = Ja and a = This


results in
t

i SMdi
0
lola, (15-8)
t

The expression 4)=M dt (or =M -t if the moments are constant) is


0
defined as the resultant angular impulse. It is a vector quantity having the
sense of >M and is expressed in ft-lb-see. Similarly the expression Jw is
defined as angular momentum. This is also a vector quantity having the
sense of w and expressed in ft-lb-sec. Components in Eq. (15-8) can be
treated exactly as in the case of linear impulse-momentum to obtain equa-
tions similar to Eq. (15-2).
The rate of change of angular momentum (Art. 15-10) is equal to the
resultant moment at any instant, 1.e.,
d(Iw)
=M =
dt
The princip le of conservation of angular momentum (Art. 15-12) may be
applied to any system of rotating bodies acted upon only by mutual reactive
forces. The principle is expressed by the equation
Te ho. b+ +: = hei’ » her’ »--: (15-13)
When applied to bodies having plane motion, impulse-momentum meth-
ods combine the translational and rotational components of the motion.
The resultant linear impulse is equal to the change in linear momentum,
and the resultant angular impulse is equal to the change in angular momen-
tum.
The angular momentum (H) about any reference axis through a point A
is given by
Hy = yo + Ua Fay (15-14)

This is reducible to simpler expressions with respect to the center of gravity


O by cae
H = Io (15-15)
and with respect to the instant center C, by
He = Ico (15-16)
Chapter XVI.
Mechanical Vibrations

16-1. Introduction. Definitions and Concepts


Vibrations in an elastic structure are caused by disturbing forces which
create a displacement of the structure from its position of static equilibrium.
Such displacements create elastic forces which tend to restore the body to its
original condition of equilibrium. When the disturbing force is removed,
the elastic forces cause the body to accelerate back to its equilibrium posi-
tion. However, the body will now possess some velocity as it passes
through its equilibrium position, and this will cause it to overshoot. Thus
vibrations of it are set up which may or may not diminish, depending upon
whether any resisting forces are present.
When these vibrations are caused by an initially applied force which is
then removed from the body, there result what are known as free vibrations.
If resisting forces also act on the vibrating body, the motion is known as a
damped free vibration. When the disturbing force continues to act at
periodic intervals upon the body, the result is a forced vibration which may
or may not be damped.
Generally speaking, a vibration is a periodic motion which repeats itself
after a definite interval of time. This time interval is called the period of
the vibration and will be designated by the symbol 7’, usually measured in
seconds. Each repetition of the motion is called a cycle. The frequency (f)
of the vibration is the reciprocal of the period, i.e., f = = and is measured
in cycles per second. The maximum displacement of the body from its
equilibrium position is known as the amplitude of the vibration.

16-2. Simple Harmonic Motion. Free Vibrations


Simple harmonic motion (shm) is the name used to describe the straight-
line motion of a body whose acceleration is proportional to the displacement
from a fixed origin and is always directed toward the origin. This motion
is a special case of rectilinear motion with variable acceleration. The
motion is essentially a vibratory displacement such as that described by a
weight which is attached to one end of a spring and is allowed to vibrate
freely.
464
Art. 16-2] Simple Harmonic Motion. Free Vibrations 465
The mathematical description of the motion is given by the equation

a= —Ks
where K is the constant of proportionality. It is preferable, however, to
denote this constant by w?, thus making

a = —w’s (16-1)
the mathematical statement of simple harmonic motion. The negative
sign indicates that the direction of the acceleration is always opposite to
that of the displacement.
The equations for simple harmonic motion may be derived from the
kinematic differential equations by means of the method developed in
Art. 10-4. Another simpler method is to describe the motion graphically by
considering a point that moves with constant speed around a circular path.
The projection of this moving point upon a diameter of the circle possesses
simple harmonic motion.
Consider, for example, a circle whose radius r is rotating at a constant rate
of w radians per second. Choose a diameter of the circle as the X axis
(Fig. 16-1), and let the initial position of a point on the circle be at A. The

v=rw

Fic. 16-1. — Auxiliary circle.

speed of this point will be constant at the valuev = rw. Its acceleration isa
normal one due only to the change in the direction of the velocity because
the magnitude of the latter is constant. In Art. 12-2 this acceleration was
given by dn = rw. After a time interval of t seconds, point A reaches posi-
tion C. The velocity and acceleration of the point in this position are rw
and rw*, directed as shown.
During this time the projection of the point on the X axis moves from
466 MECHANICAL VIBRATIONS [Chap. XVI

O to B through a displacement x. From the figure are AC projected upon


the diameter is
2 =r sind (a)
If Eq. (a) is differentiated with respect to time, we obtain

dedp =eg
cos?
or
Vz = rw cos 0 (b)
since w = & Observe that rw cos @ is the diametral projection of the ve-
locity of point C.
If Eq. (6) is differentiated with respect to time, the tol one expression
for acceleration results:

os —rw
digg atk: sin 6 we
di
or ;
az = —Trw* sin 6 (c)
The minus sign indicates that the acceleration is directed toward the
origin O. Notice that rw sin 6 is the diametral projection of the acceleration
of point C. This projection is also directed toward the origin.
However, 7 sin 6 = x from Eq. (a); hence Eq. (c) may be rewritten as

Gz = —w’r sin 8
or
az = —wr

This is the mathematical description of simple harmonic motion given in


Eq. (16-1). Hence we may conclude that the displacement, velocity, and
acceleration of a simple harmonic motion are described by the diametral
projection of the properties of a point moving with constant speed around
a circular path.! It is evident from Fig. 16-1 that the diametral projection
of such a point oscillates back and forth as the point rotates around the
circle; also that w is the constant angular velocity of the radius. A conven-
ient name for w is natural circular frequency. This is sometimes written
aS wn, the subscript being added to avoid confusion when w is used in its
usual meaning of angular velocity.
The circle in this discussion is known as the auxiliary circle. Its radius
is called the amplitude of the motion. The time required to complete one
oscillation backward and forward is called the period of the motion. The
1 A physical analogy is to whirl a stone in a horizontal circle. Look at the edge view of
the plane described by the rotating stone. The horizontal movement of the stone is
describing simple harmonic motion.
Art. 16-2] Simple Harmonic Motion. Free Vibrations 467
period is determined by the time required for the radius of the auxiliary
circle to complete one revolution, or

2G
Tue ee. (16-2)

The frequency is the number of oscillations per second, or

@
f= oe (16-3)

ILLUSTRATIVE PROBLEMS
1601. Ifasimple harmonic motion has an amplitude of 1 ft and a period of 1 see,
determine the displacement, velocity, and acceleration after 0.4 sec have elapsed,
time being measured from the right end of the path.

r=1 ft

w=2zcrad/sec

Fic. 16-2.

Solution: Construct an auxiliary circle having a radius equal to the amplitude,


as shown in Fig. 16-2. Choose diameter AOB as the path of the simple harmonic
motion. The natural circular frequency is determined from Eq. (16-2).

E = ze, i = Ag w = 27 rad per sec


Ww (6)

After 0.4 sec the radius will have swept through an angular distance
[6 = at] 6=27 X (0.4) = 0.8 7 radians
; = 0.87 X 57.3 = 144°
The moving point will then be at position C on the auxiliary circle. The properties
of its diametral projection P are given by
[s = r cos 0] s = —1 X cos 36° = —0.809 ft
[v = rw sin 6] v = —1 X 27 sin 36° = —3.70 ft per sec
[a = rw’ cos 6] a = 1X (27)? cos 86° = 31.9 ft per sec? [Cont.]
468 MEeEcHANICAL VIBRATIONS [Chap. XVI

The signs are determined by inspection of Fig. 16-2, rightward values being plus
and leftward minus.
1602. An elastic string of length 2 LZ tightly stretched
between two rigid supports, as in Fig. 16-3, carries a small
ball of weight W at its midpoint. Show that for small dis-
placements, the ball will have a simple harmonic motion;
compute the period. :
Solution: We shall assume that the initial tension P in the
string is so large that we can neglect any increase in it
caused by the displacement of the ball. If the rightward
direction from the equilibrium position is considered posi-
tive, the unbalanced force acting on the ball is

Deas Pain ge a

since sin 6 = tan 6 for small values of 6.


Substituting this value in the equation for translation,
Eq. (10-9), we obtain

=} IP ge
P SysUN
L F a

2 Pg
Tay.
a ats : : : 2 Pg
which is in the form of a simple harmonic motion (a = —w*s) where w? = VL
Hence from Eq. (16-2) the period is

27 WL
E Soke
= 4p a aePq
T wl Ans

PROBLEMS

1603. A simple harmonic motion is defined by the relation a = —36s. Deter-


mine its period and frequency. Ans. T = 1.047 sec; f = 0.957 osc per sec
1604. The amplitude of a simple harmonic motion is 2 ft and the period is 1 sec.
Determine the maximum velocity and the maximum acceleration.
1605. The amplitude of a simple harmonic motion is 9 in. and the period is
0.5 sec. Find the velocity and acceleration of a point when its displacement from
the center of the path is 3 in. What time is required for this displacement?
Ans. v = +8.88 ft per sec; a = 89.5 ft per sec?; t = 0.027 sec
1606. A particle has a simple harmonic motion defined by a = —9s and an
amplitude of 10 in. Find the velocity and acceleration of the particle when it is
6 in. away from the center of its path.
Art. 16-3] Simple Pendulum 469
1607. A particle moving with simple harmonic motion has a maximum velocity
of 12 ft per sec and a maximum acceleration of 48 ft per sec?. Determine the
velocity and acceleration of the particle when it is midway between the center and
one end of its path. Ans. v = +10.4 ft per sec; a = 24 ft per sec?
1608. A particle moving with simple harmonic motion is ob-
served to have a velocity of 16 in. per sec when it is 6 in. from
the center of its path and a velocity of 12 in. per sec when it is 8
in. from the center. Determine the maximum velocity, maximum
acceleration, and the frequency of vibration.
Ans. Umax = 20 mM. per sec; Gmax = 40 in. per sec’; f = 0.318
vib per sec
1609. A weight of W lb is attached to one end of a vertical
spring (Fig. P-1609) whose scale is k lb per ft. The weight is
displaced vertically from its equilibrium position by an exter- yg. p_1609.
nal force. It is then released and vibrates freely. By means of
: Ww : ee ;
the equation >X = —a, show that the vibration is in the form of a simple har-

monic motion and that the period of vibration is given by 7’ = 2 sal

1610. A 200-lb weight is attached to one end of a vertical spring. What must
the scale of the spring be so that the period of vibration will be 1 sec?
1611. A 161-lb weight is suspended vertically from the end of a spring whose
modulus is 12 lb per in. The weight is pulled down 6 in. below its equilibrium
position and then released. Compute the velocity of the weight as it passes through
the equilibrium position and also its frequency of vibration. Check the velocity
using the work-energy method. Ans. v = 3.47 ft per sec; f = 1.104 vib per sec
1612. The average depth of immersion of a ship is / and its area at the water
-

line is A. Using the equation 2X = aea, show that if the ship is displaced slightly
g
downward and then released, the buoyant force of the water will cause vertical
oscillations that have a simple harmonic motion. Determine the period of oscilla-
h
noe Ai: IE SB rf!

16-3. Simple Pendulum


A simple pendulum is defined as a particle at the end of a weightless
cord that is allowed to vibrate in a vertical are of a circle under the influence
of gravity and the tension in the cord. This motion is approximated by
suspending a small weight at the end of a thread, as shown in Fig. 16-4.
Taking the displacement as positive in the direction shown, we have

|=x -*a| -W sind = Ta (a)


g
470 MECHANICAL VIBRATIONS [Chap. XVI
If the oscillation is restricted to small angles of vibration, sin 6 may be
considered Pee equivalent to @ measured in radians. From the
figure, 6 = n wherefore
Seta
C —W=—a
aa b
(0)
a so that
ae g
yi i a a (c)
vi L}
Xe > 7, This is the equation of simple harmonic
a ae op eg motion (a = — ws) in which w? = a. From
x Art. 16-2, the period is given by
W
Fic. 16-4. — Simple pendulum. T= “= =a Ty A (16-4)

and the frequency by

=O LE 9,
I= om ~OaNL ed,
These results show that for small angles of vibration the period and fre-
quency depend only on the length of the cord of the simple pendulum.

PROBLEMS

1613. Determine the length of a simple pendulum whose period is 1 sec.


1614. Find the period of a simple pendulum whose length is 50 in.
Ans. T = 2.26 sec
1615. A simple pendulum is suspended from the roof of an elevator which is
accelerating at a ft per sec?. Assuming that the vibrations are small, determine the
period of the pendulum when the elevator is (a) accelerating upward, (b) accel-
erating downward, (c) falling freely.

Ans. (a) T= EEN pacers


5? 0) P= Tee (c) T = ~, pendulum sta-

tionary

16-4. Compound Pendulum

A compound pendulum is a rigid body that is free to rotate about a


horizontal axis and that oscillates under the influence of gravity. Such a
body is represented in Fig. 16-5. Point O is called the center of suspension.
Art. 16-4] Compound Pendulum 471

The position of the pendulum is defined


by its angular displacement @ from the
equilibrium position.
Applying the equation of rotation about
a fixed axis and noting that moments are
positive in the sense of angular displace-
ment, we obtain (neglecting friction at the
axis)
(2M, = Ia] —W7 sin 6 = Ia (a)

Expressing J, in terms of its mass : and


radius of gyration kz gives
AT

—W7 sin 6 = — k,’a


g Fie. 16-5. — Compound pendulum.
If the oscillations are restricted to small
angles for which sin 6 is practically equal to 6, Eq. (b) may be written in
the form

CS SS lt (c)

which is similar to a simple harmonic motion in which w? = ia Therefore

the period is given by JT = os (Art. 16-2), or

t= any ~ an /E (16-6)
gi g
Observe that the period of a compound pendulum is the same as that of a
simple pendulum (Art. 16-3) that has an equivalent length L, equal to eS
From the transfer formula relating radii of gyration, Eq. (8-6),
ae
Hence the equivalent length of a compound pendulum may be written as

Ty, = e ae iP (d)

This may be interpreted as meaning that the period is the same for all
parallel axes of suspension located at the same distance from the center of
gravity.
2

Point P on line OG at the distance L, = : from O is called the center of

oscillation. This point is also the center of percussion (Art. 12-5). Huy-
472 MECHANICAL VIBRATIONS [Chap. XVI

gens showed that if the center of oscillation is made the center of rotation,
the period will be unchanged, i.e., the centers of suspension and oscillation
are interchangeable. Thus since GO = 7 and GP = L, — 7, Eq. (d) gives
GO-GP =k (e)
If the pendulum is suspended from a horizontal axis through P and the new
center of oscillation is denoted by O’, then from-Eq. (e) we have
GO'-GP =} (f)
or O’ and O must coincide. We conclude that if P becomes the center of
suspension, O becomes the center of oscillation.
Equation (16-6) is sometimes used in experiments to determine the radius
of gyration or moment of inertia of a body that can be swung as a pendulum.
For example, if a connecting rod is swung from a knife edge that passes
through the hole for the wrist pin or crank pin and the duration of the
period is observed, the radius of gyration and hence the moment of inertia
about the axis of suspension can be computed. The center of gravity can
be determined by balamcing the connecting rod on a knife edge, and the
moment of inertia about any parallel axis can then be found by applying
the transfer formula.
PROBLEMS
1616. A circular ring is suspended from a knife edge as shown in Fig. P-1616.
The ring weighs 64.4 lb, its outer radius is 3 ft, and its inner radius is 2 ft. Deter-
mine the period for small oscillations. ANS lee OAISeG
1617. Thecompound pendulum in Fig. P-1617 consists
of a slender rod 2 ft long weighing 6 lb to which is at-
tached a solid circular disk of 12 in. diameter that weighs
8 lb. Compute the period of small oscillations.
Aig. 1" = ie see
1618. A conneéting rod weigh-
ing 9.66 lb is swung as a pendulum Tye)
from a horizontal knife edge resting
on the inner face of the wrist pin

Aw

Fig. P-1616. Fia. P-1617. Fic. P-1618.


Art. 16-5] Torsion Pendulum 473

bearing. The dimensions are given in Fig. P-1618. If the observed period of oscil-
lation is 1 sec, compute the moment of inertia with respect to the axis of the crank
bearing. Ans. I = 0.0633 ft-lb-sec?

16-5. Torsion Pendulum


A torsion pendulum consists of a body that is fastened to the lower end
of an elastic wire or rod whose upper end is clamped. The body is assumed
to be fastened so that the axis of the rod passes through the center of
gravity of the body, as in Fig. 16-6.
Let an external moment be applied to the body
that rotates it (and the rod) through an angular
displacement @ from the equilibrium position.
When this moment is removed, the body under-
goes an oscillatory rotation known as torsional
vibration. During it, the only external moment
acting on the body is supplied by the resisting
moment M exerted by the rod. From strength
of materials, it is known that the relation be-
tween the twisting moment and the angle of
twist for a circular cylindrical rod is
Fic. 16-6. — Torsion pen-
JG (a) dulum.
M = aie 6

in which J is the polar moment of inertia of the cross-sectional area of the


rod,? G the shear modulus of elasticity of the material, and L the length.
G. ; ; :
The product = is sometimes called the torsional spring constant of the rod,
i.e., the torque required to produce an angle of twist of 1 rad.
Applying the equation of rotation, we have
[2M = Ia] —M = Ia
in which J represents the mass moment of inertia of the body about the
axis of rotation. From Eq. (a) this becomes
G

_ ue i) = Shes
or ies
are t (b)
é lot? : JG
This is in the form of a simple harmonic motion in which #? = TTA In

ad
2 For a rod of diameter d, J = 32"

3 Observe that J should be expressed in in.-lb-sec?; also that L should be expressed in


inches.
474 MEcHANICAL VIBRATIONS [Chap. XVI

: ‘ 2
Art. 16-2 the period was given by T' = ms or

IL
T =20 sa (16-7)
If the axis of the rod does not pass through the center of gravity of the
attached body, the rod tends to be thrown out of plumb. However, if it is
supported in bearings at the lower end to prevent this, the attached body
will undergo torsional vibrations whose period is defined by Eq. (16-7).
Under these conditions, observing the period of vibration is an easy method
of determining the mass moment of inertia of any body with respect to an
axis coinciding with that of the rod.
Many practical applications of torsional vibration occur when the load or
power transmitted along a shaft between two heavy masses is suddenly
released. An example is the shaft connecting the engine and propeller of a
vessel that is pitching in a heavy
sea. If Fig. 16-7 represents two
heavy masses having moments of
inertia J; and J, between which a
transmitted torque is suddenly re-
leased, the elastic energy stored in
the shaft will cause the masses to
rotate in opposite directions. (See
Prob. 1567.) One section n-n of
Fic 16272 Nodal seetiou: the shaft, called the nodal section,
will remain stationary. The mo-
tion of each mass may be considered equivalent to a torsion pendulum on
a shaft fixed at the nodal section.
The position of the nodal section is located from the condition that each
segment of the shaft has the same period of oscillation. Hence by applying
Eq. (16-7) we obtain
Tha = Tob (c)

from which, since a + b = L


2 Io a ees
ee iar htt, 2

Since each segment has the same period, we may substitute either of these
lengths in Eq. (16-7) to obtain the following expression for the period of free
torsional vibration for the system:

[roe fe] oe oe i
Art. 16-6] Vector Representation of Simple
a ER GEES DANE SihNa Harmonic Motion
AN ea le et 475
ILLUSTRATIVE PROBLEM
1619. A solid circular disk weighing 32.2 lb and having a diameter of 3 ft is
suspended at its midpoint horizontally from the end of a vertical steel wire 3 ft
long and ¥ in. in diameter. The shear modulus of elasticity for the wire is @ =
12 X 10° lb per sq in. Determine the period of torsional vibration.
Solution: The moment of inertia of the disk with respect to the axis of the wire is
1W lee 82:9
|T==—Pr
oor I =) 3— X an9 .5)?
* (15) = 1.125 ft-lb-sec?
1.125 ft-lb-see

= 1.125 X 12 = 18.5in.-lb-sec?
The polar moment of inertia of the wire is

_ md4 BEX). are


l-% Beagle

Substituting these values in Eq. (16-7) gives the period

¥y
|r=2s
[71
a Ce
u i ous 2)
eee eee Ans.

PROBLEMS
1620. A horizontal bar 1 ft long that weighs 16.1 lb is suspended at its midpoint
by a vertical steel rod 4 ft long and 4 in. in diameter. If the shear modulus of
elasticity is 12 & 10°lb per sq in., compute the frequency of torsional vibration.
Ans. f = 2.21 ose per sec
1621. A certain body has a moment of inertia of 0.5 ft-lb-sec? with respect to an
axis coincident with the center line of a wire from which the body is suspended.
The period of torsional vibration is 0.6 sec. When another body is suspended from
the same wire in place of the first body, the period is 0.9 sec. Determine the moment
of inertia of the second body. Ans. I = 1.125 ft-lb-sec?
1622. A shaft 3 in. in diameter and 10 ft long connects the rotor of a steam
turbine with the armature of a generator. The rotor weighs 1932 lb and has a radius
of gyration of 2 ft. The armature weighs 966 lb and has a radius of gyration of 3 ft.
If G = 12 X 10° lb per sq in., compute the frequency of free torsional vibration of
the system. Ans. f = 3.64 osc per sec

16-6. Vector Representation of Simple Harmonic Motion


As discussed in Art. 16-2, the displacement-time relation defining a simple
harmonic motion may be written in the form
x =r sin wt (a)
Differentiating this equation with respect to the time determines the
velocity-time relation
Vv = Tw COS wt (b)
476 MECHANICAL VIBRATIONS [Chap. XVI

The maximum value of velocity is vm = rw. The acceleration-time relation


is obtained by ancther differentiation with respect to the time; this gives

a= —ro? sin wt (c)

from which the maximum value of acceleration is found to be a, = rw”.


One method of visualizing the variations in these equations is by means
of the auxiliary circle described in Art. 16-2. Another method, to be
described, permits the displacement-time, velocity-time, and acceleration-
time curves to be plotted easily. This technique is useful when analyzing
mechanical vibrations, especially when combining vibrations of the same
frequency.
Consider a point that moves with constant speed around a circle of
radius 7, as shown in Fig. 16-8. Let the position of this point be represented
+

seelocity -Time
—_-— Se eee
—_ Displacement -Time

Fe

teaN
\ e He‘ ‘ ~ |=
‘ NS Z

ctvig Sitters -Time

Fig. 16-8. — Vector representation of simple harmonic motion.

by the position vector 7 which is rotating at the constant angular velocity w.


Taking time as zero when the position vector is horizontal, we can write
the vertical projection of r as x = rsin wt, which is the simple harmonic
motion expressed by Eq. (a). Similarly the velocity can be represented by
the vertical projection of a vector of length rw that rotates with the same
angular velocity w as the position vector but is always 90° ahead of it. The
acceleration is represented by the vertical projection of a vector of length
rw that rotates with the same angular velocity w as the position and velocity
vectors but is 180° ahead of the position vector and 90° ahead of the velocity
vector. Figure 16-8 indicates how the projections of these rotating vectors
Art. 16-7] Free Vibrations Without Damping. General Case 477

are used to plot the values of displacement, velocity, and acceleration


against time.

16-7. Free Vibrations Without Damping. General Case


Let us consider a spring-suspended weight which is displaced from its
equilibrium position and allowed to vibrate freely. This is described in
Fig. 16-9. It is assumed that all resistances such as air resistance and in-
ternal friction in the spring are neglected. This means that the spring will
vibrate indefinitely under the action
of the variable spring force exerted
upon the weight. This condition is
:k=spring constant
called free vibration.
The differential equation of motion
for this case is found by applying Bes
the relation between the unbalanced —+—
force and the resulting acceleration.
= static elongation
Considering downward forces and
displacements as positive, we obtain Equilibrium position

or, since a = 9 a a
dt? eee

dx kg y wY Free-body diagram
de = oy. ARO aaa om 8 Hh (a) |

where |
ae
va kg (b) Fia. 16-9. — Free vibration of a spring-
W suspended weight.
Equation (a) is the differential equation of a free vibration. It indicates
that the displacement z is a function of time such that, upon being dif-
ferentiated twice with respect to time, it equals itself multiplied by a nega-
tive constant of value w*. The sine and cosine functions repeat themselves
in this manner. Substitution of the relations = sin wt and x = cos wt in
Eq. (a) shows that they are solutions of that equation. A more general
solution is obtained by multiplying these solutions by arbitrary constants
C, and (2 which are evaluated to fit the actual condition of the vibration.
With this procedure, the complete solution of Eq. (a) is given by the sum of
these solutions, or
z = C, sin wt + C2 cos wt (c)
To determine the constants C; and C, it is necessary to understand that
Eg. (c) describes all the possible motions that the system of weight and
478 MEcHANICAL VIBRATIONS [Chap. XVI

spring shown in Fig. 16-9 can have. If we now specify that the weight has a
velocity v. when given an initial displacement 2, there result two conditions
which can be used to determine the constants of integration. Measuring
the time as zero under the given conditions, at t = 0 we obtain
Lae andy — Op
Substituting the first condition in Eq. (c) yields
py = (Ch OW) SE Gp 0 1 or Cs (d)

The second condition is substituted in the velocity-time relation obtained


by differentiating Eq. (c) with respect to the time. Thus

E= a v = wC cos wat — wC2 sin wt

Setting v = v, when ¢ = 0, we have

Vo = wfi-1
— wf,-0 or Cpa (e)
(4)

Under the specified conditions, the vibration is described by the equation

gS . sin wt + x, cos wt (16-8)

The period is the time required for the position vector describing the
vibration (Art. 16-6) and moving with a constant angular velocity w, to
sweep through one revolution, or

i as
= a = 7p UE
yo oe est
7 (16-9)

where the static elongation is

Ww
Ct = Y)
and represents the elongation produced by the weight when hanging freely
from the spring. The frequency is

EL ee ee Deen, Levene
S= 3" om — on NW 7 an New 1619)
It will be observed that Eq. (16-8) really shows the combined effect of two
vibrations of the same frequency. If the position vectors of these vibrations
are plotted 90° apart, as in Fig. 16-10, the sum of their projections on the
vertical X axis will be

x = x,coswt + “ sin otf = Acos (wt — B) (16-11)


Art. 16-7] Free Vibrations Without Damping. General Case 479
e e ee
The figure shows that the sum of two simple harmonic motions of the same
frequency is another simple harmonic motion of the same frequency. The
amplitude of the resultant vibration is

Annee +(3)
a vo se (9)

which lags behind vibration x, cos wt by the phase angle 8, the value of
which is determined from the condition
Vo
canoe (h)
Wo
The phase angle 8 indicates that the resultant vibration reaches its maxi-

mum values of displacement, velocity, and acceleration at eCseconds be-


hind vibration x, cos wt. The construction shown in Fig. 16-10 may be
~ used to represent the displacement curves of each vibration. The resultant

tm x°
x=A cos (wt-@)

Amplitud
fr

(a) Initial conditions (b) Vector representation (c) Displacement-time curves

Fic. 16-10. — Combination of vibrations showing phase angle @ and resultant vibration.

vibration is determined by adding these displacements either geometrically


or directly from the position vector A.
The following conclusion can be drawn from this discussion: The period
and frequency of free vibration of a body are independent of the initial
conditions of the motion. The period and frequency of the vibration depend
only upon the weight W of the body and the scale k of the spring, or upon
the static elongation e. caused by the weight.
The system of weight and spring shown in Fig. 16-9 represents a con-
ventionalized diagram for vibrating bodies in which the supports act like
springs, regardless of whether this is their actual purpose. For such cases
the period and frequency can also be given by Eqs. (16-9) and (16-10) by
480 MECHANICAL VIBRATIONS [Chap. XVI

substituting in them the static elongation es caused by the weight of the


body whose vibrations are being considered or the equivalent spring
constant of that weight, whichever is more convenient.

ILLUSTRATIVE PROBLEMS
1623. The deflection produced at the free end of the cantilever beam in Fig.
16-11 by a static load of 200 lb is 0.20 in.
{ If a weight W = 300 lb is dropped on the
h free end of the beam from a height h = 2
~ Kk in., compute the frequency of vibration.
ae ’st What will be the maximum deflection?
at Assume that the weight stays in contact
ae with the beam after striking it.
Solution: Using Eq. (f), we find that the spring constant, determined from the
deflection produced by the static load, is
200 ;
[= = | k= 0207 1000 lb per in.

The frequency with which the 300-lb weight will vibrate is determined from
Eq. (16-10).
E ==af v2 f= ee1 <i100 12
ae as
32.2 :
= 5.72 vib persec Ans.

The amplitude of vibration is given by Eq. (g). Since the initial conditions of the
: W —=- 5 k
motion are%, = — | and v, = »/2 gh, and also since w = = Eq. (g) may be re-
written

Since the amplitude is measured from the position of static equilibrium, the
maximum deflection will be the sum of the static deflection plus the amplitude of
vibration. Hence
€max = €st + A
_W Ey 2Wh _ 300 il300 i 2 x 300 x 2
k ¢ ane Saez = 1000 ¢ Ni \1000), 5 aunloco
= 0.30 + 1.136 = 1.436in. Ans.
A check on this result, as well as a more direct determination of the maximum
deflection, is afforded by the work-energy method illustrated in Prob. 1415. The
weight has zero velocity when initially dropped, and zero velocity when the canti-
lever beam is deflected through emax in. Equating the resultant work on W to the
zero change in kinetic energy, we have
[W(h + emax) — % kemax? = 0] 300(2 + emax) — X 1000 X emax? = 0
€max = 1.4386 in. Ans.
Art. 16-7] Free Vibrations Without Damping. General Case 481
e e
1624. Consider a body of weight W supported by a system of two springs, as
shown in Fig. 16-12. Determine the equivalent spring constant for each arrange-
ment.

(a) (b) (c)


Fic. 16-12. — Springs in parallel and in series.

Solution: In Fig. 16-12a, it is evident that a displacement of the body through


a unit distance stretches each spring a unit distance. Hence the equivalent spring
constant for the system is
k=kh+hke (a)

This arrangement is known as springs in parallel.


A variation of it is shown in Fig. 16-12b. At first it is not so evident that the
spring constant here is given by Eq. (a), but it will be observed that a unit displace-
ment of the weight will stretch one spring a unit amount while compressing the
other a unit amount. Hence the spring force exerted on the weight when displaced
a unit distance is also
k=ht+he (a)
The arrangement in Fig. 16—12c is known as springs in series. A unit force applied
to W will be transmitted through both springs. Since the displacement of the weight
is the sum of the deflection of both springs, a unit force will cause a displacement
of the amount
it 1
ie ies
Ie : ;
From Hooke’s law applied to a spring, P = ks, we find k = ry, or for a unit force

BT
=
1
(b)b
ky ke
Eq. (b) can be rewritten in the form
1 1 1
ee eT (c)
482 MEcHANICAL VIBRATIONS [Chap. XVI

This form shows clearly that the reciprocal of the equivalent spring constant is the
sum of the reciprocals of the springs in series.
1625. Consider a U tube (Fig. 16-13) of
cross section A that contains a length L of
fluid weighing w lb per unit volume. Deter-
mine the equivalent spring constant and the
frequency of vibration when the fluid is dis-
Equilibrium turbed from its equilibrium position.
position Solution: If the fluid oscillates back and
forth, the weight in motion is W = wAL.
The unbalanced force acting on the fluid
when it is displaced a distance 6 from its
equilibrium position is 2 wAb. This results
Fig. 16-13. in a “gravity spring’ which tends to restore
the fluid to its equilibrium position. The
spring constant or force required to displace the fluid through a unit distance is
therefore
= Pan (a)
Applying Eq. (16-10) gives the frequency

Loe 1 eee
p=] f 2 7 wAL Qn L (2)
Observe that the frequency of vibration is independent of the fluid. A column
of mercury, for example, vibrates with the same frequency as a column of water.
1626. The system shown in Fig. 16-14 consists of a stiff rod of length L that has
a body of weight W at its free end and is supported by a spring having a constant

Fic. 16-14.

ks. Determine the frequency of vibration of the weight W if it is displaced vertically


from its position of equilibrium.
Solution: It was shown in Eq. (16-10) that the frequency of vibration can be
expressed in terms of the static deflection produced by the body whose vibrations
are being considered.
Art. 16-7] Free
aVibrations Without Damping. General Case
htl a hd Sa i de 483
From the static equilibrium of the rod, moments about O determine the force
: WL
exerted on the spring to be P = ei This stretches the spring an amount

7, ~ P_¥L
he Kad (a)
From the geometry in Fig. 16-14, the corresponding deflection of the weight is

est = Me et ei b
Cy a: ©)
Applying Eq. (16-10), we find the frequency of vibration to be

a ek WE ae it 1 gks

[r- 22] ia N We (©)


PROBLEMS
1627. A weight W = 100 lb is suspended from two springs in series, as shown
in Fig. P-1627. If k, = 30 lb per in. and k, = 20 lb per in., compute the period of
free vibration when W is displaced vertically from its equili-
brium position. Ans. T = 0.922 sec
1628. Compute the period of vibration if the springs in
Prob. 1627 are arranged in parallel.
1629. Assume that the weight in Prob. 1627 has a velocity
of 10 ft per sec as it passes through its equilibrium position.
Compute the maximum tension produced in the springs. By
how much will this tension be changed if the lower spring is
omitted?
Ans. Max tension = 311 lb; increase in tension = 122 lb
1630. A stiff bar of length L = 4 ft and negligible weight
is supported by a frictionless hinge at its left end, as in Fig. Fic. P-1627.
P-1630. It carries a body of weight W = 10 lb at a distance
b = 1.5 ft from the hinge. The bar is supported at the right end by a spring having
a constant k, = 40 lb per in. Determine the frequency of vibration of W if it is
displaced vertically from its equilibrium position. Ans. f = 16.7 vib per sec

Fic. P-1630.
484 MECHANICAL VIBRATIONS [Chap. XVI

1631. As shown in Fig. P-1631, a weight W = 4 lb rests upon a frictionless sur-


face between two springs having constants kh; = 1.5 lb per in. and ky = 2.5 lb per in.
The weight is released from rest when it is $ in. from its equilibrium position.
Determine its period of vibration and velocity as it passes through the equilibrium
position. Ans. T = 0.82 sec; v = 14.7 in. per sec

Fie. P-1631.

1632. An elevator weighing 4000 lb is being lowered at the constant rate of 10


ft per sec when the hoisting drum is stopped suddenly. If the elastic properties of
the cable are such that 1000 lb stretches it 0.025 in., compute the frequency of
free vibration of the elevator. Determine the maximum tension produced in the
cable. Neglect the weight of the cable.
Ans. f = 9.9 vib per sec; max tension = 81,200 lb
1633. In Prob. 1632, assume that a spring which elongates 1 in. per 2000 lb is
inserted between the elevator and the end of the cable. Determine the frequency
of vibration and the maximum tension produced in the cable.
Ans. Max tension = 20,900 lb

16-8. Free Vibrations Analyzed by Work-Energy Method


The work-energy method provides an alternative technique for deter-
mining the frequency and period of vibrating systems. Stated briefly, it
consists of determining the elastic energy stored in the system at the instant
of maximum displacement and equating this energy to the kinetic energy at
the instant the system passes through the position of static equilibrium.
As an example of this method, consider the spring-suspended weight in
Fig. 16-9. The resultant work is done by the unbalanced spring force,
which is zero at the position of static equilibrium and maximum at the posi-
tion of maximum displacement zm. The velocity is maximum as the body
passes through the position of static equilibrium, and zero at the position of
maximum displacement when the body is instantaneously at rest. Express-
ing the work-energy relation between these two positions as an equation, we
obtain
oy eo
|er . V (y2— 0d) | ee ie
W
2g (0 — vm”) (a)

Since the restoring force ka is always directed toward the origin and is
directly proportional to the displacement, we can assume that the motion is
harmonic and is defined by the relation
Cae SINCE
Art. 16-8] Free Vibrations Analyzed by W. ork-Energy Method 485
eee ee
The velocity, obtained by differentiating the displacement with respect to
the time, is given by
Vv = WXm COS wh

Obviously the maximum value of the velocity is

Um = Wim

Substituting this value in Eq. (a) gives

5 Km
pea ae
2g Ln
2

from which
sou k
0)
This agrees with the value of natural circular frequency as determined in
Art. 16-7. Since z,,? cancels out, we conclude as before that the frequency
is independent of the amplitude.

ILLUSTRATIVE PROBLEM

1634. The stiff bar of length L and weight W in Fig. 16-15 is hinged freely at one
end and supported at the other by a spring for which the constant is k. Determine
the period of vibration, using the work-
energy method.
Solution: Let 6,, be the angle through
which the bar rotates from its equili-
brium position to reach the position of
maximum displacement. Assume that
the angular displacement at any in-
stant is defined by the harmonic mo-
tion
6 = Om sin wt
The angular velocity* at any instant Fig. 16-15.
is found by differentation to be

Q = sd Im COS wt
dt

from which the maximum angular velocity is seen to be Qm = wm.


When the bar is in an extreme position, the spring is stretched an amount s = L@m.
Equating the elastic energy at an extreme position to the kinetic energy as the bar
passes through its equilibrium position, we obtain
4To avoid confusion with natural circular frequency w, the angular velocity is here
denoted by ©.
486 MECHANICAL VIBRATIONS [Chap. XVI
ee
Se eee
1 1 2 1/1W
5 UL»? = 5 IK = 5 éri12(ln)?

Therefore
1W
k = 3 gq. -

or ~
3 kg
oN
The period is therefore expressed by
27 W
DN
Comparing this result with that in Prob. 1609, we see that the period is the same
as though a body equal to one-third the weight of the bar were applied directly
to the spring.

PROBLEMS
1635. In Illus. Prob. 1634, assume W = 60 lb andk = 301b perin. What addi-
tional weight Q applied at the right end of the bar will produce a frequency of 2
vib per sec?
1636. A stiff weightless bar of length
L (Fig. P-1636) is hinged at its upper end
and carries a weight W at its lower end.
Two springs, each haying the constant
k, are attached to the bar ata distance b
from the hinge. Compute the frequency
of vibration for small oscillations.
1 Gp, Zaieafee
EOE Sree Grae
1637. If the bar in Fig. P-1636 weighs
Q lb and the weight W is removed, de-
termine the frequency of vibration, as-
suming small oscillations.
Ww
1 3g 6 kgb?
Fra. P-1636. f=
as BE oe
NO Tle
1638. Let the bar in Fig. P-1636 weigh Q = 30 lb and the weight W = 16 lb. If
L = 2 ft, b = 3 ft, and k = 20 lb per in., compute the period of vibration.
Ans. T = 0.837 sec
16-9. Forced Vibrations

Thus far we have considered free vibrations in which the amplitude de-
pends only upon the initial conditions of the motion. The free vibration
Art. 16-9] Forced Vibrations
ar g ee ee 487
is maintained by the action of an unbalanced variable spring force created
by the motion itself. The vibration exists only after it is begun by an
external disturbing force which, after displacing the body from its position
of equilibrium, ceases to act on the vibrating body.
If the disturbing force is not removed but continues to act upon the body
at periodic intervals, forced vibrations are created in which the frequency
and amplitude of vibration are
affected by the frequency and
magnitude of the disturbing force.
A typical example of a disturbing
force is the force caused by lack
of balance in a rotating machine.

|
If the frequency of the disturb- |
ing force is the same as the ma- %
chine’s natural frequency of free
vibration, a condition known as
resonance arises which results in
vibrations of dangerously large
amplitudes.
In most practical cases the dis- RA
- NG
ens
turbing force varies with the time pyg. 16-16. — Forced vibration caused by an
according to a sine or cosine law. unbalanced rotating weight.
For instance, the motor in Fig.
16-16 has a weight W and is mounted on a spring-supported support which
permits only vertical movement. Denote the equivalent spring constant of
the support by k. Asmall unbalanced weight W, rotates with the faceplate
: : , WwW
of the motor, and creates a centrifugal inertia force P, = iy rar. The
vertical component of this force varies harmonically with the time, its
magnitude being P, cos wt.
If displacements from the position of static equilibrium are taken as
positive downward, the equation of motion is
W @
[2x=7 6 W + P,cosut — (W + ka) = — oF
g g
Multiplying this by 7 gives

dx _ _kg L
ae.CO*C<Csa gle
W COS es wt (a)

Before solving this equation, consider another example of forced vibra-


tion illustrated by the spring-suspended weight shown in Fig. 16-17. The
upper end of the spring is connected by an inextensible cord to a disk of
488 MECHANICAL VIBRATIONS [Chap. XVI

radius r that rotates with an angu-


lar velocity w. Assuming the cord
to be long compared with 7, this cre-
apy ates in the upper end of the spring
Deeg Position of upper end _ the simple harmonic motion
I of spring when r is
horizontal : b = r cos wt
j= The time ¢ is measured from the in-
stant radius r is vertically up, and
Pariliomoresne b is measured from the center of
equilibrium when 6=0 the disk. The spring extension is
x — b because the downward move-
ment of the top of the spring tends
to overcome the downward stretch-
ing movement x caused by the vibra-
tion of the weight. Hence at any
instant the unbalanced upward
spring force is k(x — b).
Forces, displacements, and accel-
Fic. 16-17.— Forced vibration of a erations being taken as_ positive
spring-suspended weight due to the motion , :
aiihe paintiet eunaoet downward, the equation of mo é
tion is

oF ee bie ae SS ieee
acten ge
[> breyh
If we multiply this by = and replace b by r cos wt, this becomes

2
= =— +a24 9 COS wt (b)

It will be noticed that Eqs. (a) and (b) are identical except for the form of
the constant term preceding the expression cos wt in each equation. The
only difference between the two cases is that in the second case the disturb-
ing force is transmitted to the weight through the spring instead of being
applied directly to it. Denoting the coefficient of cos wt by h, we have in
the first case
_ gPo
es W
and in the second case
_ krg
Me W

Also, as in Art. 16-7, the expression agcan be replaced by w,?2. Note that wa,
Art. 16-9] Forced Vibrations 489

is the natural circular frequency of a free vibration, whereas in each of Eqs.


(a) and (b) above, w is the circular frequency of the disturbing force. Thus
Kgs. (a) and (b) may be put into the form

— = —w,t + hcos wt (c)

The forced vibration represented by Eq. (c) may be visualized as a com-


bination of a free vibration of the type discussed in Art. 16-7 plus a new
motion caused by the disturbing force. Thus if 2; represents the displace-
ment caused by the free vibration (which is independent of the disturbing
force) and x2 is the additional displacement caused by the disturbing force,
the actual displacement is
& = Xi + Xe
With this assumption, Eq. (c) may be considered to be composed of the
following parts:
ax,

dpe — enh (4)


2
on = —w,rm + h cos wt (e)

Adding these equations gives the original equation (c); Eq. (d) is similar to
Eq. (a) in Art. 16-7. Hence the value of 2; is given by
t= Gh sin Wnt — C. cos Wnt (f)

It seems reasonable to suppose that a tentative solution of Eq. (e) is


xz. = A cos wt, in which the constant A, representing the amplitude of
forced vibration, is selected to satisfy the equation. Substituting this
solution in Eq. (e) yields
—Aw? cos wt = —Aw,* cos wt + h cos wt
from which the value of A is
—Aw’* + Aw? =h
or
Ae (16-12)

Hence a solution of Eq. (e) is

X, = Acosat = ae 2 ae
> 608 wl (16-13)

The complete solution of Eq. (c) is the sum of the solutions of its parts, or
h
X=n+uH= C, sin Gt + Cy cos Ont + Tea cos wl (16-14)
n
490 MEcHANICAL VIBRATIONS [Chap. XVI
eee
MON
This result is shown graphically in Fig. 16-18. Part (a) represents the dis-
placement-time diagram for the free vibration (assuming C= 0) sand
(b) is the diagram of the forced vibration. The summation of these two
harmonic motions (which have different periods) is shown in (c).

| (a) Free vibration

—— 28 !
Ww

(b) Forced vibration

(c) Resultant vibration

Fic. 16-18.

Since some damping of the free vibration will always exist, the free
vibration will die out, as shown in Fig. 16-19a. Combining the damped free
vibration with the forced vibration results in the steady state of forced
vibration shown in Fig. 16-19b which is maintained indefinitely by the
periodic disturbing force.
Magnification Factor. Because of the damping of the free vibration, it
is the motion of forced vibration represented in Fig. 16-19b that is of prac-
tical importance. If the free vibrations which soon die out are neglected,
Art. 16-9] Forced Vibrations 491

f (a) Damped free vibration

(b) Resultant steady state of forced vibration


Fic. 16-19.

the maximum displacement (or amplitude) of forced vibration will be given


by Eq. (16-12).
For the spring-mounted motor shown in Fig. 16-16, w,? = roand h = Ea
Hence the amplitude of forced vibration reduces to
te 1
os k ow” (g)
ee Wn

Similarly in the spring-suspended weight in Fig. 16-17, if h is replaced by


krg the amplitude of forced vibration becomes
W’

1
Jah
Ss Hi
sails! (h)
@,7

The factor in Eq. (g) represents the maximum displacement caused

by the static action of the disturbing force P,. Also r in Eq. (h) represents
492 MeEcHANICAL VIBRATIONS [Chap. XVI

the maximum displacement of W in Fig. 16-17 when the disk revolves very
slowly. In general,

A = Xst ne (16-15)

ih ==
2
On

where xs: represents the displacement of the body caused by the static ap-
plication of the disturbing force. We conclude that the dynamic effect of
the disturbing force on the maximum displacement is given by the expres-
sion

ae (2)
This is known as the magnification factor.
The effect of this factor is shown in Fig. 16-20, in which the factor has
been plotted in terms of the ratio“. Thus when the disturbing force
n

alternates slowly compared with the natural frequency 2 less than ah


the magnification factor is only slightly larger than unity. Hence the
dynamic deflection is only slightly greater than that produced if the dis-
turbing force were applied statically.

2
1 we No damping
j=

Small damping

Large damping

factor
Magnification

Fic. 16-20. — Magnification factor.

As the impressed frequency approaches the natural frequency, i.e., as


w becomes equal to w,, the magnification factor increases rapidly, reaching
infinity when w equals w,. Physically this means that when — = 1, the
Wn
frequency of the disturbing force coincides exactly with the natural fre-
quency; 1.e., the disturbing force tends to push the body always at the
Art. 16-9] Forced Vibrations 493
right time in the right direction so that the amplitude increases without
limit. The condition existing when the impressed frequency is equal to the
natural frequency is known as resonance. In a machine, the speed of rota-
tion at which resonance occurs is called the critical speed. Obviously a
machine should be designed to operate well above or below this speed.
As is increased above resonance, the magnification factor becomes
negative. Physically this means that the disturbing force is 180° out of
phase with the motion; i.e., it is of opposite sense to the displacement.
Usually this phase relation is of only little interest and the sign of the mag-
nification factor is neglected. It will be observed that when the impressed
frequency is well above the natural frequency the magnification factor
approaches zero and the body tends to stand still in space. This is due to
the fact that the disturbing force moves up and down so rapidly that the
body cannot follow it because of its inertia.
It can be shown that the effect of a relatively small and a relatively large
viscous damping can be represented by curves (a) and (6) in Fig. 16-20.
Note that there is practically no change in the magnification factor except
near resonance and that the resonant frequency is practically unchanged.
For torsional forced vibrations, equations similar to those discussed
above can be obtained. To do this, we replace force by torque, mass by
mass moment of inertia, and linear values of displacement, velocity, and
acceleration by corresponding angular values.

PROBLEMS
1639. A horizontal shaft rotates in bearings at its ends. At its midpoint is keyed
a disk weighing 40 lb, whose center of gravity is 0.1 in. from the axis of rotation.
If a static force of 200 lb deflects the shaft and disk
through 0.1 in., determine the critical speed of rotation A
of the shaft. Ans. w = 547rpm
1640. The motor in Fig. 16-16 has a weight W = 1000
lb and is supported by four springs each having a con-
stant of 250 lb per in. When the motor is rotating at
1800 rpm, the centrifugal force caused by the eccentric
weight is P, = 200 lb. Compute the maximum ampli- Neutral
tude of forced vibration. Ans. A = 0.0022 in. Position |
1641. The spring-suspended weight in Fig. 16-17
weighs 60 Ib and the spring constant is k = 100 lb per 7
in. The disk of radius r = 1 in. rotates at 24 rad per sec. =a 0.05
Compute the maximum tension produced in the spring. Fic. P-1642.
Ans. 1013 lb

1642. The bob in Fig. P-1642 weighs 8.05 lb and is fastened to the upper end of
a flexible rod. The lower end of the rod is embedded in a block that oscillates hori-
494 MECHANICAL VIBRATIONS [Chap. XVI

zontally with an amplitude of 0.05 in. and a frequency of 1 vib per sec. The elastic
properties of the rod are such that a horizontal force of 10 lb applied at the upper
end deflects the bob through 1 in. Determine the amplitude of forced vibration of
the bob.

SUMMARY
Vibrations are essentially periodic motions which repeat themselves after
a definite time interval called the period. The frequency is the reciprocal
of the period and represents the number of vibrations per second.
Most vibrations have simple harmonic motion. The mathematical de-
scription of simple harmonic motion is

PALE (16-1)
in which w is the natural circular frequency of the motion. If a motion
can be expressed in this form, the period and frequency are respectively

al (16-2)

f=5 (16-3)
For purposes of comparison, the period and frequency for various types of
pendulums are summarized in the following table:

Pendulum Period Frequency Reference

Simple Y =
D Pee
1 he Art. 16-3
: i ¥E 27 NL 3

Compound 20 hy" i esI Art. 16-4


gi Qari Nays

Torsion 2
TD
= —
ee
« |E= Art. 16-5
"NIG 2a NIL :

The manner in which displacement, velocity, and acceleration vary with


time can be visualized by projecting upon a diameter the length of the
position, velocity, and acceleration vectors. These vectors rotate about a
common axis with the same angular velocity w, which is the natural cir-
cular frequency of the vibration. The magnitude of the position vector
is r, the amplitude of the vibration. The magnitude of the velocity vector
Summary 495
is rw located 90° ahead of the position vector; the magnitude of the accelera-
tion vector is rw? located 90° ahead of the velocity vector.
The general solution of the differential equation of the free vibration of
a spring-suspended weight is

x= ~ sin wf + x, cos wf (16-8)

in which 2, is the initial displacement from the position of static equilibrium


and v, is the initial velocity at 2».
The combined effect of the two vibrations of the same frequency ex-
pressed in Eq. (16-8) is equivalent to
x = Acos (ot — B) (16-11)

in which A is the amplitude of a resultant vibration that has the same


frequency as each of the combined vibrations. The magnitude of the
amplitude A and of the phase angle 6 is determined from

21 (%)”
Air le +(¥)
Vo
Pr ae

Article 16-7 shows that the period and frequency of a free vibration are
independent of the initial conditions of the motion. They depend only
upon the weight W and the constant k of the spring support or upon the
static elongation e. caused by the weight as expressed by

T=? ™ lig NG; (16-9)


W est
—= 27r./— 16-9

meas kg _ 1 gs
Lom NW 2m Nes Cee)
Hence the period or frequency of a body of weight W which is part of a
vibrating system may be found by determining the static elongation caused
by that weight or computing the equivalent spring constant of that weight,
whichever is more convenient.
An alternative technique for determining the frequency and period of
vibrating systems is the work-energy method. In this, the elastic energy
in the system at the instant of maximum displacement is equated to its
kinetic energy at the instant the system passes through its mid-position.
This procedure determines the natural circular frequency of the system.
496 MECHANICAL VIBRATIONS [Chap. XVI

In a forced vibration, the maximum displacement (i.e., amplitude) is


given by

(16-15)

where 2s: is the displacement caused by the static application of the dis-
turbing force, w is the circular frequency of the disturbing force, and w, is
the natural circular frequency of free vibration of the system.
Chapter XVII.
Graphic Methods

17-1. Introduction

In this chapter we shall discuss graphic methods for solving many of the
problems considered in the preceding chapters on statics. Graphic solutions
at one time enjoyed widespread popularity but now are being gradually
discarded. In most cases, the care and time needed to obtain results graph-
ically are much greater than is required for the same results analytically.
Graphic results, moreover, are frequently not as accurate as the analytical.
In spite of these objections, graphic methods are still used for the follow-
ing reasons: They are valuable as an aid in checking analytical calculations.
This check is rapid and is often sufficiently accurate when done freehand
to approximate scale. Also, in some cases of stress analysis analytical
calculations are long and cumbersome. The care required in plotting a dia-
gram to scale is justified in such instances. Moreover, although graphic
solutions are not as accurate as analytical ones, the degree of accuracy
obtained from them is often sufficiently accurate for many engineering
purposes.

17-2. Application of Parallelogram Law


In Art. 2-3 we showed how the parallelogram law could be applied to
determine graphically the resultant of a concurrent system of coplanar
forces. We showed also that the magnitude and direction of the resultant
could be obtained from a tip-to-tail addition of the free vectors of the
system. (See Fig. 2-7.)
The present article will show how this law can be applied to determine
graphically the resultant of any system of coplanar forces. It will also show
that the magnitude and direction of the resultant can be determined from
a force polygon formed by a tip-to-tail addition of the free vectors of the
forces comprising the system.
In Fig. 17—1a is shown a body acted upon by the non-concurrent forces
F,, Fo, F3, and Fy. Extend the lines of action of 7, and F to intersect at A
and construct the parallelogram shown. The resultant A; of 7; and F, can
be similarly extended to intersect the line of action of Ff; at B, and the
resultant R. determined by the parallelogram law. MR, is evidently the
resultant of the forces F:, F2, and F3. In the same way, the resultant of
497
498 GrapHic Mrruops [Chap. XVII

/ N

——->--

po

(a) (b)
Fia. 17-1. — Resultant determined from parallelogram law.

R, and F,, and consequently of the given system of forces, is determined to


be acting through point C.
Comparing the polygon of forces created by the tip-to-tail addition of
their free vectors (Fig. 17-1b) with the successive applications of the par-
allelogram law in Fig. 17—1a, we conclude that a force polygon can be used
to determine the magnitude and direction of the resultant of any system of
forces. The position of the resultant force, however, cannot be found from
the force polygon. One method of determining this position has just been
discussed, but it is tedious and cannot be used when the action lines of the
forces do not intersect within the limits of the paper. Accordingly, other
and simpler methods are necessary. Some of them will be discussed.

17-3. Resultant of Coplanar Force Systems


Let it be required to determine the resultant of the non-concurrent system
of coplanar forces shown in Fig. 17-2. The system consists of forces Fi, Fs,
and F’; having the lines of action ab, bc, and cd. (This is in accordance with
Bow’s notation, which is described in the next article.)
We begin by using the parallelogram law to resolve F; into two com-
ponents fF)’ and F,’’.. Now resolve F2 into the components F,’ and F,’’ so
that Fs’ is equal, opposite, and collinear with F’’ as shown. Similarly,
resolve Ff’; into components F3’ and F3’’ so that F./’ and F;' are also equal,
opposite, and collinear.
Art. 17-4] String Polygon 499
Se RPA a ere se mt et 2 la
Thus the original system of forces is replaced by an equivalent system
of six components. These components are chosen in such a manner that four
of them make two pairs of equal, opposite, and collinear forces which cancel
one another. The original system is thereby reduced to the two components
FY and F;’’ which can be combined to determine the resultant of the given

pee Ais

eT
ae Bs
Neate 1
ee =S NN
Vie
ee eas Rr’

ye on Ss
See"Ie ty
FE
jt hms SR SS paha Sa
io ad sx B
aad (7) =e a
<< Sn eee i
a a4 ftlp Paes Sa
SIA ee we
FE F,
a/b
ble

Fic. 17-2. — Another method of using the parallelogram law.

system. The line of action of the resultant R passes through the point of
intersection O of the action lines of F;’ and F3;’’.. The magnitude and direc-
tion of R are determined by the parallelogram law as shown in the figure.
In the next article we shall discuss another graphic method which involves
less construction and has certain advantages over this one. The present
method, however, serves to focus attention on the fundamental principle
underlying graphic methods — the reduction of the given system to two
forces. The intersection of these two forces determines a point on the line
of action of the resultant. For the special case in which the system reduces
to two equal, opposite, parallel forces, the resultant is a couple.

17-4. String Polygon


It has just been shown that the key to graphic constructions is the cancel-
ing of pairs of equal, opposite, and collinear components. This article will
present a simple method of selecting and removing these pairs of com-
ponents. The construction involves what is known as a string polygon
(also called a funicular polygon).
Here a system of notation called Bow’s notation is convenient. In it, two
lower-case letters are used to designate the line of action for a force. The
500 GrapPHic METHODS [Chap. XVII
a
corresponding capital letters denote the terminal points of the force vector.
For instance, in Fig. 17—3a the line of action of force F, is designated by the
letters be, and the force vector of F, is described by the line BC in Fig. 17-8b.
The letters BC are placed so that the sense of the force vector is obtained
from their alphabetical order.
Consider the system of non-concurrent coplanar forces Fi, F2, and F3 in
Fig. 17-3. We first construct a force polygon A BCD (Fig. 17-3b) by adding
the free vectors of the forces in tip-to-tail fashion. The magnitude and

(a)
Fig. 17-3. — String and force polygons.

direction of the resultant R are determined by the closing side AD of the


force polygon. Only the line of action of R remains to be determined.
To accomplish this, we draw lines from any arbitrarily chosen point O
(either inside or outside of the force polygon) to points A, B, C, and D on
the polygon. The arbitrary point O is called a pole, and lines OA, OB, etc.,
are called rays. The rays of the force polygon are the components into
which the forces of the given system are to be resolved. For example, from
the arrows inside triangle AOB, it is evident that F is the vector sum of AO
and OB. Similarly, F; can be considered as the resultant of BO and OC,
and F; as the resultant of CO and OD. Likewise R is the resultant of AO
and OD.
Referring to Fig. 17-3a, which represents the position of the action lines
of the forces, we replace each of the given forces by the components deter-
mined from the rays of the force polygon. Starting at any point / on the
action line of F1, we represent AO and OB by their lines of action ao and ob
Art. 17-4]
i ea String Polygon 501
respectively, parallel to AO and OB. At point 2, where ob intersects the
action line of F2, we draw bo and oc parallel respectively to BO and OC.
Since the components BO and OB are laid off on the same line of action,
they form a pair of equal, opposite, and collinear forces. (Compare with
F,’’ and F,’ in Fig. 17-2.) Similarly at point 3, where oc intersects the line
of action for F's, we draw co and od parallel to CO and OD. The intersection
of ao and od determines point 4 through which passes the line of action of
the resultant.
Lines ao, bo, co, and do in Fig. 17-3a may be compared with a string held
at the ends and stretched taut by the action of the forces F',, Fs, and F3.
The figure described by these lines is therefore called a string polygon and
the lines are spoken of as strings. The reason for the other name, funicular
polygon, is evident from the fact that funicular means “consisting of a rope.”
We conclude that the resultant of a force system can be completely
determined by constructing a force polygon and a string polygon. The force
polygon determines the magnitude and direction of the resultant, and the
string polygon determines its line of action.
If the force polygon closes, the magnitude of the resultant will be zero.
This will be the case if D coincides with A. In this case either equilibrium
will exist or the resultant will be a couple (see Art. 2-9) consisting of the
coincident forces AO and OD in the force polygon and represented by their
parallel action lines ao and od. The perpendicular distance between ao and
od will be the moment arm of the couple. However, if ao and od coincide,
the moment arm will be zero, and the string polygon is said to close. Hence
when both the force polygon and the string polygon close, the resultant is
zero, and equilibrium exists.
The force polygon may be constructed by taking any point as the pole;
the string polygon may be started at any point on the line of aetion of any
one of the forces. A change in the shape of either polygon only locates a
different point on the line of action of the resultant, unless the resultant is
a couple. In this case, such a change will produce an equivalent couple
consisting of a different moment arm with a corresponding change in the
magnitude of the forces composing the couple.

ILLUSTRATIVE PROBLEMS

_1701. Determine graphically the resultant of the parallel force system shown
in Fig. 17-4.
Solution: The force polygon in Fig. 17-4b is obtained by drawing to scale a tip-
to-tail addition of the force vectors comprising the given system. It is a straight
line because the forces are parallel. The sense of a force is indicated by reading the
letters in alphabetical order. For instance, the 60-lb force, whose line of action is
ab in Fig. 17-4a, is read from A to B in Fig. 17-4b, the downward alphabetical
502 GrapHic METHODS [Chap. XVII

sequence of the letters indicating the direction of the force. The upward 70-lb
force whose action line is de is read alphabetically upward from D to E in the force
polygon. Since the direction of a force is indicated by the alphabetical sequence
of the letters describing it, it is usually unnecessary to use arrows on the force
60 lb 80 lb 100 lb 70 |b 130 lb

x0 alf 5 atc
SS re <a G
ss ee
S (b)
R=100 lb

(a)
Fig. 17-4.

polygon. The resultant R is represented by the vector AF drawn from the tail A
of the first vector to the tip F of the last one. From the force polygon, AF scales
100 lb, and its direction is downward from A to F.
Choose a pole O and draw rays AO, BO, etc. The string polygon is started at any
point, say point / on the action line of ab. Through this point draw strings ao and
bo parallel to rays AO and BO. At point 2, the intersection of bo with bc, draw
string co parallel to ray CO. Continuing in this fashion, construct the string polygon.
All the components of the forces neutralize each other except AO and OF. These
two intersect at point 6 which is a point on the action line of the resultant. Ac-
cordingly, show the resultant passing through point 6. From Fig. 17—4a the distance
from the line of action of ab to the line of action of R is found, by scaling, to be 9.6 ft.
1702. Determine graphically the resultant of the non-concurrent system of
coplanar forces shown in Fig. 17-5.
Solution: Draw the force polygon to scale as shown in Fig. 17-5b. The resultant
R is drawn from A to E and scales 87 Ib. Choose a convenient pole O, and draw rays
AO, BO, etc. Starting at any point / on the line of action of ab in Fig. 17-5a, draw
strings ao, bo, etc., parallel to the respective rays. All the components of the forces
neutralize each other except AO and OZ. The strings of these components intersect
at point 5, through which is drawn the line of action of R. By scaling from Fig.
17—5a, the action line of the resultant intersects the X axis at a point 6.62 ft to the
right of the origin of the axes.
Art. 17-4] String Polygon 503

Fic. 17-5.

1703. Determine graphically the resultant of the non-concurrent system of


forces shown in Fig. 17-6.
Solution: Plot the force polygon to scale as shown in Fig. 17-6b. Here the poly-
gon closes because E coincides with A. The resultant therefore has zero magnitude.
Construction of a string polygon will determine whether the moment of the resultant
is also zero. If it is, there will be equilibrium; if not, the resultant will be a couple.
Choose a pole O, and draw the rays. Construct the string polygon as in Fig.
17-6a. Strings ao and oe are found to be parallel because they are parallel to the
(251b) — (25 Ib) B

ee
a\o ale

Hie. 17-6.
504 GRAPHIC METHODS [Chap. XVII

same ray, AO or OF, of the force polygon. They represent the action lines of forces
AO and OE. Since the strings do not coincide, these two forces constitute a couple.
Its moment arm is the perpendicular distance between strings ao and oe, which
scales 2.56 ft. Each force of this couple is found to be 25 lb by scaling AO in Fig.
17-6b. Therefore the moment of the resultant couple is 25 X 2.56 = 64 lb-ft
acting in a counterclockwise sense, as shown in Fig. 17-6a.

PROBLEMS
1704. Check graphically the resultant of the parallel force system described in
Prob. 236 repeated here as Fig. P-1704.
50lb 40lb 20 lb 60 lb
30 1b 60lb 201b 40 lb

Fic. P-1704. Fig. P-1705.

1705. Determine graphically the resultant of the four parallel forces on the
rocker arm of Prob. 237 repeated here as Fig. P-1705.
1706. Determine graphically the resultant of the four parallel forces shown in
Fig. P-1706.

10lb 401b

:20]lb 30]b
Fic. P-1706. Kie. P=1'707.

— F=6000lb
P=10,000 lb

Fic. P-1708.
Art. 17-5] =Equilibrium of Force Systems by Graphic Methods 505

1707. Determine graphically the x and y intercepts of the resultant of the force
system described in Prob. 265 repeated here as Fig. P-1707.
1708. Determine graphically the resultant of the three forces acting on the dam
shown in Fig. P-1708, and locate its intersection with the base AB.

17-5. Equilibrium of Force Systems by Graphic Methods


The graphic condition for equilibrium is that both the force polygon and
the string polygon must close. This is sufficient to reduce the resultant of
any coplanar force system to zero because a closed force polygon means
that the resultant has zero magnitude, and a closed string polygon means
that the resultant has zero moment.
The force polygon may be constructed in any order, and the pole O chosen
at any point. Although the string polygon can be started at any point in
the plane of the forces, it is usually best to begin it on the line of action of
one of the unknown forces. In case one of the unknown forces is unknown
as far as both magnitude and direction are concerned, the string polygon
must be started at the point of application of the force. The reason is
that the apexes of the string polygon lie on the action line of the forces;
hence if only the point of application of an unknown force is specified, the
string polygon must be started at that point of application in order to have
the apex of the string polygon lie on the action line of this undetermined
force.
ILLUSTRATIVE PROBLEMS
1709. Determine graphically the reactions on the beam loaded and supported
as shown in Fig. 17-7.
120 lb 80 1b
] 60 1b
alb b A
: d \
eS Se eS ee ees
s
\ \

a Eko.
1 4 \
Bree
ps Yo \
Ne
SEAN
ire
\\
C= a
0
igs

0) 4 ge
ST ian D
5 (b)
506 GrapHic Mrruops [Chap. XVII

Solution: In determining the reactions, the uniformly distributed load can be


replaced by its resuliant of 120 lb having the line of action ab. We begin by plotting
to scale the force polygon in Fig. 17-7b. Reactions DE and EA close the force
polygon, but the position of point Z is still undetermined so that ray OH cannot be
drawn yet. Its direction will be determined from the closing side of the string
polygon.
A characteristic of the string polygon is that its apexes lie on the action lines of
the forces. Further, the strings intersecting any action line (such as ab) are those
defined by the letters (a and 6) of the action line of the force combined with the
letter o (thus, ao and bo). Therefore, starting at point / on the action line of the
unknown reaction ea, draw strings ao, bo, etc., respectively parallel to rays AO, BO,
etc. Close the polygon 1, 2, 3, 4, 5 by drawing the closing side eo from point 4 to
point 7.
The position of point EF in the force polygon can now be found by drawing the
unknown ray OF parallel to the closing string oe. Vector DE represents the magni-
tude and sense of reaction R», and vector HA similarly determines reaction A.
By scaling from the force polygon, we obtain
R, = EA = 921b; Re = DE = 168lb Ans.
1710. The Fink truss shown in Fig. 17-8 carries the given loads and is supported
by a roller at A and a hinge at B. Determine graphically the reactions at A and B.

(a)
Fia. 17-8.

Solution: Using a convenient scale, plot the force polygon ABCD as shown in
Fig. 17-8b. The given loads are represented by the portion ABC. The reaction
at roller A, whose line of action is designated by ad, is constrained to be vertical
because of the roller support. Represent this reaction in the force polygon by a
vertical line through A for vector DA. However, the magnitude of this vector is
unknown; consequently the position of point D remains to be determined. Once
Art. 17-5] =Equilibrium of Force Systems by Graphic Methods 507

this is determined, the magnitude and direction of the unknown hinge reaction Rg
will be given by vector CD which closes the force polygon. Choose a pole O, and
draw the known rays AO, BO, and CO. The direction of the unknown ray OD,
which locates D on the force polygon, must be determined from the closing side of
the string polygon.
As discussed in Illus. Prob. 1709, a characteristic of the string polygon is that its
apexes must lie on the action lines of the forces. In the present problem, hinge B
is the only known point on the action line of Rg, and therefore the hinge must be
an apex of the string polygon. In other words, to make B an apex, the string poly-
gon must be begun at B.
Beginning at point B in Fig. 17-8, draw strings co, bo, and ao, thereby establishing
the closing side do. The unknown ray DO in the force polygon can now be drawn
parallel to the closing side do of the string polygon. The intersection of ray DO
with the vertical line previously drawn through point A determines point D in
Fig. 17-8b. The magnitudes of the reactions are obtained by scaling from the force
polygon. These values are:
if) = (Dy = A os a S C1) = PAG le, Alas
The directions and senses of the reactions can also be obtained from the force
polygon.
PROBLEMS
1711. Check graphically the reactions on the beam described in Prob. 332 and
repeated here as Fig. P-1711.
300 lb 400 Ib
2

100 lb/ft.

Fig. P-1711.

1712. Check graphically the reactions on the truss described in Prob. 335 and
repeated here as Fig. P-1712.
200 lb

600 |b 1500 lb
Fig. P-1712.
508 GrapHuic MrrHops [Chap. XVII

1713. Check graphically the reactions on the truss described in Prob. 350 and
repeated here as Fig. P-1713. Hint: The reactions are unchanged if the load at F
is applied at H.

600 lb 1200 Ib 500 lb


Fig. P-1713.

1714. Check graphically the reactions to the truss described in Prob. 355 shown
again here as Fig. P-1714.
1000 lb
1200 Ib
2000 Ib

1000 lb

Fig. P-1714.

17-6. Graphic Analysis of Trusses. Maxwell Diagrams


Consider the truss loaded as in Fig. 17-9 and supported by a roller at A
and a hinge at D. The values of the reactions are assumed to have been
determined. The stresses in the various members of the truss may be de-
termined graphically by constructing the polygon of forces acting on each
joint. From Art. 4-3, the forces at a joint are concurrent, the unknown
stresses being directed along the axes of the members. Since no more than
two unknown forces can be determined to maintain equilibrium of a con-
current force system, the solution is begun at a joint at which there are only
two unknown stresses. Selecting joint A, lay out R, to some convenient
scale. Through the tip of R; draw a line parallel to AB and, through the
tail of 1, a line parallel to AC. The intersection of these two lines deter-
mines the magnitudes of the stresses in members AB and AC. Proceeding
next to joint B and starting with the now known stress AB, determine the
Art. 17-6] Graphic Analysis of Trusses. Maxwell Diagrams 509
a a a a i
je

Joint D

Stress diagram
Fic. 17-9. — Maxwell diagram. we

stresses in BC and BD. Continue to draw force polygons until the stresses
at all joints have been found.
Notice that the force polygons for joints A and B have the common
stress AB representing the reaction of member AB upon the pins at A
and B. These reactions are equal but oppositely directed. Also observe
that the force polygons for joints A and B can be superimposed because of
the common vector for stress AB. Hence the stress in AB need be repre-
sented by only one line.
The force polygon for joint D can be similarly superimposed on the
common diagram for joints A and B to form the single stress diagram shown
in the figure. One procedure, however, must be observed if the force
polygons are to be superimposed on each other — in constructing the
polygons, the vectors of loads and stresses must be drawn consecutively
in the order in which the loads and stresses are met in a clockwise direction
around each joint.! It is immaterial whether the order of drawing the
1 As shown in Art. 2-3, the order in which the vectors of a force polygon are added is
immaterial. Unless a special sequence is observed, however, the vectors for one joint
will not be in the correct order so that those for succeeding joints can be attached with-
out redrawing some of them.
510 Grapuic METHODS [Chap. XVII

loads and stresses is taken by a clockwise or counterclockwise direction


around a joint. But the same order must be taken around each joint in order
to combine the several force polygons into a common one. This combined
diagram of loads and stresses is known as a Maxwell diagram, after James
Clerk Maxwell, who published the method in 1864.
In constructing a Maxwell diagram for stress analysis, the following
type of Bow’s notation is convenient. The space enclosed between two
adjacent external forces is denoted by a lower-case letter, and each force is
designated by the capital letters of the two spaces separated by its line of
action. The spaces enclosed by the truss members are denoted by numerals.
Thus any vector on a Maxwell diagram which includes a numeral in its
designation is automatically recognized as a stress in a truss member as
distinguished from an applied load whose designation includes only letters.
For example, consider the truss shown in Fig. 17-10a. Load P is de-
noted by the letters a—b; load Q by b-c; reaction R; by c-d, etc. The mem-
bers of the truss are similarly named; thus the upper chord member is b—2.
When constructing a Maxwell diagram, each load or member stress is
designated by the corresponding upper-case letters or numerals at the end
of the force vector.
To construct a Maxwell diagram, we begin by drawing the force polygon
for the truss, considered as a free body; this is shown in Fig. 17-10b. Let
us assume that the clockwise order is to be used throughout in drawing the
vectors acting on a free body. (The reactions on the truss are assumed to
have been previously determined.) Considering joint e—a—/ as a free body,
we construct the polygon of forees H-A-/ by drawing through point A
vector A—1 parallel to member a-—/, and through point F vector /—E parallel
to member /-e. Note that these three vectors follow each other in the
clockwise order in which they are met in going around joint e—a-/, starting
with the known force H—A.
Considering next joint /—a—-b—-2, we begin with the known vectors 1—A
and A-—B (Fig. 17-10c) and, in clockwise order, draw through point B
vector B-2 parallel to member b—2, and through point 7 vector 2-1 parallel
to member 2-1. The intersection of these two vectors establishes point 2.
At joint 2-b-c-3, the known vectors (in clockwise order) are 2-B and
B-C. These are already established. The force polygon 2-B-C-3 (Fig,
17-10d) is completed by drawing through points C and 2 vectors C-3 and
3-2 parallel respectively to members c-3 and 3-2, thereby establishing
point 3 on the Maxwell diagram.
Finally, joint 3-c-d-c is considered as a free body. Here there is only one
unknown, that in member e-3. Since points H and 3 are already estab-
lished, the accuracy of the construction may be checked by drawing vector
E-3 (Fig. 17-10e) and noting whether it is parallel to member e-3 as it
Art. 17-6] Graphic Analysis of Trusses. Maxwell Diagrams oLt
a

D c
Fic. 17-10. — Construction of a Maxwell diagram.

should be. Although joint e-/—2-3 has not been considered a free body, it
is represented by the force polygon H-1—2-3.
The stress in any member can now be obtained by scaling the length of
the corresponding line in the completed Maxwell diagram (Fig. 17-10e).
The character of the stress in any member is obtained by using Bow’s
notation in the following way. Select a joint and name a member by read-
512 GrapHic MretuHops [Chap. XVII

ing the letters in a clockwise direction around the joint. Now, maintaining
the same order of letters, read the letters which designate this member’s
stress in the Maxwell diagram. The reaction of the member upon this
joint is in the direction in which the stress is read. For example, consider
the upper chord member which is read as b—2 at joint /-a-b-2 and as 2-b
at joint 2-b-c-3. In the Maxwell diagram (Fig. 17-10e), reading this mem-
ber in direction B toward 2 shows that the stress acts toward joint 1—a-b-2,
whereas reading it in direction 2 toward B shows that it acts toward joint
2-b-c-8. In either case, the stress in member b—2 is compressive, according
to the convention established in Art. 4-2.
A convenient aid in drawing a Maxwell diagram is to observe that there
is a corresponding point in the diagram for each space in the truss diagram.
Furthermore, through each point on the Maxwell diagram must pass lines
parallel to all the lines that bound the corresponding space in the truss
diagram. Thus the Maxwell diagram can be drawn by starting with the
force polygon A-~B-C—D-E and drawing through point EF lines parallel to
all those bounding space e in Fig. 17-10a. By doing the same thing at points
A and C, points 7 and 3 will be established. Then lines through points 7
and 3 drawn parallel to all those that bound spaces / and 3 will determine
point 2 on the Maxwell diagram. The accuracy of the construction can
be checked by observing whether stress B—2 is parallel to member b-2.

ILLUSTRATIVE PROBLEM
1715. The triangular Fink truss, loaded as in Fig. 17-11, is supported by a roller
at A anda hinge at O. Determine graphically the stress in each member of the truss.
Solution: The truss diagram is first drawn to a convenient scale. To avoid con-
fusion with the letters denoting the joints, use a Bow’s notation in which the spaces
between loads and truss members are all described by numerals instead of letters.”
The Maxwell diagram is numbered to correspond.
Since the loading is symmetrical, the reaction at A, denoted by 9-1 is 35 kips.
Also as a result of symmetry, only the stresses in one-half of the truss need be
found. Accordingly, start the Maxwell diagram with the force polygon 9-1—2-3-4,
using a clockwise order.
Following the method outlined in the article, locate point 10 on the Maxwell
diagram by drawing lines 1-10 and 10-9 through points / and 9 parallel to members
AB and AC respectively. Reading the members in clockwise order on the Maxwell
diagram indicates that AB and AC act on pin A as shown by the arrows on the
truss diagram. Accordingly, AB isin compression and AC isin tension. In asimilar
way, locate points 11 and 12, and mark the stress directions on the truss diagram.
? This departure from the accepted convention described above is adopted here be-
cause of the convenience of describing a particular joint by a single letter, and because
of the student’s familiarity with defining a member in terms of the joints that it con-
nects. This shows that there is nothing sacred about the type of notation adopted,
although some admittedly have advantages over others.
Art. 17-6]
ee Graphic ee
Analysis of Trusses. Maxwell Diagrams
ee a Oe 513
10 kips

It now is evident that all the remaining joints of the left half of the truss have more
than two unknowns. If an analytical solution were being made, this would happen
with the method of joints. It would be necessary to take a section a—a and compute
the stress in FH (4-15) by the method of moments. The solution by the method
of joints could then be completed.
The same procedure can be accomplished graphically as follows: If the stress
in FH is found by the method of sections (by taking moments about point £), it is
514 GRAPHIC MrTHops [Chap. XVII

evident that this stress will be independent of the arrangement of the members to
the left of section a—a. Therefore if members /G and DG are temporarily replaced
by a substitute member FE (shown dotted), there will be no change of stress in FH.
With a substitute member, the stress in FH can be found by the Maxwell diagram
because there will be only two unknowns at joint D. With member DG (13-14)
assumed removed, construct polygon 12-11-2-3-X. (The substitute member
creates a new space x, bounded by joints D, F, and #.) Then at joint F, with
member FG (14-15) assumed removed, construct polygon X—3-4-15, thereby lo-
cating point 164.
With point 15 located on the Maxwell diagram, the substitute member may
be discarded and the regular construction resumed. In some cases, it may be
necessary to substitute two members for three or three for four, in order to complete
the graphic solution.
PROBLEMS
1716. Determine graphically the forces in the truss described in Prob. 413 and
repeated here as Fig. P-1716.

5200 1b
D Fic. P-1716.

1717. Determine graphically the forces in all members of the truss described in
Prob. 414 and shown here again as Fig. P-1717.

300 Ib 300 lb 900 Ib


Tee 1AM Ali7(

1718. Determine graphically the forces in all members of the nacelle truss
shown in Fig. P-1718.
1719. Determine graphically the forces in each member of the triangular Fink
truss shown in Fig. P-1719.
Ans. DF = 1200 lb C; DG = 173 1b T; EG = 346 lb T; EK = 693 lbT
Art. 17-6] Graphic Analysis of Trusses. Maxwell Diagrams 515

600lb 12001b 1200lb 12001b

200 Ib

Fig. P-1719.

1720. Determine graphically the force in each member of the triangular Howe
truss shown in Fig. P-1720.
Ans. BD = 800 lb C; DG = 264 lb C; FG = 600 lb T

200 Ib 200 lb 200 lb 200 Ib 200 lb

6 panels at 10’= 60’


Fie. P-1720.
516 GrapHic METHODS (Chap. XVII

1721. Determine graphically the force in each member of the triangular Pratt
truss shown in Fig. P-1721.
Ans. BD = 900lbC; EF = 336 lb T; EG = 450 lbT

200 Ib 200 lb 200 lb 200 lb 200 lb

6 panels at 10'=60'
Fig. P-1721.

SUMMARY
The resultant of any coplanar force system can be determined in mag-
nitude and direction by the tip-to-tail addition of the free vectors of the
forces. The force polygon thus created does not, however, determine the
position of the resultant force. This position is determined by applying the
parallelogram law in the following manner:
Select pairs of equal, opposite, and collinear components of forces com-
posing the force polygon by drawing rays from a pole O to the ends of the
force vectors. These rays form the pairs of components into which the
given forces are resolved.
Start at any convenient point on the action line of a force and resolve it
into components parallel to the rays drawn to its ends. These components
are represented in direction, but not in magnitude, by lines called strings,
At the intersection of a string with the action line of the second force for
which it represents a common component, this second force is also resolved
into components.
This procedure is continued until all forces have been replaced by pairs
of components, all of which, except two, will cancel. These last two inter-
sect in a point on the action line of the resultant. As previously noted, the
magnitude and direction of the resultant are determined from the force
polygon.
If the force polygon closes, the magnitude of the resultant is zero. In
this case either equilibrium exists or the resultant is a couple consisting of
the rays to the closing point of the force polygon and a moment arm which
Summary Ol7

is the perpendicular distance between the parallel strings representing these


rays. However, if the strings coincide, the moment arm will be zero and
the string polygon is said to close. We conclude that for the case in which
both the force polygon and the string polygon close, the resultant is zero
and equilibrium exists.
A Maxwell diagram is used to determine the stress analysis of a truss.
This is made by combining the force polygons acting at each joint into a
common polygon. Note that the same order of loads and stresses must be
taken around each joint in order to combine the several force polygons into
a common one. A convenient help in drawing the Maxwell diagram is to
observe that for each space in the truss diagram there is a corresponding
point in the Maxwell diagram. Furthermore, through each point on this
diagram there must pass lines parallel to all lines bounding the correspond-
ing space in the truss diagram.
= - er
th ase ¥, a oe Th

iY ier bit ‘ '

) toni? geese able tone


va ae

ai ety
rie
Ody ore
on
ufia@iecds hi tpledt ay
vigil ered vil oi ii
i Avid) eng
Tena teat Inflseg 2 wth

: ov Yom — 4
Siok

~ rf
Index

Acceleration, 239 Body, center of gravity of, 188


angular, 313 considered as a particle, 247, 271, 301
constant, 251, 314 freely falling, 253
in curvilinear motion, 291, 296 motion of center of gravity of, 245
in plane motion, 342, 343, 347 rigid, 237
in rectilinear motion, 251, 256, 262 Bow’s notation, 499, 510, 512
in rotation, 314, 317 Brake, differential band, 138
normal component of, 296, 313
of instant center, 356 Center, instantaneous, 354
rectangular components of, 291, 298 acceleration of, 356
tangential component of, 296, 313 application of, 358-360
variable, 256, 262, 317 kinetic energy with respect to, 416, 417
Acceleration-time diagram, 261 loeation of, 355
displacement from, 264 Center of gravity, defined, 168
velocity from, 263 motion of, 245
Adhesion, 115 of bodies, 188
Amplitude of vibration, 464, 466, 479, 491 summary of, 193
Analysis of structures, 82 Center of moments, 25
by method of joints, 85 Center of oscillation, 471
by method of members, 105 Center of percussion, 321
by method of sections, 93 Center of suspension, 470
Angle of friction, 116 Centrifugal inertia force, 302, 328, 329
Angular acceleration, 313 Centroidal rotation, 322
Angular displacement, 311 Centroids, by integration, 171, 190
from acceleration-time curve, 318 by tabular computation, 181, 191
from velocity-time curve, 318 defined, 170
vector representation of, 314 for geometric shapes, 178
Angular impulse, 446 formulas for, 170, 176, 190
Angular momentum, 446, 454-456 importance of, 171
conservation of, 451 of angles and channels, 179
rate of change of, 447, 459 of areas and lines, 169
Angular velocity, 312 of composite figures, 176
from acceleration-time curve, 318 of volumes, 188
Areas, centroid of, see Centroids signs for, 177
moment of inertia for, see Moments of summary of, 193
inertia for areas theorems of Pappus, 185
properties of, 178 Circle, auxiliary, 465, 476, 479
Auxiliary circle, 465, 476, 479 Mohr’s, 217
Axions of mechanics, 4 Circular frequency, 466
Axis, of moments, 25, 149 Coefficient of friction, 116
of rotation, 311; instantaneous, 354 Coefficient of restitution, 442
principal, of inertia, 211, 220 Cohesion, 115
Components, of acceleration, 291, 296, 342
Ballistic pendulum, 440 of displacement, 238, 289, 338
Banking of highway curves, 306 of force, 14, 143
Bearing reactions, 329, 331, 410, 411 of velocity, 291, 342
Belt friction, 136 Composite areas, centroids of, 176
Bodily contact, types of, 49 moment of inertia for, 202
519
520 INDEX

Compound pendulum, 470 Dynamic equilibrium—(Continued)


center of oscillation, 471 in rotation, 327, 328
center of suspension for, 470 application of, 330-332, 410, 411
equivalent length of, 471 in translation, 280, 301
used to determine moment of inertia, Dynamics, applying the principles of, 247
472
Concurrent forces, equilibrium of, 54-56, Effective force on a particle, 243
60, 152, 505 Efficiency, 399
resultant of, 19, 146, 498 Elastic impact, 442
Cone of friction, 117 Engineering mechanics, defined, 3
Conical pendulum, 302 Equilibrium, by graphic methods, 505
Conservation of momentum, 439, 451 defined, 49
Constant acceleration, 251, 314 equivalence of force and moment sum-
Conversion of units, 9 mations for, 61
Counter diagonals, 102 meaning of, 49
Couples, 36 of concurrent forces, 54, 60, 152
effect of, 37 of non-concurrent forces, 72, 157, 505
independence of moment center, 37 of parallel forces, 67
transformations of, 37
Critical speed, 493 Flight of projectiles, 292
Curvilinear motion, displacement in, 289 Force, characteristics of, 4
of a particle, 289 components of, 14, 143
rectangular components of acceleration defined, 4
in, 291 intercepts of line of action of, 28, 44
velocity in, 290, 400 internal and external effects of, 4, 85
Curvilinear translation, 289 magnitude, inclination, and direction of,
dynamic equilibrium in, 301 15, 144, 147
kinetics of, 301 moment of, 24, 29, 149
work-energy in, 400 on a spring, 394
resolved into a force and a couple, 39
D’Alembert’s principle, 244 resultant, 19, 31, 41, 148, 146, 498
applied to plane motion, 373 signs of components of, 14, 146
applied to rotation, 327 units of, 4
applied to translation, 281, 282, 283, 301 Force-displacement diagram, 393
Differential band brake, 138 by superposition, 396
Differentials and integrals, 13 for a spring, 394
Dimensions, check for homogeneity of, 9 Force polygon, 497, 500
Direction cosines, 144 Force systems, classification of, 4
Direction of motion determined, for plane Force-time diagram, 430
motion, 368, 418 displacement from, 433, 447, 450
for rotation, 322 Forced vibrations, 486
for translation, 272, 274 amplitude of, 491
Displacement, defined, 238, 311 critical speed of, 493
from acceleration-time diagram, 264, magnification factor of, 490, 492
318 resonance caused by, 487, 493
from force-time diagram, 433 torsional, 493
from velocity-time diagram, 263, 318 Forces in space, direction cosines of, 144
in plane motion, 338 equilibrium of concurrent, 152
in rotation, 311, 314, 317 equilibrium of non-concurrent, 157
in translation, 251, 256, 260, 289, 292 resultant of, 146
Displacement-time curve, 261, 318 summary of, 166
Dummy diagram, 43 Free-body diagrams, 5, 49
Dynamic equilibrium, defined, 244 method of constructing, 50
explained, 281 table of, 50
in plane motion, 373 Free-rolling bodies, kinematics of, 343
alternate method of, 375 kinetics of, 365
ee
INDEX 521
Free vibrations, 464, 477 Impulse, 427
amplitude of, 464, 466, 479 angular, 446
by work-energy method, 484 linear, 428
frequency of, 464, 467, 478 Impulse-momentum method, in plane mo-
period of, 464, 466, 478 tion, 454
phase angle of, 479 in rotation, 445, 447
Freely falling bodies, 253 in translation, 427, 429, 435
Frequency, 464, 467, 478 summary of, 462
natural circular, 466 Inertia couple, 328, 374
Friction, angle of, 116 Inertia force, 243
belt, 136 centrifugal, 302, 328, 329
coefficient of, 116 in plane motion, 374
defined, 114 in rotation, 328
kinetic, 116 in translation, 280, 302
laws of, 118 tangential, 302, 328, 374
rolling, 140 Instantaneous axis of rotation, 354
screw, 134 Instantaneous center, 354
static, 116 acceleration of, 356
summary of, 141 kinetic energy with respect to, 416, 417
theory of, 114 loeation of, 355
wedge, 130 use of, 358-360 -
Friction force on highway curves, 307 Intercepts of resultant force, 44
Frictionless path, velocities along, 401 Internal force in a truss member, 85
Funicular polygon, 499, 501
Jackserew, 135
Galileo, 237, 241 Jets, dynamic action of, 435, 437
Geometric addition, 6 Joints, method of, 85
Geometric shapes, centroids for, 178
moments of inertia for, 203 Kilowatt, 399
Graphic methods, 497 Kinematics, defined, 2388
acceleration by, 342 graphic interpretation of, 260, 296, 341
analysis of trusses by, 508 of a particle, 238, 289
Bow’s notation for, 499, 510, 512 of curvilinear motion, 289-296
equilibrium by, 505 of free-rolling bodies, 343
force polygon in, use of, 497, 500 of plane motion, 338, 341, 343, 354
Maxwell diagrams for, 508, 510, 512 of rectilinear motion, 251, 256, 260
resultant of coplanar forces by, 19, 498 of rotation, 312, 314, 317
simple harmonic motion by, 465, 476, Kinetic energy, defined, 385
of plane motion, 416
479
string polygon for, 499, 501 of rotation, 404
of translation, 385, 400
summary of, 516
Kinetic friction, 116
velocity by, 341
Kinetics, defined, 238
Graphical differentiation, 262
of a particle, 241
Guldinus, theorem of, 185
of curvilinear translation, 301
Gyroscope, 458
of plane motion, 363, 365, 373
Harmonic motion, simple, 464, 475 of rectilinear translation, 271, 280
Highway curves, banking of, 306 of rotation, 319, 322, 327
Hooke’s law, 394
Laws, of friction, 118
Horsepower, 399
of motion for a particle, 241
Huygens, 237, 470
Lead of square-threaded screw, 134
Ideal angle of banking, 306 Line, centroid of, 170
Impact, direct central, 442 Linear displacement, 238, 289, 338
elastic, 442 from acceleration-time diagram, 264
inelastic, 443 from force-time diagram, 433
oblique, 443 from velocity-time diagram, 263
522
em
INDEX

Linear impulse, 428 Motion—(Continued)


Linear impulse-momentum, 427, 429, 435 of particle, 238, 241, 289
Linear momentum, 428 of projectiles, 292
conservation of, 439 plane, 338
rectilinear, 251, 256, 260
Magnification factor, 490, 492 simple harmonic, 464
Marked member, 86 uniformly accelerated, 251, 314
Mass, defined, 222, 242 Motion curves, 260
Mass moments of inertia, 222 application of, 264, 266-269, 318
by integration, 224 geometric significance of, 263
determined by compound pendulum, relations between, 262
472
for composite bodies, 228 Natural circular frequency, 466
radius of gyration for, 223 Newton’s laws of motion, 241
transfer formula for, 223 Nodal section, 474
units of, 223 Non-centroidal rotation, 327, 409
Maximum and minimum moments of in- Non-concurrent forces, equilibrium of, 72,
ertia, 220 157, 505
Maxwell diagrams, 508, 510, 512 resultant of, 41, 498, 499
Members, marked, 86 Non-coplanar force systems, 143
three-force, 104 equilibrium of concurrent, 152
two-force, 84 equilibrium of non-concurrent, 157
Method, of joints, 85 resultant of, 146
of members, 105 summary of, 166
of sections, 93 Normal acceleration, 296, 313
of solving problems, 8 Notation, Bow’s, 499, 510, 512
Mohr’s circle for moments of inertia, 217 Numerical accuracy, 10
application of, 220
rules for applying, 218
Moment, of area, 171 Oblique impact, 443
sign of, 179 Oscillation, center of, 471
of a couple, 36 Over-all efficiency, 399
of a force, 24, 29, 149
vector representation of, 149 Pappus, theorems of, 185
Moment of inertia of bodies, see Mass Parallel, springs in, 481
moments of inertia Parallel forces, equilibrium of, 67
Moments of inertia for areas, 195 resultant of, 31
by integration, 199 Parallelogram law, 6, 20, 497
by tabular computation, 205 Particle, body considered as, 247, 271, 301
defined, 195 curvilinear motion of, 289
for composite areas, 202 defined, 237
for structural sections, 204 effective force on, 2438
maximum and minimum, 220 laws of motion for, 241
Mohr’s circle for, 217 rectilinear motion of, 238, 251, 256, 260
polar, 196 Pendulum, ballistic, 440
principal axes for, 211, 220 compound, 470
radius of gyration for, 197 conical, 302
transfer formula for, 197 simple, 469
units and signs, 196 torsion, 473
with respect to inclined axes, 215 Percussion, center of, 321
Momentum, 427 Period of vibration, 464, 466, 478
angular, 446 Phase angle, 479
rate of change of, 447 Pitch of square-threaded screw, 134
conservation of, 439, 451 Plane motion, defined, 338
linear, 428 dynamic equilibrium in, 373
Motion, curvilinear, 289 equations of, 363, 365
of center of gravity, 245 impulse-momentum applied to, 454
aS ES SEE EEE EE eee
INDEX eee ee 523
ean

Plane motion—(Continued) Restitution, coefficient of, 442


kinematics of, 341, 348, 354 Resultant effective force, in plane motion,
kinetic energy of, 416 374
kineties of, 363, 365, 373 in rotation, 321
of free-rolling bodies, 343, 365 in translation, 281, 302
summary of, 382 Resultant force, defined, 7
work-energy applied to, 416 Resultant of, concurrent forces, 19, 20,
Point diagram, 247 146, 498
Polar moment of inertia, 196 non-concurrent forces, 72, 157, 505
Polygon, force, 497, 500 parallel forces, 31, 498
string, 499 Reversed effective force, 281
Power, 398 See also Inertia force
Precession, 458, 459 Reversed normal effective force, 302
Principal axes of inertia, 211, 220 Reversed tangential effective force, 302
Principle, d’Alembert’s, 244, 281, 327, 373 Right-hand rule, 149, 314, 447
fundamental, of graphic methods, 499 Rigid body, 3
of conservation of momentum, 439, 451 Rigid structure, 82
of moments, 26, 28, 44, 149 Rolling bodies, kinematics of, 343
of transmissibility, 4, 25 kinetics of, 365
Problems, solution of, 8 Rolling resistance, 140
Product of inertia, defined, 210 Rotation, axis of, 311
relative to inclined axes, 217 centroidal, 322
transfer formula for, 211 center of percussion for, 321
units and signs, 210 characteristics of, 311
with respect to axes of symmetry, 210 defined, 311
Projectiles, flight of, 292 dynamic equilibrium in, 327, 328
Properties, of angles and channels, 179 effective force in, 321
of areas, 178 impulse-momentum applied to, 447
of bodies, 229 kinematics of, 312, 314, 317
of structural sections, 204 kinetic energy of, 404
kinetics of, 319
Radius of gyration, 197, 223, 472 non-centroidal, 327, 409
Rate of change of angular momentum, 447 reference axes for, 319
Rated speed of highway curves, 307 sign convention for, 314, 321
Rectangular components, of acceleration, summary of, 336
291, 298 unsymmetrical, 329
of force, 15, 21, 143 work-energy applied to, 403, 405, 409
of velocity, 291
Rectilinear motion, 251, 256, 260
Rectilinear translation, analysis as a Scalar and vector quantities, 6
particle, 271 Screws, see Square-threaded screws
analysis as a rigid body, 280 Second moment of area, see Moments of
defined, 250 inertia
direction of motion for, 272, 274 Sections, method of, 93
dynamic equilibrium in, 280 Series, springs in, 481
impulse-momentum applied to, 427, 435 Significant figures, 10
kinematics of, 251, 256, 260 Simple harmonic motion, 464
kinetics of, 271, 280 amplitude of, 466
plan of solution, 271 auxiliary circle of, 465
sign convention, 271 frequency of, 466, 467
summary of, 287 period of, 466
work-energy applied to, 387, 393 vector representation of, 475
Redundant members, 102 Simple pendulum, 469
Relative motion, equations of, 340 Simple trusses, assumptions in, 83
Resolution of a force into a force and a construction of, 82
couple, 39 defined, 83
Resonance, 487 loading of, 84
524 INDEX

Simple trusses—(Continued) Three coplanar forces in equilibrium, 61


method of joints applied to, 85 Three-force members, 104, 105
method of sections applied to, 93 Torque axis, 458
redundant members in, 102 Torsion pendulum, 473
Solution of problems, 8 period of, 474
Space forces, see Forces in space Torsional forced vibrations, 493
Speed, 240 Torsional spring constant, 473
critical, 493 Transfer formulas, 197, 211, 223
maximum, on banked highway, 307 Transformations of a couple, 37
rated, of highway, 307 Translation, see Curvilinear translation;
Spin axis, 458 Rectilinear translation
Spring, force on, 394 Transmissibility, principle of, 4, 25
force displacement diagram for, 394, 396 Triangle law, 7, 20
Hooke’s law for, 394 Triangular loading, 34
work done on, 395 Trigometric functions, 12, 13
Spring constant, equivalent, 481 Trusses, see Simple trusses
torsional, 473 Two-force members, 84
Springs, in parallel, 481 rule for tension or compression in, 85
in series, 481 Types of bodily contact, 49
Square-threaded screws, 134
lead of, 134
Unbalanced wheel, 3866
multiple-threaded, 134
work-energy applied to, 419
pitch of, 134 Uniform motion, 252
Static friction, 116
Uniformly accelerated motion, 251, 314
Statics, principles of, 3
Units, absolute system of, 243
String polygon, 499, 501
conversion of, 9
Substitute member, 514
gravitational system of, 243
Summary of, analysis of structures, 112
Unsymmetrical rotation, 329
centroids, 193
curvilinear translation, 309
equilibrium of coplanar forces, 80 Variable acceleration, 256, 262, 317
forces in space, 166 Varignon’s theorem, 26
friction, 141 applications of, 28, 44
graphic methods, 516 Vector addition, 6, 19
impulse and momentum, 462 Vector polygons for velocity and accelera-
mechanical vibrations, 494 tion, 347
moments of inertia, 232 Vector representation, of acceleration, 342,
plane motion, 382 347
principles of dynamics, 248 of moment, 149
rectilinear translation, 287 of simple harmonic motion, 475
resultants of force systems, 47 of velocity, 342, 347
rotation, 336 Velocity, 239
work and energy, 424 angular, 312
Suspension, center of, 470 from instant center, 355
in curvilinear motion, 290
Table of, centroids, 178, 179 in frictionless path, 401
free-body diagrams, 50 in plane motion, 341
moments of inertia, 203, 204, 229 in rectilinear motion, 251, 256, 260
Tabular computation, 181, 191, 205 in rotation, 314, 317
Tangential acceleration, 296, 313 rectangular components of, 291
Velocity-time diagram, 261
Tangential inertia force, 302, 328, 374
displacement from, 263
Tension in truss members, 85
Vibrations, amplitude of, 464, 466, 479,
Theorems of Pappus, 185, 186 491
Theory of friction, 114 defined, 464
Theory-solution method, 8 forced, 486
INDEX 525

Vibrations—(Continued) Work—(Continued)
free, 464, 477, 484 in rotation, 404
frequency of, 464, 467, 478 meaning of, 386
period of, 464, 466, 478 units of, 385
torsional, 473, 493 Work-energy method, advantages of, 388
Volumes, centroids of, 188 applied to curvilinear translation, 400
applied to free vibrations, 484
Watt, 399 applied to plane motion, 416
Wedges, 130 applied to rectilinear translation, 385,
Work, defined, 385 387, 393
done by gravity, 387, 401, 410 applied to rotation, 403, 405, 409
done by variable forces, 393 general plan of using, 388
done on a spring, 395 summary of, 424
in plane motion, 416 use of force-displacement diagram, 394
a:

acy x BOk
Minivan Var Bt aslen fo
foes & «Dep wie’

Gite serapabsdiscos t-s9


. Tae =e wan 9 39
aoe | an AGS ee

wes i - =
te
Sra & ae or, ee

hae eA f

‘ Pf -

te (i ae A, %
-J =

P - -

a Virte? is pestowighs bsSit


: \ J eee Vier SY
x , RPrGen tine@, S487
wv) Sone jolie,
Oy 1G, :
( ree i vey Gt panes teenie
a @ : tr, 2A f 5
‘- ~ ty wf en,
> _ -: =
hw Vee :
< eve (gee iim, 8
i + Seth, EE :
a ae F
= Ma) = Ate te =
meee eT yore’ -,

ho ite 2
: & (Wien ee a.
oar! one eas
eat) aes
eT ee aT ee ee ee ek ee em i re et ae tree
- t D ede Y a =
au
ts Sette:
eae
7
mata 2
paamenioteas
ssa i an
het ere

lore
ests etoie Sete henenehete hetelOiglete
be oteryret
Sepetelstelsteteio! terd tate (otylabectea riotpie
hee,
ot
= se! tate retelerer
Ree le DOoSH te slerpig sielpie
~ Ss
rateteletet
tat e 1974 phperel
elake'g
eryIetecp yegtaseretetoiers Isgintpipebe
esaa tae sate eng ease Sgaina ano
a Sesear
oe arheses get tote rertiptetststete
stp tprets astaretiei
40
Seapets

alalpia
tel ates
Bestelgteiatate eis ni letelaiclelelet
>
4} ofanei
eresraler
ees Riolale

elofeiei@eniestereie
Perens pislal
SS bleton 4 it lete
pg psleletetcietsie
32453en lstclotatasotete
pipte
teee ee leptotole hal ighaies
tae acitcy ernie
reseen pivtehelsg
Pig
cs
tp ‘shel bsaresns
Sur SOH8
a 6394 : reir areare
iciisieesiotci
sess estetstonrpen!
ot
pivigieieitigighg
ee Ae
) ieigh
Epeteaage iataleiatibar ~ pbetateie tts
ippttt
teterstot eti? te iehhn siatarsiateeie
iester ae:
. e! . shots tekstati)rthtT terest tetetergt
ete ele gant.
e+ gietia taeee
55 eigtetetete hne Seiket,
bstatsis teiels oos
Iaheigis Releintoreioisict“ey
stole?aaahty’ape race
sa digoSash£9 4th wh, ane, tieieiede
Sic
E hs ota
tete tetera etenene‘ ;
iphecrtelieiginleletetetetetet eeis!s Midle aie e
atelate SENSIS
te aie
relatos ne thy onpeta siaisle
sleighsint
lslicte ighisise etietie pasicle
itis tate aie tehai
atetysple Utgeprtet siaiete typre elete pastry plethiele: letete ? eheiel pes ch le
salgn
slelerpiogtelete tiotss eretoiclohionionn eieibis
piplhs inl eetistet tePeis eres peeog Pel
rel sisigugia os betel
aeTapeanstely s + ahinieesre : SE
seenhq able TTMON30 ISTE 3 BO oie eretehata
; ne)
* so erie roi<4, Nitasetehitee es SanhpisheIA epee: ape FAAS
dates enseyeeepiriss Sooo
pecsas pieneesisosror ei
: ae tel tate tacnreded
te te acacsnanah Pages
ay se ae AP Ste
plotetetat e!
DeSeseda
Seseres teas 2e Bash Ssea
Ree
x Te ngsa Pem ea h
rnd54 SSeS
oiergth Pata MODEaleoeladibiatehte
at te seatsa
seas ayaa jeseleiopola level
lglelele lsiersingioty eiargigiealates eteteisien pelptetghelc
alus aes
re ra bes ee epee ea eee eaten man este ELE csle)
gahubhE
a igip-ein
sights large
ete!leplatel
fete
phlei
potshelene ee ee Sesto
Se 2 lee eS Sloth paid a Tote,tee
ta
preter” o-4 9g aby ton rats n eseti pers.) piebste tot -pieitisit
io sisig
ol it etste-p-obore; pet 27Ste aes Seas ts le
hye Pe oie Sele acegieletersts
Meatem tipitigivle
SeDepesashssioe te Tgteay
= Tatece' Pela begat
eereteet geen rerelgtereetes
tre there
et
SSK letelotetee
Ssemtoatsars icieiererersiete giateletel
algeeee lelerele etpre
bichibeiinnn
Sek 8 D5 egese ae leioteteletster
sy etetetpietats Velgtatateteicionebmininn
aes AS $4 A oie L> > + BESO Sieistersbt
bg etttebters? aa. aneERG
pa nee
oo 3 giigigigiers: R
sets 4 eo en . esis ralebololgieteters!
si gislpare
telat srislsisietvigns
heisig ps Staton) Shi2s es
wigislelety
e Eteeles ar eyes
Fore! gis sieitielieiet tates a Mloletety ret
phe ionihen a at- oigres
: isl gre5pre hishhtesihteen
2 ote pe Ietosat Seipiiece St
2
ho sheigie
er te wey chet 508m,28 rete
caen
se ohneeee Pee ws
lorclebelerebereiciotel enemas
teeseas Sipe peteeetear Ete he
earedeberste!
Ase seit a . eee! ree ee: me
so!gies Rs
sly tir Voit
ttiphelste naan
una3 ereiteiet _ .
ae,to o@ Peteleetetetets
esa
5558we
2 6 tetons? ta etadutaleheital telat
AAR S
a folelesiplersiete betes : eietess sles teietoneres
ehetet
aera recetese
ghar t tes eis icieeieien
pe ST 3 bataereete
hr g ielarg
IPislgleipacl
pisses ebay
5
alels-o] May

th tetelonste
teeiipielelatetalelstm sis ih ieietst
ies ccletelg
eo ” =
leisighioh in!
site utr SUR Pietstneies
ts S
SSeS Tas tetas bebe
ie lprete
wig pela neisheietsieighohpia?
erate
ase rscan s - atae Sere
uses salsa Sintu 4
fsa Sista aaos Serek ttete tetera
eee
Say gayrs
Tass a
ta tare
y nse see nescseeg aie
sedeteTrasaaeeed *etbia hehehe
eaeaig desgsh seie Beteae bigieigisisic
catans pasesgtecslelelgiob
niecig lary
tetentLate
l
pialgieioieien
r
Tet
er
more pepiprelaigie
ere
ttre
tlatoia: npign
ep ster torts ee * arene tre ee inlghsisteleletareletaterste ieethtepet
st eeep area eti Seipratartatatetele le o te teat neha
iret tenpe estes waginigl
eee aisioiei oinei peeteres
t teente eee
tern tetate Mise ninaitetitlars!
gels locetrsent
eterseialeieretehe) ere Solace )e: pei Soba? pres aeoe Pape e: eleteie
i. c es ‘ete latetaerotate Taietetereiel
ps= aaa aeseseece
es leleteretsiel, Gialeetelgret
hei pieieiS
etetpe feet
ee Age >
asec
estas,tee Petes
ageaagoasc
ngs g2tsneg
eeadn LU
isielaichelessietgtett
eee LE
te taleeo are
8 Sele
# = SSaBe?a, Prater
aie) enteleelalats
16)e)SS
6 >a rete tiara ighyia 2ka atplelotetict
feritisetetety er gis
er aon
Aare nnrarePree a kelare
ata ?
pigs arpiele
Se pit stelsviele siereteteeet
petepit
tortie
2 riPit
sleinieiplarpi
thy ES ulsletetersivi
+ >’ gtlehete:
Sa do de40 * eae ema dare
Shere
ie plepreie
Pi tere
sisttereaters tapangeneage
th theese
tase tetetemian
“ee
e- ne ber
p WER <“
selene Tia tales le
eters sehclehete tvisleipterersie epreeyertetd slatelokebetelakelenteteadsieeietioteil
lelebe|
‘ere
eee fas ag it
NSS Sena ‘ : BOee gies neseeara
+
se aeas
toads
se Maas Madea
Nake dets gnats
eaieisiviel
pi lel ele viel NSTR a
RRIa
a last aaae eke aseaag
oo se SS eg Se MaBe ssena fotstaletatots sepereratebate lane INHG> 205re ges Baae
ps eaatg
eettebe
els ie te teen. lelereietend
aga)peseieigielel plete De +52 tires ttt sithe hp saa ae encas
dageaides
ie Wigislels rararase
rd Paresa
slobsleleleds
isisightl Horansadd
rose istelorsieiei
aeetanitchien arbFoR PECL95%
e-elaleieislo taletetstowiets 5 -s ae4 me —
rihtsecitwctsioeleseisiipa
Pasrrrenees
tei atete 4 terotehecttit otelehonsd thvoseees
Shes leg clog ciglere tela + fe
re poles! ~ SR SStslorslerd-p
ae tete! otsIs ASSESS
peters
ai ities petit ied ee =
eae. ‘ery Ce
SaIEEASMARE SOga
t tots sipiniarsiep
Siete
ppalegtn eae se peePe cre eeipere piete]leet- eleig
Vg lahat J lsleleisidicivslelglentehci loielersteteeet
tatacteiten
RTD
s slllelt
erate 7 aaeatetete:
lela rletep lahotetitete?
ely retetatr Tobey hhhthisrr
pphrcreeserete
re alihotehotohboieteke
see ete
te oe etaress bacnda
tess cena
a Paso
oa Pes n SohppicitiSeterarecrsteryeen thoes sig areterpieteleipi
: ° io) - = piererers’ tglerstetere:
seenpedeeeeegesegagens
cups lagercnersoon
hearse sctcsesegtees
Sinz prea : Sank ; loislataleretticherienn
ena eae
en olelanb
reise ens eceraten
Sp
tTPep
te aatetrstet
paar statetee g iol
peerees J ss eg eg Jap
oy, gate!
sag plese
bp po $6 eo tolers! pleis
Pi ee ttePRT TED inlelen
fetalTit laste ieietei
oe PTs Siplebalpla :
elpieiess
tp 4'9-2'9 ht
e ele
sarnnk STIs esare Hates
de adn
te. np eple-gigiy ao ds ak rt
4 Ke tise
ele ere
a eme
ei ghelgigiele
ate gio lgietsies
bere’ aeeee Relsielslelsiieetteeta
pie
ele, galetse - sie! Sehebe.
ther
eteearetog aLee pe
cco
(x tater
Hea feet Metat
Rie
loathe
3e daas ete “ole-Biahp
t
pale Ss" ele9 >6 0-9) 2 25 Pent
piers ereigel Spee
t 3S statea NESE
ate p pipet
atahtehtolaten, Spte one sitet reteteteinns elsiele! letetyteteteneiehee store
OrbttePeefa.ettee ee eReoats 2Poispislelt
te
thrttotsTatee > pie
PotseEe PAISeele
Tolereheis pieizishstel v7 fs es ae
leletehe.e
> Nets!
= yaa Tete
sie feerpieierr
ei@ligiet
paaaeesonne Beat
rereebetele

aietpleteretelers
2 tists
irstebe
spat
5 = relate
: 5
estes
aa -
On CaaSRA
nese
SSRN Let
a 336 SN BS

late leig
ese 737555 @.
eer
ee

.
IS SE
ile - To? - z
eiptetete
_
SEES
seerripteret
. tate steko “gale

eSrseesess
iteetsis;
Ho « org
tele
em
ir

You might also like